S001

Increased Bleeding Risk with Enoxaparin Venothromboembolism Prophylaxis Compared with Heparin in Patients Undergoing Bariatric Surgery

Audra J. Reiter, MD, MPH 1; Joanne Prinz, RN, BSN1; Alexander P. Nagle, MD1; Eric S. Hungness, MD1; Ezra N. Teitelbaum, MD, MEd1; 1Northwestern University Department of Surgery

Introduction: Perioperative venothromboembolism (VTE) chemoprophylaxis is an established tenant of bariatric surgery. However, few data exist comparing efficacy and bleeding risk of VTE prophylaxis medications. In March 2018, our bariatric program switched from heparin to enoxaparin for perioperative VTE prophylaxis, creating a natural experiment. The objective of this study was to determine if this change in VTE prophylaxis was associated with differing rates of postoperative bleeding and VTE occurrence after bariatric surgery.

Methods: This retrospective cohort study was performed using a prospective clinical database maintained by a trained abstractor for the Metabolic and Bariatric Surgery Accreditation and Quality Improvement Program. All patients 18 years or older who underwent primary bariatric surgery (sleeve gastrectomy (SG) or Roux-en-Y gastric bypass (RYGB)) at a single institution between March 2012 and December 2021 were included. Heparin was given from March 2012 through February 2018 and enoxaparin from March 2018 through December 2021 for pre and postoperative VTE prophylaxis. Postoperative bleeding was defined as requiring a blood transfusion or reoperation for bleeding within 30 days of surgery. Chi-square test was used to test for differences between groups.

Results: There were 2,159 patients who underwent bariatric surgery with 1,324 (61%) patients in the heparin group and 835 (39%) in the enoxaparin group. Overall, 1,505 (70%) patients underwent SG and 654 (30%) RYGB. Most patients were female (n = 1,709, 79%) with a median age of 43 years (interquartile range (IQR): 36–52), and median BMI of 45 (IQR: 41–50). There were no significant differences in demographics or type of procedure (SG vs. RYGB) between the groups. Overall postoperative bleeding occurred more frequently in the enoxaparin group (n = 27, 3%) compared with the heparin group (n = 11, 0.8%) (p < 0.01). Additionally, reoperation for bleeding was more frequent with enoxaparin (enoxaparin 0.8% vs. heparin 0.2%, p = 0.04). In both groups, postoperative bleeding occurred more frequently after RYGB as compared with SG (heparin group: RYGB 1.8% vs. SG 0.4%, p = 0.01, enoxaparin group: RYGB: 5.6% vs. SG 2.1%, p = 0.01). There was no difference in VTE occurrence between the two groups (heparin: n = 14, 1.1%, enoxaparin: n = 5, 0.6% (p = 0.27)).

Conclusion: An institutional change from subcutaneous heparin to enoxaparin for bariatric surgery perioperative VTE prophylaxis was associated with a significant increase in postoperative bleeding, with no difference in VTE complications. Further studies examining the specific dosing and timing of varying VTE prophylaxis regimens are needed to clarify the optimal regimen for minimizing bleeding while protecting against VTE after bariatric surgery.

S002

A Prospective Multicenter, Standard of Care Study of Outpatient Laparoscopic Sleeve Gastrectomy

Daniel Cottam 1;Aurora Pryor, MD, MBA, FACS, FASMBS2; Amit Surve1; Robert Michaelson, MD, PhD, FACS, FASMBS3; Thomas Umbach, MD, FACS4; Michael Williams, MD, FACS5; Hossein Bagshahi, MD6; Laura July, MD4; Racquel Bueno, MD4; Devorah Chock, MD, FACS, FASMBS3; Matthew Apel, MD, FACS, FASMBS4; Christopher Hart, MD, FACS5; William Johnson, MD, FACS5; Brendon Curtis, MD, FACS5; Brian Long, MD, FACS7; Thomas Roshek, MD, FACS7; Amy Rosenbluth, MD2; Konstantinos Spaniolas, MD2; Nick Nicholson, MD, FAC7; Walter Medlin, MD, FACS1; 1Bariatric Medicine Institute; 2Stony Brook University Hospital; 3Northwest Weight & Wellness Center; 4Blossom Bariatrics; 5Atlanta General and Bariatric Surgery Center; 6HB Health PLLC; 7Collin Surgical Group

Background: Outpatient ambulatory management following many surgical procedures is increasing in frequency. Recently, bariatric surgeons have begun to perform outpatient ambulatory surgery. However, there are no prospective outcomes trials to judge the safety and efficacy of this approach for bariatric patients. This study aimed to report on the prospective 30-day outcomes of outpatient primary laparoscopic sleeve gastrectomy (LSG) performed in six U.S. centers.

Methods: This study was funded by a grant from the Society of American Gastrointestinal and Endoscopic Surgeons (SAGES) and has been approved by the Advarra institutional review board (Pro00055990). Six centers from across the U.S. agreed to prospectively enter all LSG performed as an outpatient from August 2021 through September 2022. The data were centralized in a single registry for research purposes (ArborMetrix).

Results: Two hundred eighty-five patients were included in this analysis. The mean preoperative BMI was 43.6 ± 6.1 kg/m2, with 17.5% male and 82.4% female. Hypertension, gastroesophageal reflux disease, obstructive sleep apnea, hyperlipidemia, and diabetes mellitus were seen in 32.6%, 28.4%, 24.9%, 21.4%, and 13.6% patients, respectively. In 15% of patients, a concurrent procedure was performed. No intraoperative complications or conversion to another approach were noted. The 30-day follow-up rate was 100%. The 30-day complication, reoperation, readmission, emergency department visit, and urgent care visit rates were 2.1%, 0.7%, 0.3%, 0%, and 0%, respectively. In regard to short-term complications, three (1%) patients had Clavien-Dindo grade I complications, two (0.7%) patients had grade IIIb complications, and one (0.3%) patient had grade II complications. No mortality and no leaks were noted.

Conclusion: Same-day discharge following LSG seems to be safe in highly selected patients at experienced U.S. centers.

S003

Safety and Cost of Laparoscopic Sleeve Gastrectomy as an Outpatient Procedure at a Large Academic Hospital

Joshua P Landreneau, MD 1; Divyansh Agarwal, MD, PhD2; Elan Witkowski, MD1; Ozanan Meireles, MD1; Matthew Hutter, MD1; Denise Gee, MD1; 1Massachusetts General Hospital, Harvard Medical School; 2Massachusetts General Hospital

Introduction: Laparoscopic sleeve gastrectomy (LSG) is the most common surgical treatment for morbid obesity. While certain specialized ambulatory surgery centers offer LSG on an outpatient basis, patients undergoing LSG at most academic centers are admitted to hospital for initial postoperative convalescence and monitoring. Our institution has begun to offer LSG with same-day discharge (SDD) in select patients, with the belief that a robust home hospital program can achieve similarly excellent outcomes as inpatient admission. We aimed to compare the perioperative outcomes and costs for patients undergoing LSG with inpatient admission versus SDD.

Methods: Eligibility requirements for our institution's LSG with SDD program included BMI less than 50 kg/m2, adequate medical control of comorbidities, residence in local service area, and attestation of adequate social support. All patients enrolled in the SDD program from December 2020 through July 2022 were identified from a prospectively maintained database. Patients enrolled in this pathway were analyzed on an intention-to-treat basis even if ultimately admitted postoperatively. Propensity scoring was used to match these patients 1:1 to those with planned inpatient recovery based on age, BMI, and ASA classification.

Results: Seventy-five patients were enrolled in the LSG with SDD program during the study period. Among these, 62 patients (82.7%) had successful immediate postoperative discharge. Reasons for cancellation of planned SDD included anxiety (n = 5), pain (n = 3), nausea (n = 2), and one patient each with hypotension, urinary retention, and bleeding. The cohort was 72% female with a mean age of 39.6 years and mean BMI of 42.9 kg/m2. After matching, there were no differences in age, BMI, or ASA classification in a comparison group of patients with planned inpatient recovery. Obesity-related comorbidities were similar between the two groups. Serious adverse events occurred in no patients with planned inpatient admission and one patient (1.3%) in the SDD cohort, which was bleeding identified immediately following the procedure requiring reoperation with evacuation of hematoma. There were no readmissions or requirements for outpatient intravenous fluids among patients with SDD, compared to n = 3 (4.0%) and n = 2 (2.7%) in the inpatient cohort, respectively. The total perioperative cost for patients undergoing LSG with planned SDD was 6.1% less than those with inpatient recovery.

Conclusions: With appropriate protocols, LSG with same-day discharge can safely be performed at large academic surgery centers without increased morbidity or need for additional services in the perioperative period. SDD may be associated with decreased costs and allows for more efficient hospital bed allocation.

S004

Assess the Combined Diagnostic Accuracy of Transient Elastography and Visual Liver Score in Assessing NAFLD or NASH and Compare it with Intra-operative Liver Biopsy in Morbidly Obese Patients Undergoing Bariatric Surgery: An Observational Study

Deborshi Sharma 1; Gautam Anand, Dr1; 1Lady Hardinge Medical College

Introduction: Non-alcoholic fatty liver disease (NAFLD) is one of the major causes of chronic liver disease worldwide. The prevalence in general population varies between 25 and 30%, and increases up to 70–90% in the morbidly obese. The gold standard for diagnosis of NAFLD and NASH is liver biopsy. In this study, we have combined preoperative transient elasto-graphy (TE) and intra-operative standardized visual liver score (VLS) compared with intra-operative liver biopsy for prediction of NAFLD and NASH in patients undergoing bariatric surgery.

Aim: To evaluate the combined diagnostic accuracy of Transient Elastography and Visual liver score in assessing NAFLD or NASH and compare it with liver biopsy in morbidly obese patients undergoing bariatric surgery.

Methods: Prospective cohort study of 70 morbidly obese patients undergoing bariatric surgery from October 2019 to October 2021 in a tertiary care medical college. Sample size was based on the sensitivity and specificity of VLS in predicting NAFLD and NASH (70% and 79.5%, Geraldine JO et al.) and precision of 20%, 80% power of study and 5% level of significance. All patient underwent preoperative workup including TE. Intraoperative VLS based on liver color, size and surface was calculated. Both findings of TE and VLS were compared with histology from intraoperative liver biopsy.

Result: There were 44(62.85%) patients with histologically diagnosed NAFLD (≥ S1). Significant steatosis was seen in 20 (28.57%) while significant fibrosis was seen in 18(25.71%). The AUROC for TE for diagnosis of NAFLD in the study was excellent (0.844, p = 0.001). At an optimal cut-off of 8.1, it had a PPV of 92.9% and diagnostic accuracy (DA) of 90.6%. Total VLS had a sensitivity of 90.9% for NAFLD. The combined sensitivity of TE + VLS was 95.5% which was excellent for ruling out the NAFLD. Fourteen (20%) patients had NASH. VLS had the highest DA of 97% in identifying NASH in comparison to TE. The AUROC for VLS was 0.987, p =  < 0.001 with sensitivity of 100%. The overall sensitivity of combined TE and VLS was 100% with a NPV of 100%.

Conclusion: Non Invasive preoperative test (TE) when combined with intraoperative VLS is comparable to liver biopsy and can be used for the diagnosis of NAFLD and NASH in patients with morbid obesity undergoing bariatric surgery.

S005

Prevalence and Risk Factors for Secondary Hyperparathyroidism (SHPT) in Patients Undergoing Bariatric Surgery

Cooper Little 1; Dustin Cochran1; McKenna Box1; James Bothwell1; Grayson Domingue, MD1; Kenneth Stewart, PhD1; Fernando Mier-Giraud, MD1; Rishi Thakral, MD, MSc1; Laura E Fischer, MD, MS1; 1University of Oklahoma College of Medicine

Introduction: Secondary hyperparathyroidism (SHPT) after bariatric surgery is common with a 5-year prevalence of 63%. SHPT has significant adverse implications on bone metabolism, including increased risk of osteoporosis and fragility fractures. Our aim was to characterize SHPT prevalence and identify risk factors at the University of Oklahoma.

Methods: We performed a single institution, retrospective chart review of patients who underwent Roux-en-Y gastric bypass (RYGB) or vertical sleeve gastrectomy (VSG) from June 2017 through June 2021. Demographic and clinical data were collected, including serum values of parathyroid hormone (PTH), calcium (Ca), and vitamin D3 (D3) at enrollment, 3, 6, and 12-months post-operatively. Chi-square or Fisher's exact tests were used to analyze categorical data and Student's t-test for continuous data. Univariate and multivariable analyses assessed risk factors for SHPT. P-values < 0.05 were considered significant.

Results: We retrospectively collected data on 325 patients after institutional IRB approval. Mean BMI decreased from 46.3 ± 8.1 pre-operatively to 30.7 ± 5.4 at 1-year (33.9% total body weight lost). SHPT prevalence at any time-point was 77.5%. 70% had pre-operative SHPT (preSHPT) which correlated with higher BMI (44.2 vs 42.3, p = 0.008) while 49.3% had 1-year SHPT (BMI comparison NS). Patients with D3 deficiency (p = 0.002), calcium deficiency (p < 0.0001), or joint replacement history (p = 0.19) had higher baseline SHPT (p = 0.002, p = 0.0001, p = 0.019). Those with diabetes (p = 0.004), NASH (p = 0.048), or CKD (p = 0.027) were more likely to have SHPT at any time-point in multivariable analysis. PreSHPT resolved with treatment in 40.7% of patients, while 19.8% of patients had recurrent SHPT. Overall SHPT resolution was 33% at 1-year. 13.3% had new post-operative SHPT (postSHPT). Of these, only 27.6% had resolution at 1-year. D3 deficiency and higher BMI correlated with SHPT at 3 and 6 months, but only D3 deficiency correlated with SHPT at 1-year. D3 deficiency was significantly higher in baseline SHPT (75.8% vs 2.9%, p < 0.0001) suggesting an alternative mechanism for the disease process.

Conclusions: SHPT prevalence is elevated even before undergoing bariatric surgery. Most patients did not have SHPT resolution despite appropriate treatment. PostSHPT patients were significantly less likely to have D3 deficiency than preSHPT, suggesting different disease mechanisms before and after surgery. Further studies are needed to develop an effective treatment strategy to mitigate the long-term adverse effects of this disease.

*Funding: This project was funded in part by the 2021 SAGES Medical Student Summer Research Award ($2,000) awarded to Cooper Little on June 18, 2021.

S006

Standalone Hiatal Hernia Repair is Effective Treatment for Reflux After Sleeve Gastrectomy

Karan R. Chhabra, MD, MSc 1; Annabelle Jones, MD1; Yali Liu, PAC1; David Spector, MD1; 1Brigham and Women's Hospital

Introduction: Sleeve gastrectomy is the most common bariatric operation in the US, but carries a risk of worsening reflux in up to 30% of patients. The standard treatment for GERD after sleeve gastrectomy is conversion to gastric bypass, however gastric bypass carries major long-term complications. We hypothesize that, when a hiatal hernia is present in patients with post-sleeve gastrectomy reflux, repairing the hiatal hernia may address reflux and obviate conversion to gastric bypass.

Methods: Single center retrospective study of patients who underwent laparoscopic sleeve gastrectomy complicated by postoperative GERD and were found to have hiatal hernias. All hiatal hernias were repaired laparoscopically at an academic referral center. Data were obtained by chart review and analyses were performed in Stata 15.

Results: Of 33 patients who underwent hiatal hernia repair from July 2015 – July 2022, the mean age was 46 and 91% were female. Mean BMI was 40.9 at the time of the index sleeve gastrectomy and 32.6 at the time of hiatal hernia repair. Thirteen (39%) of hiatal hernias were recurrences after sleeve gastrectomies with concurrent hiatal hernia repairs. Thirty-two repairs (97%) were performed with biologic mesh reinforcement. Data were reviewed from a median 22.2 months since surgery (interquartile range 16.-32 months). Of 33 patients, 85% (95% CI 68%-94%) experienced immediate improvement in reflux symptoms and 91% (95% CI 74%-97%) did not require conversion to gastric bypass for refractory reflux. In the long term, 13 were weaned completely off antireflux medications, and another 4 experienced long-term symptomatic improvement in reflux symptoms, for a total of 17/33 patients (51.5%, 95% CI 34%-68%) with durable long-term improvement. Of the 12 with ongoing reflux symptoms, 4 (33%) had a recurrent hiatal hernia, and are being assessed for a possible second repair.

Conclusions: For patients with reflux after sleeve gastrectomy who are found to have hiatal hernias, repairing the hiatal hernia eliminates the need for conversion to gastric bypass in the vast majority of patients. Conversion to gastric bypass entails long-term risks. Standalone hiatal hernia repair should be considered an effective surgical option for post-sleeve gastrectomy reflux.

S007

Long-term Follow-Up in a Sleeve Gastrectomy Leak Cohort: Outcomes of Endoscopic and Salvage Surgical Management

Alexis Deffain, MD 1; Felix Thibeault, MD1; Abdullah Almunifi, MD2; Waël Dimassi, MD1; Pierre Garneau, MD1; Ronald Denis, MD1; Anne-Sophie Studer, MD1; Radu Pescarus, MD1; 1Hopital du Sacré Coeur de Montréal; 2Department of Surgery, College of Medicine, Majmaah University, Saudi Arabia

Introduction: Sleeve gastrectomy is the most frequently performed bariatric procedure worldwide. The most worrisome postoperative complication is the gastric leak. The objective of this study is to examine the efficacy and morbidity of the different therapeutic strategies undertaken, as well as the long-term outcomes of a sleeve leak cohort.

Methods: This is a retrospective observational study that included 32 patients treated for sleeve leaks between 09/2014 and 07/2022 treated in our tertiary bariatric surgery center. All patients were offered initial endoscopic management, followed by surgical treatment when the endoscopic management failed. An analysis of the procedures and their efficacy is performed.

Results: Average follow-up was 31 months [1.9–96.1]. Average age was 43 years. There were 26 (76%) women and 6 (19%) men. The mean preoperative BMI was 44.2 kg/m2. 8 (25%) patients required emergency surgical management at the time of diagnosis. 25 (78%) patients were treated exclusively by endoscopy afterwards and 7 (22%) patients required subsequent revisional surgery. In 60% of cases the axis/caliber of the sleeve was considered to be abnormal. When looking specifically at the revisional surgery group, 6/7 (86%) of patients had a sleeve stenosis. Overall, the median number of endoscopic procedures was 2 [1–12]. In addition, an achalasia balloon dilatation was performed in 28% of the procedures to alleviate a severe distal obstruction. The endoscopic armamentarium included pigtails (35%), long fully-covered bariatric stents (35%), septoplasty (12%), endovac (7%) and partially-covered stents (6%). Complications include esophageal stenosis (2), stent migration (1) and kinking (1) and pigtail migration (3). Beyond 4 endoscopic procedures, the success rate was 0%. Majority (86%) of patients that failed endoscopic treatment were referred more than 30 days after index surgery. Indeed, the success rate of endoscopic management drops to 33% when treatment begins more than 30 days after the index surgery. Laparoscopic management included in 67% cases proximal gastrectomy and Roux-en-Y esophagojejunal anastomosis, 17% a Roux-en-Y fistulo-jejunostomy and 17% a classic Roux-en-Y gastric bypass proximal to a sleeve stenosis. At maximal follow-up, the average BMI was 31 kg/m2, 6% of poorly controlled GERD and no malnutrition cases.

Conclusions: Endoscopic management was successful in 78% cases. After 4 failed endoscopic procedures, surgical approach should be considered. Delayed (> 30 days post-op) endoscopic management as well as the presence of concomitant distal sleeve stenosis appear to be risk factors for failure of endoscopic management. In the long term, treated patients seem to reach the normal postoperative course without significant morbidity.

S008

Comparable Improvement and Resolution of Obesity Related Comorbidities in Endoscopic Sleeve Gastroplasty vs Laparoscopic Sleeve Gastrectomy: Single Center Study

Stephanie Joseph, MD, MPH 1; Vanessa N Vandruff, MD2; Kristine Kuchta, MS3; Julia R Amundsin, MD, MPH2; Simon Che, MD3; Shun Ishii, MD3; Chris Zimmermann, MD3; Mason Hedberg, MD, MS3; Woody Denham3; John Linn, MD3; Michael B Ujiki, MD, FACS3; 1Wayne State University; 2The University of Chicago; 3NorthShore University HealthSystem

Background: Despite excellent surgical outcomes, a minority of qualified patients undergo weight loss surgery. Endoscopic Sleeve Gastroplasty (ESG), an incisionless procedure, has proven to be effective in achieving weight-loss and comorbidity improvement. We aim to compare outcomes of ESG to that of Laparoscopic Sleeve Gastroplasty (LSG).

Method: A retrospective review of a prospective database of patients who underwent ESG and LSG at NorthShore University HealthSystem from 2016 to 2022 was completed. Demographic and outcome data was extracted and analyzed from the NorthShore Bariatric Quality Database. Pre and post-surgical data for ESG and LSG were compared including Total Body Weight Loss (%TBWL) and Excess Weight Loss (%EWL) using chi-square and two-sample t-tests. Improvement or resolution of obesity related comorbidities were also assessed. Statistical significance was set at p < 0.05.

Results: A Total of 100 LSG and 49 ESG patients were reviewed. ESG patients were older. (48 ± 10 vs. 44 ± 12, p = 0.030). Initial BMI of ESG patients was significantly lower (35.4 ± 3.2 vs. 45.8 ± 7.3, p < 0.001). Median length of stay after ESG was 0 days and LSG 2 days (P < 0.0001). Major adverse events were seen less frequently in ESG group (1/49; 2.04% LSG group had 5 Major adverse events (5/100; 5.00%). LSG achieved more significant total body and excess weight loss at 6 months, 1 year and 2 years post op compared to ESG (LSG: %TBWL 26.0 ± 10.2, 28.2 ± 10.2, 25.9 ± 7.4, ESG: %TBWL 16.2 ± 7.0, 12.3 ± 9.8, & 11.9 ± 7.9, all p < 0.0001); LSG: %EWL 46.4 ± 17.8, 50.1 ± 18.6, 47.9 ± 15.4; ESG: %EWL 36.5 ± 16.2, 27.8 ± 23.0, 25.9 ± 16.9, p = 0.018, 0.0001, 0.001 respectively). There were no statistically significant differences in the percent of improvement or resolution of diabetes after LSG vs ESG (76.9%, 50.0% p = 0.7177; 53.8%, 33.3% p = 0.629), obstructive sleep apnea (27.3%, 28.6% p = 0.717; 22.7%, 28.6% p = 0.798) hyperlipidemia (55.0%, 50.0% = 0.752; 45.0%, 50.0% p = 0.752), or hypertension (61.3%, 45.5% p = 0.483; 29.0%, 36.4% p = 0.713) between LSG and ESG respectively. Rate of GERD improvement was statistically significantly greater in LSG than ESG at 2 years post op. (61.5% vs. 14.3%, p = 0.043). No statistical significance in complete resolution of GERD was noted (53.8% vs.14.3%, p = 0.158).

Conclusion: Endoscopic Sleeve Gastroplasty is an effective procedure for weight loss and comorbidity resolution. Obesity related comorbidities are comparably improved and resolved following ESG vs LSG. Although the weight loss in LSG is significantly higher, after ESG, patients can expect a shorter hospital length of stay and a lower rate of complications. ESG continues to show promise for long term weight loss and improvement in health.

S009

A Study Analyzing Outcomes After Bariatric Surgery by Primary Language

Claire B. Rosen, MD 1; Sanford E. Roberts, MD1; James Sharpe, MS1; Victoria Gershuni, MD, MSMTR1; Maria S. Altieri, MD, MS1; Rachel R. Kelz, MD, MSCE, MBA1; 1Hospital of the University of Pennsylvania

Introduction: Communication is key to success in bariatric surgery. This study aims to understand how outcomes after bariatric surgery differ between patients with a non-English primary language and those with English as their primary language.

Methods: This retrospective, observational cohort study of bariatric surgery patients age ≥ 18 years utilized the Michigan, Maryland, and New Jersey State Inpatient Databases and State Ambulatory Surgery and Services Databases, 2016 to 2018. Patients were classified by primary spoken language: English and non-English. Primary outcome was complications. Secondary outcomes included length of stay (LOS) and cost, with cost calculated using cost-to-charge ratios provided by Healthcare Cost and Utilization Project and converted to 2018 United States dollars using the Consumer Price Index for medical care. Multivariable regression models (logistic, Poisson, and robust) were used to examine associations between primary language and outcomes. Given the uneven distribution of race by primary language, interaction terms were used to examine conditional effects of race.

Results: Among 56,330 bariatric surgery patients, 2,146 (4%) spoke a non-English primary language. Patient characteristics and unadjusted outcomes significantly differed by primary language (Table 1). After adjustment, non-English primary language was not associated with significantly increased odds of complications (OR 1.38, p = 0.078) though was associated with significantly longer LOS (+ 0.11 ± 0.05 days, p = 0.03). Mean healthcare costs were not significantly higher among non-primarily English speaking patients (+ $507 ± 311, p = 0.103). There were no significant conditional effects of race among outcomes.

Conclusion: Controlling for observable characteristics, non-English primary language was associated with significantly longer LOS, but not greater risk of complications or costs despite a trend towards statistical significance. Further, there were no significant conditional effects of race on any outcomes. As such, this study suggests merit in further investigation regarding the association between non-English primary language and adverse outcomes after bariatric surgery.

Table 1 Representative covariates/univariate outcomes by primary language

Covariates/univariate outcomes

English

Non-English

p-value

Age, mean(SD)

44.3(12.4)

44.6(12.2)

0.205

Race, n(%)

  

 < 0.001

 White

33,622(62)

469(22)

 

 Black

12,254(23)

87(4)

 

 Other

8,308(15)

1590(74)

 

Complications, n(%)

882(1.6)

42(2)

0.275

LOS (days), mean(SD)

1.86(2.95)

1.85(4.1)

 < 0.001

Cost, mean(SD)

$13,187($13,347)

$18,665($46,991)

 < 0.001

S010

Internal Hernia Trends Following Gastric Bypass Surgery

Victoria Ende, BS 1; Nina Devas, BS1; Xiaoyue Zhang, MS1; Jie Yang, PhD1; Aurora Pryor, MD, MBA, FACS, FASMBS1; 1SUNY—University at Stony Brook

Introduction: Internal hernia is a well-known complication of laparoscopic Roux-en-Y gastric bypass (LRYGB), with reported rates ~ 5% within three months to three years after surgery. Internal hernia through a mesenteric defect can lead to small bowel obstruction. Mesenteric defects began to be more routinely closed, often considered standard practice by 2010. To our knowledge, there are no large population-based studies looking at rates of internal hernia post-LRYGB from multiple centers, with a follow-up period greater than three years. This study utilizes a statewide database to better characterize the trends of LRYGB and internal hernia over the last two decades in multiple centers.

Methods: Gastric bypass procedure records between January 2005 and September 2015 were extracted from the New York SPARCS database. Exclusion criteria included age < 18, in-hospital deaths, bariatric revision procedures, and internal hernia repair/resection during the same hospitalization as LRYGB. The time to internal hernias was calculated from the initial hospital stay for LRYGB to the admission date of the first internal hernia repair/resection record. Cumulative incidence of internal hernia repair/resection post-LRYGB with in-hospital death as a competing risk event were estimated. A multivariable proportional sub-distribution hazards model with adjustment of potential confounding factors was utilized to analyze the trend of incidence within three years after bypass.

Results: 46,918 gastric bypass patients were identified between 2005–2015, with 2,950 (6.29%) undergoing internal hernia repair/resection post-LRYGB by the end of 2018. The cumulative incidence of internal hernia repair/resection at the 3rd year post-gastric bypass was 4.80% (95% CI: 4.59%—5.02%). By the end of the 13th year, which was the longest follow-up period in this study, the cumulative incidence of internal hernia was 12.00% (95% CI: 11.30%—12.70%). Overall, there was a decreasing trend over time of undergoing internal hernia repair/resection within three years post-gastric bypass (HR = 0.94, 95% CI: 0.93—0.96), after adjusting for confounding factors. Interestingly, only 72.78% of patients had bypass and internal hernia surgeries at the same hospital, supporting the need for large studies like this to adequately assess hernia incidence.

Conclusion: This multi-center study maintains the rate of internal hernia following LRYGB reported in smaller studies and provides a longer follow-up period demonstrating decreasing occurrences of internal hernia after bypass as a function of time. This supports the benefit of routine mesenteric closure.

S011

Opioid Prescription and Consumption After Hospital Discharge Following Laparoscopic Bariatric Surgery: A Prospective Cohort Study

Shrieda Jain, BSc 1; Maxime L Gagner, BSc1; Anne-Sophie Poirier, BSc1; Hiba Elhaj, MSc1; Pepa Kaneva, MSc1; Naser Alali, MD1; Lawrence Lee, MD, PhD1; Mohsen Alhashemi, MD1; Liane Feldman, MD1; Michel Gagner, MD2; Amin Andalib, MD1; Julio F Fiore, PhD1; 1Steinberg-Bernstein Centre for Minimally Invasive Surgery and Innovation, McGill University Health Centre; 2Clinique Michel Gagner (Westmount Square Surgical Center)

Introduction: The primary aim of this study was to assess the extent to which opioids prescribed at discharge after bariatric surgery are consumed by patients. In the current opioid crisis, patients undergoing bariatric surgery may be at an increased risk of harms related to opioid overconsumption due to impulse control deficits and postoperative changes in drug absorption mechanisms. Therefore, surgeons should minimize these risks while ensuring effective postoperative pain management. While previous studies have addressed patterns of opioid prescribing and consumption in other surgical populations, there is a lack of research focused on bariatric surgery. This knowledge gap prevents the development of evidence-based quality improvement initiatives to optimize analgesia prescribing for bariatric patients.

Methods and Procedures: In this multicenter prospective cohort study, we recruited adult patients (≥ 18yo) undergoing laparoscopic bariatric surgery at two academic hospitals and a private surgical clinic in Montreal, Canada. Preoperative assessments included demographic information, Pain Catastrophizing Scale, Patient Activation Measure, and PROMIS-29 anxiety and depression scales. Information regarding surgical and perioperative care interventions (including discharge prescriptions) were obtained from medical records. Self-reported opioid consumption was assessed weekly up to 30 days after discharge. The total number of opioid pills prescribed and consumed were compared using Wilcoxon signed-rank test. Multiple linear regression (with log transformation) was used to identify predictors of post-discharge opioid consumption.

Results: We analyzed 351 patients (mean age 44 ± 11, BMI 45 ± 8.0, 77% female, 71% sleeve gastrectomy, length of stay 1.6 ± 0.6). The quantity of opioids prescribed at discharge (median 15 pills [IQR 15–16], 112 MMEs [IQR 80–112]) was significantly higher than patient-reported consumption (median 1 pill [IQR 0–5], 7.5 MMEs [IQR 0–35.5]) (p < 0.001). Overall, 48% of patients did not take any opioids post-discharge and 77% of the opioid pills prescribed were unused. In multivariate analysis, higher preoperative depression score (+ 0.03 [95%CI + 0.01 to + 0.04]), preoperative BMI (+ 0.01 [95%CI + 0.0004 to + 0.03]), and current smoking (+ 0.72 [95%CI + 0.22 to + 1.23]) were associated with increased post-discharge opioid consumption; sleeve gastrectomy was associated with decreased opioid consumption in comparison to bypass procedures (-0.31 [95%CI -0.57 to -0.07]).

Conclusion: This study supports that most opioid pills prescribed to bariatric surgery patients at discharge were not consumed. We identified patient and procedure characteristics that may predict opioid consumption. Our findings suggest that post-discharge analgesia strategies with minimal or no opioids may be feasible and should be further investigated in future research aimed at mitigating opioid-related harms after bariatric surgery.

S012

Perioperative Use of Low Dose Haloperidol Safely Reduces Episodes of Postoperative Nausea/Vomiting and Length of Stay Following Elective Minimally Invasive Bariatric Surgery

Priscilla Lam, MD 1; Nicholas Druar, MD, MPH1; Santosh Swaminathan, MD1; Tian Sheng Ng, MD1; Shohan Shetty, MD, FACS1; 1Saint Mary's Hospital

Introduction: While total intravenous anesthesia protocols include Decadron and Ondansetron prophylaxis, bariatric patients continue to be considered high risk for postoperative nausea/vomiting (PONV). A multimodal approach is recommended by the Bariatric Enhanced Recovery After Surgery (ERAS) Society however, there remains a lack of consensus on the optimal strategy to manage PONV in these patients. As a community hospital bariatric center of excellence, we sought to identify which perioperative strategies were most effective at reducing episodes of PONV and the effect on length of stay (LOS).

Methods: An institutional bariatric database was created by retrospectively reviewing patients undergoing elective minimally invasive bariatric procedures from 2018–2022. Demographic data included age, gender, preoperative body mass index (BMI) and procedure type. Primary endpoints included reported episodes of PONV, total doses of Ondansetron administered, need for a second (rescue) antiemetic, complication rate (ICU transfer, readmission within 30 days) and LOS. Fisher's exact test and Mann–Whitney test were used to evaluate the effect of perioperative management on various endpoints.

Results: A total of 320 patients were reviewed with Haloperidol being utilized in 17.5% of all patients. Patients receiving Haloperidol were less likely to require Ondansetron outside of the perioperative period (26.8% vs 50.6%, p = 0.004), experienced less PONV (30.4% vs 61.3%%, p < 0.0001) and had decreased median length of stay (22.66 vs 40.33 h, p < 0.0001).

Conclusions: Addition of low dose Haloperidol to Bariatric ERAS protocols decreases episodes of PONV and need for additional antiemetic coverage resulting in a significantly shorter length of stay, increasing the likelihood of discharge on postoperative day 1.

Table 1 Use of Haloperidol decreases episodes of PONV, overall antiemetic requirement and shortens length of stay

Variable

No haloperidol

Haloperidol

p-value

Male

6.4%

9.1%

 > 0.999

Female

93.6%

90.9%

 > 0.999

Average Age (years)

42.5

42.2

0.944

Average BMI

42.98

43.49

0.406

Complication Rate

11.3%

3.6%

0.13

Reported Episodes of PONV

50.45%

15.0%

0.03

Need for ≥ 3 doses of Ondansetron

50.57%

26.79%

0.004

Need for Rescue Antiemetic

35.71%

19.64%

0.02

Length of Stay (hours)

40.33

22.66

 < 0.0001

S013

SAGES Acquisition of Data for Outcomes and Procedure Transfer (ADOPT) Course for Foregut Surgery

Kristen A Wong, MD 1; Jayleen Grams, MD, PhD1; Erin Schwarz, BA2; Jonathon Dort, MD, FACS3; Caroline Reinke, MD, MSHP4; 1University of Alabama at Birmingham; 2Society of American Gastrointestinal and Endoscopic Surgeons; 3Inova Health System; 4Atrium Health Carolinas Medical Center

Background: The Acquisition of Data for Outcomes and Procedure Transfer (ADOPT) program was established by the Society of Gastrointestinal and Endoscopic Surgeons (SAGES) to develop and expand individual surgeons' comfort with specific, complex operations using hands-on teaching and longitudinal mentoring. The Foregut (Dominating the Hiatus) section of the course focused on standardized hiatal hernia dissection and gastric fundoplication techniques.

Methods: 16 surgeons enrolled in the program. The hands-on course using a cadaveric model occurred in March of 2022 at the SAGES annual meeting, where each surgeon was matched with an expert mentor (2:1 ratio, trainee to mentor). The mentors walked the surgeons through steps of a laparoscopic hiatal hernia repair and fundoplication. After this initial session, the trainees had the opportunity to attend a monthly group webinar and receive individual coaching from their assigned mentor. Each individual was given a pre-course survey with a 3 month follow up questionnaire.

Results: The majority of participants are employed at non-academic settings (87.5%). 12.5% practice in a rural setting, while 50.0% were in a small urban setting, and 37.5% were in a large urban setting. The years in practice ranged from 1–28, and the majority had completed a fellowship. 9 of the 16 participants (56.3%) completed the 3 month survey. 56.6% surgeons said that the course has already changed their practice with 44.4% specifically stating increased comfort in mediastinal dissection and with large paraesophageal hernias. When asked about change in confidence level for steps of a fundoplication, the majority of surgeons (55.5%-77.8%) reported becoming confident to significantly more confident in each of the 6 steps assessed. 44.4% of surgeons have attended at least one of the monthly webinars, and 22.2% have kept in contact with their mentors.

Discussion: The ADOPT program has proven since its inception to successfully provide expert instruction for practicing surgeons to learn new techniques or improve their confidence in performing operations. The data for the 2022 ADOPT Foregut course shows that three months of participation is already making a positive impact on these surgeons' practice. This helps to fill in the learning gap that occurs after formal surgical training ends. Future reporting on the status of this course will include 6 month and 12 month follow-up data.

S014

Outcomes Following Anti-reflux Surgery Stratified by Definitions of GERD: DeMeester Score vs Acid Exposure Time

Julia R. Amundson, MD, MPH1; Haris Zukancic2; Christopher Zimmermann, MD 2; Kristine Kuchta, MS2; Vanessa N. VanDruff, MD1; Stephanie Joseph, MD, MPH2; Simon Che, MD2; H. Mason Hedberg, MD, MS2; Michael B. Ujiki, MD, FACS2; 1University of Chicago; 2NorthShore University HealthSystem

Introduction: Definitions of gastroesophageal reflux disease (GERD) vary. The American Gastroenterology Association (AGA) Expert Review on GERD, published May 2022, focuses on acid exposure time (AET) rather than DeMeester score. We aim to review outcomes in patients who have undergone anti-reflux procedures at our institution, stratified by our historical GERD definition, focusing on DeMeester score, and the AGAs updated definition, focusing on AET.

Methods and Procedures: This is a retrospective review of a prospective gastroesophageal quality database. All patients who underwent anti-reflux surgery and preoperative BRAVO testing ≥ 48 h were included. Group comparisons were made using two-tailed Wilcoxon rank-sum and Fisher's exact tests, with two-tailed statistical significance of p < 0.05.

Results: Between 2010 and 2022, 255 patients underwent evaluation for GERD and preoperative BRAVO testing. The majority, 91.4%, had LA grade C/D esophagitis, Barrett's, or a positive DeMeester score on any day of testing, meeting our institutions historical GERD definition. Fewer, 67.5%, met the new AGA GERD definition, with LA grade B/C/D esophagitis, Barrett's, or AET ≥ 6% on 2 + days. A subset, 23.9%, met our historical GERD definition but did not meet the AGA's new GERD definition. Zero patients met the AGA GERD definition and did not meet our historical definition. Demographic differences between groups included lower BMI, less concurrent hiatal hernias, and less DeMeester score positive days in the cohort meeting the historical definition only. There were no significant differences in perioperative outcomes or percent symptom resolution at follow up between groups. Patient reported quality of life (PRQOL) metrics, including GERD-HRQL and Reflux Symptom Index (RSI) did not differ significantly between groups at the preoperative, 3-week, 6-month, or 1-year postoperative timepoints. Those who met only our historical definition reported worse RSI scores at a 2-year postoperative timepoint (18.6 ± 9.5 vs. 11.8 ± 11.0, p = 0.039). (Fig. 1).

Conclusion: Updated AGA GERD guidelines exclude a portion of patients who historically would have been diagnosed with and surgically treated for GERD. This cohort appears to have less severe GERD, but equivalent symptomatic improvement and PRQOL outcomes up to 1-year postop, with more atypical GERD symptoms at 2-years postop.

figure a

S015

Smoking Status is Not Associated with Increased Morbidity Following Laparoscopic Paraesophageal Hernia Repair

Patrick J Sweigert, MD 1; Theresa N. Wang, MD1; Tarik K. Yuce, MD, MS1; Roukaya T. Hassanein, MD1; Kelly R. Haisley, MD1; Kyle A Perry, MD1; 1Ohio State University Wexner Medical Center

Introduction: Smoking negatively impacts healing, and patients are often advised on smoking cessation prior to elective surgical interventions. However, patients with paraesophageal hernias (PEH) often present with significant symptoms and require more timely repair with limited ability for smoking cessation measures. The clinical impact of active smoking on outcomes of minimally invasive PEH repair remains unclear. The objective of this retrospective cohort study was to evaluate the impact of active smoking on short-term outcomes following PEH repair.

Methods: Adult patients who underwent primary or recurrent laparoscopic PEH repair at a single academic institution between 2011 and 2022 were retrospectively reviewed. Elective, urgent, and emergent cases were included. Patient demographic data, operative factors, and 90-day post-operative outcome data were collected and maintained in an IRB approved database. Cohorts were stratified based on active smoking status. Primary outcomes included overall complication rate, 30-day readmission, and symptomatic recurrence. Bivariate analyses were performed to compare cohort characteristics and outcomes. The association between smoking and outcomes were evaluated using multivariable logistic regression models. Data are presented as mean ± SD or median (interquartile range) as appropriate and a p-value < 0.05 was considered statistically significant.

Results: 576 patients underwent PEH repair during the study period (76.4% primary, 23.6% recurrent). 441 patients were female (76.6%) and median age was 66 (58–74) years. A total of 33 (5.7%) patients were identified as smokers. Rates of respiratory compromise (Non-smokers: 6.2% vs Smokers: 0.0%, p = 0.14), wound issues (Non-smokers: 0.2% vs Smokers: 0.0%, p = 0.80), hernia recurrence (Non-smokers: 3.5% vs Smokers: 3.0%, p = 0.89), and overall complications (Non-smokers: 16.0% vs Smokers: 9.1%, p = 0.29) did not differ significantly among groups. No difference was observed in mean length of stay (Non-smokers: 2.5 ± 3.4d vs Smokers: 1.8 ± 1.3d, p = 0.26), or readmission rate (Non-Smokers: 7.6% vs Smokers: 6.1%, p = 0.75). On multivariable analysis adjusting for age, American Society of Anesthesiologists (ASA) status, Body Mass Index, gender, and recurrent status, no differences were seen in risk of readmission (OR 0.69, 95%CI 0.15–3.12), recurrence (OR 0.61, 95%CI 0.08–4.95), or overall complications (OR 0.68, 95%CI 0.19–2.34) among smokers when compared to non-smokers.

Conclusion: For patients with symptomatic PEH repair, smoking status does not confer a clinically significant increased risk of postoperative complications following laparoscopic PEH repair. Active smoking status should not delay definitive surgical management of paraesophageal hernias.

S016

Need for Frequent Dilations After Magnetic Sphincter Augmentation: An Assessment of Associated Factors and Outcomes

Inanc S. Sarici, MD 1; Sven Eriksson1; Ping Zheng1; Toshitaka Hoppo1; Blair Jobe1; Shahin Ayazi1; 1Esophageal Institute, Allegheny Health Network

Introduction: Persistent dysphagia is the most common complaint after magnetic sphincter augmentation (MSA) with nearly one-third of patients requiring at least one dilation following MSA. A small subset of patients require frequent dilations, but there is paucity of data on characteristics of this group of patients. The aim of this study was to identify factors associated with need for frequent dilations and to compare the outcomes of these patients to others.

Methods: This is a retrospective review of patients who underwent MSA at our institution between 2013 and 2020. Patients aged 18 and older with no history of foregut surgery were included. Frequent dilations were defined as 2 or more dilations within 1-year of surgery. Patients completed baseline and 1-year postoperative GERD-HRQL questionnaires. Heartburn, regurgitation, dysphagia, and odynophagia were defined as a score > 3 on the symptom-specific item within the GERD-HRQL. Baseline demographic, clinical characteristic and objective testing data were compared between patients who did and did not require frequent dilations.

Results: A total of 697 (62.7% Female) patients with a median (IQR) age of 56.0 (45.0–64.0) and BMI of 28.9 (25.8–32.1) underwent MSA. Of these, 62 (8.8%) required frequent dilations. Demographic characteristics were comparable between groups.

Frequent dilations was associated with a higher rate of preoperative dysphagia (43.9% vs 28.7%, p = 0.017), odynophagia (31.6% vs 15.3%, p = 0.002), and lower DeMeester scores [21.3 (10.7–35.1) vs 29.9 (17.2–43.2), p = 0.018]. Preoperative manometry showed less percent intact swallows [85.0 (75.0–100.0) vs 100.0 (90.0–100.0), p = 0.017], higher resting LES pressure [26.5 (20.7–37.8) vs 20.3 (13.0–30.0), p < 0.001], higher integrated relaxation pressure (IRP) [8.0 (4.3–11.4) vs 6.1 (2.8–10.2), p = 0.028] and higher rate incomplete relaxation (IRP ≥ 15 mmHg) (16.1% vs 7.4%, p = 0.018). No other preoperative characteristics were different between groups.

Outcomes at one-year demonstrated an association between frequent dilations and failure to achieve 50% improvement in GERD-HRQL scores (57.8% vs. 18.2%, p < 0.001), despite a similar rate of pH normalization (73.8% vs. 73.8%, p = 0.998). Frequent dilations were also associated with postoperative heartburn (21.3% vs. 6.4%, p < 0.001), regurgitation (40.4% vs. 5.8%, p < 0.001), dysphagia (48.9% vs. 12.4%, p < 0.001), odynophagia (27.7% vs. 8.0% p < 0.001), and dissatisfaction (46.7% vs 11.6%, p < 0.001). Device removal was also more common with frequent dilations (25.8 vs 2.2%, p < 0.001).

Conclusion: The need for frequent dilations after MSA is marker for poor symptom control, dissatisfaction, and device removal. Patients with preoperative dysphagia, odynophagia, high LES pressures and poor esophageal motility should be counselled of their risk for these poor outcomes.

S017

Preoperative Patient Factors and Anatomy Do Not Predict Who Will Develop Reflux After Per Oral Endoscopic Myotomy

Joshua Lyons, MD 1; Christina Boutros, MD1; Saher-Zahra Khan, MD1; Jamie Benson, MD1; Daniel Hashimoto, MD1; Jeffrey Marks, MD1; 1University Hospitals

Background: Per oral endoscopic myotomy (POEM) has been shown to be an efficacious and safe therapy for the treatment of achalasia. Compared to laparoscopic Heller myotomy however, no antireflux procedure is routinely combined with POEM and therefore the development of symptomatic or silent reflux is of concern. This study was designed to determine if various patient factors and anatomy would predict the development of gastroesophageal reflux disease post-operatively.

Methods: This was a retrospective cohort study of all patients who underwent a POEM at a single institution by a single surgeon over an eight year period (2014–2022). It has been our practice to obtain a postoperative ambulatory pH test on all patients 6 months after POEM off all acid reducing medications. Patients without a postoperative ambulatory esophageal pH monitoring test were excluded. Age, sex, obesity (BMI > 30), type of achalasia, presence of a hiatal hernia, history of prior endoscopic achalasia treatments, and history of a previous myotomy (Heller or POEM) were then analyzed using univariate analysis as predictive factors for the development of postoperative GERD (DeMeester score > 14.7 on ambulatory pH monitoring).

Results: There were 179 total patients included in the study with 42 patients (23.5%) having undergone postoperative ambulatory pH testing. The majority of patients (137 or 76.5%) were lost to follow up and did not undergo ambulatory pH testing. Twenty-three out of those 42 patients (55%) had evidence of GERD on ambulatory pH testing. Age, sex, obesity (BMI > 30), type of achalasia, presence of a hiatal hernia, history of prior endoscopic achalasia treatments, and history of a previous myotomy (Heller or POEM) did not correlate with the development of post-operative GERD. (p = 0.51, 0.24, 0.66, 0.44, 0.43, 0.51, and 0.86 respectively).

Conclusions: Despite the high rate of reflux after POEM, there does not appear to be any reliable preoperative indicators of which patients have a higher risk of developing post-operative GERD after POEM.

figure b

S018

10-Year Follow-up of Endoscopic Muscal Resection Versus Esophagectomy for Esophageal Intramucosal Carcinoma in the Setting of Barrett's Esophagus: A Canadian Experience

Alisha R. Fernandes, BAS, MPH, MD, FRCSC 1; Chao Li, MD, FRCSC1; Daniel French, MD, FRCSC1; James E. Ellsmere, MS, MD, FRCSC, FACS1; 1Dalhousie University

Introduction: Endoscopic mucosal resection (EMR) is a safe and effective treatment for esophageal intramucosal adenocarcinoma (IMC), with similar recurrence and disease-specific mortality rates to esophagectomy, and lower morbidity in short-term follow-up. However, the long-term outcomes of EMR have not been well-studied in this population. This single-centre, retrospective study investigates IMC eradication rates, recurrence rates and morbidity at 10-years following EMR versus esophagectomy.

Methods: Patients diagnosed with IMC from 2005–2013 were included in this study. Esophagectomy represented the treatment standard. In 2009 EMR was introduced for the treatment of IMC, with the intent of achieving endoscopic and pathologic eradication of adenocarcinoma. Follow-up per protocol occurred every 3 months for 1 year, every 6 months for 2 years, and every 12 months thereafter. Categorical variables were expressed as percentages, continuous variables expressed as median with interquartile range. Comparisons were made using student's t-test and Fischer's exact test.

Results: 23 patients were included in the study (10 esophagectomy and 13 EMR). Patient and tumor-specific characteristics were similar between groups. All patients had T1a disease (m1-3) except for 1 non-operative candidate offered EMR for T1bsm1. Median follow-up for EMR was 7 years (IQR 3–8.5) and for esophagectomy was 10.5 years (IQR 7–11). EMR eradicated IMC in 12 patients (92.3%). A median of 1 EMR (IQR 1–1) was required to achieve eradication. Recurrence occurred in 3 (23.1%; 2 local and 1 metastatic), of which 2 presented after being lost to endoscopic follow-up. Two local recurrences were treated with subsequent esophagectomy. No post-procedure complications occurred. Esophagectomy eradicated IMC in 10 patients (100%). Recurrence occurred in 2 (18.2%, metastatic). Major, early procedure-related complications affected 5 esophagectomy patients (50%). In long-term follow-up, 7 (70%) required endoscopic intervention for stricture. EMR and esophagectomy were associated with similar eradication (p = 0.369) and recurrence rates (p = 0.287). Only 1 EMR was required to achieve IMC eradication. Esophagectomy was associated with significantly more complications than EMR (p < 0.001).

Conclusions: EMR and esophagectomy for the treatment of IMC are associated with comparable eradication and recurrence rates in 10-year follow-up. EMR was associated with lower rates of procedure-associated morbidity. If close endoscopic follow-up is possible, EMR should be used preferentially to treat patients with T1a distal esophageal adenocarcinoma to minimize procedure-related morbidity while maintaining acceptable rates of disease-related outcomes.

S019

Intraoperative Impedance Planimetry and Panometry (EndoFLIP) Can Replace Manometry in Preoperative Assessment for Anti-Reflux Surgery

Vanessa N. VanDruff, MD 1; Julia R. Amundson, MD, MPH1; Stephanie Joseph, MD, MPH2; Simon Che, MD3; Chris Zimmermann, MD3; Kristine Kuchta, MS3; H. Mason Hedberg, MD3; Michael B. Ujiki, MD3; 1University of Chicago; 2Wayne State University; 3NorthShore University HealthSystem

Background: Impedance planimetry and panometry (EndoFLIP™) has been gaining momentum as an advanced tool to assess esophageal motility and distensibility. Since the functional luminal imaging probe (FLIP) is placed under sedation, it may provide means to 'tailor' crural closure and fundoplication without requiring unpleasant high-resolution manometry (HRM) preoperatively. However, it is currently unknown whether panometry can be conducted under general anesthesia, and whether secondary motility induced by volumetric distension can be used to predict HRM-defined primary peristaltic dysfunction.

Methods: Single institution prospective data was obtained from 100 patients with gastroesophageal reflux disease (GERD) undergoing anti-reflux procedures from August 2021 to September 2022. A single FLIP motility rater blinded to preoperative motility studies assessed planimetry and panometry immediately prior to surgery. We previously determined halogenated gases disrupt secondary esophageal peristalsis, therefore all testing was performed with total intravenous anesthesia. Esophageal contractile response to stepwise FLIP distension was classified in real-time as normal (NCR), diminished/disordered (DDCR), or absent (ACR), and compared to preoperative HRM and timed barium esophagram (TBE) results.

Results: One hundred patients were evaluated with HRM and FLIP panometry, of whom 60% had paraesophageal hernias. Panometry was abnormal in 71/100 patients, and concordant with HRM results in 71% of patients. No statistical differences in age, body mass index, diabetes mellitus, % acid exposure time, DeMeester score, or hernia size were found comparing patients with concordant versus discordant HRM and FLIP findings. There were 27 of 54 patients with normal HRM and FLIP dysmotility, yielding a 50% specificity rate. Of the 46 patients with dysmotility on HRM, 44 had FLIP dysmotility (DDCR, n = 42, ACR, n = 2) resulting in a 93.1% negative predictive value (NPV). Only 2 patients with normal FLIP (6.9%) had abnormal HRM. Furthermore, only 4.3% of patients with abnormal HRM failed FLIP detection, corresponding to a 95.7% sensitivity rate. In comparison, TBE (n = 65) had sensitivity and specificity rates of 57.1% (16/28) and 56.8% (21/37) respectively. Only 21 of 33 patients with normal TBE motility had normal HRM equating to a 63.46% NPV. Dysphagia and globus sensation had low sensitivity rates of 47.8% and 19.6%, respectively, to predict abnormal manometry.

Conclusion: FLIP Panometry can be used under general anesthesia and can accurately detect peristaltic dysfunction in GERD patients with higher sensitivity than TBE. FLIP therefore should be considered an effective tool for preoperatively identifying patients who may forgo manometry, and/or intraoperatively assessing those who cannot obtain or tolerate preoperative manometry.

S020

Can Endoscopic Impedance Planimetry Replace High-resolution Manometry (HRM) for the Detection of Preserved Motility During Pre-operative GERD Evaluation?

Theresa N Wang, MD 1; Jennifer Underhill, MD1; Robert Tamer, MPH, MPA1; Kyle A Perry, MD2; Kelly R Haisley, MD2; 1The Ohio State University Department of Surgery; 2The Ohio State Department of Surgery, Center for Minimally Invasive Surgery

Introduction: Pre-operative evaluation of patients with gastroesophageal reflux disease (GERD) includes high-resolution manometry (HRM), the gold standard for evaluating esophageal motility. However, many patients experience dissatisfaction with the awake HRM procedure. Endoscopic impedance planimetry (IP) may provide an alternative approach to identify preserved peristalsis during sedated endoscopy, minimizing discomfort. We hypothesized that the presence of repetitive antegrade contractions (RACs) on pre-operative impedance planimetry would correlate with normal esophageal motility assessed by traditional HRM, and tested this hypothesis in a prospective cohort study.

Methods: Adult patients presenting between 9/2020–10/2021 for surgical evaluation of GERD were prospectively enrolled under an IRB approved protocol. Patients with prior gastric surgery, motility disorders, or large paraoesophageal hernias were excluded. Patients underwent both HRM and IP with normal motility defined by the Chicago 3.0 classification for HRM and by the presence of repetitive antegrade contractions (RACs) on IP analysis. Data regarding successful procedure completion and procedure tolerance were also collected. Descriptive statistics, chi-squared test and t-test were used to analyze the data as appropriate and a p value < 0.05 was considered significant.

Results: Of 63 patients enrolled, 48 successfully completed both Endoflip and HRM testing (5 failed Endoflip, 5 failed HRM, 5 lost to follow up). The patient population was 50% male with a median age of 52.5 [42.0,66.0] years, mostly ASA class 1–2 (75.1%, n = 36) and had an average BMI of 31.4 ± 6.3 kg/m2. Normal motility tracings were identified in 62.5% of IP procedures and 79.2% of HRM tests. Using HRM findings as the gold standard, IP was able to detect normal motility with a sensitivity of 69.4% and a specificity of 58.3% (positive predictive value = 83.3%, negative predictive value = 38.8%). Tolerability of Endoflip was significantly better than HRM with lower rates of discomfort (10.9% vs 93.4%, p < 0.0001) and higher willingness to repeat testing (100% vs 47.8%, p < 0.0001).

Conclusion: Esophageal motility testing with Endoflip 2.0 is well-tolerated by patients and detects normal motility with a positive predictive value of 83.3%. However, the low specificity (50%) and poor negative predictive value (27.8%) raises concern for the reliability of this test as a stand-alone replacement for HRM. Larger studies will be needed to determine if there is a subset of patients in whom Endoflip may be more reliable.

S021

Approach to failed fundoplication: is conversion to gastric bypass superior for symptom resolution?

Naga S. Gunturu, MBBS1; Rocio Castillo-Larios, MD1; Spencer W. Trooboff, MD, MBA 1; Steven P. Bowers, MD1; Enrique F Elli, MD1; 1Mayo Clinic Florida

Introduction: We evaluated the efficacy of re-operative surgery for failed fundoplication with respect to symptomatic, pharmacologic and anatomic outcomes.

Methods and procedures: This was a single institution retrospective cohort study involving chart review of all patients having surgery after failed fundoplication from 2016–2021. The primary outcome was new or persistent reflux or dysphagia at last follow-up. Secondary outcomes included persistent anti-reflux medication requirement and radiographic recurrence of hiatal hernia. Univariate analyses were performed to evaluate patient, operative and postoperative factors associated with choice of surgical approach and/or the primary outcomes. This informed multivariable logistic regression to identify factors associated with persistent symptoms.

Results: The analytic cohort comprised 161 patients having surgery after failed fundoplication. Median follow-up was 12 months (interquartile range 5 – 24 months). Most patients had previously undergone either Nissen (83%) or Toupet (11%) fundoplication and 21% had one or more prior re-operations. Rates of preoperative reflux and dysphagia were 73.9% and 62.7%, respectively. The re-operative procedures included roux-en-Y gastric bypass (RYGB, n = 38), Nissen (n = 44) and Toupet (n = 72) fundoplication, and fundoplication takedown (n = 7). Patients undergoing RYGB had significantly higher body mass index, were more likely to have prior re-operation and had more preoperative reflux but less dysphagia as compared to those undergoing redo fundoplication. Length of stay was significantly longer following RYGB (mean 5.1 ± 1.3 days vs. 2.2 ± 0.1 days for other procedures). There was no significant difference in complication rate (21.1% RYGB vs. 13.0% other procedures, p = 0.224). At last follow-up most patients were free from reflux and dysphagia, however this favored patients who received RYGB (Table 1). On multivariable modeling we found prior re-operative surgery, BMI ≥ 35 kg/m2 and recurrent hiatal hernia were positively associated with new or persistent reflux while re-operative RYGB was protective against reflux. Prior re-operative surgery was the only factor associated with persistent dysphagia.

Conclusions: Re-operation after failed fundoplication improves reflux and dysphagia for most patients. Conversion to Roux-en-Y gastric bypass may offer superior resolution of reflux than redo fundoplication, especially for obese patients.

figure c

S022

The Impact of Frailty on outcomes in Elective Paraesophageal Hernia Repair

Theresa N Wang, MD 1; Bryan W An, BS2; Tina X Wang, BS2; Molly McNamara, BS2; Victor Heh, PhD1; Courtney E Collins, MD3; Kelly R Haisley, MD3; Kyle A Perry, MD3; 1The Ohio State University Department of Surgery; 2The Ohio State University College of Medicine; 3The Ohio State University Department of Surgery, Center for Minimally Invasive Surgery

Introduction: Paraesophageal hernia repair (PEHR) improves reflux symptoms and quality of life scores. Previous studies have described risk factors such as emergent operation or advanced patient age, but few have examined the relationship between pre-operative frailty and peri-operative outcomes. The goal of this retrospective cohort study was to determine whether markers of frailty predict poor peri-operative outcomes in elective PEHR.

Methods: Patients who underwent elective PEHR between 2011 and 2022 by a single high-volume surgeon were identified. Patient demographics, operative details, post-operative complications, length of stay (LOS), readmissions, modified frailty index (mFI) and discharge level of care (DLOC) were recorded and maintained in an IRB-approved database. Descriptive statistics and chi-squared test were used to analyze the data. Data are presented as mean ± SD or median[IQR] as appropriate and p-values of < 0.05 were considered statistically significant.

Results: 576 patients underwent elective PEHR during the study period. The mean age was 64.9 ± 12.4 years, and 76.3%(n = 440) were female. Median LOS was 1[1,2] day. The youngest and oldest patients (age 19–34 in 2.4%, 35–49 in 8.3%, 50–65 in 36.6% and > 65 in 52.6%) were associated with a higher overall complication rate (28.6%,8.3%,10.4%,19.8%; p = 0.007), prolonged LOS (> 3 days) (21.4%,10.4%,19%,27.4%; p = 0.022) and increased DLOC (14.3%,0%,1.9%,11.3%; p < 0.001). The youngest patients were associated with early PEH recurrence (< 6 months) (14.3%,2.1%,2.4%,1%; p = 0.005). Higher ASA (ASA 3–4, 66.1%; n = 381) was associated with prolonged LOS (26.8% vs. 14.9%; p = 0.001). Higher mFI (mFI 0 in 35.2%, 1 in 27.8%, >  = 2 in 19.6%) was associated with prolonged LOS (18.2%,21.9%,32.7%; p = 0.012), noninvasive complications (10.8%,10.8%,21.2%; p = 0.012) and DLOC (3.5%,6.2%,32.7%; p < 0.001). Patients who required assistance with activities of daily living at baseline 2.6%(n = 15) were more likely to have prolonged LOS (66.7% vs 21.6%; p < 0.001) and DLOC (80% vs 5%, p < 0.001). Conversion to open 0.2%(n = 1) and mortality 0.3%(n = 2) were rare. No examined pre-operative factors were associated with readmission or all-comer recurrence.

Conclusion: Frailty is associated with prolonged post-operative stay, peri-operative complications, and the need for a higher level of care after elective paraesophageal hernia repair. Factors associated with negative perioperative outcomes included increasing Frailty Index score, ASA, age > 65 years old and baseline assistance for activities of daily living. Interestingly, we found that young age (19–34) was associated with increased length of stay, complications, and early recurrence, which may reflect a different disease process in symptomatic paraesophageal hernia in young patients. This provides additional information to counsel patients pre-operatively, as well as an opportunity for targeted pre-habilitation.

S023

Demographic, Socioeconomic and Clinical Risk Factors Associated with Functional Esophageal Disorders versus Gastroesophageal Reflux Disease

Moniyka Sachar, MD 1; Marina Wizentier, BS2; Emma Risner, BS2; Hannah Asmail, BS1; Matthew Omara, BS1; Abraham Khan, MD1; Rita Knotts, MD, MSc1; 1NYU Langone Medical Center; 2NYU Department of Biostatistics

Introduction: Despite the prevalence of gastroesophageal reflux disease (GERD), approximately 30% of patient who undergo esophageal pH testing have a functional esophageal disorder (FED), specifically either reflux hypersensitivity (RH) or functional heartburn (FH). We investigated demographic, medical co-morbidities and socioeconomic factors in relation to FED versus pathologic GERD.

Methods: This was single center study examining adult patients who underwent 24-h pH-impedance testing from 11/2019 to 3/2021. Participants were categorized into pathologic GERD, FH or RH using pH-impedance data and reported symptom correlation. Demographic and medical comorbidity data was retrieved from the electronic medical record on all participants. Multivariate logistic regression analysis was utilized to identify predictors of FED.

Results: 229 patients were included. Female gender (OR = 1.84; 95%CI 1.08–3.15), non-Hispanic Asian ethnicity (OR = 5.65; 95% CI 1.57–20.33), being underweight (OR 7.33; 95% CI 0.91–59.39), chronic pain (OR = 2.33; 95% CI 1.29–4.23), insomnia (OR = 2.83; 95% CI 0.98- 8.22), and allergic rhinitis (OR = 3.90; 95% CI (1.60–9.50) were associated with a greater risk for FED, while being overweight or obese (OR 0.48 and 0.4; 95% CI 0.27–0.87 and 0.18–0.93) and alcohol use (OR 0.57; p = 0.06) were protective. When comparing FH to RH, a greater percentage of those with FH had history of alcohol use (48% vs 24%; p = 0.04).

Conclusion: To date this is the largest study examining predictors of specific functional esophageal diseases. Low BMI, insomnia, chronic pain, allergic rhinitis, and Asian race appears to have greater odds of having functional esophageal disease. Our findings enable clinicians to better screen patients with reflux for these disorders.

S024

The Use of Indocyanine Green (ICYG) Angiography Intraoperatively to Evaluate Gastric Conduit Perfusion During Esophagectomy: Does It Impact Surgical Decision Making?

Gabrielle M LeBlanc, MDS1; Caitlin Takahashi, MD2; Jamie Huston, MS3; Ravi Shridhar, MD, PhD4; Kenneth Meredith 5; 1Florida State University; 2East Carolina State University; 3Sarasota Memorial Hospital; 4Advent Health Orlando; 5Jellison Cancer Institute/Sarasota Memorial Hospital

Introduction: Anastomotic leak (AL) after esophagectomy is associated with significant morbidity and mortality. Ischemia is known to be a major contributor for anastomotic leaks and indocyanine green fluorescence angiography (ICYG) has been utilized to assess perfusion. However, experienced esophageal surgeons have clinically assessed the gastric conduit with acceptable outcomes for years. We sought to examine the impact of ICYG in a surgeon's decision making during esophagectomy.

Methods: We queried a prospectively maintained database to identify patients who underwent esophagectomy. The use of ICYG was implemented in 2019. Time to initial perfusion(IPT), time to maximum perfusion (TMP) and residual ischemia(RI) were measured and used as a guide to resection of residual stomach. During esophagectomy the surgeon identified the anticipated line of ischemic demarcation (LOD) prior to ICYG injection. The distance between the surgeon's LOD and ICYG LOD was measured. Patient demographics and operative outcomes were compared using Mann–Whitney U, Kruskal Wallis and Pearson's Chi-square test as appropriate.

Results: We identified 1024 patients who underwent esophagectomy, 963 without ICYG and 61 ICGY with median ages of 66, and 70 respectively, p < 0.001. There were no differences in gender, BMI, histology, stage, or use of neoadjuvant therapy between groups. The ICYG group did have higher ASA scores 3.2 vs 2.6, p < 0.001. Outcomes including operative time, estimated blood loss, lymph node harvest, length of hospitalization, length of ICU and readmissions did not differ between groups. There were 59 (5.8%) AL (55 (5.7%) in the non ICYG and 4(6.6%) in the ICYG, p = 0.78. The initial perfusion time was ≥ 10 s and max perfusion was > 25 s in all of the patients in the ICYG that developed anastomotic leaks. In addition, all patients were noted to have at least 1 cm of residual gastric ischemia. Complications were noted in 32.4% of patient in the non ICYG and 31.1% in the ICYG, p = 0.84. Fifteen patients underwent independent surgeon evaluation of the ischemic LOD prior to ICYG. Differential distances were noted in 12/15 (80%) of patients with a mean distance between surgical line of demarcation and ICYG LOD of 0.77 cm.

Conclusions: While the implementation of ICYG during esophagectomy demonstrates no significant improvements in anastomotic leak rates compared to historical controls, surgeon's decision making is impacted in 80% of cases resulting in additional resection of the gastric conduit. Additionally, elevated times to initial perfusion and maximum perfusion were associated with increased gastric ischemia and anastomotic leaks.

S025

Physiologic Tension of the Abdominal Wall

Ryan C. Ellis, MD 1; R. M. Walsh, MD1; Daniel Joyce, MBBCh1; Robert Simon, MD1; Nima Almassi, MD1; Byron Lee, MD, PhD1; Robert DeBernardo, MD1; Scott Steele, MD, MBA1; Samuel Haywood, MD1; Benjamin T Miller, MD1; 1Cleveland Clinic Foundation

Introduction: Tension-free hernia repair is a primary tenet of abdominal wall reconstruction (AWR). However, this concept neglects the baseline physiologic tension present in the native abdominal wall. Ideally, hernia repairs should be tailored to restore native physiologic tension. We sought to quantify the baseline physiologic tension needed to re-establish the linea alba in patients undergoing exploratory laparotomy.

Methods: Patients without ventral hernias undergoing laparotomy at a single institution were enrolled from December 2021 to September 2022. Patients who had undergone prior laparotomy were included. Exclusion criteria included presence of an ostomy, large-volume ascites, and large intra-abdominal tumors. After laparotomy, a sterilizable tensiometer measured the quantitative tension needed to bring the myofascial edge to the midline. Outcomes included the force needed to bring the myofascial edge to the midline and the association of BMI, incision length, and prior lateral incisions on abdominal wall tension.

Results: There were 86 patients included in the study, for a total of 172 measurements (right and left for each patient). Median patient age was 64.5 (IQR 58;69.8), median BMI was 26.4 kg/m2 (IQR 22.9;31.5) and median incision length was 17 cm (IQR 14;20). Median tension needed to bring the myofascial edge to the midline was 0.75 lbs (left IQR 0.5;1; right IQR 0.25;1). Mixed effect modeling found that increasing BMI and greater incision length were associated with higher abdominal wall tension (SE 0.01, 95% CI [0.01,0.07], p = 0.004); SE 0.01, 95% CI [0.01,0.07], p = 0.006), respectively.

Conclusion: In patients without ventral hernias, the physiologic tension of the abdominal wall is approximately 0.75 lbs. Increasing BMI and longer incisions are associated with higher abdominal wall tension. A quantitative understanding of baseline abdominal wall tension may help surgeons tailor hernia repairs to achieve physiologic tension in AWR.

S026

Clinical Outcomes and Costs of Retromuscular and Intraperitoneal Onlay Mesh Techniques in Robotic Incisional Hernia Repair

Omar Y. Kudsi, MD, MBA, FACS1; Georges Kaoukabani, MD, MSc 2; Naseem Bou-Ayash, MD3; Fahri Gokcal, MD2; 1Good Samaritan Medical Center, Tufts University School of Medicine; 2Good Samaritan Medical Center; 3Tufts Medical Center

Objective: To compare clinical outcomes and financial cost of intraperitoneal onlay mesh(IPOM) versus retromuscular (RM) repairs in robotic incisional hernia repairs (rIHR).

Methods: Patients who underwent either IPOM or RM elective rIHR from 2012 to 2022 were included. Patients' demographics, operative details, postoperative outcomes and hospital costs were directly compared.

Results: 69 IPOM and 55 RM were included. Mean ± Standard-Deviation of age and body mass index did not differ between both groups(IPOM vs RM: 59.3 ± 11.2 vs. 57.5 ± 14, p = 0.423; 34.1 ± 6.3 vs. 33.2 ± 6.9, p = 0.435, respectively). Comorbidities and hernia characteristics were comparable. Comparison of intraoperative variables is presented in the table below. Extensive lysis of adhesions was required more often in IPOM(18 vs. 6 in RM, p = 0.034). Defect closure was achieved in 100% of RM vs. 81.2% in IPOM (p < 0.001). Median(interquartile range) postoperative pain score was higher in RM than in IPOM[5(3–7) vs. 4(3–5), respectively, p = 0.006]. Median length of stay (0 days) and same-day discharge rate did not differ between groups (p = 0.598, p = 0.669, respectively). Six(8.7%) patients in the IPOM group versus one (1.8%) patient in the RM group were readmitted to hospital within 30-day postoperatively (p = 0.099). Perioperative complications were higher in IPOM (p = 0.011; 34.8% vs. 14.5% in RM) with higher Comprehensive Complication Index® morbidity scores [0(0–12.2) vs 0(0–0) in RM, p = 0.008)], Clavien-Dindo grade-II complications (8 vs 0 in RM, p = 0.009), and surgical site events (17 vs. 5 in RM, p = 0.024). Recurrence rate was higher in IPOM(7.2%) vs. RM(1.8%) but did not statistically differ between both groups (p = 0.226) with a follow-up of 57 (± 28) months. Hospital costs did not differ between groups [IPOM: $9,978 (7,031–12,926) vs. RM: $8,961 (6,701–11,222), p = 0.300]. Although postoperative complication costs were higher in IPOM ($2,436 vs RM: $161, p = 0.020), total costs were comparable [IPOM: $12,415(8,700–16,130) vs. RM: $9,123(6,789–11,457), p = 0.080].

Conclusion: Despite retromuscular repairs having lower postoperative complications than intraperitoneal onlay mesh repairs, both techniques offered encouraging long-term results in robotic incisional hernia repair at a comparable total cost.

Comparison of Intraoperative Variables.

Variables [median (IQR), n(%)]

IPOM (69)

RM (55)

p

Defect Size (cm2)

9.6 (7.1–18.8)

15.7(15.7–31.4)

 < 0.001

Mesh Size (cm2)

150 (144–225)

300 (225–450)

 < 0.001

Total Operative Time (min)

72 (54–112)

96.5 (71–132)

0.023

Estimated Blood Loss (mL)

5 (5–5)

5 (5–5)

0.561

Intraoperative Complications

2 (2.9)

2 (3.6)

0.492

S027

Sufficient Mesh Overlap for Ventral Hernia Repair in Current Practice

Desmond Huynh, MD 1; Anne Ehlers, MD, MPH1; Sean O'Neill, MD, PhD1; Jenny Shao, MD1; Michael Englesbe, MD1; Justin Dimick, MD, MPH1; Dana Telem, MD, MPH1; Ryan Howard, MD, MS1; 1Michigan Medicine

Introduction: Sufficient overlap of mesh beyond the borders of a ventral hernia helps prevent hernia recurrence. Guidelines from the European Hernia Society and Americas Hernia Society recommend ≥ 2 cm overlap for open repair of < 1 cm hernias, ≥ 3 cm overlap for open repair of 1-4 cm hernias, ≥ 5 cm overlap for open repair of > 4 cm hernias, and ≥ 5 cm overlap for all laparoscopic ventral hernia repairs. We evaluated whether current practice reflects this guidance.

Methods: We used the Michigan Surgical Quality Collaborative Hernia Registry to evaluate patients who underwent elective ventral and umbilical hernia repair between 2020–2022. Mesh overlap was calculated as [(width of mesh – width of hernia)/2]. The main outcome was "sufficient overlap," defined based on published EHS and AHS guidelines. Explanatory variables included patient, operative, and hernia characteristics. The main analysis was a multivariable logistic regression to evaluate the association between explanatory variables and sufficient mesh overlap.

Results: 4,451 patients underwent ventral hernia repair with a mean age of 55.4 (14.1) years, 1,905 (42.8%) females, mean body mass index (BMI) of 33.3 (7.4) kg/m2, and mean hernia width of 3.7 (3.4) cm. Mean mesh overlap was 3.6 (2.5) cm and ranged from -6.0 cm to 21.4 cm. Only 1,137 (25.5%) ventral hernia repairs had sufficient mesh overlap according to published guidelines (Table). Operative factors associated with increased odds of sufficient overlap included myofascial release (adjusted odds ratio [aOR] 4.52 [95% CI 3.43–5.95]), mesh type (absorbable aOR 2.89 [95% CI 2.03–4.10]), laparoscopic approach (aOR 1.78 [95% CI 1.52–2.07]), mesh location (onlay aOR 1.32 [95% CI 1.09–1.60]) (Figure). Patient factors associated with increased odds of sufficient overlap included prior hernia repair (aOR 1.56 [95% CI 1.30–1.87]).

Conclusion: Although sufficient mesh overlap is recommended to prevent ventral hernia recurrence, only a quarter of ventral hernia repairs in a statewide cohort of patients had sufficient overlap according to evidence-based guidelines. Factors strongly associated with sufficient overlap included myofascial release, mesh type, and laparoscopic repair.

Mesh overlap

Defect size

Recommended mesh overlap

Repairs with sufficient overlap

Open repair

  

 < 1 cm

 ≥ 2 cm

21/57 (37%)

1-4 cm

 ≥ 3 cm

307/1634 (19%)

 > 4 cm

 ≥ 5 cm

157/572 (28%)

Laparoscopic Repair

  

Any size

 ≥ 5 cm

652/2188 (30%)

figure d

S028

Life After No: Barriers to Behavior Change for Persons Declined Hernia Repair Due to High-Risk Features

Anne P Ehlers, MD, MPH 1; Wilson Nham, MPH, PMP1; C. Ann Vitous, MA, MPH1; Krisinda P Palazzolo, PAC1; Ryan Howard, MD, MS1; Lia Delaney, MD2; Jenny M Shao, MD3; Dana A Telem, MD, MPH1; 1University of Michigan; 2Stanford University; 3University of Pennsylvania

Introduction: It is well-established that a surgical episode can serve as a "teachable moment" for high-risk patients with potentially modifiable risk factors (e.g., active tobacco use, obesity). Elective ventral hernia repair is one example of a common surgical condition where patient optimization factors largely into outcomes. To address this, we developed a pre-operative optimization clinic to support high-risk patients seeking elective ventral hernia repair.. Unfortunately, few patients progressed to surgery. Within this context, we sought to understand the barriers to behavior change among high-risk patients with a goal of improving care as well as scaling interventions to future patients.

Methods: Using a qualitative study design, we performed semi-structured interviews with 20 patients who were declined ventral hernia repair due to either a BMI > 40 or active tobacco use. Patients were recruited from a hernia-specific preoperative optimization clinic at a single academic health center. Interviews sought to characterize the patients' perceived barriers and facilitators to behavior change (e.g., tobacco cessation, weight loss) in the pre-operative setting. We used an inductive thematic analysis to analyze the data. Interviews were concluded once thematic saturation was reached. All data analysis was performed using MAXQDA software.

Results: Among 20 patients (mean age 50, 65% female, 65% White), none had yet undergone surgical repair of their hernia. Data revealed three dominant themes that impacted the patient's perceived ability to make behavior changes necessary to undergo surgery:

Sense of responsibility: Some patients voiced a sense that they were responsible for behavior change, others felt that it was beyond their control.

Access to resources: Many patients could not afford needed resources (e.g., bariatric surgery, transportation to receive care) while others chose not to access resources that were offered (e.g., smoking cessation program).

Individualism: While patients understood that they were high risk for surgery, they felt that there were other factors specific to their case not being considered by the provider, but that were potentially more important in the decision to proceed with surgery.

Representative quotes are demonstrated in the Table.

Conclusion: Behavior change prior to elective surgery is complex and multifaceted. While improving access to resources (e.g., expanded coverage for bariatric surgery) may help some patients, our data demonstrates other factors that may be more challenging to solve. Future studies should evaluate the effectiveness of evidence-based behavior change interventions to address patient-level barriers related to sense of responsibility and importance of individualism in care.

figure e

S029

Evaluation of 30-Day Outcomes for Open Ventral Hernia Repair Using Non-self-gripping Versus Self-gripping Mesh

Anoosh Bahraini 1; Justin Hsu1; Nick Cochran1; Shanelle Campbell1; David W Overby1; Sharon Phillips2; Arielle Perez1; Ajita Prabhu3; 1UNC Hospitals; 2Vanderbilt University; 3Cleveland Clinic

Introduction: The purpose of the study is to determine early 30-day outcomes when comparing non-self-gripping mesh (NSGM) and self-gripping mesh (SGM) in ventral hernia repair (VHR). SGM has been rapidly adopted for VHR due to its purported ability to obviate the need for mesh fixation. Although most VHR utilize synthetic mesh, there is limited data regarding the efficacy of SGM in VHR and, to our knowledge, there is no literature comparing SGM to NSGM. We aimed to determine whether there is a difference in early rates of surgical site occurrence (SSO), surgical site infection (SSI), surgical site occurrence requiring procedural intervention (SSOPI), recurrence, and patient reported pain scores between the meshes after VHR. We hypothesized there will be no difference in SSO, SSI and SSOPI, recurrence, and pain scores after 30 days.

Methods: We performed a retrospective analysis of prospectively collected data from January 2014 to April 2022 using the Abdominal Core Health Quality Collaborative (ACHQC). We included patients over 18-years of age who underwent elective open VHR using SGM or NSGM and who had 30-day follow-up. Propensity matching was utilized to control for variables including hernia width, wound class, BMI, age, smoking, COPD, ASA, mesh location. Matched groups were analyzed to identify differences in SSO, SSI, SSOPI, recurrence, and pain scores measured by the HERQles and PROMIS scales at 30-days.

Results: 11,860 patients were identified. The mean hernia width to mesh width ratio was 9 cm:22 cm in NSGM compared to 7 cm:15 cm in SGM. The most common location for mesh placement was in the sublay position for both NSGM (86.2%) and SGM (76.4%). After propensity matching, 2072 patients were included for outcomes analysis. There was no difference in 30-day rates of SSO or recurrence. SGM had lower rates of SSI (3.1% vs 5.1%; p < 0.05) and SSOPI (3.1% vs 6.7%; p < 0.005). There was no difference in patient-reported pain scores at 30-days.

Conclusions: In patients undergoing VHR with mesh, SGM performs well in the early postoperative period. Use of SGM may be associated with lower rates of SSI and SSOPI in the early postoperative period when compared to NSGM. More long-term follow-up is required to compare the performance of NSGM vs SGM over time.

figure f

S030

Use of Statewide Financial Incentives to Improve Documentation of Hernia and Mesh Characteristics in Ventral Hernia Repair

Sean M O’Neill 1; Brian T. Fry, MD1; Wenjing Weng, MS1; Michael Rubyan, PhD, MPH1; Ryan A. Howard, MD1; Anne P. Ehlers, MD, MPH1; Michael J Engelsbe, MD1; Justin B. Dimick, MD, MPH1; Dana A Telem, MD, MPH1; 1University of Michigan

Introduction: Documentation of intraoperative details is critical for understanding and advancing hernia care, but on a broad scale, it is poorly done. The provision of tools such as synoptic operative notes can help, but have inconsistent uptake. Therefore, to improve data capture on a statewide level, we implemented a financial incentive targeting documentation of two essential intra-operative elements of hernia surgery – defect size and mesh use. The Abdominal Hernia Care Pathway (AHCP), a pay for performance (P4P) initiative, was introduced within the statewide Michigan Surgical Quality Collaborative (MSQC) during the year 2021.

Methods: The AHCP consisted of a voluntary organizational-level financial incentive that was given for achieving 80% performance on 8 specific measures for pre-, intra-, and post-operative ventral hernia care. Two measures in the pathway included complete documentation of ventral hernia defect size (length/width or diameter) and location (European Hernia Society classification), as well as mesh characteristics (length/width or diameter; product name/ID; brand), placement (onlay, inlay, retrorectus, preperitoneal, intraperitoneal/underlay), and fixation technique. We hypothesized that inclusion of these measures in a P4P program would result in significant increases in rates of complete documentation. Comparisons were made between AHCP and non-AHCP sites in 2021. Additionally, the performance of 8 AHCP pilot sites was compared between 2020 and 2021.

Results: Of 69 eligible sites, 47 participated in the AHCP in 2021. There were N = 5,362 operations (4,169 at AHCP sites; 1,193 at non-AHCP sites). At AHCP sites, 69.8% of operations had complete hernia documentation, compared to 50.5% at non-AHCP sites (p < 0.0001). At AHCP sites, 91.4% of operations had complete mesh documentation, compared to 86.5% at non-AHCP sites (p < 0.0001). The site-level hernia documentation goal of 80% was reached by 14 of 47 sites (range 14–100%). The mesh documentation goal was reached by 41 of 47 sites (range 4–100%). Among the eight pilot sites, hernia documentation was 39.6% in 2020 and 76.5% in 2021; mesh documentation was 72.1% in 2020 and 95.2% in 2021.

Conclusion: Addition of an organizational-level financial incentive produced marked gains in documentation of intra-operative details across a statewide surgical collaborative. The relatively large effect size–19.3% for hernia—is remarkable among P4P initiatives. This result may have been facilitated by surgeons' direct role in documenting hernia size and mesh use. These improvements in data capture will foster understanding of current hernia practices on a large scale and may serve as a model for improvement in collaboratives nationally.

figure g

S031

Deep Learning Model (DLM) Utilizing Clinical Data Alone Outperforms Image-Based Model for Hernia Recurrence Following Abdominal Wall Reconstruction (AWR) with Long Term Follow-up

Hadley H Wilson, MD 1; Chiyu Ma, BS2; Dau Ku, MS1; Gregory T Scarola, MSPH1; Vedra A Augenstein, MD1; Paul D Colavita, MD1; Todd Heniford, MD1; 1Carolinas Medical Center; 2Duke University

Introduction: Image-based DLMs have shown promise in predicting surgical complexity and wound complications for AWR with the potential to augment other clinical information in surgical decision-making. The purpose of this study is to evaluate a preoperative image-based DLM in predicting hernia recurrence.

Methods and Procedures: A prospective single-institution database was queried for open AWR patients with preoperative CT imaging. Patients with < 18 months of follow up were excluded. Clinical data including age, sex, BMI, history of diabetes or tobacco use, and postoperative recurrence were collected. Patient data were batched into training (80%) and test (20%) sets. A convolutional neural network was trained to evaluate image characteristics alone. A feedforward neural network (FNN) was also trained to evaluate clinical data alone. Another FNN was designed to incorporate the outputs from both models. The primary outcome was the ability to accurately predict hernia recurrence, as assessed by the area under the curve (AUC) for each model.

Results: There were 190 patients in this dataset who underwent AWR. Median age was 58 [49, 65] years old, 125 (65.8%) were female, median BMI was 32 [29, 39], 55 (28.9%) had a history of diabetes, and 32 (16.8%) had a history of tobacco use. Overall, 28 (14.7%) had a recurrence with a median follow up of 85 [56, 113] months. Preoperative CT imaging alone returned an AUC = 0.500. Clinical data alone performed better with an AUC = 0.667. Incorporating both models resulted in an AUC = 0.604.

Conclusions: In this study, the DLM with clinical data alone outperformed an image-based DLM in predicting recurrence. Clinical data alone also outperformed the model incorporating both clinical data and preoperative imaging. DLMs have shown significant potential to predict AWR outcomes and may be used in the future to guide care and aid in patient consent for surgery.

S032

A Prospective Study To Evaluate The Abdominal Wall Dynamics Using Surface Electromyography In Patients With Incisional Hernia In Comparison To Healthy Controls

Krishna Asuri 1; Nilanjan B1; Omprakash Prajapati1; Renu Bhatia1; Virinder Bansal1; 1AIIMS, New Delhi

Background: Incisional hernia leads to alteration of abdominal wall dynamics which can be measured using surface electromyography (sEMG). Component separation and other newer techniques promise to restore this abdominal wall dynamics, but this has not yet been studied objectively. This study was conducted to objectively measure and compare abdominal wall dynamics between patients with incisional hernia and healthy controls.

Methods: This prospective comparative study was conducted between May 2020 and November 2021. Twenty patients with incisional hernia (Group A), and twenty age and sex-matched healthy controls (Group B) were recruited in the study. Patients with complicated incisional hernia, underlying collagen disorders and with severe cardiac or respiratory insufficiency were excluded. sEMG was performed for three anterior abdominal wall muscle groups bilaterally, i.e., rectus abdominis (RA), external oblique (EO) and internal oblique (IO) muscles. All individuals were asked to perform predefined movements for each muscle group. Maximum voluntary contraction (MVC), time of contraction and area under the curve were recorded using sEMG.

Results: The median MVC and area under the curve for each of the muscle groups, i.e., rectus abdominis (p = 0.002), external oblique (p = 0.003) and internal oblique(p = 0.001) muscle were statistically significantly higher in healthy controls as compared to patients with incisional hernia. The impairment of abdominal wall dynamics in patients with incisional hernia was not associated with age, sex, location and size of the hernia.

Conclusions: sEMG is a technically feasible method to measure abdominal wall dynamics in terms of MVC and area under the curve, as indirect measures of muscle strength and power, respectively. Abdominal wall muscle strength and power is severely impaired in patients with incisional hernia, in comparison to the healthy population.

S033

Does Increasing Obesity Class Result In Untoward Outcomes In Robotic-Assisted Ventral Hernia Repair: An ACHQC Analysis

Antoinette Hu, MD 1; Debarati Bhanja, BS1; Elizabeth M. Sodomin, MD1; Sharon Phillips, MSPH1; Eric M. Pauli, MD1; Charlotte M. Horne, MD1; 1Penn State Health Milton S. Hershey Medical Center

Background: Obesity is considered a risk factor for hernia formation and for poor outcomes after hernia repairs. Limited outcomes data exist for patients with obesity undergoing robotic-assisted ventral hernia repair (RVHR). We hypothesized that obesity class would correlate with increasing rates of surgical site occurrences (SSO) and hernia recurrences following RVHR.

Methods: Data from the Abdominal Core Health Quality Collaborative (ACHQC) registry was obtained for patients with obesity who underwent RVHR from 2012 to 2022 and had recorded postoperative follow-up. Patients were stratified based on their body mass index (BMI) into Class I (BMI 30–35 kg/m2), Class II (BMI 35–40 kg/m2), or Class III (BMI > 40 kg/m2). Primary outcomes were SSO (including surgical site infection (SSI)) and hernia recurrence rates.

Results: 4784 patients were included with 2258 Class I (47.2%), 1422 Class II (29.7%), and 1104 Class III (23.1%) patients. Patients with Class III obesity were more likely to be female (62%), undergo an emergency surgery (2.08%), and have a recurrent hernia (28%). 30 day follow up rate was 100% and 1 year follow up rate was 13.7%. At 30-day follow-up, there was no difference in recurrence rates but there were significant differences in total SSO rates between obesity classes; Class I 6.6%, Class II 8.8%, and Class III 10.7% (p < 0.001). Despite higher SSO rates, there were no differences in rates of specific wound complications (eg. SSI, SSO requiring procedural intervention) between classes. On multivariate regression analysis, the predicted probability (PP) of 30-day SSO/SSI increased with hernia width across all obesity classes with an odds ratio of 1.25. As hernia width increased from 2 to 6 cm, the odds of SSO/SSI increased by about 25% (p < 0.001). At 1 year, there were no differences in SSO or SSI rates.

Conclusion: Using a national hernia database, we found that RVHR in patients with obesity appears to be safe in terms of 30-day complications and SSO/SSI rates, irrespective of obesity class. While statistically significant, the difference in PP of SSI/SSO between obesity class I and class III is only 2% at any given hernia width, which may not be clinically significant. Additionally, statistically significant baseline characteristics between each obesity class may affect SSO/SSI outcomes. Lastly, overall follow up at 1 year was low so limited conclusions about recurrence can be made. Longer follow-up is necessary to assess recurrence rates in these patients.

S034

Surgical Site Occurrence After Prophylactic Use of Mesh for Prevention of Incisional Hernia in Midline Laparotomy: Systematic Review and Meta-analysis of Randomized Clinical Trials

Irma Lisseth Maldonado Barrios, MD 1; Efren Lozada, MD1; 1Regional Hospital of High Speciality of Bajio

Background: Use of the mesh is the standard for prevention of incisional hernia (IH). However, the effect of surgical site occurrence (SSO) has never been compared. The aim of this meta-analysis was to evaluate the prevalence of SSO and measure its negative effect through the calculation of the number needed to treat for net effect (NNT net).

Methods: A meta-analysis was performed according to the PRISMA guidelines. The primary objective was to determine the prevalence of SSO and IH, and the secondary objective was to determine the NNT net as a metric to measure the combined benefits and harms. Only published clinical trials were included. The risk of bias was analyzed, and the random effects model was used to determine statistical significance.

Results: 17 studies comparing 2,603 patients were included. The prevalence of IH was significantly lower in the mesh group than in the control group, with an OR of 0.53 (95% CI 0.43–0.65 p = 0.0001). The prevalence of SSO was significantly high in the mesh group than in the control group, with an OR of 1.65 (95% CI 1.3–2.03 p = 0.0001). So, the way to compare the benefits and risks of each of the studies was done with the calculation of the Number Needed to Treat for net effect NNTnet, which is the average number of patients who are needed to be treated to see the benefit exceeding the harm by one event, and the result was 6, which is the average number of patients who are needed to be treated to see the benefit exceeding the harm by one event.

Conclusion: Use of the mesh reduces the prevalence of IH and increase the prevalence of SSO, but NNT net determined that the use of mesh continues to be beneficial for the patient.

Funding: No funding was received.

figure h

S035

The Association of Active Smoking at the Time of Minimally Invasive Inguinal Hernia Repair with Mesh and 30-Day Wound Events and Additional Morbidity Outcomes: An Analysis of the ACHQC Database

Ivy N Haskins, MD 1; Felicia C Thorson, APRNNP1; Robert Tamer, MPH, MPA2; Vishal M Kothari, MD1; Arielle J Perez, MD, MPH, MS3; 1University of Nebraska Medical Center; 2Ohio State University; 3University of North Carolina at Chapel Hill

Introduction: No study to date has looked specifically at the association of active smoking on wound events following minimally invasive (laparoscopic or robotic) inguinal hernia repair (IHR) with mesh. We aimed to determine if active smoking at the time of minimally invasive IHR with mesh was associated with worse 30-days wound events and additional morbidity outcomes using the Abdominal Core Health Quality Collaborative (ACHQC) database.

Methods: All adult patients undergoing elective, minimally invasive IHR with mesh who had at least 30-day follow-up data available were identified within the ACHQC database. Smokers were defined as having smoked at least one cigarette within the 30 days prior to surgery. A 1:1 propensity score matched analysis was performed comparing smokers to non-smokers, controlling for age, sex, body mass index, and comorbidities. The effect of smoking on 30-day wound events and additional morbidity outcomes following minimally invasive IHR with mesh was investigated using Chi-square or Fisher's exact test for categorical data and Wilcoxon ranked test for continuous data.

Results: A total of 15,325 patients met inclusion criteria; 1,673 (10.9%) were active smokers at the time of minimally invasive inguinal hernia repair with mesh. A total of 3,286 patients were used for the matched analysis. A majority of the patients in both groups underwent unilateral IHR for pain with an associated bulge. With respect to 30-day outcomes, active smokers were more likely to experience a postoperative seroma but were not more likely to experience a surgical site infection, surgical site occurrence requiring procedural intervention, or a 30-day inguinal hernia recurrence.

Conclusions: Active smoking at the time of minimally invasive IHR with mesh is not associated with worse 30-day wound or additional morbidity outcomes. Further studies are needed to determine long-term effects of smoking on minimally invasive IHR with mesh.

S036

A Novel Combination of Endoscopic Anterior Component Separation as an Aid to Laparoscopic Ventral Hernia Repair for Large Ventral Hernia

Sameer Rege, MS1; Sulay Shah, MS 1; Vivek Salvi, MS1; Jayati Churiwala, MS1; Vishal Sangade1; 1Seth GS Medical College and KEM Hospital Mumbai

Introduction: Laparoscopic Ventral Hernia Repair (LVHR) along with defect closure has been the standard norm in recent times. In cases of large defects (> 6 cm), LVHR is technically challenging thus needing either open or endoscopic component separation. Component separation technique (CST) has its own set of technical and post-operative complication especially in unexperienced hands. In recent times, transversus abdominis release(TAR) is used widely for CST and anterior component separation has fallen out of favour owing to local complications and poor outcomes. We present a novel combination of Endoscopic subfascial anterior component separation (EACS) with laparoscopic ventral hernia repair for medium sized defects.

Materials and Methods: An observational study of all ventral hernia cases operated using EACS followed by LVHR from 2010 to 2022 was conducted in a tertiary care centre. In all cases, subfascial EACS was performed bilaterally, laparoscopic closure of defect was achieved, followed by an Intraperitoneal composite mesh repair. Demographic data, intraoperative and post-operative parameters were analysed.

Results: 69 patients with 30 male and 39 females were operated with mean age of 46.8 years (36–62) and BMI of 38 kg/ m2. 48 patients had defect size at midline between 6–8 cm while 21 had between 8- 10 cm. Mean operative time was 100 min (85–125 min) with mean post operative hospital stay of 3.4 days. Defect closure was possible in all cases after EACS. Postoperative SSO were seen in 8 cases (11.6%), out of which there were 6(8.7%) cases of seroma and 2(2.89%) superficial SSI with none requiring intervention. There was no surgical mortality. Recurrence at mean follow up of 24 months was a single case of lateral recurrence (1.45%). No other long term complications including mesh infection were reported.

Conclusion: EACS combined with LVHR for 6–10 cm defects is an attractive alternative to open repair with lower operative times and minimal morbidity for the patient. Outcomes including SSO and recurrences are comparable to standard open and endoscopic repairs. Subfascial EACS has advantage of having minimal local complication and easy learning curve compared to other techniques. This combined with LVHR can achieve defect closure with an easier minimal access technique. The novel combination also has the advantage of avoiding a technically challenging posterior component separation like TAR, thus easing the surgical challenges and having the potential of larger uptake amongst all surgeons alike.

S037

Extent of lymph node harvest: a comparison of intracorporeal vs. extracorporeal anastomosis in right hemicolectomy

Katlin Mallette, MD 1; Jeffrey Hawel, MD, MSc1; Ahmad Elnahas, MD, MSc1; Christopher M Schlachta, MDCM1; Nawar A Alkhamesi, MD, PhD1; 1CSTAR (Canadian Surgical Technologies & Advanced Robotics), London Health Sciences Centre, and Department of Surgery, Western University, London, Canada

Introduction: Colorectal cancer remains one of the most common cancers in North America despite robust screening programs. Minimally invasive surgery has been demonstrated to have clear advantages in colon cancer management, through a decrease in the morbidity and mortality associated with surgery. With the introduction of intracorporeal anastomosis, the mesenteric dissection is performed under direct visualization and may lead to superior lymph node harvest.

The aim of our study is to evaluate lymph node harvest in patients undergoing totally laparoscopic right hemicolectomy with intracorporeal anastomosis (ICA) compared to laparoscopic assisted right hemicolectomy with extracorporeal anastomosis (ECA).

Methods and Procedures: This is a single institution retrospective cohort study. Eligible patients underwent laparoscopic right hemicolectomy at our institution between 2012 and 2022. Patients were identified using a hospital database, and surgeon office databases. Patients included in the study underwent laparoscopic right hemicolectomy for neoplastic lesions (colon cancer/unresectable polyps), or benign etiologies. We excluded patients who underwent laparotomy (planned or intra-operative conversion), resection without anastomosis, resection for IBD or lack of documented lymph node number. Data was compared using two sided t-test evaluation with a 95% confidence interval.

Results: A total of 679 patients were included, 493 ECA (72.6%) and 186 ICA (27.4%). Patient demographics (age, biologic sex, ASA and BMI) were not significantly different. Lymph node harvest was significantly higher in those with ICA when compared to ECA (EC:21 ± 11 vs. IC:24 ± 14; p-value: < 0.05). In subgroup analysis, this difference was maintained in patients with malignant processes (EC: 23 ± 10 vs. IC: 27 ± 14, p-value: < 0.05). The proven benefits of ICA have been documented in our previous study including length of stay, anastomotic leak, SSI and incisional hernia.

Conclusion: Overall, our study demonstrated that ICA has higher lymph node harvest in comparison to ECA, which in turn provides significant improvement in patient management and the application of adjuvant therapies.

S039

Comparing Advanced Platforms for Local Excision of Distal Rectal Lesions

Deborah S Keller, MS, MD1; Taylor Ikner1; Henry Schoonyoung1; Hela Saidi1; Gerald Marks1; John H Marks 1; 1Lankenau Medical Center

Background: Transanal surgery facilitates organ-preservation in select patients benign and early malignant distal rectal lesions to avoid the functional consequences of radical surgery. The transanal endoscopic microsurgery (TEM) platform created a standard for local excision, offering a magnified view and greater access to proximal lesions with lower margin positivity, specimen fragmentation, and recurrence rates than traditional transanal excision. The single port robot (SPR) presents a promising alternative transanal platform. As early adopters, we aim to find the best applications for this new technology. The goal of this study was to compare peri-operative and pathologic outcomes of TEM and SPR for excision of distal rectal lesions.

Methods: A review of consecutive patients who underwent local excision of distal rectal lesions (within 5 cm of the anorectal ring) at a single tertiary referral center from 1/2001–5/2022 was performed. Cases were stratified into TEM or SPR in a 1:1 propensity score-matched cohort, adjusting for all baseline characteristics. Clinical, tumor-specific, and perioperative outcomes were compared using χ2, t-tests and Mann–Whitney U-tests. The main outcome were oncologic quality measures, complications, and operative time.

Results: Matching resulted 50 SPR and 50 TEM patients. Groups had similar age, gender, body mass index, comorbidity, diagnosis, lesion characteristics, and neoadjuvant chemoradiation rates. There were identical rates of intraperitoneal entry across groups. Three SPR patients were converted intraoperatively- 2 to TEM and 1 to an abdominal approach. The SPR had significantly shorter operative times. The rate of specimen fragmentation and positive distal, radial, and proximal margins were low and similar across groups. Postoperatively, the groups had comparable morbidity and no short-term mortality. The rate of local recurrence was minimal and alike in both groups.

Conclusions: This SPR provided high quality outcomes that were similar to TEM for local excision of distal rectal lesions. The SPR offered a faster operative time with comparable clinical and oncologic outcomes to TEM. While longer-term follow-up is needed for recurrence, the SPR offered an excellent alternative to TEM. As we looks for options in patients with complete and near complete responses to treatment, this early data is promising for expanding use of the new SPR technology.

figure i

S040

Survival Paradox Between Stage IIB/C and Stage IIIA Colon Cancer: Is It Time to Revise the American Joint Committee on Cancer TNM System?

James Sahawneh, MD 1; Whiyie Sang, MD1; Tingting Li, PhD2; Mei-Chin Hsieh, PhD2; Yong Yi, PhD2; John Gibbs, MD3; Xiao-Cheng Wu, MD, MPH3; Joseph Gallagher, MD4; Quyen D. Chu, MD5; 1Orlando Health Colon & Rectal Surgery; 2Louisiana Tumor Registry and Epidemiology Program, School of Public Health, Louisiana State University Health Sciences Center-New Orleans, Louisiana; 3Louisiana State University Health Science Center Department of Medicine-Shreveport, LA; 4Colon & Rectal Clinic of Orlando; 5Orlando Health Surgical Oncology

Introduction: A survival paradox between T4N0 (Stage IIB/IIC) and Stage IIIA colon cancer exists, even after adjusting for adequate lymph node (LN) retrieval and receipt of adjuvant chemotherapy (C). We conducted a large hospital-based study to re-evaluate this survival paradox based on the newest 8th edition staging system.

Methods: The National Cancer Data Base was queried to evaluate 35,606 patients diagnosed with Stage IIB, IIC, and IIIA colon cancer between 2010 and 2017. The Kaplan–Meier method and log-rank test were used to compare unadjusted overall survival (OS). Multivariable Cox proportional hazards model was used to determine the association of stage with hazard ratios adjusted for relevant demographic and clinical variables including ≥ 12 LNs retrieved and receipt of adjuvant chemotherapy. P-value < 0.05 was considered statistically significant.

Results: The 5-year OS for optimally treated stage IIIA colon cancer (receipt of Chemotherapy) was 84.3%, which was significantly higher than stage IIB/C (≥ 12 LNs retrieved + Chemotherapy) (72.8%; P < 0.0001). Stage was an independent predictor of OS. Among optimally treated Stage IIIA patients, T1N1 had the best survival (90.6%) while stage T4bN0 (Stage IIC) had the worst (70.9%) (P < 0.0001). Compared to Stage IIB, stage IIC had a 17% increased risk of overall death while stage IIIA had a 21% reduction in death (P < 0.0001).

figure j

Conclusions: Stage IIB/C and Stage IIIA survival paradox persists even after accounting for adjuvant chemotherapy and adequate lymph node retrieval. Consideration of this paradox should be taken into account with future editions of the TNM system.

S041

Long-term Quality of Life, Urinary and Sexual Function following Laparoscopic and Robotic Total Mesorectal Excision with Lateral Pelvic Node Dissection in Locally Advanced Rectal Cancer

Gyu-Seog Choi1; Hye-Jin Kim 1; Jun Seok Park1; Soo Yeun Park1; Seung Ho Song1; Min Hye Jung1; 1Kyungpook National University Chilgok Hostpial, Daegu, Korea

Background: Quality of life (QoL) and functional outcomes are at risk of being impaired after rectal surgery, but there has been no large prospective study to thoroughly assess QoL according to total mesorectal excision (TME) with lateral pelvic node dissection (LPND).

Purpose: We have investigated the impact of this operation on long-term quality of life (QoL) and urogenital functions.

Methods: Patients undergoing TME with LPND after preoperative chemoradiotherapy (CRT) for rectal cancer between 2015 and 2019 were prospectively enrolled in this questionnaire-based survey of QoL together with variations in urinary and sexual function. QoL was assess with the European Organization for Research and Treatment of Cancer Quality of Life Questionnaire (EORTC-QLQ)-C30 and CR29. The International Prostate Symptom Score (IPSS) was used to assess bladder function. And the International Index of Erectile function (IIEF) and the Female Sexual Function Index (FSFI) were used to assess male and female sexual function, respectively. The survey was performed preoperative (preop) and at least 12 months after surgery (postop).

Results: A total of 101 patients (82.1%) completed the survey. Global health status/QoL was not compromised after TME with LPND (preop vs. postop; 61.9 ± 19.8 vs. 70.8 ± 19.9; P = 0.065). In EORTC QLQ-CR29, only fecal incontinence was significantly increased after 12 months (9.9 ± 20.3 vs. 33.3 ± 29.6; P < 0.001). IPSS was significantly increased after surgery (13.0 ± 9.0 vs. 16.2 ± 9.9; P = 0.048): but it was only significantly different in 22 patients who had unsolved complications during the survey (12.1 ± 11.6 vs. 21.0 ± 10.0; P = 0.001). Among 65 male patients, only 21 patients (32.3%) replied to have sexual activity before and after 12 months. Erectile function decreased, but did not reach statistical significance (19.5 ± 8.1 vs. 14.9 ± 6.8; P = 0.107). The other domains were similar after 12 months. 14 female patients (38.9%) completed FSFI survey. Total score and all domains were not different (total score of preop vs. postop; 19.3 ± 8.9 vs. 18.5 ± 9.5; P = 0.836).

Conclusion: Patients undergoing TME with LPND have comparable QoL and urogenital function after 12 months postoperatively, based on the questionnaire-survey. But patients who have unsolved complications had compromised urinary function after surgery. Further studies are still needed for assessing sexual function after TME with LPND.

S042

Does Patient Activation Impact Remote Digital Health Follow-Up and Same-Day Discharge After Elective Colorectal Surgery

Tiffany Paradis, MD 1; Stephan Robitaille, MD1; Teodora Dumitra, MD1; A. Sender Liberman, MD1; Patrick Charlebois, MD1; Barry Stein, MD1; Julio F Fiore Jr, PhD1; Liane S. Feldman, MD1; Lawrence Lee, MD1; 1McGill University

The objective of this study was to determine the impact of patient activation (PA) on the effectiveness of digital health remote post-discharge follow-up among patients undergoing elective colectomy. Low PA (PA = ability to manage their own health conditions) is associated with worse outcomes after major surgery. We have previously shown that remote post-discharge follow-up using a digital health app decreased unnecessary emergency department (ED) visits after colorectal surgery, and it is now an important component of our same-day discharge (SDD) program. However, the impact of PA on the effectiveness of remote discharge follow-up is unknown.

We administered the Patient Activation Measure (PAM) to elective colorectal surgery patients that received a digital health (DH group) remote post-discharge follow-up app from 03/2019–08/2022, including a subset of patients undergoing SDD colectomy. The app included patient education material, daily health check questionnaires, and patient-physician communication. The DH group was compared to PAM scores from a control colorectal surgery cohort from 10/2017–04/2018 (CC group) without the remote post-discharge follow-up app. The PAM score (0–100) categorizes patients into low (≤ 55.1) or high (> 55.1) PA. The main outcome measure was 30-day postoperative ED visits. Secondary measures included total length of stay (LOS), and 30-day complications.

A total of 152 patients were included [77DH(including 39SDD); 75CC], with no differences in patient or operative characteristics (mean age 58.5yrs(SD15.2), 49% male, 71% malignancy, 88% laparoscopic, 28% proctectomy, 11% new stoma). The proportion of high PA was similar in DH and CC(82%vs. 73%, p = 0.21), and SDD and non-SDD (88%vs. 76%, p = 0.25). High PA was associated with less ED visits (14%vs. 32%, p = 0.02) overall and in the SDD subgroup (6%vs. 60%, p = 0.001) but not within the DH or CC groups. There were no differences in total LOS or complications between high and low PA overall, or within the DH or CC groups. Total LOS was shorter for high PA in the SDD subgroup (0d(SD0.2) vs 5.8d (SD12.4), p = 0.006). On multiple regression, high PA remained independently associated with ED visits and total LOS only in the SDD group.

Overall, PA did not impact the effectiveness of remote monitoring on outcomes after colectomy. However, in a subgroup analysis of SDD patients, low PA was associated with a higher incidence of ED visits and longer LOS. Therefore, PA may have an impact on the success of SDD and may be considered as a criterion for SDD.

S043

Which Video Technology Brings the Higher Cognitive Burden and Motion Sickness in Laparoscopic Colorectal Surgery: 3D, 3D–4K or 2D–4K? A Propensity Score Study

Giuseppe Portale 1; Ylenia C Spolverato1; Alice Tonello1; Patrizia Bartolotta2; Gianfranco Frigo3; Marco Simonetto4; Dario Gregori2; Valentino Fiscon1; 1General Surgery ULSS 6 Euganea, Cittadella; 2Unit of Biostatistics, Epidemiology and Public Health, Department of Cardiac Thoracic Vascular Sciences and Public Health, University of Padova; 3Cardiology, ULSS6 Euganea, Cittadella; 4Neurology, ULSS6 Euganea, Cittadella

Background: Technological development has offered laparoscopic colorectal surgeons new video systems to improve depth perception and perform difficult tasks in limited space. The aim of this study was to assess the cognitive burden for surgeons during 3D, 3D-4 K or 2D-4 K laparoscopic colorectal procedures and to report post-operative data with the different systems employed.

Methods: Patients were assigned to either 3D, 3D-4 K or 2D-4 K video system and two questionnaires (Simulator Sickness Questionnaire-SSQ- and NASA Task Load Index -TLX) were used to collect data on cognitive burden and motion sickness during elective laparoscopic colorectal resections (October 2020-August 2022) from two operating surgeons. Short-term results of the operations performed with the three different video systems were also analyzed.

Results: A total of 113 consecutive patients were included: 41 (36%) in the 3D Group (A), 46 (41%) in the 3D-4 K Group and 26 (23%) in the 2D-4 K Group (C). Weighted and adjusted regression models showed no significant difference in cognitive load amongst the surgeons in the three groups when using the NASA-TLX. An increased risk for slight/moderate general discomfort and eyestrain in the 3D-4 K group compared with 2D-4 K group (OR = 3.5; p = 0.0057 and OR = 2.8; p = 0.0096, respectively) was observed. Further, slight/moderate difficulty concentrating was lower in both 3D and 3D-4 K groups compared with 2D-4 K group (OR = 0.4; p = 0.0124 and OR = 0.5; p = 0.0341, respectively), and higher in the 3D-4 K group compared with 3D group (OR = 2.6; p = 0.0124). Patient population characteristics as well as operative time, post-operative staging, complication rate and length of stay were similar in the three groups of patients.

Conclusions: 3D and 3D-4 K systems, when compared with 2D-4 K video technology, have a higher risk for slight/moderate general discomfort and eyestrain, but lower difficulty concentrating. Short post-operative outcomes do not differ, whichever imaging system is used.

S044

Does Specimen Extraction Incision and Transversus Abdominus Plane block Affect Opioid Requirements After Laparoscopic Colectomy?

Stephan Robitaille, MD 1; Anna Wang, MD1; Naser Alali1; Hiba Elhaj1; Sender Liberman1; Patrick Charlebois1; Barry Stein1; Liane S Feldman1; Julio F Fiore Jr.1; Lawrence Lee1; 1McGill University Health Center

Introduction: Advances in regional anesthetic techniques such as the transversus abdominus plane (TAP) block have been associated with decreased post-operative opioid use following colorectal surgery. Studies evaluating incision type in open surgery have also reported associations with post-operative pain and opioid use but there are few data in laparoscopic surgery. We sought to evaluate the effect of TAP block and specimen extraction incision(SEI) on opioid requirements after elective laparoscopic colectomy.

Methods: A prospectively collected database of patients undergoing elective laparoscopic colectomy without a new stoma creation at a single university affiliated colorectal referral center between 2017–2022 was queried. All patients were managed by a mature enhanced recovery pathway with an emphasis on opioid-sparing multimodal analgesia including regular NSAIDs. Patients were grouped according to whether they received an intra-operative TAP block (bilateral infiltration of 20 ml 0.25% bupivacaine + 5 mg dexamethasone) and by SEI (midline, Pfannenstiel, or transverse). Post-operative opioid use was measured by postoperative day (POD) up to POD3 and reported in morphine milligram equivalents(MME).

Results: Overall, 485 patients were included with 42% TAP block versus 58% without. SEIs were midline (11%), transverse (11%) and Pfannenstiel (78%). Patients were 47% female and mean age 65(SD14) years. There was no difference in patient characteristics between TAP vs. no TAP. Patients undergoing right-sided resections were more likely to have a midline SEI when compared to left-sided or rectal resections (80% vs. 46%, p < 0.001). There were no differences in MME or proportion opioid-free between SEIs on any POD. MME was lower in the TAP group on POD0 (27.9 vs. 35.6, p < 0.001) and POD1 (13.5 vs. 28.4, p < 0.001). The TAP block group also had a higher proportion of opioid-free patients on POD0 (13% vs. 4%, p < 0.001) and POD1 (39% vs. 24%, p = 0.001) but not beyond. Overall, 42% of patients were opioid free on POD2 and 53% on POD3. On multiple linear regression, TAP was independently associated with -6.8 MME (95%CI: -11.2,-2.3) on POD0 and -13.0 MME (95%CI -18.0,-8.1) on POD1 after adjusting for SEI and other confounders. SEI was not independently associated with a difference in MME on any POD.

Conclusion: Specimen extraction incision did not affect post-operative opioid consumption following laparoscopic colectomy, regardless of regional anesthesia. TAP blocks were independently associated with a significant decrease in opioid use on both POD0 and POD1, but not beyond. These findings suggest TAP blocks are more important than selection of extraction incision in reducing opioid consumption following laparoscopic colectomy.

S045

Risk Stratification of AKI Following Ureteral Stent Insertion for Colorectal Surgery

Thomas G. Matkov, MD 1; Laurel S. Curry1; Alma L. Ochoa, APNP1; 1Aurora Medical Center, Kenosha, WI

Introduction and Objectives: Ureteral stents have long been utilized during colorectal resections to assist in the identification of ureters intraoperatively and mitigate risk of ureteral injury. As these procedures have shifted toward robot-assisted laparoscopic methods, lighted stents have increasingly been used. The incidence of acute kidney injury (AKI) following bilateral ureteral stent placement has been reported to be as high as 41.9%. We sought to identify our single institution risk, and determine the extent to which age, sex, and stent type affected incidence of AKI.

Methods: A retrospective analysis was performed at a single community hospital of all open and robotic-assisted laparoscopic colorectal surgeries from October 2012 to April 2022. If requested, ureteral stents were placed bilaterally by a urologist and later removed by the general surgeon. Non-lighted stents used were 5 Fr whistle-tip (BARD); lighted stents were 6 Fr with a fiberoptic core (STRYKER). Kidney failure was described as a rise of creatinine to 31.5 times the preoperative value, per KDIGO guidelines. Statistical analysis was performed using R.

Results: 633 consecutive colorectal surgeries were evaluated, with no stents placed in 237 cases, non-lighted stents placed in 137 cases, and lighted stents placed in 259 cases. No ureteral injuries were observed. Overall incidence of AKI for non-stented surgeries was 0.8% vs 5.8% for non-lighted stents and 5.8% for lighted stents. Patient age was the most significant factor in AKI incidence: for patients under 60, there was no statistical difference in AKI incidence for stented vs non-stented procedures (2.2% vs 1.1%). For patients over 60, the risk of AKI was 10.5% for stented vs 0.7% for non-stented. Female patients had statistically significant risk differences, with AKI incidence of 7.1% stented vs 0.0% non-stented. AKI completely resolved in all cases, regardless of cohort.

Conclusions: In patients under age 60, the use of stents was not associated with an increased risk of AKI. For women and those over 60, stents pose a higher risk of transient AKI. Overall incidence of AKI in our larger and single-institution community hospital population was significantly lower than reported in other studies. No statistical difference was observed, overall, in AKI incidence between lighted and non-lighted stents.

Source of Funding: None.

S046

Modified Pull-Through Coloanal Anastomosis to Prevent Temporary Ileostomy and Reduce Postoperative Complications for Lower Rectal Tumors

Shinichiro Mori 1; Kan Tanabe1; Masumi Wada1; Yuki Hamada1; Ryutaro Yasudome1; Takaaki Arigami1; Ken Sasaki1; Hiroshi Kurahara1; Takao Ohtsuka1; 1Department of Digestive Surgery, Breast and Thyroid Surgery, Graduate School of Medicine, Kagoshima University

Introduction: Sphincteric-preserving ultralow anterior resection (ULAR) for lower rectal tumors often requires temporary ileostomy to avoid anastomotic leak leading sepsis complications. Recently, the second-stage Turnbull-Cutait pull-through hand-sewn coloanal anastomosis has been reintroduced to avoid those problems. We performed modified pull-through coloanal anastomosis via immediately performing coloanal hand-sewn anastomosis after sphincteric-preserving ULAR (pull-through ultra) in the first stage for lower rectal tumors to avoid the temporary ileostomy and the postoperative complications.

Aim: The aim of this study was to compare clinical outcomes of the pull-through ultra versus the standard coloanal anastomosis with temporary ileostomy after sphincteric-preserving ULAR for lower rectal tumors.

Methods: We retrospectively analyzed the prospectively collected data of 106 (rectal cancer 94; rectal NET 9; rectal GIST 3) consecutive patients with rectal cancer who underwent pull-through ultra (n = 23) or standard coloanal anastomosis with temporary ileostomy (SCAA, n = 83) from January 2011 to September 2022. Clinical outcomes were assessed.

Surgical procedure of the pull-through ultra: In a first stage of hand-sewn coloanal anastomosis was prepared for anastomosis, dissecting the mesocolon at 4 cm proximal to the colonic stump. And then blood perfusion was confirmed using indocyanine green fluorescence imaging. On the other hand, rectal stump was prepared for anastomosis. Sixteen sutures using 4–0 monofilament suture were stitched rectal stump and hold it to the TMP suture-ring (Tokai medical products Inc, Kasugai, Aichi, Japan). Then, a hand-sewn side to end coloanal anastomosis was immediately performed circumferentially. Thus, we created colostomy at the anal site in the first stage. In 6–8 days after primary surgery for the second stage, after setting the self-retaining anal retractor, amputation of the prolapsed colon was carried out and then additional sutures using a 4–0 monofilament suture was done if needed.

Results: The median of operative time in pull-through ultra group and SCAA group were 392 min and 404 min, respectively. The rate of the complications in pull-through ultra group and SCAA group were 21% and 44%, respectively. The rate of the complications was significant lower in the pull-through ultra group (p = 0.04). The rate of the anastomotic leak in pull-through ultra group and SCAA group were 4.3% and 6%, respectively. We did not create any stomas for patients in pull-through ultra group.

Conclusion: The unique technique of pull-through ultra via immediately performing coloanal anastomosis is promising technique to avoid temporary ileostomy and to reduce postoperative complications after sphincteric-preserving ULAR following for lower rectal tumors.

S047

A Nationwide Readmissions Database (NRD) Analysis Assessing the Timing of Readmissions for Complications Following Emergency Colorectal surgery: Why Limiting Follow-Up to POD30 Underserves Patients

Natasha G Caminsky, MDCM, MSc; Jeongyoon (Jenny) Moon, MDCM, MSc; Daniel Marinescu, MBA, MEng; Allison Pang, MD, MSc, FRCSC; Carol-Ann Vasilevsky, MDCM, FRCSC, FACS; Marylise Boutros, MDCM, FRCSC; Division of Colon and Rectal Surgery, Sir Mortimer B. Davis Jewish General Hospital, Montreal, Quebec, Canada

Introduction: Emergency procedures are associated with a higher risk of complications than those performed electively. The literature lacks a critical assessment of complications occurring beyond post-operative day (POD)30. The aim of this study was to assess the readmission rate and factors associated with readmission 6-months following emergency colectomy.

Methods: A retrospective cohort study using the Nationwide Readmissions Database of adult patients who underwent emergency colectomies (2010–2018) was performed. Cases performed January-June were retained, allowing 6-months follow-up. The cohort was divided into: (i)no readmission and (ii)readmission(s) for complications related to colectomy (ICD-9/10 codes). Readmissions were categorized as either "early" (POD0-30) or "late" (> POD30). Differences between patients admitted for early vs late complications after colectomy were described and multivariable regression controlling for relevant covariates defined a priori were used to identify factors associated with timing of readmission and cost.

Reults: Of 141,532 patients, 33.7% (N = 47,702) were readmitted within 6-months of emergency colectomy. Of these, 71.7% (34,180) were readmissions for colectomy-related complications, of which 73.7% (N = 25,195) were "late" (≥ POD30), with a median time to readmission of 97 (IQR62-133) days [compared to 17 (IQR12-22) days in the early group]. Compared to early readmissions, late readmissions were more likely to occur following sigmoidectomy (45.9% vs 29.7%), a longer length of stay for their initial surgery [10 (IQR7-15) vs 9 (IQR6-12) days, p < 0.0001], open colectomies (85.8% vs 81.6%, p < 0.0001), and an admission estimated to result in "extreme loss of function" by All Patients Defined Diagnosis Related Groups (APD_DRG; 23.8% vs 18.3%, p < 0.0001) compared to the early group. Early readmissions were more likely to occur following right hemicolectomy (RHC; 48.0% vs 30.0%, p < 0.0001), The most common reasons for late readmission were stoma-related complications, infection, and gastrointestinal complications (35.1%, 23.5%, and 4.2%, respectively) and infection, anastomotic leak, gastrointestinal complications (36.5%, 16.2%, and 13.6%, respectively) were the most common causes of early readmission. On multiple logistic regression, open procedures (OR1.28, 95%CI 1.18–1.40), extreme APD_DRG severity (OR1.71, 95%CI 1.523–1.919), alcohol use disorder (OR1.32, 95%CI 1.09–1.56), and sigmoidectomies (OR2.51, 95%CI 2.34–2.70, relative to RHCs) were the strongest predictors of late readmission. On multiple linear regression, late readmissions were associated with a $4,936.09 (95%CI $4,936.06–4,936.13 USD) increased cost of readmission compared to early readmissions.

Discussion: The majority of readmissions following emergency CRS occur beyond POD30 and are associated with open procedures, specifically sigmoidectomies, and increased comorbidity. Given the associated increased cost of care, mitigation of such readmissions by close follow-up beyond POD30 is advisable.

S048

The Analysis of the Relationship Between Skeletal Muscle Radiodensity and the Continuity of Adjuvant Chemotherapy in Colorectal Cancer

Keigo Yokoi, PhD 1; Kazuko Yokota, PhD1; Toshimichi Tanaka, PhD1; Hirohisa Miura, MD1; Takahiro Yamanashi, PhD1; Takeo Sato, PhD, Prof2; Takeshi Naitoh, PhD, Prof, FACS1; 1Kitasato University School of Medicine; 2Kitasato University School of Medicine, Department of Clinical Skills Education, Research and Development Center for Medical Education

Background: We have been reported the potential of skeletal muscle radiodensity (SMR) as the prognostic marker for colorectal cancer. There have been a few reports about the association between the continuity of adjuvant chemotherapy in colorectal cancer and SMR.

Aim: The aim of this report is to clarify the association between the continuity of adjuvant chemotherapy by CapeOX regimen in Stage II and III colorectal cancer and SMR.

Methods: We retrospectively reviewed 143 colorectal cancer patients who underwent curative laparoscopic surgery in our hospital and following adjuvant chemotherapy by CapeOX regimen between January 2012 and December 2021. SMR was measured based on preoperative CT images. Patients were classified as low SMR group (bottom quarter) and high SMR group (top three quarters). We compared the number of the cycles of performed chemotherapy, the amount of the dose of capecitabine and oxaliplatin, relative dose intensity (RDI), and adverse effects in each group.

Results: In high SMR group, the ratio of the patients who performed more than 3 cycles, 5 cycles and 8 cycles were significantly higher than low SMR group (72% vs 96%, 53% vs 75%, and 44% vs 68%, respectively). The amount of the dose of capecitabine and oxaliplatin were also higher in high SMR group. The incidence of Grade 2 or Grade 3 adverse effect were not different in each group. The frequency of discontinuation of the treatment because of adverse effects was significantly low in high SMR group. No significant difference was seen in each group according to the RDI of capecitabine and oxaliplatin (78% vs, 81% and 80% vs 79%). Logistic regression analysis revealed the factors related to successful treatment completion were Stage III (odds ratio 9.30, 95% CI 1.07–80.54), and high SMR (odds ratio 3.09, 95% CI 1.07–8.94). Additionally, the ratio of the patients who were treated lower than 2 cycles was lower in high SMR group (50% vs 12%).

Conclusion: High SMR group showed higher completion rate and kept higher drug dose in adjuvant chemotherapy. Higher SMR was the independent factor of the treatment completion.

S049

The Pressure is Rising; An Analysis of Medical Malpractice Claims Related to Abdominal Compartment Syndrome

Asad Choudhry, MD; Ronald Zerna Encalada, BS; Reema Choudhry, MD; SUNY Upstate University Hospital

Background: Abdominal Compartment Syndrome (ACS) refers to organ dysfunction caused by a rise in intra-abdominal pressure and may be a life-threatening emergency. A delay in diagnosis in ACS along with failure to treat especially after an abdominal operation (open or laparoscopic) can lead to severe morbidity, and even mortality. Few studies have examined medicolegal outcomes related to ACS. Using a large legal database, we evaluated reasons for medical litigation related to abdominal compartment syndrome.

Methods: Westlaw® (Thompson Reuters), an online legal dataset in the United States was queried for all medical malpractice cases from 1980 to 2022 related to the diagnosis of abdominal compartment syndrome. We excluded all duplicate cases and cases where the primary legal action was not related to management of abdominal compartment syndrome. Data was collected on patient/case characteristics, procedural variables, defendant information, alleged reason(s) for litigation, and case/trial outcome(s).

Results: The search criteria yielded 20 case briefs. Mean (± SD) patient age was 43 ± 18 years; 10(50%) patients were male. General surgery was the most common surgical specialty implicated in cases. The most common reason for litigation was delayed diagnosis of abdominal compartment syndrome leading to a delay in management. California was the state with the highest number of reported cases 5(25%). An outcome in favor of the defendant was reached in 13 (65%) cases; a plaintiff outcome was reported in 4 (20%) of cases. A settlement was reached in 3 (15%) of cases. Median award payout was $ 2,301,285.

Conclusion: Abdominal compartment syndrome can lead to significant morbidity and may trigger legal redress. Although most cases were decided in favor of the defendant, award payouts were costly. Delayed detection of abdominal compartment syndrome and failing to timely treat the patient was the most common reason for litigation. These medicolegal findings further support the clinical axiom of identifying and addressing abdominal compartment syndrome in a timely and effective manner.

S050

Laparoscopic Peritoneal Dialysis Catheter Insertion-An Oasis Harnessed: Experience and Results at a Tertiary Care Centre in India

Jayati Jagdish Churiwala, MS; Sameer Rege, MS; Yash Jain, Dr; Seth GS Medical College & KEM Hospital

Introduction: The concept of peritoneal irrigation for the treatment of renal failure evolved into peritoneal dialysis for end stage renal disease in the latter half of the twentieth century. By 1980, continuous ambulatory peritoneal dialysis was being offered as renal replacement therapy in several countries throughout the world. Its highlighted merits of improved patient motility and independence, better patient acceptance and home based implementation served as an impetus for the development of open and minimal access techniques for the insertion of peritoneal dialysis (PD) catheter. However, despite an array of available insertion techniques and catheter designs, access to skilled and timely insertion remains a limiting factor to its widespread use in India. Open peritoneal catheter placement has been practiced with notable complications as peri-drain leak, exit site infections, incisional hernia and genital edema.

Case Series: 48 patients were referred from the department of nephrology at our centre for laparoscopic PD catheter insertion. 11 of these had undergone open PD catheter insertion and catheter removal for infection or catheter blockage and failure at least a month prior. Out of rest 37 patients, three had to be operated in an emergency setting due to failed vascular access. A two port technique was used in 32 patients and three port technique was implemented in the rest for simultaneous adhesiolysis and/or omentectomy. A straight tip catheter was tunneled through the rectus muscle in all patients. Two patients had incisional hernia from a previous abdominal surgery which was repaired concomitantly with onlay meshplasty.

Results: The operative time ranged between 20 to 35 min under general anaesthesia. One patient developed peridrain leak as an early complication. All other patients were commenced on peritoneal dialysis two weeks after surgery. None of our patients developed bleeding, peritonitis, catheter blockage requiring removal or surgical site occurrences during a 6 month follow up with the department of nephrology.

Conclusion: The laparoscopic approach is feasible for widespread and safe use for PD catheter insertion. It allows precise placement of the catheter under vision and simultaneous diagnosis and management of other surgical conditions like ventral hernia. Fixation of the catheter tip to pelvic structures is possible in patients undergoing reinsertion following a past history of catheter blockage and failure due to tip migration. The benefits of keyhole surgery can thus be made available to patients at civil hospitals even with a basic laparoscopy setup.

S051

Using the Modified Frailty Index to Predict Post-operative Morbidity and mortality in Patients Undergoing Surgery for Adhesive Bowel Obstruction: A Retrospective Cohort Study

Gaurav Talwar, BSc, MD 1; Tyler McKechnie, BSc, MD1; Janhavi Patel, BMSc, MD1; Reva Qiu, BSc2; Kathy Wu, HBA2; Yung Lee, BHSc, MD1; Dennis Hong, MD, MSc, FRCSC3; Aristithes Doumouras, MD, MPH, FRCSC3; Jessica Bogach, MD, MSc, FRCSC4; Cagla Eskicioglu, MD, MSc, FRCSC, FASCRS3; 1Department of Surgery, Division of General Surgery, McMaster University; 2Michael G. DeGroote School of Medicine, McMaster University; 3Department of Surgery, Division of General Surgery, McMaster University, St. Joseph’s Healthcare; 4Department of Surgery, Division of General Surgery, McMaster University, Juravinski Hospital

Introduction: There is a paucity of studies identifying patient-specific predictors of post-operative outcomes in adhesive small bowel obstruction (ASBO). This study investigates the efficacy of the modified frailty index (mFI) as a tool to predict postoperative morbidity and mortality among patients undergoing surgery for ASBO.

Methods and procedures: A multi-centre retrospective cohort study including patients undergoing surgery for ASBO after failed trial of non-operative management between January 2015 to December 2020 was performed. mFI consisting of 11 variables was used to stratify patients intro groups, frail (≥ 0.27) and robust (< 0.27). The effect of mFI on post-operative morbidity (graded using Clavien-Dindo), mortality, length of stay, and discharge destination was evaluated using multiple logistic regression.

Results: Ninety-two patients with a low mFI (mean age 62.4 years, 68% female) and 41 patients with a high mFI (mean age 81.7 years, 63% female) were included. The groups were well matched in pre-operative and operative characteristics. Frail patients had significantly increased 30-day morbidity (overall morbidity 80% vs. 49%, minor morbidity 73% vs. 47%, major morbidity 32% vs 13%), admission to intensive care unit (40% vs. 10%) and need for higher level of care on discharge (41% vs. 9%). Functional dependence was independently associated with worse 30-day overall morbidity (OR 3.97, CI 1.29–12.19) and lower likelihood of returning to pre-operative disposition (OR 0.21, CI 0.05–0.91).

Conclusions: The mFI, with attention to functional status, may be a convenient tool to predict post-operative outcomes following surgery for ASBO. Further adequately powered prospective studies are required.

Table 1 Post-operative outcomes

30-day outcome

mFI < 0.27

mFI ≥ 0.27

P value

Mortality [n(%)]

3(3%)

5(12%)

0.059

Any morbidity [n(%)]

45(49%)

33(80%)

0.005

Minor morbidity [n(%)]

43(47%)

30(73%)

0.005

Major morbidity [n(%)]

12(13%)

13(32%)

0.011

Post-operative length of stay days [median(range)]

7(1–42)

9(1–101)

0.064

Post-operative intensive care length of stay days [median(range)]

0(0–10)

0(0–8)

 < 0.001

Intensive care post-operative admission [n(%)]

9(10%)

18(40%)

 < 0.001

Discharge to higher level of care disposition (e.g., rehab, long-term care, inpatient facility) [n(%)]

8(9%)

17(41%)

 < 0.001

Reoperation

3(3%)

0(0%)

0.328

S052

Clinical Outcomes in Patients with Complicated Appendicitis

Isheeta Madeka, MD; Keyur Patel, MD; Peter Altshuler, MD; Amrita Iyer, BS; Sonali Dukle, BS; Anirudh Kohli, MD; Thomas Jefferson University Hospital

Introduction: Complicated appendicitis (appendicitis with abscess, perforation, or generalized peritonitis) poses a significant burden on healthcare systems, with incidence up to 28–29% overall. Current management options include up-front surgery, percutaneous drainage, or medical management with antibiotic therapy alone. There is no consensus on treatment guidelines in current literature. This study sets out to better define treatment algorithms for patients presenting with acute complicated appendicitis, evaluating clinical outcomes in those treated with or without surgery.

Methods: We performed a retrospective review of 158 adult patients (> / = 18 years old) at our institution treated for acute complicated appendicitis from January 2017 to June 2022. Clinicopathologic variables at time of initial presentation, procedure details, and clinical outcome measures at index and subsequent hospital stays were collected and analyzed. We compared patients who were managed non-operatively versus operatively. Regression modeling was used to determine factors associated with non-operative management and those predictive of failure of non-operative management on initial presentation.

Results: Our analysis showed 41% patients with acute complicated appendicitis were managed non-operatively, versus 69% managed operatively at index admission (n = 109). Within the non-operative group, 16.5% patients were treated with antibiotics alone (n = 26) versus 14.6% with percutaneous drainage (n = 23). Mean age of patients in the non-operative cohort was 52.2 years (SD = 18.1) versus 52.1 years (SD = 18.1) in the operative cohort. Mean BMI in the non-operative cohort was 29.1 (SD = 7.9) versus 28.0 (SD = 6.6) in the operative cohort. Age, BMI and comorbidities were similar between both groups. Clinical factors predictive of initial non-operative management were perforation (OR 7.60, 95% CI 3.2–17.9) and phlegmon (OR 4.09, 95% CI 1.7- 10.1) at presentation. Clinical factors predictive of drainage failure requiring surgery on index admission or within 30 days of drainage were larger abscess size (OR 1.82, 95% CI 1.1–3.1) and hypotension on presentation (OR 15, 95% CI 1.2–194.1). Younger age was the only factor associated with increased LOS (OR 0.98, 95% CI 0.97–0.99); advanced age (OR 1.03, 95% CI 1.0–1.1) and Medicare insurance when compared to private/commercial insurance (OR 2.43, 95% CI 1.2–5.2) were associated with increased complication rates for both operative and non-operatively managed patients.

Conclusion: There may be a role in identifying clinical factors in patients with complicated appendicitis that favor non-operative versus operative management. Hypotension at initial presentation and larger abscess size could be predictors of failure of non-operative management. This may warrant further studies with larger sample sizes to delineate management of this patient population.

S053

Risk Factors and Natural History of Bedside Percutaneous Endoscopic Versus Fluoroscopic Guided Gastrostomy Tubes in Intensive Care Unit Patients

Ryan Soheim, MD; Sarah Chung, BS; Lucy Ching Chau, MD; Michael Dix, MD; Miles Bowman, BS; Nadia Obeid, MD; Arielle Hodari Gupta, MD; Cletus Stanton, MD; Henry Ford Hospital

Introduction: There is a paucity of literature comparing patients receiving bedside placed percutaneous endoscopic gastrostomy (PEG) versus fluoroscopic guided percutaneous gastrostomy tubes (G-tube) in an intensive care unit (ICU) setting. This study aims to investigate and compare the natural history and complications associated with PEG versus fluoroscopic G-tube placement in ICU patients.

Methods: All adult patients admitted in the ICU requiring feeding tube placement at our center from 1/1/2017 to 1/1/2022 were identified through retrospective chart review. Patients with at least 6-months follow-up were included in this study. Descriptive statistics were used to illustrate the cohort's natural history. Adjusting for patient comorbidities, hospital factors, and indications for enteral access, a 1-to-2 propensity score matched cox proportional hazards model was fitted evaluate the treatment effect of bedside PEG tube placement versus G-tube placement on patient complications, 6-month readmission, and 6-month death. Major complications were defined as the need for operative or procedural intervention.

Results: This study included 740 patients, with 178 bedside PEG (mean age 59.9 [IQR: 47–68.3] years; 55.9% black race; 63.5% male sex) and 562 fluoroscopic G-tube (62.9 [IQR: 51.1–71.1] years; 42.6% black race; 58.5% male sex) placements. Indication for enteral access was predominantly trauma (23.7%) or respiratory (33.7%) in nature for PEG recipients and neurologic (59.6%) for G-tube recipients. The overall rate of complication was 22.3% (13% PEG, 25.2% G-tube, P = 0.003). The major complication rate was 11.2% (8.5% PEG, 12.1% G-tube, P = 0.09). Most common complications were tube dysfunction (16.7% PEG; 39.4% G-tube; p = 0.04) or dislodgement (58.3% PEG; 40.8% G-tube). The average hospital stay was 30.9 days (IQR: 22.3–45.3) for PEG and 24.7 days (IQR: 17.6–36.9) for G-tube recipients (P < 0.001). 55.9% PEG and 45.7% G-tube recipients were discharged to long-term care whereas 5% PEG and 9.6% G-tube recipients were discharged home (p < 0.001). After propensity score matching, G-tube recipients had significantly increased risk for all-cause (HR: 2.7, 95% C.I.: 1.56–4.87, P < 0.001) and major complications (HR: 2.11, 95% C.I.: 1.05–4.23, P = 0.035). There were no significant differences in 6-month rates of readmission (HR: 0.90, 95% C.I.: 0.58–1.38, P = 0.62) or death (HR: 1.00, 95% C.I.: 0.70–1.44, P = 0.7).

Conclusions: The overall rate of complications for ICU patients requiring feeding tube in our cohort was 22.3%. ICU patients receiving fluoroscopic guided percutaneous gastrostomy tube placement had significantly elevated risk of 6-month all-cause and major complications compared to those undergoing bedside percutaneous endoscopic gastrostomy.

figure k

S054

Risk Factors for Postoperative Abscess Formation in Patients with Upper Gastrointestinal Perforation

Kana Ishikawa; Siyuan Yao; Yugo Matsui; Takashi Kumode; Kesuke Tanino; Ryosuke Mizuno; Syusaku Honma; Shinichi Hosokawa; Teppei Murakami; Takatsugu Kan; Sanae Nakajima; Takehisa Harada; Shigeki Arii; Department of Surgery, Kobe City Medical Center West Hospital, Hyogo, Japan

Background/Purpose: Residual abscess is a common complication of surgical treatment for upper gastrointestinal (UGI) perforation. Postoperative abscess formation prolongs the duration of antibiotic therapy and hospitalization. To date, there have been no previous reports focusing on the risk factors for postoperative abscess formation of surgically treated UGI perforation. This study aims to identify predictive factors of postoperative abscess formation in patients with UGI perforation.

Method: This single-center retrospective study enrolled 75 patients diagnosed with UGI perforation who underwent surgical intervention from 2015 to 2022. Patient characteristics were compared between those with abscesses (n = 22) and those without abscesses (n = 53). Independent risk factors for abscess formation were determined with multivariable logistic regression analysis.

Result: There were 42 men (56%) and 33 women (44%) with a median age of 70 years (range,13–96). The causes of UGI perforations were gastric ulcers (25.3%), duodenal ulcers (65.3%), or malignancies (9.4%). Laparoscopic surgery was performed in 69 patients (92%). Postoperative abscess formation occurred in 22 patients (29.3%). Within the 22 cases, abscess formation with Clavien-Dindo grade II was observed in 7 patients, and 10 patients had grade III or worse. Mortality rate of the abscess group was 2/22 (9%). Patients who developed abscesses were older (72.5 years vs 67 years; P = 0.009), had a higher proportion of current cancer chemotherapy (22.7% vs 5.7%; P = 0.043), had larger perforations (10 mm vs 5 mm; P?0.005), and had higher C-reactive protein levels (6.03 mg/dL vs 2.21 mg/dL; P = 0.017). Multivariable analysis revealed age (OR1.08, 95%CI;1.01–1.16 P = 0.022), current history of cancer chemotherapy (OR9.55, 95%CI;1.12–81.2 P = 0.038), and perforation size (OR1.25, 95%CI;1.05–1.49 P = 0.014) as independent risk factors for abscess formation.

Conclusion: Advanced age, current history of cancer chemotherapy, and large perforation size are predictive risk factors for postoperative abscess formation following surgical treatment of UGI perforation. Patients with these factors require additional care during and after surgical intervention.

S055

The Interplay of Burnout and Professional Quality of Life Scores Among ICU Nurses: A Prospective Survey Study

Yuchen You, DO; Mattie Arseneaux, DO; Javier Romero, MD; Graal Diaz, PhD; Ventura County Medical Center

Background: It is well documented that intensive care nurses have higher burnout rates than other nursing professionals. Nursing Burnouts have shown to have significant negative impacts on patient outcomes.

Objective: To identify and evaluate the factors that cause low professional qualities of life and burnouts for ICU nurses. Additionally, to compare these factors in the setting of a Trauma Center (TC) vs a Non-Trauma Center (NTC).

Methods: This was a prospective self–report survey study. The survey instruments used were a seven-point background questionnaire, the Maslach Burnout Inventory – Human Services Survey (MBI-HSS), and the Professional Quality of Life (ProQOL) scale. MBI measures three aspects of burnout syndrome: emotional exhaustion, depersonalization, and personal achievement. ProQOL measures three constructs: compassion fatigue, burnout, and secondary traumatic stress. Descriptive, univariate, and multivariate analyses were performed using R-analytics.

Results: A total of 26 ICU nurses participated [TC = 18 (69%), NTC = 8 (31%)]; 11.5% had 3–5 years of experience, 23% had 5–10 years, 23% had 11–15 years, and 42% had > 15 years. The majority were females (96%). The mean age was 43.2 (10.2). 72% worked the day shift, and 28% worked night shifts. 91.5% of the participants reported moderate to high burnout rates, and 67.3% reported low qualities of life. To better understand the outcomes of MBI and ProQOL, a correlation matrix was constructed. The components of MBI that are most correlated with MBI are Emotional Exhaustion (EE) (0.92) and Depersonalization (DP) (0.74). Personal achievement only has a -0.15 correlation with MBI. Meanwhile, ProQOL correlates with Secondary Trauma (STS) (0.84). It has much weaker, positive correlations with Compassion Satisfaction (CS) (0.15) and Burnout (BO) (0.32). ProQOL and MBI correlate 0.19. Multivariate analysis for MBI indicates that > 15 years of work experience (p = 0.03), working in a TC (P < 0.001), and night shift (p = 0.05) were significant contributors to burnout. For ProQOL, 10–15 years of experience (P = 0.03) and working in a TC (p = 0.04) significantly contributed to low scores of professional qualities of life. Personal achievements and less than 5-years of work had protective value for MBI.

Conclusion: According to our study, 92% of participants reported burnout and 67% reported low ProQOL. Greater than 15 years of working experience and working in a trauma center setting are factors that significantly contribute to both burnout and poor quality of life. Working nights worsens burnout but does not significantly affect ProQOL. Personal achievements and less than 5 years of working experience are protective against burnout.

S056

Predictive Factors for Operative Intervention in Adhesive Small Bowel Obstruction

Tara E van Veen, MD 1; Purushotham Ramanathan, BS2; Lolita Ramsey, RN, PhD1; Arkadii Sipok, MD, PhD1; Jonathan Dort, MD1; Dina Tabello, DO1; 1Inova Fairfax Medical Campus; 2University of Virginia School of Medicine

Introduction: Small bowel obstruction (SBO) is responsible for 350,000 U.S. hospitalizations and healthcare costs of ~ $2.3 billion annually. The current standard of care for SBO is to trial 3 to 5 days of non-operative management, which is successful up to 82% of the time. It remains a clinical challenge when to change the course towards operative management. Delaying surgery increases morbidity, mortality, length of stay, and cost. This study evaluated if any clinical factors were associated with operative management in adhesive SBO.

Methods and Procedures: This retrospective cohort study included adult patients who were admitted between 2013 to 2020 with an ICD-10 code for SBO and history/risk for adhesion. Exclusions were for operative intervention within 24 h of admission or hospice/death prior to surgery. Descriptive statistics, univariable and multivariable logistic regression were conducted (SPSS: IBM, v26).

Results: 360 out of 623 patients were eligible. At baseline: mean age was 66 years, 57.5% female, 71.3% white, mean BMI 26, 38.7% with history of SBO and 98.1% had history of abdominal surgery. Patients had 1–2 days of symptom onset prior to hospitalization. 55.6% had successful non-operative management at discharge (median length of stay 3 days, symptom resolution 1.7 days) vs. 44.4% failed to improve, requiring surgery.

In a univariate analysis, BMI, history (prior SBO, abdominal surgeries, colorectal surgery, exploratory laparotomy), initial vitals (temperature, blood pressure), abdominal pain, obstipation, acute kidney injury, and small bowel feces sign on CT scan were significantly associated with operative management. In a multivariable logistic regression, after controlling for all other variables, small bowel feces sign (Adjusted Odds Ratio = 0.43, 95% CI 0.2–0.9, p = 0.03; protective against surgery) and history of exploratory laparotomy (Adjusted Odds Ratio = 0.43, 95% CI 0.2–0.9, p = 0.02) were statistically significant factors associated with treatment.

20.4% of surgeries were laparoscopic vs. 79.6% open. Time from admission to surgery averaged 3.89 days: small bowel resection (n = 55) was 4.9 days (median = 4), compared to patients without resection (3.4 days, median = 2; p = 0.00; OR = 1.2, 95% CI 1.07–1.35).

Conclusions: A lack of (i.e. negative) small bowel feces sign can be a potential indicator for operative management and should be further explored. Since the median resolution of symptoms in the non-operative management group was ~ 2 days and a 20% higher odds for bowel resection each day surgery is delayed, the conservative trial period for adhesive SBO should not exceed 3 days after admission.

S057

Virtual Triage from Freestanding Emergency Departments: A Propensity-Score Weighted Analysis of Short-Term Outcomes in Emergency General Surgery Patients

William Lorenz, MD; Maria Baimas-George, MD, MPH; Hongmei Yang, PhD; B. Lauren Paton, MD, FACS; Selwan Barbat, MD; Brent D Matthews, MD, FACS; Caroline E Reinke, MD, FACS; Lynnette Schiffern, MD, FACS; Atrium Health

Background: Freestanding emergency departments (FSEDs) have generated improved hospital metrics including decreased ED wait times and increased patient selection. Patient outcomes and process safety have not been evaluated. This study investigates the safety of FSED virtual triage in the emergency general surgery (EGS) patient population.

Methods and Procedures: A retrospective review evaluated all adult EGS patients presenting to either 1) FSED (fEGS) or 2) community hospital (cEGS) for surgical consultation between January 2016 – December 2021. Prior to disposition from the ED, the fEGS cohort was evaluated virtually without physical evaluation by a surgeon, whereas the cEGS cohort was evaluated in-person. Patient's demographics, acute care utilization history, and clinical characteristics at the onset of the index visit were used to build a propensity score model and stabilized Inverse Probability of Treatment Weights (IPTW) were used to create a weighted sample. Multivariable regression models were then employed to the weighted sample to evaluate the treatment effect of virtual triage compared to in-person evaluation on short-term outcomes including length of stay (LOS), readmission, and 30-day mortality. Variables which occurred during the index visit (such as surgery duration and type of surgery) were adjusted for in the multivariable analyses.

Results: Of 1962 patients, 631 (32.2%) were evaluated virtually (fEGS), and 1331 (67.8%) underwent an in-person evaluation (cEGS). Baseline characteristics demonstrated significant differences between the cohorts in gender, race, payer status, BMI and CCI score. Baseline risks were well-balanced in the IPTW weighted sample (standard differences ranged from 0.002–0.18). Multivariable analysis found no significant differences between the balanced cohorts in 30-day readmission, 30-day mortality, and LOS (p < 0.05 for all).

Conclusions: Patients who present to FSEDs have similar outcomes to those who undergo in-person triage for EGS diagnoses. Virtual triage at FSED for these EGS patients may be an efficient and safe means for initial evaluation.

S058

Perforated Peptic Ulcer Disease in Transferred Patients is Associated with Significant Increase in Length of Stay: A Single Institution Retrospective Study

Rachel Schneider, MPH; Rachel C Lister, BS, BA; James E Carroll, MD, FACS; University of Massachusetts Chan Medical School

Background: Perforated peptic ulcer disease (PPUD) has a prevalence of 0.004- 0.014% with an associated mortality of 23.5%0.1 In this single center study, we examined the impact upon outcomes associated with patient transfer from outside facilities to our center for definitive surgical intervention (exploratory laparotomy).

Methods: Using EPIC report workbench and SlicerDicer, we identified 27 patients between 2018 and 2021 undergoing exploratory laparotomy with a concurrent diagnosis of peptic ulcer disease. We queried this population for markers of disease severity including mortality, length of stay, intensive care unit (ICU) length of stay, surgical case classification, surgical complications, and readmission rates. Manual chart reviews were performed to examine these outcomes in more detail and identify patients who had been transferred to our facility for surgery from an outside hospital.

Results: A total of 27 patients were identified undergoing exploratory laparotomy for definitive treatment of PPUD. The majority of patients queried underwent level A operations, the most urgent level of activation. Average mortality for this patient population was 14.8%. The readmission rate was 40.1%, and average length of ICU stay post-operatively was 16 days, with 83% of non-transfer patients requiring ICU admission and 100% of transfer patients requiring ICU admission. Average length of hospital stay (LOS) was 27 days overall. For non-transfer patients and transfer patients, LOS was 20 days and 41 days respectively, which was statistically significant by one-sided t-test (p = 0.05). Two intra-operative complications identified were attributed to transfer patients.

Conclusion: Patients transferred for definitive care of PPUD in a population otherwise notable for high mortality and high readmission rates: their average length of stay compared to non-transfer patients was over twice the length, which was statistically significant. Transferred patients also had higher rates of ICU care requirement. Further inquiry to identify modifiable variables to facilitate the care of transferred patients is warranted, especially in the context of improving quality metrics known to enhance patient outcomes, satisfaction, and value.

References

Tarasconi, Antonio et al. "Perforated and Bleeding Peptic Ulcer: WSES Guidelines." World Journal of Emergency Surgery: WJES, BioMed Central, https://www.ncbi.nlm.nih.gov/pmc/articles/PMC6947898/, 7 Jan. 2020, 23 Sep. 2022.

S059

The Role of Intra-peritoneal Drains for the Prevention of Post Operative Intra-abdominal Abscesses in Adult Patients Undergoing Laparoscopic Appendectomies

Danny Poon, MD 1; Leor Alkadaa2; Panagiotis Volteas, MD1; Kevin Duh, MD1; Aurora Pryor, MD3; Samer Sbayi, MD1; Konstantinos Spaniolas, MD3; 1Stony Brook University Hospital, Department of General Surgery; 2Renaissance School of Medicine at Stony Brook University; 3Stony Brook University Hospital, Division of Bariatric, Foregut, and Advanced Gastrointestinal Surgery

Introduction: We aimed to assess the efficacy of intra-abdominal drainage placed at the time of laparoscopic appendectomy in preventing the development of post operative intra-abdominal abscesses in adult patients with acute appendicitis. The gold standard for the treatment of acute appendicitis is removal via laparoscopic surgery. While advantages of this minimally invasive procedure include reduced post operative pain and shorter hospital stays, disadvantages include complications such as the development of intra-abdominal abscesses. Strategies often used to prevent the formation of these abscesses include placement of drains that are removed at the surgeon's discretion. To date, there are no current meta-analyses on the use of intra-peritoneal drainage to help prevent the formation of intra-abdominal abscesses in adult patients undergoing laparoscopic appendectomies for acute appendicitis.

Methods and Procedures: Systematic searches were conducted in PubMed/Medline, Scopus and Cochrane Central. We included all studies comparing abdominal drainage versus no drainage in patients undergoing emergent laparoscopic appendectomy for acute complicated appendicitis. This study was performed according to the PRISMA (Preferred Reporting Items for Systematic Reviews and Meta-Analyses) guidelines.

Results: We identified 9 retrospective observational studies and no randomized controlled trials, enrolling a total of 3941 patients. We found that there was no difference in the risk of development of post operative intra-abdominal abscesses between patients with an intra-abdominal drain compared to patients with no drain at time of the procedure (OR 0.74, 95% CI 0.31 to 1.80). However, patients with intra-abdominal drain placement at the time of the procedure had an increased overall risk for post operative wound infections (OR 1.89, 95% CI 1.27 to 2.81).

Conclusion: This study suggests that the placement of intra-abdominal drains at the time of emergent laparoscopic appendectomy for acute appendicitis does not prevent the development of post operative intra-abdominal abscesses but rather increases the overall risk of wound infections. The quality of the best available evidence is moderate; therefore, future high-quality randomized controlled trials or prospective cohorts are needed to validate our results.

S060

Predictors of Complicated Gallstone Disease During the COVID-19 Pandemic

Ryan B. Cohen, MD; Moshumi Godbole, MD; Candace L. Ward, MD; Leonardo Kozian, MD; Sakib Adnan, MD; Afshin Parsikia, MD; Alexi Bloom, MD; Benjamin Moran, MD; Mark Kaplan, MD; Pak Leung, MD, MBA; Einstein Healthcare Network

Introduction: The COVID-19 pandemic introduced unforeseen challenges to surgical healthcare delivery. We hypothesized that routine surgical cases, such as laparoscopic cholecystectomy (LC), may have been prone to more complicated presentations due to unforeseen effects of the pandemic. The purpose of our study was to analyze the predictors of complicated gallstone disease during the COVID-19 pandemic.

Methods: A retrospective chart review from July 2019 to December 2020 identified all non-elective LC cases performed at a level-1 trauma center. This included patients from a 6-month pre-pandemic period (7/2019–12/2019). Patients with complicated gallstone disease were compared to those without complicated gallstone disease in terms of baseline factors, in-room time (combined anesthesia and operating room time), complications, and outcomes. We then used multivariate regression to identify predictors of complicated gallstone disease in both pre-pandemic and intra-pandemic periods.

Results: In total, 201 patients were reviewed with 83 in the pre-pandemic period and 118 patients in the intra-pandemic period. Compared to patients without complicated gallstone disease, patients with complicated gallstone disease were older, predominantly male, had a higher mean American Society of Anesthesiologists (ASA) classification, more likely presented during the pandemic, had more complications, longer length-of-stay, and longer in-room time (P < 0.05). On multivariate regression, prolonged in-room time was the strongest predictor of complicated gallstone disease in the intra-pandemic period (P < 0.05). We could not identify any significant predictors for complicated gallstone disease in the pre-pandemic period.

Conclusions: As hypothesized, complicated gallstone disease may have been more prevalent throughout the COVID-19 pandemic. In-room time was a significant predictor of complicated gallstone disease in the intra-pandemic period but not in the pre-pandemic period. We suggest that delayed presentations, staffing shortages, and prolonged intubation times, may have contributed to longer in-room times resulting in more complicated gallstone disease during the COVID-19 pandemic.

S061

Creation of a Side-to-Side Magnetic Compression Anastomosis to Achieve Duodeno-Ileostomy Diversion in Adults with Severe Obesity with or without Type-2 Diabetes

Michel Gagner, MD, FRCSC, FACS 1; Guy-Bernard Cadiere, MD2; Andres Sanchez-Pernaute, MD3; David Abuladze, MD4; Lamees Almutlaq, MD, FRSCSC1; Antonio J Torres, MD, FACS3; 1Westmount Square Surgical Center, Westmount, QC, Canada; 2CHU St-Pierre, Brussels, Belgium; 3Hospital Clinico San Carlos,Madrid, Spain; 4Innova Medical Center, Tbilisi, Georgia

Introduction: Classical gastrointestinal anastomoses have been made with sutures and/or metal staples, but have resulted in significant bleeding and leak rates. The study aim was to evaluate the safety and effectiveness of a compression anastomosis using magnets to achieve weight loss and remission of type 2 diabetes (T2D).

Methods: Patients with severe obesity (body mass index [BMI] ≥ 35 kg/m2 with or without T2D [HbA1C > 6.5%]) underwent duodeno-ileostomy (DI) diversion. A linear magnet was delivered by flexible endoscopic catheter to a point 250 cm proximal to the ileocecal valve, and a second magnet was positioned in the first part of the duodenum; the bowel segments containing the magnets were apposed to initiate gradual incisionless compression. Laparoscopic assistance was used to obtain accurate bowel measurements, obviate tissue interposition, and close mesenteric defects.

Results: Ongoing from November 18, 2021 in four centers, 27 patients were enrolled and underwent magnetic compression DI. Respective mean weight (kg) and BMI (kg/m2) at baseline were 117.6 ± 15.8 and 44.4 ± 4.9; at 6 months, 85.8 ± 5.9 and 31.9 ± 3.3. Total weight loss of 31.8 ± 9.5 and BMI reduction of 12.5 ± 3.3 corresponded to 62.0% excess weight loss (range 55.0–77.0%). Mean HbA1C (%) dropped from 6.82 ± 1.76 to 4.74 + 0.61 at 6 months; glucose (mg/dL), from 134.3 ± 40.0 to 85.64 ± 12.37 (mean reduction, 48.7 ± 44.7). No anastomotic bleeding or leakage, and no bowel obstruction were observed.

Conclusion: Creation of a side-to-side magnetic compression anastomosis to achieve duodeno-ileostomy diversion in adults with severe obesity appeared to be safe, achieved dramatic weight loss, and ameliorated T2D in the short term.

S062

Perioperative outcomes of laparoscopic low anterior resection using ArtiSential® versus Robotic surgery in patients with rectal cancer

Chul Seung Lee, PhD 1; 1Seoul St. Mary’s Hospital

Background: Even for experienced surgeons, total mesorectal excision (TME) for low anterior resection (LAR) might be technically difficult. Multi-joint mechanics and ergonomics are benefits of robotic systems, but they are costly. As an alternative to robotic systems, several laparoscopic joint tools have been introduced. The objective of this study was to compare perioperative outcomes between laparoscopic low anterior resection(LAR) using ArtiSential® and Robot assisted LAR for rectal cancer.

Methods: Medical records of patients who underwent minimally invasive surgery (robotic or laparoscopic) for rectal cancer at Seoul St. Mary's Hospital, The Catholic University of Korea from September 2018 to December 2021 were reviewed. We excluded patients who underwent concomitant surgeries and emergency surgery and conversion to an open approach. Of 682 LAR cases for rectal cancer, 82 patients underwent laparoscopic low anterior resection using ArtiSential® (group 1) and 201 patients underwent Robot assisted LAR. The daVinci Xi Surgical System (Intuitive Surgical, Sunnyvale, CA, USA) was used for all robot-assisted procedures.

Results: Mean ages of patients in groups 1 and 2 were 67.4 ± 12.0 years and 60.9 ± 11.8 years (P = 0.001), respectively. Mean BMI in group 1 and 2 were 23.9 ± 3.5 kg/m2 and 24.0 ± 3.6 kg/m2 (P = 0.790), respectively. With regard to perioperative outcomes, postoperative hospital stay was not significantly different between group I (6.1 ± 4.5 days) and group II (6.4 ± 4.8 days) (P = 0.702). However, the operative time was significantly shorter in group I (166.9 ± 70.7 min) than in group II (255.1 ± 84.5 min) (P < 0.001). Postoperative complications, re-operation and re-admission within 30 days were similar between the two groups. There were no postoperative deaths in all groups.

Conclusion: Compared to robotic LAR, LAR with ArtiSential® reduced the length of the procedure without increasing complications or readmission rates. ArtiSential® may serve the feasible alternative approach for rectal cancer.

S063

A Novel 4-in-1 Medical Device for the Treatment of Esophageal Atresia

Heather Wanczyk, MS1; Joanne Walker, MS1; Liisa Kuhn, PhD1; Christine Finck, MD, FACS 2; 1University of Connecticut Health Center; 2Connecticut Children’s

Introduction: Esophageal atresia is a congenital defect where the proximal and distal ends of the esophagus fail to connect [1, 2]. Surgical repair is complicated by postoperative anastomotic strictures, which occurs in 32–59% of cases [3]. Currently, serial balloon dilations are required for treatment, which carries the risk of repetitive anesthetic usage in neonates. To address this, we developed a 4-in-1 medical device that contains conduits for feeding, drug delivery and balloon dilation (Fig. 1).

figure l

This all-in-one device is a feeding tube and balloon dilator, which can dilate strictures as needed. Contrast in the balloon identifies the location at the anastomotic site. Additionally, agents such as mitomycin C, which treat strictures [4], can be delivered through a port at the anastomotic site. Finally, an aspiration port is present to monitor the healing environment of the esophagus. We sought to determine feasibility of the device in a cadaveric and in vivo animal model of esophageal resection.

Methods: Functionality of the device was tested in cadaveric rabbits (n = 2) evaluating length and dimensions of the device and balloon functionality. In vivo feasibility testing of the device was performed in a rabbit esophageal resection model to assess tolerability, ease of insertion, balloon dilation, and the ability to continuously deliver feeds (IACUC AP-200543–0824; n = 1). One male New Zealand rabbit (3.2 kg) underwent a thoracotomy with esophageal transection and re-anastomosed mimicking esophageal repair. A pharyngostomy was performed and the device was inserted and secured. Prior to chest closure, the balloon was inflated with saline to ensure appropriate position and function. During the post-op period, the rabbit received 20 mL of Ensure via the feeding tube three times a day for a total of three days.

Results: The cadaveric and in vivo studies showed that the tube was easily inserted and that both balloon and drug delivery ports worked properly. For the in vivo study, the rabbit recovered and received feeds via the device for three days. The device was well tolerated and at post-mortem analysis, there was no inflammation of tissue at the site of placement or within the stomach.

Conclusion: This preliminary study demonstrates both the functionality and tolerability of a novel 4-in-1 device. Next steps will be to test ability of the device to reduce stricture formation in a large animal model. The reduction in repeat interventions and anesthesia exposure afforded by this device will positively affect neonatal development and reduce medical costs.

S064

Kinematic Data Profile and Clinical Outcomes in Robotic Inguinal Hernia Repairs

Sarah Choksi, MD, MPH 1; Daniel P Bitner, MD1; Katie Carsky, MD1; Poppy Addison, MD1; Rachel Webman, MD2; Robert Andrews, MD2; Rebecca Kowalski, MD2; Shane Dawson, MD2; Valery Dronsky, MD2; Andrew Yee, PhD3; Anthony Jarc, PhD3; Filippo Filicori, MD1; 1Lenox Hill Hospital, Northwell Health, Department of Surgery, Intraoperative Performance Analytics Laboratory; 2Lenox Hill Hospital, Northwell Health, Department of Surgery; 3Intuitive Surgical

Background: The Intuitive Data Recorder (IDR), (Sunnyvale, CA) allows for determination of technical skills using Objective Performance Indicators (OPIs) from kinematic and event data. Our goal was to determine whether OPIs change with increasing surgeon experience and whether they are correlated with outcomes of Robotic Inguinal Hernia Repair (RIHR).

Methods: Twenty-seven RIHRs from 7 different surgeons were recorded with the IDR from February to August 2022. Year-to-date caseload ranged from 3–88 (Median: 32) while lifetime RIHR caseload ranged from 28 to 194 (Median: 134). Patients were called on postoperative days 5–7 and administered the Carolina Comfort Scale (CCS) survey to evaluate acute clinical outcomes. A Pearson test was run to determine correlations between OPIs from the IDR with a surgeon's yearly and overall RIHR experience. A Pearson test was also performed between OPIs and CCS scores to evaluate correlations with clinical outcomes.

Results: Multiple OPIs were correlated with surgeon experience. For the task of peritoneal flap exploration, 38 OPIs were significantly correlated with surgeons' experience. Figure 1 shows the wrist articulation movements of the robotic instrument, called roll, pitch and yaw. Roll of arm 2 had a correlation of -0.508 (95% CI -0.737,-0.174) with RIHR yearly caseload, and -0.422 (95% CI -0.683,-0.066) with lifetime RIHR caseload. Pitch of arm 2 had a correlation of -0.457 (95% CI -0.706,-0.109) with yearly RIHRs and -0.419 (95% CI -0.681,-0.062) with lifetime RIHRs. Yaw of arm 2 had a correlation of respectively -0.467 (95% CI -0.712,-0.121) and -0.429 (95% CI -0.687,-0.0774). Roll and pitch of arm 4 were also correlated with yearly RIHRs with a correlation of -0.420 (95% CI -0.682, -0.064) and -0.414 (95% CI -0.677, -0.056), respectively. Figure 2 shows correlations of surgeon experience with different OPIs for the task of peritoneal flap exploration. For clinical outcomes, the CCS sensation of mesh score correlated with 5 different OPIs during the creation of peritoneal incision task. Wrist articulation of the surgeon's console positively correlated with acute sensation scores from the CCS with a correlation of 0.453 (95% CI 0.013, 0.746).

Conclusions: Multiple OPIs correlate with surgeon experience and surgical outcomes. More experienced surgeons appear to articulate surgical instruments less during peritoneal flap exploration. Using this knowledge, surgical simulation platforms can be designed to teach patterns to surgical trainees that are associated with increased surgical experience and improved postoperative outcomes.

figure m
figure n

S065

A Novel High Accuracy Model for Automatic Surgical Workflow Recognition Using Artificial Intelligence in Laparoscopic Totally Extraperitoneal Inguinal Hernia Repair

Monica Ortenzi, PhD 1; Alenka Antolin1; Omri Bar1; Maya Zohar1; Dotan Asselmann1; Tamir Wolf1; 1Theator Company

Introduction: Artificial intelligence and computer vision are revolutionizing the way to perceive video analysis in laparoscopic surgery. This emerging technology has been increasingly and successfully employed for video segmentation, documentation, education, and formative assessment. Video analysis per se is a tool allowing the surgeons to gain valuable insights into their surgical practice, however, since full video review might be time consuming, we also created a completely new, easily accessible platform that can be used to navigate in the uploaded videos, extrapolating single specific pre-labeled fragments of their choice without the need to visualize the entire videos. The aim of this study is to show the accuracy of a novel computer vision algorithm for automated surgical steps detection in Totally Extraperitoneal (TEP) Inguinal Hernia Repairs.

Methods: Videos of TEP procedures were manually annotated by a team of experts, expressly trained to annotate the surgical workflow, according to the following major steps: Balloon dissection, access to the preperitoneal space, dissection, Hernia Sac reduction, Mesh placement and Mesh fixation. For bilateral hernias an additional step named Changed of focus was also considered to identify the switch of the side during the operation. Then, the videos were used to train a computer vision AI algorithm. Performance accuracy, defined as the accuracy of the AI algorithm to autonomously recognize the different steps was assessed by comparison to the human annotations.

Results: A total of 536 full-length TEP videos were analyzed: 322 were used to train the model, 80 for internal validation, and the remaining 134 as a test set, to evaluate the algorithm accuracy. The overall accuracy for the complete AI model was 89%. Per-step accuracy reached the highest value for the hernia sac reduction step (94.3%) and the lowest level for the preperitoneal dissection step (73%).

Conclusions: These results showed how the new AI model proved to be able to provide a fully automated video analysis with a high accuracy level. The model is fully exploitable for surgical training, surgeon self-assessment, performance review and quality improvement. As machine learning continues to rapidly advance, its next application will evolve from retrospective analysis to online or real-time analysis, with the final purpose to even augment surgeons' intra-operative decisions with big data. The same algorithm is also being used to reliably locate critical events, translating it to specifically clinical purposes, and so opening the path to a completely new asset of future possibilities to exploit.

S066

Surgical Approach to Core Abdominal Injuries, a Retrospective Cohort Study through the Abdominal Core Health Quality Collaborative (ACHQC)

Ryan C Ellis, MD 1; Benjamin T Miller, MD1; Molly A Olson, MS2; Sara M Maskal, MD1; Lucas Beffa, MD1; Clayton C Petro, MD1; Ajita S Prabhu, MD1; Michael J Rosen, MD1; David M Krpata, MD1; 1Cleveland Clinic Foundation; 2Weill Cornell Medicine

Introduction: Core abdominal injury (CAI) is a broad term that describes injuries causing chronic groin pain in active adults, and can be isolated or associated with inguinal hernias. CAI is captured as a variable in the ACHQC, and we aimed to describe how high volume hernia surgeons within the QC address this problem surgically and the short term outcomes.

Methods: We queried the ACHQC registry for patients undergoing surgery for CAI, inguinal hernias (IH), or both. Operative approach and quality of life (using EuraHS survey scores) was abstracted from patients identified as having a CAI.

Results: 29,451 patients underwent surgery for IHs, CAIs, or both within the registry. Twenty patients underwent surgery for isolated CAI (median age 29, 90% males, median BMI 26.9 kg/m2). Fifteen underwent unilateral operations; five underwent bilateral operations. Eleven patients (55%) underwent surgery with mesh placement (4 Lichtenstein, 3 TAPP, and 4 TEP). Nine patients (45%) underwent tissue-based repairs (4 closure of floor, 1 Bassini, 1 McVay, 1 Shouldice, 1 femoral orifice exploration, and 1 laparoscopic-to-open conversion). There were no postoperative complications, reoperations, or recurrences within 30 days. EuraHS scores showed improvement at 30 days from baseline (median 29 [6.75–41.75] from 42 [29.42–57.61]).

Sixty additional patients had both IHs and CAIs (median age 31, 97% males, median BMI 26.2 kg/m2). All approaches consisted of mesh-based hernia repair (38 unilateral, 20 bilateral, 2 bilateral repair with unilateral mesh). Thirty-one patients (52%) underwent open surgery (22 Lichtenstein, 1 Shouldice, 1 closure of floor, 1 plug, 7 TREPP) and 29 underwent minimally invasive repairs (23 TAPP, 6 TEP). There was one seroma at 30 days. EuraHS scores showed improvement at 30 days from baseline (median 16 [5.17–27.33] from 37.5 [26.44–46.58]).

Conclusions: Despite technical variability, CAIs with or without concomitant IH generally undergo operations commonly used for IH repairs. Within our series, there was inconsistency with approach and mesh placement. Future work should be focused on standardizing the approach to CAI and capturing long-term data within the ACHQC.

figure o

S067

Developing of AI-powered Skill Evaluation System for Endoscopic Surgery

Shin Takenaka 1; Kyoko Ryu1; Atsuki Tanaka1; Yumi Kinebuchi1; Yuki Furusawa1; Nobuyoshi Takeshita1; Masaaki Ito1; 1National Cancer Center Hospital East Japan

Background: Surgical skill evaluation around the world is a form of subjective evaluation by experts using a scoring chart. A lot of labor is required, and there is a variance among evaluators. This is because these processes are performed by humans. To solve this problem, we are building an "objective automatic skill evaluation system" based on artificial intelligence (AI).

Methods: Our research is divided into three steps. In step one, we searched for evaluation tools in medical literature and then determined skill assessment categories. Using these categories, we reconstructed the assessment criteria of the Japanese Society of Endoscopic Surgery (JSES). Then, in step two, we determined the skill parameters for evaluating skillfulness by watching surgical videos and then created the AI models for image recognition. We set up three cohorts. High-skill group is + 2SD grade on JSES certification exam, Intermediate-skill group is -2SD, and Low-skill group is novice surgery. We evaluated surgical videos from each cohort and quantified surgeons' performance by using AI image recognition models. Parameters that showed significant differences between cohorts were adopted as parameters of the system.

In step three, the five years of JSES certification scoring results are reconstructed in skill assessment categories. Then AI parameters of the five years of JSES certification videos are measured. Using them, a skill evaluation system is constructed.

figure p

Results: We extracted nine skill evaluation tools and defined five categories of skill assessment using qualitative research methods; Tissue handling, Psychomotor skill, Dissection quality, Exposure quality, and Efficiency. We developed fifteen AI models to evaluate surgical skills. We could calculate the following skill parameters; Bleeding errors, Invalid grasping numbers, dissection by activating electrode (DAE), Retraction quality, Surgical phase time, Surgical phase transition, etc. Some specific examples are shown below. The accuracy of the image recognition model for DAE was 91%. We found that the density of the DAE time of the high-skill and intermediate-skill groups were higher than that of the low-skill groups. It can be said that they efficiently dissected the tissue. The accuracy of a surgical phase recognition model was 89%. We found that the high-skill group took significantly less time for specific processes than the intermediate and low-skill group and that the number of phase transitions was significantly less than the intermediate and low-skill group. In all fifteen of our models, we found significant differences in scores among the three cohorts. We are currently working on step three.

figure q

S068

Global Assessment of Surgical Skills (GASS): Validation of a New Instrument to Measure Global Technical Safety in Surgical Procedures

Peter Nau, MD 1; Erin Worden, MD1; Ryan Lehmann, MD1; Kyle Kleppe, MD2; Gregory J Mancini, MD2; Matt L Mancini, MD2; Bruce Ramshaw, MD3; 1Department of Surgery, Section of Bariatric Surgery, University of Iowa Hospitals & Clinics; 2Department of Surgery, Section of Foregut Surgery, University of Tennessee—Knoxville; 3CQInsights PBC, Knoxville, TN and Medical Advisor, Caresyntax Corp

Introduction: Current video-based assessments (VBA) of surgery, such as the global objective assessment of laparoscopic skills (GOALS) assessment, focus on determining level of skill — as opposed to safety — in generic domains such as instrument handling, efficiency of motion, and respect for tissue. Our goal was to develop and validate a tool to document safe completion of generic technical surgical activities to support awarding individual autonomy based upon competence, consistent entrustable professional activities (EPAs) goals.

Methods and Procedures: Five board-certified, minimally-invasive surgery (MIS) surgeons developed real-world, experientially-driven general assessment of surgical skill (GASS) tool using binomial safe vs. unsafe constructs to document an individual's safe completion of general surgical technical activities. Items listed in Table 1 were assessed on a three-point scale: 1 = Poor (unsafe); 2 = Adequate (safe); 3 = Good (safe). The jejunojejunostomy (JJ) portion of 30 consecutive gastric bypass procedure videos was collected from surgeons at a single, university-based bariatrics program between Aug. 2021 and Jan. 2022 using a proprietary SaaS-based software platform used for automated video capture and VBA to improve surgical quality and safety. Videos were de-identified and randomized. Four board-certified bariatric surgeons from a different university and uninvolved in GASS design, having a mean practice duration of 17.8 years, independently completed a blinded review of each video using GASS. The primary analysis was percent agreement among all raters (inter-rater reliability) based upon percent of tasks assessed as safe. Percentage agreement was calculated according to whether the generic activity was scored safe or unsafe (safe = GASS score ≥ 2).

Results: There were a total of 120 video reviews for analysis (reviewed by all reviewers).

Table 1 Items and percent agreement on safe completion, by item, in the GASS validation study.

figure r

Concordance between the four raters in concluding the surgical technical activities were safe was high, with a low of 73% (economy of motion), high of 100% (achievement of hemostasis), and an average of 90%.

Conclusions: GASS had a high percentage of agreement regarding safety among raters, indicating the GASS tool a valid measure of a surgeon's safe conduct in five generic technical activities and may be used to help determine surgeon autonomy for generic skills, providing a basis for granting autonomy.

S069

A Multi-national video-Based Qualitative Study to Refine Training Guidelines for Assigning an “Unsafe” Score in Laparoscopic Cholecystectomy Critical View of Safety

Gina Adrales, MD, MPH 1; Francesco Ardito, MD, PhD, FACS2; Pradeep Chowbey, MD, FACS, MNAMS, FRCS, FIMSA, FAIS3; Salvador Morales-Conde, MD4; Alberto Ferreres, MD, PhD, JD, MPH5; Chrys Hensman, FRCS, MS6; David Martin, MD7; Hanno Matthaei, MD8; Bruce Ramshaw, MD9; Keith J Roberts, PhD, FRCS10; Harald Schrem, MD11; Anil Sharma, FRCS, MD12; Stephen Tabiri, MD, PhD, MEd13; Eric Vibert, MD14; Michael S Woods, MD, MMM15; 1Chief of Division of Minimally Invasive Surgery, Director of Minimally Invasive Surgical Training and Innovation Center (MISTIC), Johns Hopkins Hospital; 2Hepatobiliary Surgery Unit, Fondazione Policlinico Universitario Agostino Gemelli, IRCCS, Catholic University, Rome, Italy; 3Institute of Laparoscopic, Endoscopic and Bariatric Surgery, Max Super Specialty Hospital, Saket, New Delhi, India; 4Unit of Innovation in Minimally Invasive Surgery, University Hospital Virgen del Rocío. University of Sevilla (Spain); 5Department of Surgery, University of Buenos Aires; 6Monash University Department of Surgery& LapSurgery Australia; 7University of Minnesota; 8Department of Surgery, University Medical Center Bonn, Bonn, Germany; 9CQInsights PBC, Knoxville, TN and Medical Advisor, Caresyntax Corp.; 10Liver Transplant and HPB Surgery, University Hospitals Birmingham NHS Trust; 11General, Visceral and Transplant Surgery, Medical University Graz, Austria; 12Max Institute of Laparoscopic, Endoscopic & Bariatric Surgery, Max Healthcare, Saket, New Delhi; 13Consultant General Surgeon, University for Development Studies-School of Medicine and Health Sciences, Tamale Teaching Hospital; 14Centre Hépato-Biliaire, Paul Brousse Hospital, AP-HP, Villejuif, France; 15Caresyntax, Corp

Introduction: The critical view of safety in laparoscopic cholecystectomy (LC-CVS) is key to avoiding severe injuries to the biliary tree and is a critical element of training. Our goal was to identify variability in experts' assessment of safe vs. unsafe tasks in LC-CVS, promoting standardization of training in the use of an objective procedure-specific assessment (OPSA).

Methods: The LC-CVS OPSA (Table 1) measures surgical technical safety of six tasks. Each item can be scored "poor (unsafe)", "adequate (safe)", or "excellent (expert-level performance"). Ten expert biliary tract surgeons evaluated ten de-identified LC operative videos, from prior to dissection of the Triangle of Calot until the division of the cystic artery and duct, scoring each task and, if rated "unsafe", reported the reason. Authors conducted qualitative analyses of variability in ratings and reasons for "unsafe" ratings with the goal of improving training in the tool's use.

Table 1 LC-CVS OPSA Measurement Items.

figure s

Results: Qualitative review of expert comments identified common topics. The more specific the task's rating criteria the less variability in assessment. Some raters assessed both immediate risk of unsafe technique and potential risk of future problems associated with safe but suboptimal technique. Poor ratings were not always based on visualizing unsafe techniques but rather on the individual's skill level in completing the task. Further, LC-CVS can be achieved without simultaneous visualization of the cystic artery and duct; each structure can be safely identified independently of the other. Finally, medialization of the cystic artery is a safe practice consistent with LC-CVS.

Conclusions: Expert biliary tract surgeons applied different criteria to assess safe surgical techniques enabling the creation of improved training guidance to increase the accuracy of assessment when using this "safe" or "unsafe" construct.

S070

CUSUM Learning Curves: What They Can and Can’t Tell Us

Feibi Zheng, MD, MBA, FACS; Penglin Lin, MS; Minkyung Shin, MA, MPhil; Xi Liu, PhD; Daniel Oh, MD, FACS; Daniel Attilio, MA; Intuitive Surgical

Introduction: With the proliferation of new minimally invasive surgical techniques and technologies, there has been increased interest in assessing the surgeon learning curve for new skill acquisition. While there is no consensus around the best methodology, one of the most frequently used learning curve assessments in the surgical literature is the cumulative sum curve (CUSUM) of operative time. We assessed CUSUM console time across cohorts of surgeons with differing case acquisition rates, while varying the total number of cases used to calculate the CUSUM, and demonstrate the limitations of this methodology.

Methods: We compared the CUSUM curves of the average console times of surgeons who completed their first 20 robotic assisted (RAS) cases in 13, 26, 39, and 52 weeks, respectively for their first 50 cases and again for their first 100 cases. This analysis was performed for prostatectomy (1094 surgeons), malignant hysterectomy (737 surgeons), and inguinal hernia (1486 surgeons). We compared the curves across cohorts in each procedure type using the Kolmogorov–Smirnov test.

Results: In all procedures, the CUSUM curve of the cohort of surgeons who completed their first 20 procedures in 13 weeks demonstrated a lower slope and different curve shape than cohorts of surgeons with slower case acquisition rates. The case number at which the peak of the CUSUM curve occurs uniformly increases when the total number of cases used in generation of the CUSUM chart changes from 50 to 100 cases.

Conclusion: The CUSUM analyses of these three procedures suggests that surgeons with fast initial case acquisition rates have less variability in their operative times over the course of their learning curve. The peak of the CUSUM curve, which is often used in surgical learning curve literature to denote "proficiency" is predictably influenced by the total number of procedures evaluated, suggesting that defining the peak as the point at which a surgeon has overcome the learning curve is subject to routine bias. The CUSUM peak, by itself, is an insufficient measure of "conquering the learning curve."

figure t
figure u
figure v
figure w
figure x
figure y
figure z
figure aa

S071

SAGES Safe Chole Program Changes Surgeons Practice in France: Results of the FCVD Implementation of Safe Chole in France

JM Fabre 1; F Borie1; JF Gravie1; A Deleuze1; C Sabbagh1; E Magne1; J Gugenheim1; P Fuchshuber2; H Asbun2; 1FCVD; 2SAGES

Background: The Federation of Visceral and Digestive Surgery (FCVD) is tasked by the French government to certify surgeons for continuing medical education (CME). These CME programs are tailored using voluntarily reported adverse events by surgeons which are recorded into a FCVD maintained database (REX).

Based on REX data prevention of bile duct injury (BDI) during cholecystectomies has become a priority for the FCVD. In 2020 and in collaboration with SAGES, the FCVD has implemented the SAFE CHOLE modules as part of the mandatory CME curriculum for every participating surgeon. Each surgeon completing the SAFE CHOLE CME is certified by the FCVD.

Methods: The purpose of this study is to evaluate success rates, participant satisfaction and the practice impact of the SAFE CHOLE program for participating surgeons. To get CME certification each participant must fill out a FCVD validated double questionnaire. Demographics, number of cholecystectomies performed per year, percentage of outpatient and emergency surgery, and frequency of BDI were obtained. Data addressing the impact on the surgeon's practices was also gathered, including preoperative evaluation, dissection technique, cholangiography use, as well as strategy facing difficult cholecystectomy and bile duct injury.

Results: By the end of 2021 290 surgeons had completed the program. The satisfaction rate was 89%. Eighty percent declared having acquired new knowledge on cholecystectomies. Fifty-two percent changed their practice. Of those that changed practice 69% systematically adopted the critical view of safety and 59% used a time-out prior transection of vital structures. While 59% were already practicing systematic cholangiography, 11% introduced systematic cholangiography into their practice. In case a critical view of safety was not obtainable, 61% report performing a sub-total cholecystectomy. If faced with BDI, 51% would transfer to higher level of care, 27% would seek help from a colleague, and 9% would proceed with a repair. Ninety-two percent recommend adoption of SAFE CHOLE program by all digestive surgeons and 86% reported improvement of patient safety.

Conclusions: Implementation of the SAFE CHOLE program in large scale is feasible and appears to have a significant impact in the practice of most participating surgeons. This may translate in enhanced patient safety with the goal of decreasing the incidence of BDI. By the end of 2022 over 600 surgeons will have completed the program. This data should serve to encourage other surgeons and health systems to engage in this program.

S073

Trends and Disparities in Access to Minimally Invasive Distal Pancreatectomy (MIDP): An Eight-Year Analysis from the National Cancer Database

Faisal S Jehan, MD, FICS 1; Asad Azim, MD2; Gregory Veillette, MD, FACS1; Muhammad Khan, MD1; Hassan Aziz3; 1Westchester Medical Center; 2Fox Chase Cancer Center; 3University of Iowa Hospitals and Clinics

Introduction: Laparoscopic and robotic approaches to distal pancreatectomy are becoming the standard of care. The aim of our study was to evaluate the trends of utilization and disparities in access to minimally invasive approaches in distal pancreatectomy.

Methods: We queried the National Cancer Database (NCDB) and analyzed all the patients who underwent distal pancreatectomy from 2010 to 2017. Patients were divided into groups those who had open distal pancreatectomy (ODP) and those who had laparoscopic or robotic distal pancreatectomy (MIDP = minimally invasive distal pancreatectomy). Our outcome measures were trends of MIDP and disparities in access to MIDP. Cochran Armitage trend analysis and multivariate regression analysis were used to evaluate outcomes.

Results: A total of 13,537 patients with distal pancreatectomy were identified in the NCDB from 2010 to 2017. 7548(55.8%) underwent ODP while 5989(44.2%) underwent MIDP. The MIDP rate increased from 25% in 2010 to 52% in 2017 (p < 0.01) as demonstrated in Fig. 1. On regression analysis when controlled for age, gender, diagnosis, tumor size, grade, staging, and chemoradiotherapy; African American patients were 30% less likely to undergo MIDP compared to White (OR 0.7, 95%CI[0.5–0.8], p < 0.01). Similarly, Hispanic patients were 25% less likely to undergo MIDP compared to non-Hispanic patients OR 0.75, 95%CI[0.6–0.9], p = 0.02). Compared to Medicare/Private insured patients; uninsured patients were 50% less likely to undergo MIDP (OR 0.5, 95%CI[0.4–0.7], p < 0.01). Based on the medium household income, compared to patients in the fourth quartile, patients in the third quartile OR 0.9, 95%CI[0.3–0.9], p = 0.03). second OR 0.8, 95%CI[0.5–0.9], p < 0.01). and first quartile OR 0.7, 95%CI[0.5–0.8], p < 0.01) were less likely to undergo MIPD as well. On sub-analysis, African American patients even from the fourth quartile were less likely OR 0.85, 95%CI[0.2–0.9], p = 0.04). to undergo MIDP compared to White.

Conclusions: Utilization of MIDP has increased from one in every four patients in 2010 to every other patient in 2017. However, African Americans, Hispanics, the uninsured, and those from low-income quartiles are less likely to undergo MIDP. Interestingly enough, African American patients with a high median house quartile are less likely to undergo MIDP as well.

figure ab

S074

10 Years, 100 Robotic Major Hepatectomies. A Single Center Experience

Frances McCarron 1; Allyson Cochran1; Ansley Ricker1; Rohit Mantha1; Michael Driedger1; Michael Beckman1; Dionisios Vrochedes1; John Martinie1; 1Atrium Health Carolinas Medical Center

Introduction: Adoption of robotic liver resections has been gradually increasing throughout the HPB surgical community over the past decade. Currently there is limited literature which demonstrates a significant benefit of robotic surgery for major hepatectomies over open or laparoscopic. As one of the first centers to develop a robotic HPB program we have experienced improved outcomes over time with increasing utilization of robotics. Herein, we present our 10-year experience and outcomes for major robotic liver resections.

Methods: From 2012 to 2022, 361 robotic liver procedures were performed, including 100 major hepatectomies (greater than 3 segments by Brisbane classification). A retrospective data review of the electronic medical record was performed evaluating outcomes after robotic major hepatectomy. Outcomes for the first 50 (early) and second 50 (late) were compared to identify any improvements in practice. Demographic and clinical outcome variables were analyzed. Data were assessed for normality, and Wilcoxon rank-sum, Chi-square tests and a logistic regression model were performed appropriate for the data.. Stata v.17 was utilized, and significance was set as p < 0.05.

Results: There was no difference in operative time (258 vs 256 min, p = 0.55), EBL (500 vs 500 mL, p = 0.77), conversion rates (6% vs 2%, p = 0.3), median LOS (5 vs 3.5 days, p = 0.11), 90-day readmission (14% vs 24%, p = 0.20), major complications (16% vs 22%) and 90-day mortality (6% vs 4%, p = 0.64) between early and late cases respectively. ICU admissions were significantly lower in the late group (14.0% vs. 48.0%, p < 0.000).

Conclusion: Development of a robotic liver program is feasible over time. The interested HPB surgeon should be aware that perseverance and dedication is required to first overcome the learning curve of robotic surgery prior to implementing complex operations into practice.

S075

Exploring the Ability of Radiomics to Predict Oncological Response of Colorectal Cancer Liver Metastasis to Neoadjuvant Chemotherapy

A B Ricker, MD 1; MKK Niazi2; T E Tavolara2; B Annanurov2; J Davis, MD1; K Smith, MD1; J Robinson1; D A Iannitti, MD1; J B Martinie, MD1; E H Baker, MD1; M N Gurcan2; D Vrochides, MD, PhD, FACS, FRCSC1; 1Atrium Health Carolinas Medical Center; 2Atrium Health Wake Forest

Introduction: Deep learning has been effectively applied across domains of the health sciences to generate predictions from discrete inputs. As a form of artificial intelligence, deep learning cross-sectional imaging makes use of a "neural network" to generate clinically relevant predictions from discrete data inputs. This study employs deep learning on pre-chemotherapy cross-sectional imaging to predict patients' response to neoadjuvant chemotherapy for liver metastases. This may have implications for the choice of neoadjuvant chemotherapy and surgical treatment for colorectal liver metastasis.

Methods: After institutional review board approval, a patient cohort with colorectal liver metastasis who underwent colorectal and liver resection after neoadjuvant chemotherapy was generated from a retrospectively maintained institutional registry. Two machine learning models were trained. The first model trained an attention-based multiple instance learning model on imaging data extracted using TeraRecon to predict the response to neoadjuvant chemotherapy. In the second model, logistic regression in conjunction with clinical parameters, including multifocality, positive lymph nodes, pre-chemotherapy CEA level, disease-free interval until liver metastasis of < 12 months, and largest tumor size, was used to predict the response to neoadjuvant chemotherapy. Both model performances were benchmarked against a binary response variable according to the RECIST criteria. A receiver operating curve was created. The resulting area under the curve (AUC) was determined.

Results: There were 95 patients identified for study inclusion. Over 89% of patients underwent 5-FU based chemotherapy with oxaliplatin and/or irinotecan. 59% of the patients were categorized as responders to chemotherapy. Table 1 reports the result of the two models in terms of sensitivity (SEN), specificity (SPE), positive prediction value (PPV), negative prediction value (NPV), and AUC.

Table 1 Comparison of the proposed imaging-based deep learning model to a logistic regression model trained on clinical data

 

SEN

SPE

PPV

NPV

AUC

Imaging

Model

0.81

0.58

0.84

0.61

0.77

Clinical

Model

0.50

0.38

0.30

0.59

0.41

Conclusions: The clinical model does not meet the criterion for a strong machine learning model. However, a deep learning model trained on imaging data outperforms the model trained on clinical data across all performance metrics. Next steps include using larger patient cohorts to determine whether imaging-based deep learning models alone can predict response to neoadjuvant chemotherapy in patients with colorectal liver metastasis.

S076

The Consequences of Converting Robotic to ‘Open’ Pancreaticoduodenectomy

Sharona Ross, MD, FACS; Moran Slavin, MD; Iswanto Sucandy, MD, FACS; Sneha Saravanan, BS; Kaitlyn Crespo, BS; Cameron Syblis, BS; Alexander Rosemurgy, MD, FACS; Digestive Health Institute Tampa

Introduction: With dissemination of robotic pancreaticoduodenectomy, the influence of conversions to an 'open' operation on patient outcomes is ill-defined. The aim of this study was to elaborate the reasons for conversions to 'open' for robotic pancreaticoduodenectomy and to analyze the impact of such conversions.

Methods: Since 2016, with IRB approval, we prospectively followed 381 consecutive patients undergoing entire robotic pancreaticoduodenectomy (n = 339) or robotic pancreaticoduodenectomy converted to 'open' (n = 42). Data are presented as median (mean ± SD) and significance is accepted with 95% probability.

Results: Over time there was a significant decrease in the rate of conversion (p < 0.0001). More men (n = 29, 14%) than women (n = 13, 7%) underwent robotic pancreaticoduodenectomy converted to 'open' (p = 0.03). Patients whose robotic pancreaticoduodenectomies were converted to 'open' were 73(70 ± 9.1) years old, with a BMI of 28(27 ± 4.3) kg/m2 and a Charlson Comorbidity Index (CCI) of 5(5 ± 1.3) (p = 0.10, p = 1.00, and p = 1.00, respectively). Major reasons for conversions to 'open' were unexpectedly large tumor size (19%), excessive bleeding (21%), excessive intraperitoneal fat (14%), and failure to make progress over 15 min (45%).

Operative duration was shorter (376[323 ± 182.2] minutes) and estimated blood loss (EBL) (675[1,010 ± 1,168.1] mL) and need for intraoperative (0[1 ± 3.0] units packed RBCs) and postoperative transfusions (0[1 ± 2.6] units packed RBCs) were greater for patients with converted robotic pancreaticoduodenectomies (p < 0.001).

Hospital (8[11 ± 11.6] days) and ICU length of stay (LOS) (1[2 ± 5.1] days) were longer with converted pancreaticoduodenectomies (p < 0.001). There were more postoperative Clavien-Dindo ≥ 3 complications (n = 4, p = 0.01), in-hospital mortalities (n = 9, p < 0.001) and 90-day mortalities (n = 9, p = 0.003) after converted operations. Total cost was more expensive for converted operations ($35,561[50,192 ± 46,459.47], p < 0.001). Median and one-year survival for patients with ductal adenocarcinoma that underwent converted vs. entire robotic operations was 12.7 months and 50% vs. 28.2 months and 74% (p = 0.18).

Conclusion: The majority of converted operations were undertaken in our early experience. Patients with converted operations were more likely to be men with an inferior intraoperative (e.g., EBL, need for transfusions) and postoperative course (e.g., LOS, ICU LOS, and mortality/morbidity rate), decreased survival with cancer and increased cost of index admission. Most conversions were for failure to progress due to adhesions or obesity. Conversions to an 'open' operation, with all that entails (e.g., increased EBL, more transfusions, 'hostile' abdomen) seem to have an early effect on survival. Although conversions are sometimes necessary for the safety of the patient, it is advisable to take careful consideration in preoperative patient planning and proactive surgical team training.

figure ac

S077

Expanding the Role of Robotics for Pancreaticoduodenectomy While Achieving Similar Outcomes to Open. A 10-Year Review and Comparison to International Benchmarks in Pancreatic Surgery

Frances McCarron 1; Osamu Yoshino1; Yifan Yang2; Ansley Ricker1; Rohit Mantha1; Michael Driedger1; Michael Beckman1; Phillip Muller3; Pierre-Alain Clavien3; Dionisios Vrochides1; John Martinie1; 1Atrium Health Carolinas Medical Center, USA; 2McGill University, Canada; 3University of Zurich, Switzerland

Introduction: Robotic pancreaticoduodenectomy (RPD) is an emerging alternative to open pancreaticoduodenectomy (OPD). Although the robotic approach offers various theoretical advantages, it is used in less than 10% of all pancreaticoduodenectomies. The aim of this study was to report our 10-year experience and compare our clinical and oncologic RPD outcomes with the published international benchmarks for OPD.

Methods: A retrospective review was performed of our prospectively maintained institutional database of consecutive patients who underwent RPD between January 2011 and December 2021. Patients were categorized into low-risk and high-risk groups according to the selection criteria set by the 2019 published international benchmark study. Their outcomes were compared to the benchmark cutoff values from the published International Benchmarks in Pancreatic Surgery study. Outcomes were also retrospectively compared internally to establish improvements in practice during the study period. Standard statistics were applied, with p < 0.05 indicating statistical significance.

Results: A total of 201 RPDs were performed at our institution during the study period. 37 low-risk and 164 high risk patients were compared against the established benchmarks for OPD. Both cohorts showed results well within the benchmark cutoffs in all parameters. The comprehensive complication index was similar at 20.9. Improvement in outcomes peaked around 100 cases with a POPF rate as low as 5.6% at that time. Of all patients with pancreatic cancer, oncologic outcomes were superior compared to benchmark cutoffs including, a median survival of 33.7 months (95% CI 16.5–50.9 m) and 1-year disease free survival of 69.6%.

Conclusion: RPD yields results comparable to the established benchmarks for OPD. With more patients in the high-risk group including those with borderline resectable pancreas cancer, the indications for RPD may be expanded in high-volume centers to include more complex disease. Superior oncologic outcomes such as improved lymph node harvest and 1-year disease free survival, indicates RPD has benefits that potentially outweigh the presumed obstacle of cost.

S078

Sleeve Gastrectomy Facilitates Weight Loss and Permits Cardiac Transplantation in Patients with Severe Obesity and a Left Ventricular Assist Device (LVAD)

Catherine Tsai 1; Patrick Dolan1; Noah Moss1; Alejandro F. Sandoval1; Julie Roldan1; Daniel Herron1; 1Mount Sinai Medical Center

Introduction: Patients suffering from advanced heart failure may undergo left ventricular assist device (LVAD) placement as a bridge to cardiac transplantation. However, those with a BMI above 35 kg/m2 are not considered eligible for transplant. Until recently, such patients were not considered for bariatric surgery due to their elevated cardiac risk and anticoagulated status. We review our experience with bariatric surgery in this high-risk patient population to assess its safety and efficacy in reducing BMI to permit cardiac transplantation.

Methods: After IRB approval was obtained, we retrospectively reviewed all heart failure patients with LVAD who underwent bariatric surgery at Mount Sinai Hospital between Aug 2018 and June 2022. Electronic medical records were reviewed to analyze patient demographics, surgical details, and outcome with regard to weight loss and heart transplantation.

Results: A total of 10 LVAD patients who underwent bariatric surgery were identified. All patients underwent sleeve gastrectomy (SG), 3 performed laparoscopically and 7 via robotic approach. Four patients (40%) subsequently underwent an orthotopic heart transplant (OHTx). Half of these patients were female, half were male. For patients who underwent OHTx, mean age at LVAD placement was 41 years (range 30.6–52.2), at SG was 43.9 years (range 32.7–55) and at OHTx was 45.3 years (range 33.3–56.8). For these patients, mean BMI increased from 38.8 at the time of LVAD placement to 42.5 immediately prior to SG. After SG, this group had a mean total weight loss of 28.4%. Mean time from SG to OHTx was 17.9 months (range 6.7–27.5) during which BMI decreased to 32.8 by the time of OHTx (Table 1). At most recent follow-up, patients who underwent transplant had a mean BMI of 31.9 (R 29.8–37). All patients were anticoagulated prior to surgery; 1 patient required return to the operating room on post-operative day 1 after SG for bleeding. There were no deaths.

Conclusion: SG is a safe and effective operation in patients with severe obesity and heart failure requiring an LVAD. 40% of our cohort achieved target BMI and underwent heart transplantation. Patients maintained their weight loss at most recent follow-up. Longer term follow-up is needed to clarify full bridge to transplant rate and long-term survival outcomes.

Table 1 Outcomes for all patients who underwent LVAD placement and sleeve gastrectomy and for patients who subsequently underwent cardiac transplantation.

figure ad

S079

Ligamentum Teres Cardopexy for Post-sleeve Gastrectomy Gastroesophageal Reflux

Emily E Mackey, MD; Fiona J Dore, MD; John F Kelly; Allison Crawford, MS; Philip A Cohen, MD; Donald R Czerniach, MD; Richard A Perugini, MD; John J Kelly, MD; Nicole B Cherng, MD; University of Massachusetts

Introduction: Vertical sleeve gastrectomy (VSG) is the leading choice for metabolic surgery, however, despite its durability for weight loss and improvements in co-morbidities, patients with gastroesophageal reflux disease (GERD) have been counseled against VSG due to the increased risk of reflux symptoms post-operatively1. One potential etiology for this relationship may be attributed to the onset of a hiatal hernia following VSG weight loss.

Until recently, the only option for patients with prior VSG and medically refractory GERD has been conversion to Roux-en-Y Gastric Bypass (RNYGB) due to a lack of gastric cardia redundancy precluding a traditional fundoplication2. Given the risks associated with RYNGB, and patient's preference, we have adopted hiatal hernia repair with ligamentum teres cardiopexy (HH/LTC) as a surgical option for patients with intractable GERD following VSG. This study investigates the efficacy of HH/LTC in improving reflux symptoms in this patient population.

Methods: A retrospective chart review was conducted of patients who had prior laparoscopic VSG and subsequently GERD symptoms refectory to pharmacological management who underwent HH/LTC between 2017 and 2022. Pre-operative GERD disease burden, post-operative GERD symptomatology and changes in H2 blocker or PPI requirements were reviewed. Descriptive statistics and chi squared tests were calculated using Microsoft Excel.

Results: Of the study's 44 patients, all had a diagnosis of GERD through pre-operative assessments and 84% had a hiatal hernia identified in pre-operative assessments. 37 of 44 patients underwent intra-operative hiatal hernia repair with their ligamentum teres cardiopexy. Thirty-three patients completed one-year follow-up.

Prior to undergoing HH/LTC, 86% (39 of 44) patients were taking a proton pump inhibitor (PPI) and 41% (18 of 44) were on an H2 blocker (H2B). At one year post-operatively, PPI use decreased to 27% (9 of 33 patients, p < 0.0001), and H2B use had a declining trend (10 of 33). Among H2B users, 36% either stopped the medication (n = 8) or decreased their dose (n = 4). 79% of patients had either a decrease in dosage (n = 6) or complete cessation of PPI or H2B (n = 29) at 1 year following HH/LTC.

Only two patients in the study required re-operation for dysphagia, necessitating loosening of the wrap and only two patients required conversion to RYNGB at 1 year.

Conclusion: Ligamentum teres cardiopexy in conjunction with hiatal hernia repair is a safe and effective alternative to RYNGB in patients with a prior VSG and medical refractory GERD.

S080

The Effect of Changing the 1st Post Operative Follow Up Visit from 2-Weeks to 1-Week on Emergency Department Visits and IV Hydration in Sleeve Gastrectomy [SG] and Roux-en-Y Gastric Bypass [RYGB]

J Eagleston, MD; D Slack, MD; K Thompson, PhD; R Bauman, MD; K Gersin, MD; T Kuwada, MD; S Barbat, MD; A Nimeri, MD; Atrium Health

Background: Up to 15% of patients after metabolic and bariatric surgery [MBS] present to the emergency department (ED) within thirty days of MBS. Historically, at one of our MBSAQIP sites, patients were seen for their initial post-operative visit at two weeks[BSD1], and one week for the other site. We reviewed our 2019 ED visits and noted a higher ED visit rate from the site seeing patients at two weeks following MBS. The mean time for ED visits was between 10–12 days for various reasons including dehydration. The aim of this study was to evaluate the effect of changing the postoperative follow up visit from 2-weeks to 1 week on the rate of ED visits after MBS.

Methods: Using our institutional MBSAQIP registry data, we evaluated patients after primary SG and RYGB performed at our two MBSAQIP centers of excellence. MBSAQIP site A evaluated patients for the first post operative follow-up visit at 1-week post-surgery while Site B evaluated patients at 2-weeks. All revisions and duodenal switch surgeries were excluded. Preliminary data showed that ED visits from site A were 9.3% and from site B were 18%. We quired our electronic medical record for patients who had ED visits, received IV, or readmissions after MBS. In addition, we used the MBSAQIP registry to examine demographics, 30-day complications, and reoperations or mortality after primary MBS.

Results: Our study period included 1031 patients, which included 294 SG and 691 RYGB patients between April 2019 to April 2021. Preoperative demographics were similar between both groups. After surgery, 560 patients were evaluated at 1-week and 471 patients were evaluated at 2-weeks. Specifically, for RYGB, 390 patients were seen at 1-week and 301 at 2-week follow-up. Patients seen at the 2-week follow up were more likely to require IV hydration vs 1-week follow-up (9.3% vs 5.4%; p = 0.046). There was no difference between the groups in thirty-day ED visit rates or readmission rates. Specifically for SG 143 patients were seen at 1-week follow-up and 151 patients at 2-week follow-up. There were no differences in IV-hydration, 30-day ED visits, or readmission rates in SG patients.

Conclusion: Changing the post-operative visit from two weeks to one week decreased RYGB patients need for post-operative IV-hydration. These changes were not identified in SG patients. There were no changes in ED visit or readmission rates between all groups. Closer follow up in RYGB patients can decrease dehydration and additional hydration visits postoperatively.

S081

Use of GLP-1 Agonists in High Risk Patients Prior to Bariatric Surgery: A Cohort Study

Malynn Ilanga, MD; Robert B Lim, MD; Zhamak Khorgami, MD; Geoff Chow, MD; Danielle Lewis, MD; Greg Martin; Carah Horn, RN; Jesse Richards, MD; Oklahoma University School of Medicine at Tulsa

Introduction: Patients with a BMI > 50 kg/m2 are high-risk for post-operative complications from bariatric surgery (BS). This population is prevalent but there is a reluctance to consider these patients for BS because of the risk and the long-term outcomes. GLP-1 agonists combined with lifestyle modification have been very effective for weight loss. The aim of this study is to determine the safety and efficacy of using GLP-1 agonists for prehabilitation and weight reduction prior to BS in patients with a BMI > 50 kg/m2.

Methods: Patients with a BMI > 50 kg/m2 were evaluated for BS by an MBSAQIP bariatric center of excellence team. Patients received either semaglutide, liraglutide, or dulaglutide prior to surgery. All patients received the same medical, surgical, psychiatric, and nutritional evaluation and counseling. BMI, weight, Hgb A1c, time to surgery, co-morbidities, complications, and weight change were analyzed.

Results: 18 patients were analyzed with an average BMI of 60.7 kg/m2 (range 51.1–73.0) and weight of175.8 kg (range 135.0 – 222.7). The average decrease in BMI was 4.7 kg/m2 and the average weight loss was 14.1 kg. The average length of the preoperative period was 4.9 months. 7 patients dropped below a BMI of 50 kg/m2. All cases were completed laparoscopically. Semaglutide had the biggest loss of BMI and weight while Liraglutide had the least. 11 patients underwent Sleeve Gastrectomy, 15 patients underwent RYGB, and 4 patients underwent a SADI-S. There were 3 complications from surgery: 1 VTE, 1 bleeding, and 1 TIA.

Discussion: The use of GLP-1 agonists in BMI > 50 kg/m2 patients is safe and results in significant improvement in BMI and weight prior to BS. It does not prolong the time to surgery. More studies are needed to evaluate the use of GLP-1 agonists preoperatively. Ultimately, this may improve perioperative and long-term outcomes in the very high-risk BMI population.

Medication

Average weight loss (kg)

Average BMI lost (kg/m2)

Semaglutide (ozempic)

19.3

6.2

Liraglutide (saxenda)

7.7

2.7

Dulaglutide (trulicity)

18.6

6.1

Semaglutide (wegovy)

14.1

3.9

S082

Association of Depression with Substance use and Early Postoperative Outcomes after Bariatric Surgery

Erin Kim 1; Andrew C Fregenal, BS1; Sarah E Hughes, BA1; Juliana L Ramirez, BA1; Hannah J Vigran, BS1; Christopher W Reynolds, BS1; Jonathan F Finks, MD2; 1University of Michigan Medical School; 2Department of Surgery, University of Michigan, Ann Arbor, MI, USA

Introduction: Depression is strongly associated with obesity and common among patients undergoing bariatric surgery. Previous research has focused on the effect of bariatric surgery on depression and the effects of depression on postoperative weight loss. However, little is known about the impact of depression on early postoperative outcomes or its association with substance use, a known predictor of negative health outcomes.

Methods: The Michigan Bariatric Surgery Collaborative is a statewide quality improvement program that maintains an extensive clinical registry. We evaluated patients undergoing primary Roux-en-Y gastric bypass or sleeve gastrectomy between 2017–2022. Patients self-reported depressive symptoms (PHQ-8), alcohol use (AUDIT-C), smoking, and marijuana at baseline. Preoperative PHQ-8 scores stratified patients based on depression severity: no depression (0–4), mild (5–9), moderate (10–14), or severe (15–24). 30-day outcomes and measures of substance use were compared between patients with and without depression, adjusting for patient characteristics and procedure type.

Results: Among 44,301 patients, 30.8% had some level of depression, with 19.8% mild, 7.5% moderate, and 3.5% severe. Patients with depression were more likely to have an extended length of stay (> 3 days) than those without (no depression 2.1% vs. severe depression 3.0%, p = 0.0452). However, there were no significant differences between groups in rates of complications (5.7% vs. 5.2%, p = 0.1564), reoperations (0.9% vs. 0.8%, p = 0.7394), ED visits (7.7% vs. 7.8%, p = 0.5353), or readmissions (3.2% vs. 3.9%, p = 0.3034). On the other hand, patients with depression had significantly higher rates of smoking (9.7% vs. 12.5%, p < 0.0001), alcohol use disorder (8.6% vs. 14.0%, p < 0.0001), and marijuana use (8.4% vs. 15.5%, p = 0.0008).

Conclusions: This study demonstrated that nearly a third of patients undergoing bariatric surgery have some degree of depression, with over 10% having moderate to severe depression. There was a significant association between preoperative depressive symptoms and extended hospital stays, but no significant effect on adverse events or healthcare utilization. Nevertheless, preoperative depression was associated with higher rates of smoking, marijuana use, and alcohol abuse, all of which may have significant adverse health effects. Our findings highlight the need to identify and treat depression in bariatric surgery candidates and to carefully screen for substance use in those with depression to maximize the health benefits of surgery in this population.

S083

Laparoscopic Sleeve Gastrectomy to Roux en Y Gastric Bypass Conversion versus Primary Roux en Y Gastric Bypass: A Propensity Score Matching Analysis

Michael O'Laughlin, MD1; Jorge Cornejo, MD1; Jake Tatum, MD2; Alba Zevallos, MD 1; Alissa Coker, MD2; Michael Schweitzer, MD2; Christina Li, MD1; Gina Adrales, MD2; Raul Sebastian, MD1; 1Lifebridge Health / Northwest Hospital; 2Johns Hopkins Hospital

Introduction: Secondary bariatric surgery rates have increased, accounting for approximately 19% of the total bariatric cases in 2020, most commonly conversion of sleeve gastrectomy (SG) to gastric bypass (RYGB). Using the MBSAQIP, we evaluate the outcomes of this procedure compared to initial RYGB surgery.

Methods: The new variable, conversion of SG to RYGB in the 2020 MBSAQIP database was analyzed. Patients who underwent initial RYGB and those who underwent SG to RYGB conversion were identified. Using Propensity Score Matching analysis, the cohorts were matched for 23 preoperative characteristics. We then compared 30-day outcomes and bariatric specific complications between initial RYGB and SG to RYGB conversion.

Results: There were 31,864 initial RYGB procedures performed and 4,673 conversions from SG to RYGB. The matched cohorts (n = 4,044) for the two groups have similar pre-operative characteristics. Propensity matched outcomes showed that SG to RYGB conversion was associated with more readmissions (4.9% vs 6.7%, p = 0.001), interventions (1.6% vs 2.5%, p = 0.006), conversion to open (0.2% vs 0.8%, p < 0.001), length of stay (1.63 + 1.37 days vs 1.82 + 1.79 days, p < 0.001), and operative time (118.89 + 56.94 min vs 138.93 + 66.35, p < 0.001). There were no significant differences in mortality (0.0% vs 0.1%, p = 0.727), or bariatric specific complications such as anastomotic leak (0.3% vs 0.5%, p = 0.216), intestinal obstruction (0.0% vs 0.1%, p = 0.289), internal hernia (0.1% vs 0.2%, p = 0.180) or anastomotic ulcer (0.3% vs 0.3%, p = 0.481) rates.

Conclusion: With the dramatic expansion and acceptance of sleeve gastrectomy, we anticipate that the number of conversions or revisions will continue to increase. SG to RYGB conversion is a safe and feasible operation with reasonable outcomes compared with initial RYGB.

S084

Long-Term Outcomes Following Laparoscopic Roux-en-Y gastric Bypass: Weight Loss and Resolution of Comorbidities at 15 Years and Beyond

Paul H McClelland, MD; Mohsin Jawed, MD; Krystyna Kabata, PA; Susan Basharkhah, MD; Michael E Zenilman, MD; Piotr Gorecki, MD; NewYork-Presbyterian Brooklyn Methodist Hospital

Background: Laparoscopic Roux-en-Y gastric bypass (LRYGB) is the gold-standard bariatric procedure with proven efficacy in morbidly obese populations. While the weight loss benefits of LRYGB have been well-documented in the short term, durable weight loss and long-term resolution of obesity-related comorbidities have been less clearly described, especially more than 10 years after initial surgery.

Methods: This study prospectively reports weight loss and resolution of comorbidities in patients who underwent primary LRYGB between August 2001 and September 2007 with at least 15-year attempted follow-up. All patients were treated at a single institution by a single surgeon. Weight and comorbidity data were collected at the time of surgery, 1, 3, 6, and 12 months postoperatively, and then annually thereafter.

Results: A total of 486 consecutive patients were included in this analysis. Patients were predominantly female (88.7%), and the mean age of participants was 37.4 ± 11.0 years. Patients were also ethnically diverse, including black/African American (43.6%), white/Caucasian (35.0%), Hispanic (18.3%), and other backgrounds (3.1%). Mean preoperative weight and BMI were 133.0 ± 21.9 kg and 48.4 ± 6.5 kg/m2, and the mean number of preoperative obesity-related comorbidities was 6.7 ± 2.8 conditions. Follow-up rates at 1, 5, 10, and 15 years were 75.3%, 37.2%, 35.1%, and 19.0%, respectively. On average, maximum relative weight loss from baseline occurred 2 years after surgery (-36.2 ± 9.5%), and > 25% relative weight loss was consistently achieved at 1-, 5-, 10-, and 15-year time intervals (-28.0 ± 13.0% at 15 years). Patients with comorbidities generally experienced significant improvement or resolution of their conditions by the 1-year mark, including type 2 diabetes mellitus (DM2; 84/86, 97.6%), obstructive sleep apnea (OSA; 123/128, 96.1%), hypertension (HTN; 144/153, 94.1%), and gastroesophageal reflux disease (GERD; 234/242, 96.7%). Rates of improved/resolved comorbidities remained consistently high through at least 10 years after surgery (DM2 32/33, 97.0%; OSA 21/23, 91.3%; HTN 33/35, 94.3%; GERD 39/46, 84.8%).

Conclusions: LRYGB provides durable long-term weight loss for at least 15 years after surgery, with an average relative weight loss of > 25% from baseline. This maintained weight loss facilitates sustainable resolution of obesity-related comorbidities such as DM2, OSA, HTN, and GERD for at least 10 years following the index operation. Ongoing collection of follow-up data is needed past the 15-year mark to characterize the long-term effects of LRYGB and rule out insidious weight regain or recurrence of comorbidities, particularly for patients undergoing the procedure early in life.

figure ae

S085

Concurrent Paraesophageal Hernia Repair in Revisional and Conversional Laparoscopic Roux-en-Y Gastric Bypass; Propensity Score Matched Analysis of the MBSAQIP Database

Samuel C Perez; Forrest Ericksen, MD; Andrew A Wheeler, MD, FACS, FASMBS; University of Missouri School of Medicine

Introduction: The primary aim of this study was to determine perioperative complications associated with concurrent paraesophageal hernia repair (CPHR) during revisional/conversional Roux-en-Y gastric bypass. Patients requiring CPHR have been shown to have favorable outcomes in primary bariatric surgery. However, patients requiring revisional or conversional surgery represent a group of patients with higher perioperative risk. Currently, few reports on CPHR during revisional/conversional are available.

Methods and Procedures: In this retrospective cohort study, patients undergoing revisional/conversional Roux-en-Y gastric bypass (RYGB) between 2015 and 2020 were accessed via the MBSAQIP participant use file (PUF) database. Patients were categorized based on the presence of a paraesophageal hernia repair as a concurrent procedure. Patients who underwent revisional/conversional surgery without additional procedures were utilized for controls. A propensity score matched cohort was generated and E-analysis utilized to assess unmeasured confounding.

Results: After applied exclusions, 28,851 patients were available for analysis. Patients receiving CPHR were more likely to be female (90.47% vs 86.71%;p < 0.001), have an increased frequency of gastroesophageal reflux disease (64.60% vs 47.85%;p < 0.001). However, these patients had lower frequencies of sleep apnea (27.62% vs 33.16%;p < 0.001), hypertension requiring medication (42.09% vs 45.19%;p < 0.001), and hyperlipidemia (19.69% vs 22.26%;p < 0.001). After matching, 4,291 patient pairs were developed and showed that patients undergoing CPHR were at increased risk of readmission (8.62% vs 6.97%;p = 0.007), intervention (3.57% vs 2.33%;p = 0.001), increased requirement for adhesiolysis during the procedure (23.70% vs 16.69%;p < 0.001) and overall increased operative time (157 min, IQR[116,212] vs 141 min, IQR[103,188];p < 0.001). However, there were no statistically significant increases in the rates of reoperation, death, postoperative leak complications, or bleeding complications in patients having CPHR.

Conclusion: Patients undergoing revisional/conversional RYGB with CPHR may be at higher risk for a small number of postoperative complications, but these complications are still relatively uncommon. CPHR is a safe procedure in patients undergoing revisional/conversional RYGB in the early postoperative period.

S086

Utility of using American Society of Anesthesiologist Classification in Correlating Peri-operative Morbidity and Mortality in Elective Bariatric Surgery: An MBSAQIP Retrospective Cohort Study

Sukhdeep Jatana, MDCM 1; Kevin Verhoeff, MD1; Valentin Mocanu, MD, PhD1; Uzair Jogiat, MD1; Daniel W Birch, MD1,2; Shahzeer Karmali, MD1,2; Noah J Switzer, MD, MPH1; 1Department of Surgery, University of Alberta; 2Centre for Advancement of Surgical Education and Simulation (CASES), Royal Alexandra Hospital

Introduction: The American Society of Anesthesiologist (ASA) classification is a standardized tool used to gauge peri-operative mortality and morbidity but its role in bariatric surgery is poorly understood. This study aims to characterize and correlate outcomes with ASA risk and determine the predictive value of ASA with regards to serious complications for bariatric surgery patients.

Methods: The MBSAQIP database from 2020 was analyzed and two cohorts were outlined for comparison, those of ASA class IV, deemed high-risk, and those of ASA class II and III, deemed normal-risk. Adult patients undergoing elective Roux-en-Y gastric bypass (RYGB) or sleeve gastrectomy (SG) were included. Patients undergoing revision or emergency surgery, with a history of foregut surgery, or categorized as ASA class I, V, or unknown were excluded. Univariate analysis was performed to characterize differences and assess for rate of complications between these cohorts. Multivariate logistic regression analysis was performed to determine factors associated with increased odds of 30-day serious postoperative complications and mortality.

Results: A total of 138 612 patients were identified of which 5380 (3.9%) were high-risk and 133 232 (96.1%) normal-risk. High-risk patients were more likely to be older (46.2 ± 12.0 vs. 43.4 ± 11.9, p < 0.001), male (30.9% vs. 18.4%, p < 0.001), have higher BMI (51.4 ± 10.2 vs. 44.9 ± 7.4, p < 0.001), have increased comorbidities, and undergo RYGB (30.6% vs. 26.4%, p < 0.001). High-risk patients were more likely to have increased 30-day complication rates including serious complications (4.5% vs. 2.8%, p < 0.001) and death (0.2% vs. 0.1%, p = 0.001). There were no differences in anastomotic leak (0.2% vs. 0.2%, p = 0.983) or wound dehiscence (partial or fascial, p = 0.649), but cardiovascular complications occurred more often in the high-risk group. After adjusting for comorbidities, ASA class IV patients were at higher odds for 30-day serious complications (OR 1.35, p < 0.001) but not for death (aOR 1.04, p = 0.921). The single greatest predictor of serious complication was preoperative functional status (partially dependent aOR 2.07, p < 0.001, fully dependent aOR 3.18, p = 0.034 for any serious complication; partially dependent aOR 5.08, p < 0.001 for death).

Conclusions: Elevated ASA class correlates with many serious complications but the absolute differences between high- and normal-risk group remains low and many complications, including mortality, are not significantly associated with ASA class. Pre-operative functional status appears to have a much greater contribution to odds of serious complications and mortality. These findings question the utility of using ASA to risk-stratify patients peri-operatively and provides evidence for using a simpler and more practical approach based on functional status.

S087

Primary Single Anastomosis Duodeno-Ileal Bypass with Sleeve (SADI-S) Versus Revisional SADI After Sleeve Gastrectomy: A Comparison Study of Prevalence and Safety

Juan S Barajas-Gamboa, MD1; Jerry T Dang, MD, PhD2; Andrew Strong, MD 3; Gustavo Romero-Velez, MD2; Hadika Hadika, MD2; Maryam Al-Zubaidi, MD2; Matthew Allemang, MD2; Salvador Navarrete, MD2; John Rodriguez, MD1; Ricard Corcelles, MD, PhD2; Matthew Kroh, MD2; 1Cleveland Clinic Abu Dhabi; 2Cleveland Clinic Ohio; 3Cleveland Clinic

Introduction: There is controversy concerning the safety of the revisional bariatric procedures as they are perceived to have higher complication risks. Conversion from sleeve gastrectomy to single anastomosis duodeno-ileal bypass with sleeve (SADI-S) is becoming increasingly common, however current evidence comparing primary SADI-S (p-SADI-S) to revisional SADI is scarce. As such, the objective of this study was to compare the 30-day rate of serious complications and mortality of p-SADI-S to revisional SADI.

Methods and Procedures: This retrospective cohort study analyzed the MBSAQIP database. Patients undergoing p-SADI-S and revision from sleeve gastrectomy to SADI (r-SADI) were included. Data collection was limited to 2020 as this year the MBSAQIP included details of primary bariatric procedures in revisional cases. A multivariable logistic regression analysis was performed between groups to determine if r-SADI was an independent predictor of 30-day serious complications or mortality.

Results: A total of 783 patients were included in this study, 488 (62.3%) underwent p-SADI-S and 295 (37.6%) underwent r-SADI after a previous sleeve. The mean age at the time of surgery was 43.6 years and not significantly different between groups. The mean body mass index (BMI) at the time of surgery was lower in the r-SADI cohort (45.1 vs 51.4 kg/m2, p < 0.001). There were also significantly lower rates of gastroesophageal reflux disease (GERD, 24.5 vs 37.6%, p < 0.001) in the p-SADI-S cohort. Patients undergoing p-SADI-S had higher metabolic comorbidities including diabetes (33.4 vs 18.6%, p < 0.001), hyperlipidemia (28.2 vs 20.3%, p < 0.001), and hypertension (54.5 vs 39.2%, p < 0.001). Indications for revision in the r-SADI cohort included weight recurrence (50.8%), inadequate weight loss (41.0%), other (3.0%), GERD (2.7%), and persistent comorbidities (2.5%). r-SADI had longer operative times (156.7 vs 142.1 min, p < 0.001) and were not associated with a higher rate of serious complications (5.7 vs 6.9%, p = 0.508) compared p-SADI-S. p-SADI-S was associated with a higher rate of pneumonia (1.2 vs 0.0%, p < 0.001), and r-SADI was not correlated with higher rates of reoperation (3.0 vs 3.2%, p = 0.861), readmission (5.4 vs 5.5%, p = 0.948) and death (0.0 vs 0.2%, p = 0.437).

On multivariable analysis, r-SADI was not independently predictive of serious complications (OR 0.81, 95%CI 0.43 to 1.52, p = 0.514) when adjusting for age, sex, BMI, comorbidities, and operative time.

Conclusions: The prevalence of r-SADI is high, representing 37.6% of SADI-S procedures. Our study demonstrates that revision from sleeve to SADI is safe, with a similar 30-day complication rates to primary SADI-S, although with slightly longer operative times.

S088

Losing Weight to Achieve Joint or Hernia Surgery: Is the Intragastric Balloon the Answer?

Alexndra Kovar, MD; Danielle Abbitt, MD; Kevin Choy, MD; Teresa S Jones, MD; Benjamin Cassell, MD; Hazam Hammad, MD; R Matthew Reveille, MD; Kryzstzof Wikiel, MD; Edward Jones, MD; University of Colorado School of Medicine

Background: Obesity is an American epidemic. The constellation of medical conditions that accompany obesity place patients at increased risk of postoperative complications. Studies have shown obese patients who undergo bariatric surgery before orthopedic surgery have decreased postoperative complications and lower BMI one year after surgery. It is also well established that weight loss prior to joint replacement or hernia repair results in fewer complications and lower hernia recurrence. We could find no studies utilizing an intragastric balloon to achieve weight loss prior to elective surgery. Thus, we sought to evaluate the safety and efficacy of an intragastric balloon in achieving weight loss prior to elective joint replacement or hernia repair.

Methods: Retrospective review of prospective database of all patients undergoing bariatric balloon placement at a tertiary care VA medical center from 1/2019–9/2022. Patients who had a qualifying procedure (knee or hip replacement, hernia repair) and had a BMI > 35 kg/m2 were offered intragastric balloon placement to achieve 30-50lbs weight loss. Intragastric balloons were removed 6 months after placement. All patients were required to participate in a standardized weight loss program for 12 months. Balloon removal was preferentially performed concomitantly with the qualifying procedure at the completion of therapy. Baseline demographics as well as weight loss, BMI, duration of balloon therapy and overall success at achieving weight loss and proceeding to qualifying procedure were recorded.

Results: Eighteen patients received intragastric balloon placement to achieve weight loss for elective incisional hernia repair (7 patients, 38.9%), umbilical hernia repair (5 patients, 27.8%), inguinal hernia repair (3 patients, 16.7%) or knee replacement (2 patients, 11.1%). Mean age 52 (range 34–66 years), all (100%) patients were male. Mean balloon duration was 194.7 ± 30.1 days. No (0%) balloons were removed early. There were no (0%) perioperative complications. Mean weight loss was 29.98 ± 17.4 lbs with an average BMI reduction of 4.3 ± 2.7 Mean excess body weight loss was 28.8 ± 18.8%. Fifteen (83%) patients were considered a success with 14 (93%) undergoing planned procedure and 1 (7%) no longer having symptoms from an inguinal hernia.

Discussion: Intragastric balloon placement resulted in an average 30lbs weight loss over 6 months allowing more than 80% of patients to undergo joint replacement or hernia repair at an optimal weight. Bariatric balloons should be considered in patients requiring 30-50lbs weight loss prior to elective surgery.

S089

Readability of bariatric surgery education materials

Adam Lucy, MD; Kristen Wong, MD; Jayleen Grams, MD, PhD; Richard Stahl, MD; Margaux Mustian, MD, MPH; UAB

Introduction: Over 70% of Americans are overweight, and 9% have Class 3 obesity (BMI 340 kg/m2). Although bariatric surgery is the most successful treatment for severe obesity, barriers to surgery exist, including poor health literacy. Several national organizations recommend patient education materials should not exceed a sixth-grade reading level. If not easily comprehensible, patient education materials can exacerbate barriers to bariatric surgery. This is particularly relevant in Alabama, which has one of the highest obesity and lowest literacy rates. The goal of this study was to assess the readability of institutional websites and electronic medical record (EMR) bariatric surgery patient education materials from a single center.

Methods: Patient educational materials from the bariatric surgery website and professionally written perioperative education materials from the EMR of a single institution were analyzed and compared. The readability of the text was assessed by the complexity of language and syntax utilizing the validated instruments Flesch Reading Ease Formula (FRE), Flesch Kincaid Grade Level (FKGL), Gunning Fog (GF), Coleman-Liau Index (CL), Simple Measure of Gobbledygook (SMOG), Automated Readability Index (AR), and Linsear Write Formula (LW). Averages of each readability score were calculated as means with standard deviations. Means were compared using t-tests.

Results: Thirty-two webpages and seven EMR bariatric education discharge documents were analyzed. Website materials were on average "difficult to read" with a FRE mean score of 50.5 ± 18.3 compared to EMR materials with overall "standard" readability with a mean FRE score of 67.4 ± 4.2 (p = 0.023). Much of the website material required at least a high school reading level, with a mean FKGL of 11.8 ± 4.4, GF of 14.0 ± 3.9, CL of 9.5 ± 3.2 and SMOG index of 11.0 ± 3.2. Similarly, they had an average ARI score of 11.7 ± 5.1, and LW of 14.9 ± 6.6 years of education needed, consistent with college-level entry. Webpages with highest reading levels were those containing meal plan/nutritional supplements information (FRE 22.8, very difficult to read), while the patient testimonials were fairly easy to read with a mean FRE of 73.6. Comparatively, EMR materials were above a 6th-grade reading level with an average FKGL of 6.5 ± 0.7 (p < 0.01).

Conclusion: Bariatric surgery webpages curated by surgeons have advanced reading levels above recommended thresholds for optimal patient comprehension compared to standardized education material embedded in the EMR. This readability gap may unintentionally create barriers to surgery and could affect postoperative outcomes. Efforts to make patient education more easily understandable are needed to bridge the knowledge gap.

S090

Sigmoidal esophagus is in the eye of the beholder: length-to-height ratio characterizes achalasia esophageal morphology

Soon Moon, DO 1; John Barron, MD2; Sadia Tasnim, MD2; Monisha Sudarshan, MD2; Mark Baker, MD3; Siva Raja, MD, PhD2; 1Department of Surgery, Cleveland Clinic South Pointe; 2Department of Thoracic and Cardiovascular Surgery, Cleveland Clinic; 3Department of Radiology, Cleveland Clinic

Introduction: Esophageal morphology in achalasia is thought to affect outcomes, with "end-stage" sigmoidal morphology faring poorly; however, rigorous evaluation of morphology's role in outcomes has thus far been limited by lack of its objective characterization. Hence, the goals of this study were twofold: characterize the variability of timed barium esophagram (TBE) interpretation between experts and evaluate an objective classification of esophagram morphology using esophageal length: patient height ratio (LHR). We hypothesized that the esophagus must elongate to become sigmoidal, and that sigmoidal morphology would thus demonstrate larger LHR.

Methods and Procedures: Ninety preoperative TBEs were randomly selected from an institutional achalasia database. Three experts, two thoracic surgeons and one radiologist, first independently interpreted each TBE, categorizing the esophagus as straight, intermediate, or sigmoidal. A final consensus morphology was defined by majority interpretation. Esophageal length, measured from the aortic notch to the diaphragm, was recorded. Descriptive statistics and frequency of expert agreement were calculated. Median LHR was compared between expert consensus morphologies. A receiver operating characteristic (ROC) plot was utilized to determine the optimal LHR for sigmoidal vs non-sigmoidal characterization.

Results: From a total of 90 preoperative TBEs, pairs of the three individual experts agreed on morphology in 62.2–67.8% of TBEs, with all three experts agreeing in 46.7% of cases. A total of 30 TBEs (33.3%) were categorized as sigmoidal by ≥ 1 expert, with all three agreeing on 8 (26.7%) occasions. Fifty-six (62.2%) TBEs were categorized as straight by ≥ 1 expert, with all three agreeing in 26 (46.4%) instances. Expert consensus morphology found 43 (47.8%) to be straight, 30 (33.3%) intermediate, and 17 (18.9%) sigmoidal. Median LHR was 0.79 (interquartile range 0.68–0.94). Sigmoidal TBEs demonstrated higher LHR (median 0.93; 0.87–1.06), compared to non-sigmoidal TBEs (0.74; 0.65–0.90) (P < 0.001). ROC plot demonstrated that an LHR cutoff of 0.827 was 82.4% sensitive (AUC 0.795) for ruling out sigmoidal morphology (Fig. 1).

Fig. 1 ength-to-Height Ratio (LHR) receiver operating characteristic for sigmoidal morphology with optimal LHR cutoff (AUC: area under the curve; CI: confidence interval)

figure af

Conclusions: These findings confirm our anecdotal experience, that subjective TBE morphology interpretation is highly variable, even between experts at a high-volume achalasia center. LHR provides an objective method for classification, allowing us to overcome the limitations of inter-observer variability, thus paving the way for future study of the role of morphology in achalasia outcomes.

S091

Long-Term Efficacy of Different Surgical Procedures for Gastroesophageal Reflux Disease: A Network Meta Analysis

Yung Lee, MD; Lea J Tessier, MD; Umair Tahir; Kevin Yang; Taaha Hassan; Tyler McKechnie, MD; Aristithes Doumouras, MD, MPH; John Agzarian, MD, MPH; Dennis Hong, MD, MSc; McMaster University

Introduction: Laparoscopic fundoplication is currently considered the gold standard surgical therapy for gastroesophageal reflux disease (GERD). The types of fundoplication surgeries differ based on the degree of wrap used. Additionally, several meta-analyses of short (< 1 year) and medium-term (1–5 years) studies have found no difference in GERD-related outcomes treatment effectiveness between procedures. However, the long-term (> 5 years) is yet to be explored. Thus, the current network meta-analysis (NMA) was undertaken to compare long term (> 5 years follow up) GERD-related outcomes following different fundoplication procedures.

Methods and Procedures: MEDLINE, EMBASE, and CENTRAL databases were searched up to March 2022 to identify randomized controlled trials (RCTs) comparing different types of laparoscopic fundoplications reporting GERD-related outcomes long term (> 5 years). PRISMA extension statement for NMAs and Grading of Recommendations, Assessment, Development and Evaluations (GRADE) was followed for reporting and assessing the quality of evidence. Odds ratios and 95% credibility intervals (CrIs) were calculated to produce network estimates between the different types of procedures. Treatments were ranked using the surface under the cumulative ranking curve (SUCRA) probabilities.

Results: 14 RCTs were included involving 1349, 257, and 885 patients undergoing laparoscopic Nissen (360°), Dor (anterior 180°-200°), and Toupet (posterior 270°) fundoplications, respectively. There were no long-term RCTs for Belsey Mark, Lind, and other alternative fundoplications. SUCRA rankings showed that the probability of dysphagia occurring following five-years of follow up was least likely following Toupet, followed by Dor, and Nissen, with values of 0.9008, 0.4967, and 0.1026, respectively. Network estimates for incidence of dysphagia (median follow-up 5 years (IQR 60–120 months)) showed that Toupet had significantly lower long-term odds of dysphagia than Nissen fundoplication [OR 0.285; 95% CrI 0.06–0.958]. Network meta-analysis failed to demonstrate significant differences between Toupet and Dor fundoplication (OR 0.473, 95%C CrI 0.072–2.835), as well as between Dor and Nissen fundoplication (OR 1.689, 95% CrI 0.403–7.699) in terms of long-term dysphagia. There was no difference between the three procedures for long-term incidence of postoperative reflux, regurgitation, inability to belch, abdominal bloating, reoperation, usage of GERD medications, or patient satisfaction with surgery.

Conclusion: Laparoscopic Toupet fundoplication provides best long-term durability with lowest odds of developing postoperative dysphagia compared to the Nissen or Dor fundoplication. There were no differences noted between procedures for other GERD-related patient-important outcomes.

S092

Defining Surgical Risk in Octogenarians Undergoing Paraesophageal Hernia Repair (PEHR)

Hadley H Wilson, MD; Sullivan A Ayuso, MD; Dau Ku, MS; Gregory T Scarola, MSPH; Vedra A Augenstein, MD; Paul D Colavita, MD; Todd Heniford, MD; Carolinas Medical Center

Introduction: With an aging population, the utility of surgery in elderly patients, particularly octogenarians, is of increasing interest. Our goal was to analyze outcomes of those over 80 versus younger patients.

Methods: The Nationwide Readmissions Database was queried for patients who underwent PEHR from 2016–2018. Exclusions included diagnosis of gastrointestinal malignancy, concurrent bariatric procedure, or age < 18. Patients ≥ 80 were compared to those 18–79 years old using standard statistical methods, with subgroup analysis of elective PEHR.

Results: From 2016–2018, 46,450 patients were identified: 5,425 (11.7%) were ≥ 80 years old and 41,025 (88.3%) were < 80 years old. The median age of octogenarians was 84 [81, 87] and of younger patients was 64 [54, 71]. Octogenarians had a higher Charlson Comorbidity Index (CCI) (1 [0, 2] vs 0 [0, 1], p < 0.001), were more often female (74.3% vs 71.5%, p < 0.001), and more likely to have an emergent operation (46.3% vs 18.2%, p < 0.001). Procedure types included: laparoscopic (67.7% vs 69.7%), robotic (14.0% vs 18.0%), open (18.2% vs 12.1%), and conversion to open (0.2% vs 0.2%); p < 0.001. Multivariable logistic regression revealed age > 80 was an independent predictor of readmissions at 30 (OR = 1.512 [1.348–1.697], p < 0.001), 90 (OR = 1.451 [1.314–1.603], p < 0.001), and 180 (OR = 1.471 [1.341–1.613], p < 0.001) days but was not predictive of perioperative mortality (OR = 1.373 [0.962–1.959], p = 0.081). Emergent procedure was an independent predictor of mortality for all patients (OR = 3.180 [2.492–4.057], p < 0.001). In subgroup analysis of elective procedures, there were 2,912 (8.0%) octogenarians and 33,572 (92.0%) younger patients. Octogenarians had more cardiac complications (22.5% vs 6.5%, p < 0.001), VTE (1.4% vs 0.6%, p < 0.001), pneumonia (1.9% vs 1.2%, p = 0.004), respiratory failure (6.4% vs 2.7%, p < 0.001), esophageal perforation (0.7% vs 0.4%, p = 0.010), acute renal failure (4.3% vs 1.7%, p < 0.001), sepsis (1.6% vs 0.8%, p < 0.001), and had higher mortality (1.3% vs 0.2%, p < 0.001). Octogenarians had longer LOS (3 [2, 5] vs 2 [1, 3] days, p < 0.001) and higher readmission rates at 30 (11.1% vs 6.5%, p < 0.001), 90 (16.0% vs 9.4%, p < 0.001), and 180 (19.4% vs 11.5%, p < 0.001) days.

Conclusions: Octogenarians represented a substantial proportion of patients undergoing PEHR and were more likely to undergo an emergent operation. Being an octogenarian was an independent predictor of readmission but not of perioperative mortality. Emergent status was associated with mortality for all patients. In subgroup analysis of elective procedures, octogenarians' perioperative mortality rate was reduced but higher than younger patients as were complication rates. The decision for elective repair should involve a thorough risk/benefit analysis in octogenarians.

S093

Measurements of lower Esophageal Sphincter Geometry During Anti-reflux Surgery Using Impedance Planimetry (Endoflip) and Associated Outcomes

Julia R Amundson, MD, MPH1; Kristine Kuchta, MS2; Vanessa N VanDruff, MD1; Stephanie Joseph, MD, MPH2; Simon Che, MD2; Christopher Zimmermann, MD 2; Shun Ishii, MD2; H. Mason Hedberg, MD, MS2; Michael B Ujiki, MD, FACS2; 1University of Chicago; 2NorthShore University HealthSystem

Introduction: Impedance planimetry (FLIP) shows promise as an objective tool to optimize outcomes after anti-reflux surgery (ARS). However, ideal intraoperative FLIP ranges have not yet been established. In this study we report FLIP measurements during ARS and their association with outcomes in an effort to establish optimal ranges.

Methods and Procedures: This is a retrospective review of a prospective gastroesophageal quality database. All patients who underwent Nissen or Toupet fundoplication and intraoperative FLIP evaluation with an 8 cm catheter 40 mL fill or 16 cm catheter 60 mL fill were included. Surgeons used no bougie, the FLIP balloon as a bougie, or a hard bougie during the fundoplication. Outcomes included perioperative data, Reflux Symptom Index, GERD-HRQL, and dysphagia scores, as well as need for postoperative dilation. Group comparisons were made using two-tailed Wilcoxon rank-sum and Fisher's exact tests, with two-tailed statistical significance of p < 0.05.

Results: Between 2016 and 2022, 312 patients underwent ARS and intraoperative FLIP with 8 cm or 16 cm catheters. Most patients (87.5%) underwent Toupet fundoplication, 145 with hard bougie (THB), 69 with FLIP as bougie (TFB), and 59 without a bougie (TNB). Thirty-nine patients (12.5%) underwent Nissen fundoplication, 20 with hard bougie (NHB), 19 with FLIP as bougie (NFB). No significant differences in 30-day postoperative outcomes were seen. Postoperative quality of life metrics differed only in worse Reflux Symptom Index scores following NFB compared to TFB at 2-years postop (p = 0.009). FLIP measurements with 8 cm EF325 catheters at 40 mL fill varied significantly between groups, with distensibility index at crural closure (CCDI) lower following THB vs NFB, p = 0.03. CCDI averages: THB 2.6 ± 1.2mm2/mmHg, TFB 2.8 ± 1.4mm2/mmHg, TNB 2.7 ± 1.2mm2/mmHg, NHB 3.2 ± 1.4mm2/mmHg, NFB 3.6 ± 1.5mm2/mmHg. Cross sectional area (86 ± 28mm2) & DI after fundoplication were significantly lower following TNB compared to all other groups, both p < 0.01. Post-fundoplication DI averages: NHB 3.5 ± 1.2mm2/mmHg, NFB 3.3 ± 1.4mm2/mmHg, THB 3.8 ± 1.3mm2/mmHg, TFB 3.4 ± 1.4mm2/mmHg, TNB 2.6 ± 1.2mm2/mmHg [Fig. 1]. Two patients required postoperative dilation, one following NFB (CCDI 3.3mm2/mmHg, post-fundoplication DI 2.2mm2/mmHg, preoperative manometry normal) and one following TFB (CCDI 2.0mm2/mmHg, post-fundoplication DI 2.4mm2/mmHg, preoperative manometry with severe peristaltic disorder), p = 0.07. All measurements with 16 cm EF322 catheters and 60 mL fill were taken in TNB patients, n = 38, average CCDI 3.3 ± 1.7mm2/mmHg, post-fundoplication DI 2.5 ± 1.2mm2/mmHg.

Conclusion: Intra-operative FLIP values obtained during ARS vary by fundoplication and bougie choice. Long-term follow-up of patients who have had 8 cm 40 mL fill and 16 cm 60 mL fill data collected is needed to determine ideal ranges at both crural closure and post-fundoplication for optimal patient outcomes.

S094

Laparoscopic Prophylactic Total Gastrectomy for CDH1 Gene Carriers

Mary Kate Bryant, MD, MSCR; Rachel Sillcox, MD; Brant K Oelschlager, MD; University of Washington

Background: E-cadherin(CDH1) gene mutations are the leading etiology of Hereditary diffuse gastric cancer with associated cumulative lifetime risk ranging up to 80%. Prophylactic total gastrectomy (PTG) is, therefore, recommended for carriers of the mutation. A laparoscopic approach may reduce operative risk versus an open operation, thus enticing patients with CDH1 mutations to pursue PTG prior to cancer development. However, more experience and outcome data for a laparoscopic approach are needed.

Methods: A retrospective, observational cohort study of adult patients with CDH1 mutations who underwent laparoscopic PTG between 2012–2022 was conducted at a single institution. All patients had preoperative EGD screening, and those with visible tumor lesions on surveillance EGD were excluded and not considered prophylactic. Demographics, family history, pathology, and operative course were obtained. Outcomes included complications, readmission, and postoperative weight change.

Results: Among 23 patients, mean age was 47 years (SD 11) and 15 (65%) were female. Family history for gastric and/or lobular breast cancer was present in 22(96%) patients. The median (IQR) time from positive genetic testing to PTG was 347 days (140,625). Pathologic evaluation showed five (22%) patients with foci of gastric cancer in their pre-operative EGD biopsies and 10 (44%) in their resected stomachs. Patient outcomes are reported in the table. To address early postoperative complications, EJ anastomotic technique changed from EEA to GIA over the course of the study, and feeding jejunostomy was no longer placed during PTG with minimal change in post-operative weight loss.

Conclusion: Our study reports clinical outcomes in the largest series of laparoscopic PTG for CDH1 carriers. A laparoscopic approach results in an acceptable complication rate with a relatively short hospital stay. In addition, despite no visible cancer, over half of our patients had foci of gastric cancer. Therefore, CDH1 carriers should consider laparoscopic PTG.

Variable, n(%)

PTG Cohort, N = 23

Esophagojejunostomy technique

 

 EEA

8(34.8)

 GIA

15(65.2)

Jejunostomy feeding tube at PTG

7(30.4)

Median(IQR) LOS, days

5(5,5)

Mortality

0

Readmissions(< 90 days)

4(17.4)

Patients with complication requiring intervention*

5(21.8)

Median(IQR) 1-month postoperative weight loss

4.9%(2.6,7.1)

Median(IQR) 6-month postoperative weight loss

12.5%(8.1,16.4)

  1. *Includes one or more complications per patient:reoperation [(anastomotic leak, n = 1),(hemorrhage, n = 1),SBO at jejunostomy tube(n = 1),SBO (n = 1)],myocardial infarction(n = 1), malnutrition(n = 1)

S095

The Impact of Frailty on Outcomes Following Laparoscopic Repair of 'Giant' Paraesophageal Hernias

Thomas Q Xu, MD; Jesse Maguire, BS; Jon C Gould, MD, MBA; Medical College of Wisconsin.

Introduction: Frailty is a measure of physiologic reserve and correlates with surgical outcomes in the elderly. Paraesophageal hernias (PEH) tend to increase in size with age. Patients who present with giant paraesophageal hernias (> 50% of stomach in the chest) are typically older than 65. We hypothesized that frailty in patients > 65 correlates with 30-day complications, length of stay, and discharge destination following laparoscopic giant PEH repair.

Methods and Procedures: Patients older than 65 to undergo primary laparoscopic repair of a giant PEH at a single academic medical center between 2014–2022 were included. Hernia size was assessed by review of pre-op CT scans or upper GI series and only patients with hernias containing 50% or more of the stomach above the diaphragm were included. Frailty was assessed clinically pre-op using the modified Frailty Index (mFI), an 11-item instrument that counts clinical deficits associated with frailty. A score ≥ 3 was considered frail for this study. A major complication was a Clavien 3b grade or higher. Linear regression, Fisher's exact, and t-test analysis were used to determine the relationship between frailty, outcomes, and length of stay.

Results: Of the 162 patients included in the study, mean age was 74.4 ± 7.2, and 66% of patients were female (n = 128). mFI was > 3 in 37 patients (22.8%). Average length of stay was 2.4 days. Frail patients were not more likely to experience any complication (40.5% vs. 29.6%, p = 0.22) nor a major complication (8.1% vs. 4.8%, p = 0.20). Frail patients were not more likely to be discharged to a facility (8.1% vs. 3.2%, p = 0.20). An increasing mFI did not correlate with an increase in complications, (mFI = 0 (11%), mFI = 1 (32%), mFI = 2 (44%), mFI ≥ 3 (41%), p = 0.67). Frail patients had a longer length of stay (mean = 2.47 ± 0.18, p = 0.03). Functionally impaired patients (METS < 4) were more likely to develop a major complication (17.9% vs. 2.99%, p = 0.008).

Conclusion: Increased frailty as assessed by the mFI is correlated with length of stay following laparoscopic repair of giant PEH in patients > 65. We did not find an association between morbidity and mFI, although the cohort of very frail patients was small. Giant paraesophageal hernias can be safely repaired laparoscopically in patients older than 65 with a low rate of morbidity despite increasing frailty.

S096

Partial Esophagectomy with a Chest Anastomosis is the Preferred Operative Approach Despite the Risk of a Mediastinal Anastomotic Leak

Reza Fazl Alizadeh, MD; Perisa Ruhi-Williams, MD; Michael J Stamos, MD, FACS; Ninh T Nguyen, MD, FACS; University of California, Irvine Medical Center

Introduction: Anastomotic leak (AL) remains one of the most dreaded complications following esophagectomy. Some surgeon elect to perform a total esophagectomy with a cervical anastomosis due to the fear of complications associated with an AL in the chest. Recent advances in leak management such as the use of esophageal stent have minimize the need for reoperation and reduces the mortality associated with a chest or mediastinal leak. The aim of this study was to determine the differences in 1) the leak rate and 2) the need for reoperation and mortality associated with an AL after partial vs total esophagectomy.

Methods and Procedures: The National Surgical Quality Improvement Program targeted esophagectomy files (2016–2020) were used to evaluate clinical data of patients who underwent partial and total esophagectomy operations. Total esophagectomy was defined as esophagectomy with a cervical anastomosis while partial esophagectomy was defined as an esophagectomy with a chest anastomosis. Emergent cases and disseminated cancers were excluded. Multivariate logistic regression model was used to assess the risk factors after adjustment for preoperative demographic and clinical characteristics.

Results: A total of 5,208 patients underwent esophagectomy operation with 67.3% partial and 32.7% total. Of these, 651 (12.5%) developed AL with 11.9% for partial and 13.7% for total esophagectomy. Prolonged operation (AOR 1.80, CI 1.48–2.19, P < 0.05), history of smoking (AOR 1.38, CI 1.11–1.71, P < 0.05), and BMI more than 30 (AOR 1.22, CI 1.17–1.51, P < 0.05) were the most significant risk factors associated with AL. Operation type (partial or total esophagectomy) had no significant association with AL (P = 0.14). For partial esophagectomy with AL, 46.8% required a reoperation while 43.3% required a reoperation for total esophagectomy. The overall 30-day mortality was 2.4% after partial vs. 2.9% after total esophagectomy. The 30-day leak associated mortality was significantly lower after partial esophagectomy (6.9% vs 8.6%, respectively, P < 0.01).

Conclusion(S): Anastomotic leak continues to be a common major complication after esophagectomy. Despite the risk of a chest or mediastinal leak, partial esophagectomy is now the preferred operation with a leak associated 30-day mortality lower than that of total esophagectomy.

S097

Frequency of Anemia in Patients with Paraesophageal Hernias and its Impact on Their Outcomes After Repair

Timothy C Baumgartner, BS; Steven Liu, BS; Wendy Li, MD; Mohammad Kalantar, MD, MPH; Spyridon Giannopoulos, MD; Don Selzer, MD; E Matthew Ritter, MD; Dimitrios Stefanidis, MD, PhD; Indiana University School of Medicine

Introduction: Iron deficiency anemia is a common symptom of paraesophageal hernias (PEH) and may improve after repair. When present, anemia has also been proposed to be associated with an increase in length of hospital stay, morbidity, and mortality after PEH repair. This study aimed to determine the factors related to anemia in patients with PEH, the rate of anemia resolution after PEH repair, and the risk of anemia recurrence when repair failed.

Methods: We included patients who received a PEH repair between June 2019 and June 2020 and had 24 months of postoperative follow-up. Patient age, gender, hernia size, and type (I-IV) were recorded. Anemia was defined as pre-operative hemoglobin values < 12.0 for females and < 13.0 for males, or if patients were actively receiving iron supplementation. Anemia resolution was determined at 6 months post-op and defined as a return to normal hemoglobin levels or cessation of iron supplementation. Postoperative outcomes such as length of hospital stay, morbidity, and mortality were recorded. Logistic regression and ANCOVA were used for binary and continuous outcomes respectively.

Results: Of 359 patients who underwent PEH repair during the study period, 83 (23.2%) had anemia prior to surgery. Patients with anemia had larger hernia sizes vs those without (6.72 cm ± 2.77 vs 4.41 cm ± 2.54; p < 0.001). Patients with types 3 and 4 PEH were more likely to have anemia (OR = 5.1, CI: 2.53–10.32; p < 0.01). Of 70 patients with available data at 6 months after surgery anemia resolved in 23 (33%). Anemia recurred in 9 of these patients (39.1%), 4 of whom also had hernia recurrence (44.4%). After controlling for age, gender, and PEH size, preoperative anemia was associated with a higher length of hospital stay (3.57 ± 1.29 vs 2.30 ± 0.38; p = 0.009) and all-cause mortality (OR = 2.7, CI:1.08–6.57; p = 0.035). Gastropexy and fundoplication type did not lead to the resolution of anemia (p = 0.77).

Conclusion: Anemia occurs in 1 out of 4 patients with PEH and is more frequent in patients with larger hernias. Anemia was also associated with a longer hospital stay after PEH repair and all-cause mortality during the follow-up period. Further, almost half of patients with recurrent hernias had also anemia recurrence. This knowledge can help surgeons better counsel patients and highlight the importance of addressing anemia preop to optimize postop outcomes.

S098

Endoscopic Follow-Up to Assess the Healing Rate and Helicobacter pylori Infection After Simple Closure of Perforated Peptic Ulcer

Warit Rungsrithananon, MD 1; Jadsada Athigakunagorn, MD1; Warapa Kerdmanee, MD2; Woraprat Samart, MD3; Panot Yimcharoen, MD1; 1Bhumibol Adulyadej Hospital; 2Maharaj Nakhon Si Thammarat Hospital; 3Thasala Hospital

Introduction: Endoscopic follow-up to assess the healing of peptic ulcer perforation (PUP) after surgery has not been widely demonstrated in previous literature. This study aimed to evaluate the healing rate and risk factors for unhealed ulcers in PUP patients after simple closure of perforation followed by mandatory H. pylori eradication with the first-line regimen.

Methods: A prospective observational study was carried out in PUP patients treated by simple closure with omental patch and simultaneous intraoperative tissue biopsy to assess H. pylori infection status between January 2016 to July 2022. All patients then received triple therapy regardless of infection status. Data including age, gender, comorbidities, smoking, alcohol consumption, NSAIDs and steroid use, size and location of ulcers were collected. Esophagogastroduodenoscopy with biopsy was scheduled between 4 to 8 weeks after the surgery to assess the healing rate and H. pylori infection after eradication therapy.

Results: In the course of time, 50 out of 128 patients received endoscopic follow-up and were divided into 2 groups: healed and unhealed ulcers. Healed ulcers were observed in 40 patients while unhealed ulcers were observed in 10 patients. Baseline characteristics and mean time to endoscopy were not different between groups (54.6 vs 59.1 days, p = 0.745). Factors significantly associated with healing were perforation size, presence of H. pylori before and after eradication therapy. Mean size of perforation was significantly larger in the unhealed group (0.69 vs 1.3 cm, p = 0.001). Presence of H. pylori infection was observed in 15 out of all patients (30%) both before and after treatment. Pre-treatment infection rate was significantly higher in the unhealed group (22.5% vs 60%, p = 0.048). Even after eradication therapy, H. pylori infection rate was 20% in healed ulcers and 70% in unhealed ulcers (p = 0.004).

Conclusion: Endoscopic follow-up with biopsy in PUP patients after surgery is beneficial for early detection of unhealed ulcers and failure of H. pylori eradication. Size of perforation and H. pylori infection both prior to and post eradication were significant risk factors for unhealed ulcers. Our findings reported ulcer healing rate of 80% and persistent H. pylori infection at 30% after eradication therapy. High incidence of H. pylori infection in both groups after eradication may also suggest a possible resistance to the first-line treatment.

S099

Diaphragmatic Relaxing Incision for Complex Hiatal Reconstruction: Long-Term Outcomes

Sarah C McKay, BS 1; Ahmed Sharata, MD2; Steven R DeMeester, MD3; Melissa L DeSouza, MD3; Daniel Davila Bradley, MD3; Kevin M Reavis, MD3; Christy M Dunst, MD3; 1Albany Medical College, Albany, NY, USA; 2The University of South Florida, Tampa, FL, USA; 3The Oregon Clinic, Portland, OR, USA

Background: Tension has been recognized as a factor in failure of hernia repairs at multiple sites, and over the years relaxing incisions or myofascial release procedures have been advocated for inguinal and ventral hernia repairs. Techniques for relaxing incisions have also been described at the hiatus during paraesophageal hernia (PEH) repair to reduce crural closure tension and allow hiatal reconstruction without mesh bridging of the defect. However, long-term outcomes with these diaphragm relaxing incisions (DRI) have not been reported.

Methods: A retrospective chart review was performed to identify all patients who had a complex hiatal repair with a DRI between August 2016 and September 2021. All DRI defects were repaired with permanent (PTFE) mesh remote from the hiatus and esophagus. Objective follow-up was with chest x-ray, esophagogastroduodenoscopy (EGD) or upper GI series (UGI).

Results: There were 73 patients that had a DRI during complex hiatal repair, 49 primary and 21 reoperations. The DRI was on the right in 63, left in 4, and bilateral in 6 patients. Concomitant Collis gastroplasty was used in 38 patients (52%). A single intra-operative complication occurred where the right crus tore after right DRI. In one patient also with a right DRI, several sutures holding the PTFE patch in place tore leading to herniation of small intestine into the right chest 8 months post-operatively. This was repaired without recurrence. No other complications occurred. No patient developed mesh infection. At a median of 24.5 months (range 8–45 months), 77% of patients had follow-up with chest x-ray, UGI or both. No patient had evidence of diaphragm paralysis or hernia through the DRI. An UGI or EGD for PEH follow-up was available in 45 patients (62%) at a median of 23.5 months. Recurrent hernia was seen in 3 patients (6.7%).

Conclusion: A DRI can be done safely with minimal risk of intra-operative or post-operative complications, and a low rate of herniation through the repair when closed with PTFE mesh. There was no mesh infection despite concomitant Collis gastroplasty in 52% of patients, and no evidence of diaphragm paralysis by chest x-ray or UGI. Further, there was a low rate of PEH recurrence (6.7%) in these patients requiring complex hiatal reconstruction. These excellent outcomes should encourage use of a DRI in patients with excessive crural closure tension or a difficult hiatus during PEH repair.

S100

Delayed Gastric Emptying Impacts Early But Not Long-Term Outcome After Magnetic Sphincter Augmentation

Sven Eriksson; Ping Zheng; Inanc S Sarici; Xinxin Shen; Blair Jobe; Shahin Ayazi; Esophageal Institute, Allegheny Health Network

Introduction: The impact of delayed gastric emptying (DGE) on the outcome of anti-reflux surgery (ARS) is controversial. There is concern that poor gastric motility may worsen postoperative gas-bloat and negatively affect the outcomes. Magnetic sphincter augmentation (MSA) is thought to be associated with less postoperative gas-bloat, but the relationship between DGE and MSA outcomes is unknown. This study aims to evaluate the relationship between objective DGE and MSA outcomes over time.

Methods: A total of 776 patients underwent MSA at our institution between 2013 and 2021. Patients who completed gastric emptying scintigraphy (GES) prior to surgery were selected for this study. DGE was defined as a 4-h retention > 10% or half emptying time > 90 min on GES. GERD-HRQL questionnaires were completed preoperatively, at six-months and annually, postoperatively. Clinically significant heartburn, regurgitation, dysphagia, and gas-bloat were defined as a score > 3 on the symptom-specific items within the GERD-HRQL questionnaire. Outcomes were compared between DGE and normal gastric emptying (NGE) groups at six-months and one-year. A subset of patients with two-year outcomes were also compared.

Results: The final study population consisted of 132 patients (74.7% female) with a median (IQR) age of 55.7 (44.9–62.9) and BMI of 29.1 (26.8–33.1). Of these, 27 (20.5%) had DGE on GES. DGE was associated with higher rate of 90-day hospital readmissions (18.5% vs. 2.9%, p = 0.009).

At six-months patients with DGE had higher median GERD-HRQL total scores [16.0 (9.0–29.0) vs 6.0 (3.0–16.0), p = 0.003] with more frequent gas-bloat (52.9% vs 23.9%, p = 0.035). Heartburn (p = 0.181), regurgitation (p = 1.000), dysphagia (p = 0.416), freedom from PPI (0.422) and satisfaction (70.6% vs 87.7%, p = 0.130) rates were comparable.

At one-year GERD-HRQL scores were similar between groups [10.0 (4.0–27.0) vs 8.0 (3.0–19.0), p = 0.233]. Heartburn (p = 0.582), regurgitation (p = 0.582), dysphagia (p = 0.686), gas-bloat (p = 1.000), freedom from PPI (p = 0.269) and satisfaction (p = 1.000) rates were also comparable. Esophageal acid exposure was normalized in 79.9% of patients with DGE and 75.6% with NGE, p = 1.000.

The subset of 68 (13 with DGE) patients with two-year follow-up had similar GERD-HRQL scores (0.487). Heartburn (p = 0.344), regurgitation (p = 0.471), dysphagia (p = 1.000), gas-bloat (0.676), freedom from PPI (0.661) and satisfaction (91.7% vs 85.2%, p = 1.000) rates were comparable.

Conclusion: Delayed gastric emptying results in higher risk of 90-day readmission and more bloating and diminished symptom improvement within the first six-months after MSA. However, this negative impact dissipates overtime. Continuous improvement in symptoms results in outcomes comparable to patients with normal gastric emptying by one-year that is durable at two-years.

S101

Comparing Oncologic Outcomes of Esophagectomy and Endoscopic Resection in Patients with Early Esophageal Adenocarcinoma

Uzair Jogiat; Hillary Wilson; Alexander Bedard; Pam Blakely; Warren Sun; Jerry Dang; Shahzeer Karmali; Eric Bedard; Clarence Wong; University of Alberta

Introduction: Currently, there is a paucity of data on long term outcomes of endoscopic treatment. Our primary aim was to compare the overall survival and recurrence-free survival (RFS) of patients undergoing endoscopic resection to those undergoing esophagectomy. Secondary aims were to 1) evaluate predictors of a positive margin after initial endoscopic resection and 2) compare patients with endoscopic resection who proceed to esophagectomy to patients who are successfully treated with endoscopic resection.

Methods and Procedures: For the endoscopic resection cohort, a combined retrospective and prospective database was created containing demographic, clinical, and oncologic variables for patients undergoing endoscopic resection for early stage EAC from 2009 to 2021. For the esophagectomy cohort, a pre-existing retrospective database including patients undergoing esophagectomy for esophageal cancer from 2012 to 2018 at a single institution was used. A multivariate cox proportional hazards model was developed for recurrence-free survival and overall survival using a hypothesis driven approach. A kaplan–meier (KM) curve with associated log-rank test was created to evaluate recurrence-free survival and overall survival stratified by treatment modality. A multivariable logistic regression model was used to evaluate predictors of a positive margin necessitating esophagectomy after initial endoscopic resection.

Results: A total of 108 patients were included in the analysis (73 EMR, 35 esophagectomy). Baseline characteristics including age, sex, and co-morbidities were similar among the two groups. KM curves stratified by treatment modality are provided in Fig. 1. Esophagectomy was associated with greater RFS on univariate log-rank test (p = 0.0127), but no difference in OS (p = 0.9306). There was no significant difference between esophagectomy and endoscopic resection in the cox-model for OS (HR 1.03, 95% CI 0.45–2.32, p = 0.914). Endoscopic resection was associated with increased hazards of disease recurrence in the cox model for RFS (HR 2.56, 95% CI 1.1–6.0, p = 0.032). In the logistic regression model, high grade disease (OR 5.43, 95% CI 1.1 – 26.1, p = 0.035) and submucosal involvement (OR 7.8, 1.9–31.4, p = 0.004) were identified as significant predictors of positive margin necessitating esophagectomy after initial endoscopic resection.

Conclusion: In this largely retrospective analysis, esophagectomy provides an increase in recurrence-free survival, but not overall survival compared to endoscopic resection alone. Submucosal involvement and high-grade disease were risk factors for proceeding to esophagectomy after endoscopic resection. Further prospective studies are required to identify high-risk patients who may potentially benefit from upfront esophagectomy.

S102

Retrorectus Insufflation During eTEP Hernia Repair and End Tidal CO2: a Single Institution’s Experience

Ashley Huggins, BS; Cameron Casson, MD; Manjaap Sidhu, MD; Bradley Kushner, MD; Arnab Majumder, MD; Sara Holden, MD; Jeffrey Blatnik, MD; Washington University in St. Louis

Introduction: The enhanced-view totally extra-peritoneal (eTEP) approach has shown to be a safe and durable repair for small to mid-sized ventral hernias. However, its effects on intraoperative respiratory stability, including end-tidal CO2 and tidal volume, are unknown as compared to other robotic approaches, such as a r-TAR. We hypothesize that ETCO2 levels will be higher during eTEP repairs but that it will not have a significant effect on intraoperative and postoperative outcomes. As first of its kind, we present our single institution study of intraoperative respiratory monitoring of hernias repaired with eTEP or r-TAR approaches.

Methods: A retrospective chart review was completed for patients who underwent elective robotic-assisted hernia repair at our quaternary academic center from July 2018 through December 2021. Patients were grouped by type of robotic repair, either eTEP or r-TAR. All patients had ventral hernias, although some patients also had concurrent inguinal or parastomal hernia repair. Baseline demographics, intraoperative anesthesia records, and perioperative outcomes were reviewed. Anesthesia data were collected at intubation and at 30-min time intervals throughout the duration of the case.

Results: We identified 100 patients who underwent an eTEP repair and 97 patients who underwent an r-TAR repair. As expected, the patients in the eTEP population had a lower BMI at surgery (29.3 vs. 32.2, p = 0.02), smaller hernia defects (39.0cm2 vs 143.9cm2, p =  < 0.0001), and fewer medical comorbidities, including diabetes mellitus and immunosuppression use. Intraoperatively, eTEP repairs had significantly higher ETCO2 at the beginning of the case (Intubation: 37.9 mmHg vs. 36.8 mmHg, p = 0.047, Time 2: 40.3 mmHg vs. 36.6 mmHg, p = 0.0001, Time 3: 39.2 mmHg vs. 37.1 mmHg, p = 0.025), a higher peak ETCO2 (45.1 mmHg vs. 42.0 mmHg, p = 0.005), and a shorter time to peak (71.4 min vs. 119.7 min, p = 0.0001) when compared to r-TAR repairs. This difference in ETCO2 dissipated as the case progressed, which corresponded with a significantly higher percent increase in Minute Ventilation (31.5% vs. 18.1%, p = 0.03) in the eTEP procedures. The eTEP group demonstrated shorter OR times (163.9 min vs. 297.7 min, p =  < 0.00001), decreased PCA use (16% vs. 59.8%, p =  < 0.00001), and a shorter length of stay, (0.77 days vs. 2.3 days, p =  < 0.00001). There was no significant difference in postoperative complications, including ICU admission, respiratory distress, or surgical site complications.

Conclusion: As hypothesized, eTEP hernia repairs had higher starting and peak levels of ETCO2 versus r-TAR repairs. However, these intraoperative respiratory changes did not affect patient outcomes. These variations are important to understand and good communication between the surgeon and the anesthesiologist is needed.

S103

Perioperative Outcomes Analysis of Robotic Versus Open Transversus Abdominus Release

Faiz S Khaja, MD; Theresa K Krawiec, MD; Peter Santoro, MD, FACS; Christiana Care

Introduction: Transversus abdominis release (TAR) is a widely accepted approach to abdominal wall reconstruction for complex ventral hernia repair. Historically these have been managed with open surgery, however advancements in minimally invasive surgery have allowed this to often be accomplished robotically. Although there have been few studies comparing robotic TAR (rTAR) to open TAR (oTAR), these demonstrate promising advantages to robotic repair. The purpose of this study is to review outcomes of rTAR and oTAR completed at a single institution by a single surgeon, aiming to add to the current growing body of literature on this topic.

Methods and Procedures: An IRB approved retrospective outcomes analysis was performed evaluating robotic and open TAR. Cases examined were performed by a single surgeon. Data regarding age, gender, BMI, size of mesh, type of repair, robotic console time, total operative time, postoperative length of stay, and complications were analyzed.

Results: Statistical analysis was performed on 103 patients who underwent abdominal wall reconstruction at our institution. Seventy-five consecutive patients undergoing rTAR were compared to 28 matched patients who underwent oTAR. The average age was 59 for both groups. Median length of surgery was 254 min for rTAR compared to 235 min for oTAR; however median robotic console time was 215 min which was significantly shorter than length of surgery for oTAR (p < 0.0001). Median time to discharge was 1.7 days for rTAR compared to 4.6 days for oTAR (p < 0.0001). Median EBL for rTAR was 25 mL compared to 100 mL for oTAR (p < 0.0001). Five out of the 75 patients from the rTAR group had minimal seromas, all of which resolved completely.

Conclusions: Robotic TAR is an increasingly common approach to abdominal wall reconstruction for complex ventral hernia repair, with favorable outcomes and numerous benefits compared to open TAR. Robotic TAR patients had significantly decreased time to discharge and EBL compared to oTAR. Overall length of surgery was slightly longer for rTAR, however when considering console time alone this was significantly shorter compared to length of surgery for oTAR. There were no significant postoperative complications for rTAR, and overall there appeared to be a decrease in console time over the six year study period suggesting continued improvement over time. Overall this data demonstrates that rTAR is a safe option for complex ventral hernia repair offering several advantages in appropriately selected patients.

S104

Retromuscular Drain Output at Removal Does Not Influence Adverse Outcome Rates In Open Ventral Hernia Repairs

Andrea M Meyer, BS, BA 1; Antoinette Hu, MD2; Alexander T Liu, MD2; Diane H Jang, BA1; Rolfy A Perez Holguin, MD2; Colin G Delong, MD2; Eric M Pauli, MD2; Charlotte M Horne, MD2; 1Penn State College of Medicine; 2Penn State Health Milton S. Hershey Medical Center

Background: Surgical drains are commonly used following retromuscular hernia repair (RHR) to decrease postoperative wound complications and help mesh in-growth. Drains are traditionally removed when output is low but the relationship between drain output at the time of removal and postoperative complications has yet to be delineated. The objective of this study is to evaluate outcomes in patients who underwent retromuscular drain removal at either a high or low output volume.

Methods: An institutional review board approved retrospective chart review was conducted on adult patients undergoing open RHR with mesh and at least one retromuscular drain placement between 2013 and 2022 at a single academic medical center. All wound class cases were included, as were parastomal hernia repairs. Subcutaneous drain output was not evaluated. Patients were stratified into low output drainage (LOD, < 50 mL/day) or high output drainage (HOD, ≥ 50 mL/day) groups based on the volume on the day of drain removal.

Results: We identified 334 patients meeting inclusion criteria: 54.4% LOD (n = 182) and 45.5% HOD (n = 152). Age, body mass index, gender, diabetes, current smoking status, hernia size, and type of myofascial release were not statistically different between the groups. Mean drain outputs on day of removal were greater for the HOD group than the LOD group (89 ± 49.7 mL vs 27 ± 11.7 mL, p < 0.01). Drains remained longer in the LOD group than the HOD group (6.2 ± 4.7 days vs 4.7 ± 2.2 days, p < 0.01) despite no difference in the mean length of stay (4.6 days vs 5.06 days, p = 0.6) between the groups. Mean total drain output was higher in the HOD than the LOD group (697 ± 840 mL vs 489 ± 326 mL, p < 0.01). With a 97% 30-day follow up, the rates of surgical site occurrences (SSO; wound erythema, seroma, hematoma) and surgical site infections (SSI) were not statistically significant between the two groups. Similarly, the rate of surgical site occurrences requiring procedural intervention (SSOPI) were not different between the LOD and HOD groups (5.22% vs 3.83%, p = 0.54). At 1-year follow up, hernia recurrence rates were the same between the groups (1.1% LOD vs 0.65% HOD, p = 0.6).

Conclusion: Following open ventral hernia repair with retromuscular mesh placement, there was no statistical difference in the 30 day wound complication rates based on the volume of retromuscular drain output on the day of removal. These results suggest that removing drains earlier despite higher output is safe and has no effect on short or long term hernia repair outcomes.

S105

Quality of Life Gap: Sex Based Differences in Early Outcomes after Ventral Hernia Repair

Desmond T Huynh, MD 1; Ryan Howard, MD, MS2; Anne P Ehlers, MD, MPH2; Sean O’Neill, MD, PHD2; Michael Englesbe, MD2; Justin B Dimick, MD, MPH2; Dana A Telem, MD, MPH2; 1Cedars-Sinai Medical Center; 2University of Michigan

Introduction: Male and female patients receive different hernia care ranging from delays in diagnosis to disparities in operative technique and approach. These differences in care likely matter a great deal to outcomes and highlight the need to address sex as a biological variable in hernia disease. How sex-specific practices impact quality of life following hernia repair remains unexplored. Hence, we used a population level hernia registry to characterize the way in which these differences in ventral hernia repair are experienced by female and male patients.

Methods: Using the Michigan Surgical Quality Collaborative Hernia Registry we compared quality of life outcomes between females and males at 30- and 90-days following surgery. Patients were surveyed routinely at these intervals and assessed for: satisfaction with care (0–10), quality of life (1–5), regret (1–5), surgical site pain (0–10), overall pain (0–10), and number of weeks to return to full activity. Multivariable regression was used to evaluate differences in outcomes between the sexes. To control for confounding, comorbidities as well as demographic, operative, and hernia-specific characteristics were included as explanatory variables.

Results: Survey responses were available for 1074 (47.4%) female and 1194 (52.6%) male patients. Mean age was 54.1 (σ = 14.5) and mean BMI was 32.5 (σ = 7.33). In the perioperative period female sex was significantly associated with lower satisfaction and quality of life as well as greater regret and pain. Thirty days after surgery, female sex was associated with decreased satisfaction and quality of life and increased regret, yet a faster return to activity. At 90 days, female sex was associated with greater regret, surgical site pain, and overall pain. (Table).

Conclusion: Throughout their postoperative courses, females reported a significantly worse experience after ventral hernia repair. In almost all pain and quality of life metrics, females had worse outcomes. Despite this experience female return to activity was faster. This counterintuitive finding may suggest that other environmental or social factors are the impetus for return to activity. A better understanding of sex-specific needs and optimal interventions is needed to address these outcomes disparities.

figure ag

S106

Simultaneous Mesh Placement During Stoma Takedown. An ACHQC Analysis

Megan Melland-Smith; Benjamin Miller, MD; Clayton Petro; Lucus Beffa; David Krpata; Ajita Prabhu; Molly LaBelle; Michael Rosen; Cleveland Clinic Foundation

Introduction: The most appropriate method of reconstructing the abdominal wall at the site of a simultaneous stoma takedown is controversial. The contaminated nature of the field, the concomitant GI procedure being performed and the presence of a hernia at the site all complicate decision making. We sought to describe the surgical approaches used to repair stoma site defects during stoma takedowns in a large hernia registry focusing on mesh type, anatomic location, and early outcomes.

Methods and Procedures: All patients who underwent stoma takedown with simultaneous mesh placement from January 2014 to May 2022 were identified within the Abdominal Core Health Quality Collaborative. Mesh types utilized included permanent synthetic (PS), resorbable synthetic (RS) and biologic. Association of mesh type and location (onlay, retromuscular/preperitoneal, and intraperitoneal) with 30-day wound events and other complications were evaluated.

Results: 436 patients met inclusion criteria. There were 300 patients with PS mesh, 72 with biologic mesh and 64 with RS mesh. Surgeons placed the mesh in the retromuscular preperitoneal position in the majority of cases. Breakdown of mesh type and location are listed in Table 1. All groups had similar preoperative comorbidities. The outcomes of the retromuscular preperitoneal mesh location are described. Mean length of stay was 6.6 days. Surgical site infection (SSI) was 12.9% (32/248). Surgical site occurrence (SSO) was 18.2% (45/248), mostly related to seroma and serous drainage from the wound. Surgical site occurrence requiring procedural intervention (SSOPI) was 14.5% (36/248). Three patients with PS/retromuscular mesh were reported as having infected synthetic mesh and one PS/retromuscular mesh required excision. Four patients with PS/retromuscular developed an enterocutaneous fistula. 30-day re-operation was 3.6% (9/248), including one patient with PS/retromuscular mesh with early recurrence. 30-day readmission was 10.5% (26/248) in the PS/retromuscular group, most commonly related to wound complication.

Table 1 Mesh type and location within the abdominal wall

Location

 

Retromuscular/preperitoneal

Intraperitoneal

Onlay

Mesh type

PS

285

3

12

 

Biologic

47

23

2

 

RS

44

8

12

Conclusion: Our study shows that surgeons utilize multiple types of mesh and prefer the retromuscular/preperitoneal plane for placement of the mesh during stoma takedowns. The 30-day complication rates associated with simultaneous stoma takedown and abdominal wall reconstruction are significant, and more long term evaluation is needed.

S107

Laparoscopic Assisted Rectus Diastasis Approximation (LARDA) in an Asian Population

Lydia Li Yeh Tan, MB, BCh, BAO, Ireland, MRCS, Ed; Sujith Wijerathne, MBBS, Sri, Lanka, MRCS, Ed, MMed, Surge; Davide Lomanto, MD, Rome, PhD, Rome, FAMS, General Sur; National University Health Systems

Introduction: Rectus diastasis is the separation of rectus abdominis muscles, due to thinning or stretching of the linea alba. Primary rectus diastasis occurs after strenuous exercise, while secondary rectus diastasis is seen in women after pregnancy. Classifications of rectus diastasis has been proposed by Reinpold in 2019 [1] and Hernandez in 2021 [2]. As per IEHS guidelines 2019, the goal of surgical management is to reconstruct the linea alba to restitute abdominal wall function and perhaps improve cosmesis. In patients with concomitant ventral hernia, closure of defect prior to intraperitoneal onlay mesh results in less recurrence, seroma formation and bulging [3].

Methods: A retrospective study of all laparoscopic assisted rectus diastasis approximation (LARDA) performed from two tertiary institutions from March 2012 to March 2020 was performed. All patients had concurrent rectus diastasis and ventral hernia repaired laparoscopically. Physical examination using provocative maneuvers were performed to accentuate and measure the hernia and diastasis width and length. Surgery was performed if functional impairment was present with or without cosmetic impairment.

LARDA was performed with transfascial non-absorbable sutures placed via mini stab incisions along the lateral edge of one rectus, under laparoscopic assistance with a suture passer. Pneumoperitoneum was then reduced to approximate the divarication and knots tied extra-corporeally. Subsequently intraperitoneal onlay mesh placed and anchored using transfascial sutures and absorbable tacks. All patients were discharged with abdominal compression binder to be worn for 1 month following surgery.

Results: A total of 105 patients were included in the study. Mean age was 37 years old, and mean BMI 23.3. The mean operation time was 90 min, with mean hernia defect 3.8 cm (ranging between 2-4 cm) and mean diastasis length 13 cm (ranging between 9-18 cm). A composite mesh was utilized in all patients. 25 patients received a 10 × 15 cm mesh while 80 patients received a 15 × 20 cm mesh.

Postoperative VAS score was 2 at 12 h, and mean length of postoperative stay was 2 days (range 1–3 days). Mean follow-up was 11 months and no complications and no recurrences were reported. All patients were satisfied with their cosmetic outcome.

Conclusion: In conclusion, LARDA can improve function and cosmesis in patients with ventral hernia and concurrent rectus diastasis. Our study has shown that LARDA is feasible with short postoperative stay and good outcomes.

S108

Racial and Socioeconomic Disparity of Preoperative Optimization Cut-offs in Ventral Hernia Repair

Khal-Hentz Gabriel; Dan Neal; Mazen R Al-Mansour; University of Florida.

Background: Preoperative optimization cut-offs are frequently utilized to determine eligibility for elective ventral hernia repair. Our objective was to assess the relationship between race and socioeconomic status in relation to preoperative optimization cut-offs.

Methods: We retrospectively queried our institutional database for adults with a diagnosis of ventral hernia between 2016–2021. Demographics, comorbidities and laboratory data were collected and analyzed. We elected to use following cut-offs: body mass index (BMI) of 40 kg/m, glycated hemoglobin (HbA1c) of 8% and smoking status. Socioeconomic status was assessed using the Distressed Communities Index.

Results: A total of 5,640 patients were included (Whites = 4,322, Blacks = 794, Hispanics = 318). 23% of all patients (Whites = 23%, Blacks = 26% and Hispanics = 22%) failed to meet the preoperative optimization cut-offs at the time of initial ventral hernia diagnosis. There was a higher proportion of Blacks (13%) with BMI > 40 kg/m2 compared to Whites (10%) and Hispanics (9%). Hispanics (7%) had the highest proportion of HbA1c > 8% followed by Blacks (6%) and Whites (3%). Whites (10%) had the highest proportion of current smokers followed by Blacks (8%) and Hispanics (6%). As the level of socioeconomic distress increased, there was a corresponding increase in the proportion of patients that failed to meet preoperative optimization cut-offs (16% in prosperous communities to 26% in distressed communities).

Conclusion: Nearly 1 of 4 patients with ventral hernias are affected by commonly used arbitrary preoperative optimization cut-offs. These cut-offs have disproportionate effect on patients of different races and socioeconomic statuses. These disparities need to be considered when implementing preoperative optimization protocols and allocating resources to ensure equitable access to elective ventral hernia repair.

figure ah
figure ai

S109

Surgeon Volumes: Preserving Appropriate Surgical Outcomes in Higher-Risk Patient Populations Undergoing Abdominal Wall Reconstruction

Artem Shmelev, MD1; Molly A Olson, MS2; Jordan O Bray, DO3; Vahagn C Nikolian, MD 3; 1Columbia University; 2Weill Cornell Medicine; 3Oregon Health & Science University

Introduction: It is increasingly recognized that complex abdominal wall reconstruction (cAWR) necessitates specialized training. Though anecdotal evidence suggests that surgical volume would be associated with variations in outcome, no studies have been conducted to assess whether a volume-outcomes relationship is present in cAWR. We sought to determine if outcomes for patients undergoing cAWR varied based on surgeon volume among participants in the Abdominal Core Health Quality Collaborative (ACHQC).

Methods and Procedures: All patients with ventral hernias undergoing elective cAWR with component separation (lateral component release) were selected from ACHQC database. Surgeons were ranked based on annual number of cAWR procedures performed and then group in tertiles (low, medium, high). Patient characteristics, hernia risk factors, operative details, and 30-days outcomes were evaluated.

Results: A total of 9,206 patients were identified, of which 310 (3.4%), 723 (7.9%) and 8,173 (88.7%) AWRs were performed by low (105 surgeons), medium (49) and high-volume (66) surgeons, respectively. Patients operated upon by high-volume surgeons tended to have more comorbidities (all p < 0.001 unless stated otherwise) and higher ASA class (72.5% of class ≥ III, vs 53.5%). Hernia characteristics demonstrated that high-volume surgeons more commonly operated on patients with recurrences (50.2% vs 42%), wider hernias (13.5 cm vs 10.5 cm), associated ostomies (13% vs 3.6%), and histories of surgical site infections (32% vs 26%, p = 0.035). High-volume surgeons more commonly performed transversus abdominis release procedures (92% vs 84%), utilized permanent mesh (92% vs 88%), and placed mesh in sublay position. Intraperitoneal and inlay mesh was more commonly utilized by low-volume surgeons. In spite of operating on more advanced hernias, high-volume surgeons achieved comparable rates (all p > 0.4) of 30-day surgical site infections (SSI: 6.9% vs 7.1%) and surgical site occurrences requiring procedural intervention (SSO-PI: 8.9% vs 10%). Rates of readmissions (7.3% vs 6.2%) and reoperations (2.8% vs 2.9%) were comparable among surgeons as well.

Conclusions: High-volume surgeons maintain comparable outcomes following cAWR despite performing operations on patients with more comorbidities and advanced hernia disease. These findings should be integrated into the debates related to regionalizing abdominal wall reconstruction procedures among high-volume surgeons.

figure aj

S110

Investigation of Optimal Hernia Repair Technique in Patients with Inflammatory Bowel Disease

Nikita Ramanathan, MD, MM; Antonio Lam, DO; William J Selkirk, MD; Aixa M Perez Coulter, MS, MPH; Neal E Seymour, MD, FACS; Georgios Orthopoulos, MD, PhD; Baystate Medical Center, Department of Surgery

Introduction: Patients with inflammatory bowel disease (IBD) are at high lifelong risk of multiple abdominal surgeries. History of multiple laparotomies, immunosuppressive therapy, wound infection, and malnutrition are known risk factors for the development of incisional hernias. Traditionally, the overlay technique has been used to avoid exposure of mesh to intraabdominal organs. With the development of minimally invasive techniques, and various potential mesh placement locations to repair incisional hernias, our objective was to compare the different techniques and their associated complications.

Methods: We performed a retrospective single institution analysis of all adult patients with documented IBD history who underwent incisional hernia repair (midline, parastomal, flank) from 2008–2022. Chi-square test was used to compare the frequency of outcomes, including the type of repair (onlay vs underlay), technique (open, laparoscopic, robotic), and location of mesh placement (intraperitoneal, preperitoneal, retrorectus) regarding intraoperative and postoperative complications.

Results: Eighty-eight patients were included in our cohort. Fifty-two (59.1%) patients were on immunomodulators for IBD. Of the 88 incisional hernias repaired, 30(34.1%) were repaired primarily. 26(29.5%) suffered from hernia recurrence irrespective of the technique utilized. Of the patients who underwent mesh placement, 19(32.8%) had recurrences. More specifically, hernia recurrence occurred in 30% of the patients who underwent onlay repair and 33.3% in patients with underlay repair. Further stratification to intraperitoneal, preperitoneal and retrorectus mesh placement revealed differences in the recurrence rate (35.1% vs 50% vs 14.3%, respectively). Within our cohort, 12 patients presented with postoperative small bowel obstruction (SBO). Seven (58.3%) of these patients had mesh placed, 6(85.7%) of which were in underlay position. Every patient that had SBO after underlay repair had mesh placed intraperitoneally. Postoperative seroma/abscess appeared to develop more frequently in patients that underwent overlay repair compared to those undergoing underlay repair (40% vs 2.13%, p = 0.001).

Conclusion: Mesh placement in an underlay fashion (intraperitoneal, preperitoneal, retrorectus) resulted in lower overall complications compared to overlay, when midline, parastomal and flank hernias were combined. This finding reached the level of statistical significance for the development of postoperative seroma/abscess. There were no statistically significant differences between the postoperative complications that occurred within the underlay groups with different mesh placement locations. However, it is notable that there were lower recurrence rates and no readmissions for small bowel obstruction for patients who underwent retrorectus repair, suggesting a potentially better repair in this patient population. Larger cohort studies are needed to identify the optimal hernia repair technique.

S111

Is Weight Trajectory a Better Marker of Wound Complication Risk Than BMI in Obese Hernia Patients?

Cameron Casson, MD; Jeffrey Blatnik, MD; Arnab Majumder, MD; Sara Holden, MD; Washington University in St. Louis

Objective: Complex ventral hernias are frequently repaired utilizing an open transversus abdominis release (TAR). Obesity is historically one of the strongest predictors of wound morbidity following this procedure. In this study, we aimed to determine if weight loss prior to surgery may still be beneficial in patients with persistently elevated BMIs.

Methods: A retrospective review of obese patients (BMI > 30) who underwent an open TAR at a tertiary academic medical center from 1/2018 to 12/2021 was completed. Demographics, medical history, and perioperative data were analyzed. Weight and BMI were recorded at three time points: > 6 months prior to surgical consultation, initial surgical consultation in our clinic, and time of surgery.

Results: In total, 182 obese patients underwent an open TAR. Twenty-seven patients (14.8%) underwent surgery with a BMI ≥ 40. This group, compared to those with a BMI < 40, did not have any significant differences in wound outcomes, including surgical site occurrences (SSO, 48.1% vs 32.9%, p = 0.13), surgical site infections (SSI, 25.9% vs 23.2% p = 0.76), or surgical site occurrences requiring procedural intervention (SSOPI, 14.8% vs 7.1%, p = 0.18). Similarly, patients with a BMI ≥ 35 (n = 102) did not have any differences in wound outcomes compared to those with BMI < 35 (SSO: p = 0.19, SSI: p = 0.50, SSOPI: p = 0.39). The average timeframe for weight loss was 592 days prior to surgery. On average, the group's weight did not change substantially over this timeframe, however individual patients had great variability in weight loss or gain (average percent weight change of + 0.5%, range -39.9% to + 43.6%). The majority of patients who lost weight (71%) did so independently without additional medical or surgical therapy. Patients who had at least a 3% weight loss (n = 49, 26.9%) had decreased rates of SSI compared to those that did not (12.2% vs 27.8%, p = 0.03), although the groups had similar BMIs at the time of surgery (36.4 vs 36.0, p = 0.51). Patients who only had a 1% weight loss did not see a significant decrease in SSI rate compared to those who did not (20.6% vs 25.4%, p = 0.45).

Conclusion: Percentage of weight loss may be a better indicator of an obese patient's risk for wound morbidity following TAR than BMI alone. There were no differences in wound outcomes based on BMI, however weight loss of at least 3% resulted in fewer SSIs despite similar BMIs at time of surgery. Further research into optimal timing and amount of weight loss, as well as the effects on long-term outcomes, is needed.

S112

Long-Term Outcomes of PGA-TMC Absorbable Scaffold in Both Clean and Contaminated Ventral Hernia Repairs

Estella Y Huang, MD; Ryan C Broderick, MD, FACS; Jonathan Z Li, MD; Rachel R Blitzer, MD; Joslin N Cheverie, MD, FACS; Bryan J Sandler, MD, FACS; Santiago Horgan, MD, FACS; Garth R Jacobsen, MD, FACS; UCSD

Introduction: Biosynthetic meshes afford the cost advantages of being made from fully synthetic material, but are also fully biodegradable, making them good options in the setting of contaminated wounds. The COBRA study examined outcomes after contaminated ventral hernia repair with biosynthetic mesh and found that it performed well with low hernia recurrence and postoperative wound infection rates. The aim of this study is to evaluate the safety profile and long-term outcomes of abdominal hernia repair using GORE BIO-A (PGA-TMC) in all wound classes.

Methods and Materials: A retrospective review identified patients undergoing abdominal hernia repair using PGA-TMC from October 2008 to June 2018. The primary outcome was hernia recurrence rate. Patients with at least 6-month follow-up were included when looking at recurrence rates. Secondary outcomes included 30-day morbidity categorized according to CDC Surgical Site Infection Criteria into surgical site occurrences (SSO), superficial surgical site infection (SSIS), deep surgical site infection (SSID), organ space infection (OSI), and other, return to operating room (RTOR), 30-day readmission, length of stay (LOS), and mortality. Subgroup analyses performed included stratification by recurrence, mesh placement location, wound class (clean versus contaminated), and obesity.

Results: 207 patients met criteria, CDC Wound Classification breakdown was 127 (61.4%), 41 (19.8%), 14 (6.8%), and 25 (12.1%) for I, II, III, and IV, respectively. Median follow-up was 55.4 months (range 0.2–162.4). Mean hernia defect size was 117.1cm2 (range 2.0–600.0). 51 (24.5%) of patients experienced morbidity, 19 (9.2%) had SSOs, 7 (3.4%) SSIS, 16 (7.7%) SSID, and 1 (0.5%) OSI. 172 patients had at least 6-month follow-up, of these 30 (17.4%) developed recurrence. Contaminated cases experienced higher recurrence rates (28.8% versus 10.4%, p = 0.002) at a mean follow-up of 46.9 and 60.8 months for contaminated and clean patients, respectively. Recurrent patients had higher BMI (32.4 versus 28.4 kg/m2, p = 0.0011), larger hernias (162.2 versus 106.7cm2, p = 0.10), higher LOS (11.1 versus 5.6 days, p = 0.0051), and higher RTOR rates (16.7% versus 5.6%, p = 0.053). Retrorectus mesh placement trended lower than onlay in both clean (4.5% versus 15.1%, p = 0.11) and contaminated cases (18.8% versus 30.0%, p = 0.35), but these differences were not statistically significant. BMI ≥ 30 also increased recurrence (28.4% versus 10.5%, p = 0.004), along with RTOR (11.0% versus 3.2%, p = 0.04) and 30-day readmission (19.5% versus 8.0%, p = 0.04).

Conclusion: This study affirms the use of biosynthetic mesh as an effective alternative in all wound classifications, yielding good outcomes, limited long-term complications, and low recurrence rates.

S113

Sex-Based Differences in Inguinal Hernia Development

Sarah E Pivo, MD 1; Desmond Hunyh, MD1; Cherin Oh2; Shirin Towfigh, MD2; 1Cedars Sinai; 2Beverly Hills Hernia Center

Introduction: Most patients with inguinal hernias are male, and much of our knowledge about inguinal hernias is based on the male population. Meanwhile, the development of inguinal hernia may be different based on one's sex. Pediatric literature shows the incidence of bilateral or incarcerated inguinal hernias to be higher in female infants, and genetic predisposition may play a stronger role in females. As there is a dearth of literature examining sex-based differences in hernia development in adults, we aimed to compare the incidence and characteristics of inguinal hernia development in adult females as compared to males.

Methods: An institutional database of patients undergoing repair of primary inguinal hernias was queried for incidence of a standard list of risk factors. These were compared using χ2 and Mann–Whitney U test for categorical and continuous variables. A multivariate analysis was performed including sex and controlling for other demographics to predict number of hernia defects found intraoperatively, controlling for other risk factor variables.

Results: Among 494 patients, 202 (40.9%) were female. The total number of risk factors among females was significantly higher than males (2.78 vs 1.49, p < 0.001). The incidences of constipation, GERD, and asthma were significantly higher among females (Table 1). BMI was significantly lower in females. Females had a similar incidence of family history of hernias to males. The incidence of females with bilateral inguinal hernias (55.6%) and indirect inguinal hernias (92.1%), was similar to males. The incidence of direct hernia (12.9%) in females was significantly less than males, and the incidence of femoral hernias (38.5%) in females was significantly higher than males. Females averaged 1.23 deliveries; pregnancy was not correlated with number of hernia defects found at the time of hernia repair. On multivariable analysis, neither age nor BMI correlated with number of hernia defects.

Conclusions: Females undergoing primary inguinal hernia repair had a greater number of risk factors for inguinal hernia compared to males. However, age, BMI and number of deliveries did not correlate with number of hernia defects. Interestingly, contrary to pediatric literature, we did do not show a higher incidence of bilaterality nor any more significant genetic predisposition in females as noted by family history of hernias. Our study promotes awareness of inguinal hernias in females and presents new data to quantify sex-based differences and predispositions to inguinal hernias.

figure ak

S114

Ambulatory Robotic Colectomy: Factors Affecting Postoperative Opioid Use

Michael M Vu, MD1; Anna Buzadzhi, DO2; Jace J Franko, MD1; Beau Prey, MD 1; Maksim Rusev3; Marta Lavery, MD4; Laila Rashidi2; 1Madigan Army Medical Center; 2Multicare; 3Washington State University; 4Swedish Health Services

Introduction: In the worldwide effort to combat opioid overuse, surgeons play a key role by reducing postoperative narcotic requirements. Ambulatory robotic colectomies are an emerging concept in colorectal surgery, but adequate postoperative pain control remains a pressing concern. Robotic surgery has the potential to reduce opioid needs with improved tissue manipulation, lower required insufflation pressures, and better visualization. We sought to identify key factors affecting recovery room opioid use (ROU) and additional outpatient opioid prescriptions (AOP) after ambulatory robotic colectomies.

Methods: The senior author's ambulatory robotic colon resections between 2019–2022 were retrospectively reviewed. Patients were included if they underwent same-day-discharge (SDD) or stayed overnight. Perioperative factors included age, sex, ASA, diabetes, urgent/emergent case, complexity, and procedure type. Outcomes of interest included ROU (measured in parenteral morphine milligram equivalents, MME), AOP, postoperative emergency department (ED) presentations, and readmissions. Perioperative factors and outcomes were analyzed with chi-square, unpaired t-test, ANOVA, and multivariate regression where appropriate.

Results: Two-hundred-nineteen cases were examined, 48 of which underwent same-day discharge (SDD). Overall, median ROU was 7.5 MME and 8.7% of patients required AOP. Between SDD and overnight-stay patients, there were no differences in postoperative ED presentations, readmissions, recovery opioid use, or additional outpatient opioid scripts. There was a significant difference in ROU between colectomy types, with right-sided colectomies needing the least, and left-sided and total colectomies needing the most opioids in recovery (Fig. 1). Older and higher BMI patients were more likely to have zero ROU (Fig. 2).

Notably, among SDD patients, low ROU was associated with greater need for additional outpatient opioid prescriptions, as well as readmissions (Figs. 3 and 4). These correlations were not present in overnight-stay patients.

Conclusions: Ambulatory robotic colectomies and same-day-discharge can be performed with low opioid use and readmission rates. We identified key factors associated with reduced opioid use. Notably, patients who underwent SDD seemed to have more readmissions and more outpatient opioid prescriptions if they received fewer opioids during recovery. After ambulatory robotic colectomy, optimal pain control in recovery may be crucial in reducing unnecessary readmissions and additional opioid prescriptions.

Table 1

 

SDD

Overnight-stay

p

ROU (MME)

9.53

9.87

0.845

AOP (n,%)

15(8.3%)

4(8.7%)

0.993

ED presentation (n,%)

33(18.8%)

9(19.2%)

0.946

Readmission (n,%)

9(4.2%)

2(5.2%)

0.764

figure al
figure am
figure an
figure ao

S115

Colonic Interposition in Esophagectomy: An ACS-NSQIP Study

Beatrix Hyemin Choi, MD; James Church, MD; Joshua Sonett, MD; P. Ravi Kiran, MD; Columbia University Medical Center

Introduction: For patients with cancer or injury of the esophagus, esophagectomy with reconstruction using the stomach (gastric pull-up, or GPU) or colon (colonic interposition, or CI) can be curative but is associated with high morbidity. We sought to describe the differences in outcomes between the two replacement organs using a national database.

Methods and Procedures: From ACS-NSQIP, patients with CPT codes for esophagectomy with gastric or colonic replacement between 2006 and 2020 were identified. Univariate analysis was performed on postoperative outcomes including length of stay, complications, reoperation, readmission, and mortality. The variables with P ≤ 0.2 were included in the multivariate regression. Primary outcomes were reoperation, 30-day readmission, and 30-day mortality. Data were assessed using Fisher's exact, chi-squared, and logistic regression analyses.

Results: Of 13,047 patients (12,545 GPU, 502 CI), GPU patients were older (63.4 versus 61.3, P = 0.0007), had higher BMI (27.7 versus 26.3, P = 0.00000003) and were more likely to be male (80.3% versus 70.3%, P = 0.00000006) and white (77.8% versus 69.1%, P = 0.000006). More GPU patients had independent functional status (98.4% versus 96.0%, P = 0.00006) and underlying bleeding disorders (4.0% versus 1.8%, P = 0.02) but fewer other preoperative comorbidities than CI patients.

On univariate analysis, CI patients had longer hospital stays (13.4 versus 17.8 days, P = 2.2 × 10–16); serious complications including surgical site infections, wound dehiscence, respiratory failure, renal failure, sepsis, DVT, and anemia (P < 0.05 for all); greater reoperation (14.5% versus 23.9%, P = 0.000000009); lower discharge to home (82.1% versus 70.9%, P = 0.00000005), and a higher mortality rate (2.9% versus 6.0%, P = 0.00003). There was no difference in 30-day readmission between the two groups (11.2% versus 11.6%, P = 0.89).

On multivariate analysis, reoperation was associated with CI, current smoker status, chronic wound infection, hypertension, ASA class 4 or 5, and longer operating time; it was negatively associated with a clean or clean-contaminated wound classification. For readmission, female gender, hypertension, and longer operative time were independently associated. The factors associated with 30-day mortality were age, metastatic cancer, preoperative sepsis, preoperative renal failure, ASA class 4 or 5, longer operative time, and having an underlying malignant neoplasm diagnosis.

Conclusion: CI is associated with higher rates of reoperation than GPU. However, when controlling for other characteristics such as age, BMI, and underlying comorbidities, CI alone is not predictive of higher readmission or mortality rates.

S116

ROLARR Trial Comparison to Single Institution Rectal Cancer Robotic Resection Outcomes

Jetsen A Rodriguez-Silva, MD; Carolia Martinez, MD; Jame R Williams, MD; Jorge Marcet, MD; Allen Chudzinski, MD; Robert D Bennett, MD; Thanh Tran; University of South Florida, Morsani College of Medicine

Introduction: Robotic-assisted surgery for rectal adenocarcinoma has increased in popularity over the last several years. Since the publication of the ROLARR randomized clinical trial in 2017, we have seen a continued rise in the utilization of robotic surgery for rectal cancer. The ROLARR study was a multi-institution study whose primary outcome assessment was the rate of conversion to open between laparoscopic and robotic surgery for rectal cancer. Our institution has utilized robotic-assisted surgery for rectal cancer for over ten years. This study compares a single-institution (SI) robotic surgery experience to the ROLARR robotic arm results.

Methods: SI retrospective cohort analysis from the University of South Florida assessing patients undergoing DaVinci robotic-assisted resection for rectal cancer between 5/2011–12/2018. The primary and secondary endpoints identified in the ROLARR trial were directly compared. The primary outcome was conversion to open laparotomy. Secondary outcomes were circumferential radial margin, TME quality, intra-and post-operative complications, and length of stay.

Results: After exclusions, 128 of 164 patients were analyzed. The mean age was 59, which was younger than the ROLARR robotic arm (59.1 vs. 64.4, p < 0.001). The SI conversion to open rate was significantly lower (2.3% vs. 8.1%, p = 0.036). There was no difference in intra-operative damage to adjacent structure (3.9% vs. 4.7%, p = 0.73), hemorrhage (3.9% vs. 1.7%, p 0.28) or anastomotic complications (4.7% vs. 3.0%, p = 0.39). There was no difference in circumferential radial margin (4.0% vs. 5.1%, p = 0.626) and quality of complete TME (79.7% vs. 75.4%, p = 0.08). Lymph node harvest was lower in the SI group (16.1 vs. 23.2, p < 0.0001). There was no difference in the overall 30d post-operative complication rate and mortality rate (36.7% vs 33.1%, p = 0.48 and 0.8% vs 0.8%, p = 1.0), respectively. Meanwhile, the 30d SSI rate was lower in the SI group, 3.1% vs. 8.9% (p = 0.037), as was LOS (6.8D vs. 8.0D, p = 0.024). The overall 30d-6mos complication rate was not different (18.7 vs. 14.4, p = 0.27).

Discussion: This study supports the use of robotic-assisted surgery for rectal adenocarcinoma. The study conversion to open rate was significantly less compared to the multi-center RCT ROLARR trial. However, surgical pathology outcomes were similar. Although the lymph node harvest in this study was less, it was within the standard of care for adequate staging. A high percentage of patients in this study received neoadjuvant chemoradiation, which may have affected the lymph node harvest. Robotic-assisted surgery for rectal cancer remains a field of ongoing investigation but continues to demonstrate promising outcomes for rectal cancer.

S117

Objective Comparison of Short-Term Surgical Outcomes from Total Mesorectal Excisions Between RAPIDO Protocol, Standard Long Course Radiotherapy and Upfront Surgery: A Prospective Cohort Study in a Single Asian Institution

Hui Lin Tan, MBBS; Frederick Koh, FRCS; Leonard ML Ho, FRCS; Sharmini Su Sivarajah, FRCS; Cheryl XZ Chong, FRCS; Jia Lin Ng, FRCS; Darius KL Aw, FRCS; Jasmine Ladlad, MRCS; Fung Joon Foo, FRCS; Winson JH Tan, FRCS; Sengkang General Hospital

Aim: The advent of the RAPIDO protocol has increased the fidelity and popularity of total neoadjuvant therapy (TNT) in the management of locally advanced rectal cancers. However, technical difficulty of the total meso-rectal excision (TME) after TNT remains unknown. We evaluated the surgical outcomes of 3 groups of patients with rectal cancers – RAPIDO vs standard long course radiotherapy (LCRT) vs upfront surgery to objectively determine the effects of TNT on TME.

Methods: A review of prospectively collected data was performed for patients who have rectal cancer and underwent low anterior resection from January 2016 to May 2022. Data on patient demographics, disease staging, peri-operative details and up to 2-year follow-up outcomes were analysed. The surgical and oncological outcomes were compared. Patients were followed up until 31 May 2022.

Results: A total of 57 patients were included in this review, 36 (63.2%) patients underwent upfront surgery, 9 (15.8%) underwent RAPIDO TNT protocol and 12 (21.1%) underwent LCRT. Forty-two patients (73.7%) were male and median age of patients was 65 years-old (interquartile range: 57–71). Patients who underwent upfront surgery had a statistically higher proportion of smokers (58.3%, p = 0.04) and patients with diabetes (55.6%, p = 0.03). There were significantly more patients with family history of colorectal cancer in the RAPIDO group (33.3%, p = 0.04). Median follow up period for the cohort was 24 months (interquartile range: 14–35). There were no statistical differences in operative times (335 vs 360 vs 286, p = 0.26), with a median duration of 305.5 min (interquartile range: 250.5–364.25). Intra-operative blood loss showed no significant differences (250 vs 200 vs 150, p = 0.31) with a median blood loss of 200 ml (interquartile range:150–200).

Subgroup analysis of the 2 types of neoadjuvant therapy, RAPIDO and LCRT, showed that both had similar peri-operative outcomes of operative time (335 vs 360, p = 0.94) and blood loss (200 vs 150, p = 0.71). There was a significantly higher proportion of patients achieving tumour regression grading of 3 who had underwent RAPIDO compared to LCRT (33% vs 0%, p = 0.02) and no difference in distal (22.2% vs 18.2%, p = 0.82) or local (11.1% vs 18.2%, p = 0.66) recurrence rates.

Conclusion: Based on the objective short-term surgical outcomes, there does not appear to be any significant technical differences in RAPIDO and LCRT patients. However, there was a significantly improved tumour regression seen in the RAPIDO group which may impact the longer-term local and distant recurrence rates.

S118

3D Anatomic Reconstruction for Safe Delivery of Laparoscopic Complete Mesocolic Excision

A Moynihan 1; E Kearns1; MF Khan1; J Dalli1; L Lawler2; RA Cahill1; 1UCD Centre for Precision Surgery, University College Dublin, Ireland; 2Department of Radiology, Mater Misericordiae University Hospital, Dublin, Ireland

Introduction: Complete Mesocolic Excision (CME) has demonstrated superior oncological outcomes compared to standard resection for right colon cancer. However, it is a technically challenging operation and there remains concerns regarding risk of damage to vascular structures during the central vascular ligation (CVL) component of this operation. 3D-reconstructed mesenteric angiogram could assist in improving the safety of this technique by providing a personalised road-map of patient-specific anatomy. We conducted an observational study regarding our learning curve CME/CVL including cases aided by 3D recons studied preoperatively and displayed intraoperatively during each case.

Methodology: We performed a retrospective review of a prospectively maintained database of patients undergoing laparoscopic right hemi-colectomy in our hospital from 2014 to 2022. Baseline characteristics, operative details and post-operative course were recorded. 3D-reconstructed mesenteric angiograms (VisiblePatient, Strasbourg, France) were introduced in 2019. Descriptive and comparative statistics were calculated using SPSS.

Results: During the study time period 33 patients underwent CME, of whom 26 had VP reconstructions performed, while 49 patients underwent non-CME, all for cancers of the right or transverse colon. Mean operative time was longer for the CME group (216 ± 44 min vs 169 ± 53 min. P&lt;0.01). However this difference was only significant for the first quartile of the study. Mean length of the resected specimen was longer in the CME group compared to the non-CME group (268 mm vs 206 mm. p = 0.007). Mean operative time was shorter in the 3D-reconstruction group compared to no 3D reconstruction for CME (214 ± 44 min vs 235 ± 45 min) and length of large bowel was greater in the 3D-reconstruction CME group compared to the non-3D reconstruction CME group (279 ± 102 mm vs 251 ± 66 mm), however these differences were not statistically significant. Intra-operative and peri-operative complications were similar between groups without significant vascular injury in any group. 3D reconstructions matched patient anatomy as determined intraoperatively in every case except one (96% at patient level). In these patients, the ileocolic artery ran posterior to the superior mesenteric vein in 76% of cases with 20% having a right colic artery present as an isolated branch of the superior mesenteric artery running posteriorly to the superior mesenteric vein in 80%.

Conclusion: Significant variation is seen in the vascular anatomy of the right colon and 3D mesenteric reconstruction can effectively identify this. Such 3D-reconstructions may aid safe adoption of CME/CVL in units seeking this step-advance.

S119

Perioperative Outcomes for Single-Port Robotic Versus Single-Incision Laparoscopic Surgery: A Comparative Analysis in Colorectal Cancer

Deborah S Keller, MS, MD; Taylor P Ikner, BA; Hela Saidi, BS; Henry Schoonyoung; John H Marks, MD; Lankenau Medical Center, Marks Colorectal Surgical Associates

Background: Single-incision laparoscopic surgery (SILS) is reported to offer improved cosmesis, reduced postoperative pain and faster recovery after colorectal surgery. However, widespread implementation was limited by its technical demands. The single-port robot (SPR) was recently introduced, assuming the robotic ergonomics and advanced vision would further enhance the SILS benefits. No study to date has compared the 2 platforms to validate benefits, though. Our practice has the largest experience with this new technology for colorectal cancer (CRC), and continues to evaluate advantages early in its adoption curve. The goal of this study is to compare the perioperative outcomes of SPR and SILS for CRC.

Methods: A review of Stage II and III CRC patients who underwent curative surgery through a SILS or SPR approach at a single tertiary referral center from 2010–2022 was performed. Baseline demographic, clinical, tumor-specific characteristics and perioperative outcomes were compared using χ2, t-tests and Mann–Whitney U-tests in a 1:1 propensity score-matched cohort, adjusting for baseline characteristics. The main outcome measures were complications, operative time, and oncologic quality measures.

Results: Matching resulted 50 SPR and 50 SILS patients. The groups had similar mean age, gender, body mass index, comorbidity, diagnosis, rates of prior abdominal surgery and neoadjuvant chemoradiation, and procedure performed. The SPR had similar operative times to SILS, with a smaller size of the largest incision (1.000 vs. 2.988, p = 0.049). The rate of intraoperative and postoperative complications was comparable. The intraoperative conversion rate, total mesorectal excision status, distal margins, and lymph node yield were similar across groups. The proximal margin was significantly longer in the SPR than SILS group (23.875 vs. 15.935 cm, p = 0.034). There were no positive margins in either group. Postoperatively, the groups had analogous day of return of bowel function, comparable morbidity, and discharge destination. The length of stay was significantly shorter with the SPR than SILS (4.135 vs. 5.282 days, p = 0.045).

Conclusions: This SPR provided high quality oncologic surgery, adding the technical benefits of the robotic platform to clinical and cosmetic benefits of single-port surgery. There were comparable operative time, postoperative and oncologic outcomes in colorectal cancer cases with the SPR to SILS, with shorter hospital stays. This early data is encouraging for expansion of the new SPR technology.

figure ap

S120

The Robotic NICE Procedure versus Conventional Laparoscopic Left-sided Colorectal Resection: Results of a Matched Cohort Analysis

Thais Reif de Paula, MD1; Roberto Secchi del Rio, MD 2; Daniel Margain, MD2; Jose Ortiz de Elguea, MD3; Stefano Presacco, MD4; Jean Paul Lefave, FACS, FASCRS2; Eric M Haas, FACS, FASCRS2; 1Houston Colon, Houston TX; 2Houston Colon PLLC, Houston, TX; 3The University of Texas Medical Branch; 4University of Houston College of Medicine, Houston, TX

Introduction: We introduced the Robotic NICE Procedure for left-sided colorectal resection in 2018 in which the necessity of an abdominal wall incision was obviated by performing a transrectal extraction of the specimen followed by an intracorporeal anastomosis. We compare the results of the NICE procedure versus conventional laparoscopic resection, which was our standard approach prior to 2018.

Methods: A matched pair case–control study compared patients following the NICE Procedure versus those who underwent laparoscopic left-sided colorectal resection. Cases were performed at an Academic Medical Center and recorded in a prospective database to analyze perioperative outcomes.

Results: Total of 83 patients were matched in each group. There were no significant differences in age, sex, body mass index, ASA, diagnosis, or type of surgery. Operative time, blood loss, intraoperative complications, and conversion rates were similar in both groups. The NICE procedure was associated with significantly lower opioid use, earlier return of bowel function, and shorter length of stay. Overall, there were no differences in post-operative complications.

Conclusion: When compared to conventional laparoscopic resection, the NICE procedure is associated with short-term benefits including less opioid use and earlier recovery without any increased risk of complications. Multicenter studies are recommended to validate benefits and limitations of this technique.

Table 1 Postoperative outcomes

 

Lap EAS n = 83 (%)

Robotic NICE n = 83 (%)

p-value

Bowel function return, in hours mean ± SD

40.7 ± 15.7

23.6 ± 15.6

0.005

LOS in days, mean ± SD

3 ± 2.1

2.2 ± 1

 < 0.001

Overall postoperative complications

19(22.9)

14(16.9)

0.331

Postoperative Ileus

6(7.2)

1(1.2)

0.117

Surgical site infection

2(2.4)

3(3.6)

0.317

30-day readmissions

5(6)

6(7.2)

0.755

30-day reoperations

0(0)

4(4.8)

0.120

30-day mortality

0(0)

0(0)

 

Morphine Milligram Equivalents

   

Intraoperative

33.5

37.8

0.120

PACU

25.2

21.7

0.317

Postoperative

36.2

11

 < 0.001

Total

94.6

70.5

0.010

SD Standard Deviation, significant p-values in bold

S121

Rural Versus Urban Commuting Patients with Stage III Colon Cancer: Is There a Difference in Treatment and Outcome?

Michaela Gaffley, MD 1; Tingting Li, PhD2; Mei-Chin Hsieh, PhD2; Yong Yi, PhD2; John Gibbs, MD2; Xiao-Cheng Wu, MD, MPH2; Joseph Gallagher, MD1; Quyen D Chu, MD, MBA3; 1Colorectal Surgery, Orlando Health; 2Louisiana State University Health Sciences Center; 3Surgical Oncology, Orlando Health

Introduction: To evaluate if there are differences in outcomes for patients with stage III colon cancer in those from urban vs. rural commuting areas.

Methods and Procedures: We analyzed data on patients diagnosed with stage III colon cancer between 2011 and 2018 from the Louisiana Tumor Registry, an NCI-SEER and CDC-NPCR member. Patients were classified to be metro (urban) vs. nonmetropolitan (rural) on a continuum based on population size, urbanization, and adjacency to metro areas. Data on race/ethnicity, age, stage, primary tumor site, extent of surgery, surgical margins and timing of surgery and chemotherapy, hospital volume, and survival were extracted. Kaplan–Meier methods were utilized for survival outcomes, and multivariate Cox analysis of outcomes were adjusted for rural/urban residence, year of diagnosis, poverty level, nodal status, surgical margins, and facility volume.

Results: Of 2,060 patients identified, 1707 were urban (82.9%) and 353 rural (17.1%). No age difference between rural and urban patients (p = 0.56). Stage IIIB accounted for 65.2%, followed by IIIC (23.3%) and IIIA (22.5%), with no difference between rural and urban on stage (p = 0.08). No stage difference in the location of primary tumor, with the majority located in the sigmoid, followed by in cecum and right colon (p = 0.99). There was no stage difference in extent of surgery (p = 0.12) or tumor size (p = 0.42) between urban and rural settings. However, there was a statistically significant difference in chemotherapy, with 68.3% of urban patients receiving chemotherapy vs. 60.9% of rural (p = 0.03). There was a statistically significant difference in treatment of rural vs. urban patients based on hospital volume, time from diagnosis to surgery, and time from surgery to adjuvant chemotherapy (p < 0.0001). No difference in Kaplan Meier survival curves comparing rural versus urban patients who had surgery (p = 0.77) or those who had surgery and adjuvant chemotherapy (p = 0.72).

Conclusions: There was a statistically significant difference in chemotherapy and treatment delay between urban and rural community patients. There was no difference in time to surgery, time to adjuvant chemotherapy, extent of surgical resection, or survival curves between rural and urban commuting patients undergoing treatment for stage III colon cancer.

S122

Defining Risk Factors for Clostridium Difficile Infection After Ileostomy Reversal

Heather Carmichael, MD; Samuel H Lai, MD; Ana Gleisner, MD, PhD; Jon Vogel, MD; Elisa Birnbaum, MD; Brandon Chapman, MD; University of Colorado

Introduction: Patients undergoing ileostomy closure are at increased risk of Clostridium difficile infection (CDI) compared to other elective colorectal procedures, occurring in up to 6% of patients. It is hypothesized that changes in the gut microbiome in the setting of fecal diversion may result in susceptibility to infection postoperatively. This aim of this study was to examine risk factors for CDI following ileostomy reversal.

Methods and Procedures: This was a retrospective review of adult patients undergoing ileostomy closure at an urban tertiary medical center between January 1, 2020 and July 1, 2022. A postoperative CDI was defined as a positive stool PCR test within 30 days of ileostomy closure. Demographics, details of prior surgery including extent of prior colon resection, and duration since fecal diversion were abstracted from the medical record.

Results: Among 143 cases of ileostomy closure identified, 17 patients (11.9%) underwent testing postoperatively and 7 patients (4.9%) were positive. CDI was diagnosed a median of 8 days [IQR 6–20] following ileostomy closure and the majority were diagnosed after hospital discharge (n = 4, 57%). There were no significant differences in patient age, sex, race/ethnicity, underlying diagnosis including inflammatory bowel disease, or history of prior CDI between patients who did or did not develop infection postoperatively (all p > 0.05). Ileostomy diversion without bowel resection was associated with an increased risk of CDI compared to ileostomy diversion with bowel resection (p = 0.001). A longer duration of fecal diversion was associated with an increased risk of CDI (344 days vs. 75 days, p = 0.006). Although not statistically significant, an end ileostomy configuration was more common in patients who developed a CDI compared to a loop ileostomy (29.6% vs. 4.4%, p = 0.06).

Conclusions: The incidence of CDI following ileostomy closure in this study was 5%. Patients undergoing fecal diversion without concomitant bowel resection and with prolonged duration of diversion were at increased risk for CDI. Further studies are needed to evaluate the impact of these risk factors on the gut microbiome. Prophylactic treatment could be considered for patients who have been diverted for a prolonged period of time.

figure aq

S123

Comparison Between Retroperitoneal Approach by Robotic Right Hemicolectomy and Pincer Approach by Laparoscopic Right Hemicolectomy with Japanese D3 Lymph Node Dissection

Takeshi Yanagita, MD; Hiroki Takahashi, MD, PhD; Kaori Watanabe; Takuya Suzuki; Hajime Ushigome; Kazuyoshi Shiga; Ryo Ogawa; Yoichi Matsuo, MD, PhD; Shuji Takiguchi, MD, PhD; Nagoya City University

Background: Since robotic colectomy was covered by insurance in April 2022 in Japan, we have now two approaches for colon cancer; robotic or laparoscopic colectomy. In our hospital, "a pincer approach" in which a cranial approach precedes retroperitoneal (caudal) approach is preferred to perform in laparoscopic right hemicolectomy, whereas retroperitoneal approach alone is preferred in a robotic procedure from the viewpoint of making a fine surgical field and efficient maneuver by robotic arms.

Patients and Methods: 229 patients with right-sided colon cancer who were performed with ileocecal resection (ICR) or right hemicolectomy (RHC) with D3 lymph node dissection were reviewed retrospectively. As for Japanese D3 dissection, mesentery was excised from the left boarder of superior mesenteric vein and ileocolic artery/vein (and right branch of middle colic artery/vein if necessary) are ligated at their roots. Inversed probability weighting was used for evaluating the quality and safety of both robotic and laparoscopic procedures.

Result: From January 2018 to August 2022, 25 patients were performed with robotic ICR/RHC, whereas 204 patients with laparoscopic ICR/RHC. Operative duration was 35.4 min longer in robotic procedures than in laparoscopy. The amount of blood loss was smaller in robotic ones than in laparoscopy by 4.1 ml. The number of dissected lymph node was larger in robotic ICR/RHC than laparoscopic ones by 2.37. Complications greater than clavien-dindo classification grade 3, anastomotic leakage, postoperative ileus were reduced in robotic procedure, respectively. Superficial incisional surgical site infection (SSI) and deep incisional SSI were slightly increased in robotic procedures.

Discussion: As several studies revealed higher quality and less complication in robotic surgery compared with laparoscopy, our study showed the same results, even though several change in maneuver of mobilization of right-sided colon and the same technique in D3 lymph node dissection between robotic procedures and laparoscopic ones. Robotic ICR/RHC may be better in terms of quality of lymph node dissection and prevention of bleeding intraoperatively. Postoperative complications such as anastomotic leakage, postoperative ileus, and space/organ SSI were reduced in robotic ICR/RHC. On the other hand, superficial and deep incisional SSI were increased in robot. This might be because pressure on tissues around port sites is higher in robotic surgery than in laparoscopic one. This study has several limitations: small number of robotic cases, retrospective study, and so on. We need further prospective study in the future.

S124

Laparoscopic Management of IATROGENIC Colonoscopic Perforation

Boaz Loberman, MD; Gideon Sroka, MD, MSc; Eviatar Kuhnreich, MD; Ibrahim Matter, MD; Bnai Zion Medical Center

Introduction: Iatrogenic colonoscopic perforation (ICP) is a severe complication and is associated with significant morbidity and mortality. The global estimated incidence of ICP is 0.8% and 8% for diagnostic and therapeutic colonoscopies respectively. Treatment options include endoscopic repair, conservative therapy, and surgery. Treatment decision is based on the time and the setting of the diagnosis, the type and location of the perforation, the presence of related pathologies, the clinical status and characteristic of the patient, and surgeon's skills. We present our experience in the treatment of ICPs.

Methods: A retrospective review was undertaken of all patients suffered from ICP at Bnai-Zion medical center between 1/1/2010 and 1/3/2021. Clinical presentation, therapeutic approach and short-term outcomes were analyzed.

Results: There were 48 cases of ICPs. 14(29%) were diagnosed by the gastroenterologist during the procedure, 2 of whom were treated with endoscopic clips. The rest the patients (71%) were diagnosed in the ER after a CT scan. Forty patients (83%) went on to operative management: 5 (12.5%) operations started with laparotomy – All were conducted in the early study period (until 2013). All other operations (87.5%) started with a diagnostic laparoscopy, 4 of whom (11%) were converted to laparotomy. Out of the 35 laparoscopic cases 28(80%) were treated with primary suturing. 6 patients went on to colon resection (4 of whom with primary anastomosis). 6 patients required ICU admission – with 1/35(2%) from the laparoscopic cases, and 5/9(55%) from the laparotomy cases. 46/48 patients fully recovered and were discharged. Length of stay (LOS) for operated patients was 6 ± 2 days for the laparoscopic procedures and 20 ± 5 days for the laparotomy procedures.

Conclusion: Laparoscopic treatment of ICP is safe and feasible in most cases. Our data supports a laparoscopic attempt at any such scenario.

S125

Antiperistaltic Anastomosis with Partial Stomach Partitioning is the Optimal Surgical Approach for Palliative Gastrojejunostomy for Malignant Gastric Outlet Obstruction

Siyuan Yao, MD, PhD; Kana Ishikawa; Takashi Kumode; Keisuke Tanino; Ryosuke Mizuno; Yugo Matsui; Shusaku Honma; Teppei Murakami; Takatsugu Kan; Sanae Nakajima; Takehisa Harada; Kobe City Medical Center West Hospital

figure ar

Background: Gastrojejunostomy (GJ) is a surgical option for malignant gastric outlet obstruction (mGOO) caused by gastric cancer, pancreatic cancer, and other types of cancer. However, the optimal surgical approach remains unknown, as surgical techniques vary from facility to facility. Isoperistaltic or antiperistaltic anastomosis, and whether partial stomach partitioning (PSP) should be added are still unsolved clinical questions.

Methods and Procedures: This multicenter retrospective review included 108 patients who had undergone GJ for mGOO from two medical centers in Japan. The medical records were reviewed including background characteristics and surgical complications. The rates of delayed gastric emptying (DGE) and aspiration pneumonia were compared between different anastomotic procedures. The predictive factors for DGE were also assessed with the multivariable logistic regression model.

Results: There were 62 men (57.4%) and 46 women (42.6%) with a median age of 73 years (range, 42–91). The causes of obstruction were gastric cancer (40.7%), pancreatic cancer (33.3%), hepatobiliary cancer (9.3%), lymphoma (1.9%), and metastasis or recurrence from other origins (14.8%). Laparoscopic GJs were performed in 69 (63.9%) patients. The incidences of DGE and aspiration pneumonia were 12.0% (n = 13) and 5.6% (n = 6), respectively. In-hospital mortality was observed in four patients (3.7%): two for aspiration pneumonia and two for cancer progression. With regard to the surgical procedure, antiperistaltic anastomosis with PSP yielded the lowest occurrence rate of DGE (3.4%) and aspiration pneumonia (1.7%) (Figure). The multivariable analyses identified antiperistaltic anastomosis with PSP (OR, 0.122; 95% CI, 0.024–0.613; P = 0.011) and age ≥ 80 years (OR, 6.820; 95% CI, 1.847–25.183; P = 0.004) as independent predictors for DGE.

Conclusions: Antiperistaltic anastomosis with PSP should be employed for palliative GJ to reduce the surgical complications.

S126

Readability of Spanish-Language Bariatric Surgery Patient Education Materials Available in the United States

George Padilla, BS 1; Sabrina Awshah, BS, BA1; Joseph A Sujka, MD2; Salvatore Docimo, DO2; 1University of South Florida; 2Tampa General Hospital

Introduction: The Hispanic population in the United States is the fastest growing ethnic minority, contributing to nearly half of the population growth over the last decade. It is known that this population has lower than average health literacy rates, and that lower health literacy leads to poorer health outcomes. Because of this, the American Medical Association and National Institutes of Health recommend patient education materials (PEMs) be written at no higher than a 6th grade reading level. Given that US Hispanic adults are the racial/ethnic group with the second-highest obesity prevalence, this study aims to analyze the readability of Spanish-language PEMs regarding bariatric surgery available in US-based academic and medical centers.

Methods and Procedures: A total of 25 patient education materials (PEMs) were found via the query ""cirugía de pérdida de peso" site:(.edu OR.org)" on the Google search engine and subsequent inclusion of all US-based academic or medical center websites. Text that contained information regarding the indications for bariatric surgery, descriptions of the types of bariatric surgery, what to expect before and after surgery, and the risks and benefits of bariatric surgery was included. The excerpts were analyzed according to three readability formulas designed specifically to evaluate Spanish-language:

(1) Fernandez-Huerta Formula: Difficulty = 206.84 − (0.6 × Total Number of Syllables in 100 words) − 1.02 × (Total Number of Sentences in 100 words).

(2) Spaulding Formula: Difficulty = 1.609 (Average Sentence Length) + 331.8 (Density*) + 22.0

(3) Fry Readability Graph corrected for Spanish:x-axis = # syllables / 100 words – 67; y-axis = # sentences / 100 words

* Density refers to the percentage of words in a passage not present in "Children's Spanish Word List" (Fountain-Chambers, 1982).

Each excerpt was evaluated for its reading grade level.

Results: All 25 sources were at the college reading level per the Fry graph corrected for Spanish. Per the Spaulding formula, 24 sources were "Grade 12 + " and one source was "Grade 8–10". Per the Fernandez-Huerta formula, 10 sources were at the 8th/9th grade reading level & 15 sources were at the 7th grade reading level.

Conclusions: The Spanish-language bariatric surgery PEMs available online from US-based academic and medical centers are above the recommended sixth-grade reading level. Failure to meet the recommend sixth-grade reading level decreases health care literacy for Spanish speaking patients within the United States seeking bariatric surgery.

S127

Continued Fallout from the Opioid Epidemic: The Environmental Impact and Associated Health Effects of Excess Opioids Prescribed Following Inguinal Hernia Repairs

Joseph M Blankush, MD; Joel F Bradley, MD; Meredith C Duke, MD; Richard A Pierce, MD; Michael D Holzman, MD; Joseph R Broucek, MD; Vanderbilt University Medical Center

Introduction: Deaths from prescription opioid overdose have quadrupled since 2000, now nearing 17,000 annually. Recent studies demonstrated adequate analgesia in patients prescribed zero opioids following inguinal hernia repair. Still, 91% of surgical patients are prescribed opioid analgesics, and 92% report excess pills. Simultaneously, the Lancet Commission on Health and Climate Change declared, “Climate change is the biggest global health threat of the 21st Century.” Climate impact resulting from U.S. health care delivery, 70% of which stems from its massive supply chain, is directly responsible for a loss of 614,000 disability adjusted life-years (DALYs) annually. While social and economic impacts of opioid excess are well-documented, little is known regarding the environmental impact of excess opioid production and resulting impact to human life.

Methods and Procedures: Opioid prescriptions prescribed to opioid-naïve patients following 25 consecutive cases each of open, laparoscopic, and robotic inguinal hernia repair from March-September 2022 were reviewed. Institutional trends were benchmarked against recent meta-analyses. Point-of-sale prescription cost was set at the lesser of average wholesale price minus 14% or federal upper limit queried from Micromedex (IBM). Environmental impacts associated with raw materials, production, transportation, and distribution of opioid medications were analyzed employing Economic Input–Output Life-Cycle Assessment methodologies (EIO-LCA database: www.eio-lca.net).

Results: 99% of patients were prescribed post-operative opioids. There was no significant difference between MMEs prescribed for open (mean 75, range 37.5–90), laparoscopic (74,37.5–90), or robotic (66,0–90) procedures (p = 0.45). All prescriptions were in excess of average MME utilization of 10 and 15 MMEs per MIS and open repair, respectively, but well below published prescribing averages of 150 MMEs per inguinal hernia repair. Average cost per MME was $0.077, for an average of $3.87 excess per prescription. Excess prescribed opioids were associated with an incremental 1.2 kg of greenhouse gas emissions per hernia repair. Each excess opioid prescription requires an additional 1 ton of raw materials and products transported 4 km (ton-km) and begets a loss of 5.4 × 10^-6 DALYs. Extrapolated across 800,000 inguinal hernia repairs performed annually in the U.S., excess opioid prescriptions produce an excess 1,400 metric tons of CO2e emissions, 9.2 million ton-kilometers in global transportation, energy expenditure of 6,900 barrel of oil equivalents, and 12 life years lost.

Conclusion: Downstream deleterious effects of excess opioid prescribing behaviors persist and extend to adverse health impacts secondary to the climate change associated with production, transportation, and sale of opioid medications.

S128

Factors Influencing Surgeon Well-Being: Exploring the Joy of Surgery

Dorothy Hughes, PhD, MHSA1; Melissa N Hanson, MD, FRCSC 2; Adnan Alseidi, MD, MEd, FACS3; John Romanelli, MD, FACS4; Melina Vassiliou, MD, FRCSC2; Horacio Asbun, MD, FACS5; 1University of Kansas School of Medicine- Salina; 2McGill University Health Centre; 3University of California San Francisco; 4University of Massachusetts Chan Medical School—Baystate Medical Center; 5Miami Cancer Institute

Introduction: There has been considerable research into surgeons' burnout but less into how surgeons thrive and what drives joy. The purpose of this study, conducted by the SAGES Reimagining the Practice of Surgery Task Force (RPS), was to explore factors influencing surgeon well-being with the eventual goal of translating these findings into tangible individual-, practice-, and system-level changes to help restore the joy in surgery.

Methods and Procedures: This was a qualitative, descriptive study. From a list of potential interviewees generated by RPS members, we utilized purposive sampling to ensure representation of different age groups, genders, ethnic backgrounds, practice types, and geographies. Semi-structured interviews were conducted on Zoom, recorded, and professionally transcribed verbatim. We coded inductively, finalized the codebook by consensus, then organized codes into a thematic network: basic, organizing, and global themes. The global themes formed our conclusions; organizing themes gave additional detail. Analysis was facilitated by NVivo (QSR International).

Results: We interviewed 17 surgeons from the United States and Canada. 8 (47.1%) identified as women and 9 (52.9%) as men. 8 identified as non-white and 9 as white. All career stages were equally represented. Total interview time was 15 h. Our global themes (organizing themes in parentheses) were:

  • Values (Independence, Knowledge, Patients)

  • Stressors (Administration, Work-Life Integration, Time/Productivity Pressures, and Operating Room Factors)

  • Satisfaction (Autonomy, Challenge, Leadership, Respect and Recognition, and Service)

  • Support (Leaders, Institutions, Team, and Personal)

  • Suggestions (Individual, Practice, and System)

Values, Stressors, and Satisfaction influenced perspectives on Support. Experiences of Support shaped Suggestions. All participants reported stressors and satisfiers, depicting varied experiences. Stressors in particular varied by age and gender. Surgeons at all stages enjoyed operating and being of service to their communities. While Supports and Suggestions included compensation and infrastructure, overwhelmingly human resources were most critical. To experience joy, surgeons needed high-functioning teams in clinics and operating rooms, good leaders/mentors, and supportive family/social networks.

Conclusions: Our results indicate organizations could 1) better understand values driving surgeons, like autonomy and independence; 2) provide surgeons more time for satisfiers, like patient relationship-building; 3) minimize stressors, like time and financial pressures; and 4) at all organizational levels focus on 4a) building teams and leaders and 4b) giving surgeons the time and space to develop healthy family/social lives. Next steps include developing a survey tool to be used at individual institutions to build "joy improvement plans" and in surgical associations to inform advocacy efforts.

S129

Postoperative Gastrointestinal Motility After Neuromuscular Blockade Reversal With Sugammadex Versus Cholinesterase Inhibitors in Patients Undergoing Gastrointestinal Surgery; A Systematic Review and Meta-Analysis

Sahil Sharma 1; Tyler McKechnie1; Gaurav Talwar1; Janhavi Patel1; Luke Heimann2; Aristithes Doumouras1; Dennis Hong1; Cagla Eskicioglu1; 1McMaster University; 2Liberty University, Pennsylvania

Cholinesterase inhibitors have historically been used for reversal of neuromuscular blockade following gastrointestinal surgery. Sugammadex, a γ-cyclodextrin molecule is a novel reversal agent associated with improved postoperative bowel function. A comparative review between these agents on bowel function following gastrointestinal surgery is currently lacking. Our study aimed to systematically review the effects of sugammadex on postoperative ileus, nausea and vomiting, compared to cholinesterase inhibitors following gastrointestinal surgery.

Comprehensive search of MEDLINE, EMBASE, CENTRAL was performed from database inception to July 2022. Articles were included if they compared sugammadex with cholinesterase inhibitors in patients undergoing gastrointestinal surgery in terms of postoperative bowel function. Secondary endpoints included length of hospital stay, readmission, and postoperative morbidity. Inverse variable random effects meta analyses were performed.

Following screening of relevant articles, 2 randomized controlled trials and 3 retrospective cohorts met inclusion criteria. Overall, 717 patients were in the sugammadex group (mean age 59 ± 13 years, 53.4% female) and 692 patients were in the cholinesterase inhibitors group (mean age 59 ± 14 years, 50% female). Significantly lower rates of postoperative ileus (OR 0.44, 95% CI = 0.25, 0.77, p < 0.05) and morbidity (OR 0.72, 95% CI = 0.58, 0.91, p < 0.05) were seen with sugammadex. No significant difference in postoperative nausea and vomiting (OR 0.58, 95% CI = 0.31, 1.09, p = 0.09), readmission (OR 0.92, 95% CI = 0.57, 1.5, p = 0.74), or length of stay (mean difference -0.01, 95% CI = -0.38,0.37, p = 0.97) was observed.

The use of sugammadex for reversal of neuromuscular blockade following gastrointestinal surgery is significantly associated with lower rates of postoperative ileus and morbidity compared to cholinesterase inhibitors. However, these do not translate into a significant reduction in length of stay or readmission rates. Our results were limited due to the number of studies included and missing data points. Therefore, future prospective studies are needed to delineate the role of sugammadex in recovery after gastrointestinal surgery.

S130

It’s not the Arrow, It’s the Archer: The Role of the Surgeon Leader in a Safety-Driven Era

Hillary Lia, BCompH; Melanie H Mobilio, MA; Frank Rudzicz, PhD; Carol-anne Moulton, MD, PhD; University of Toronto

Introduction: The purpose of this study is to examine the role of the surgeon as leader of the operating room (OR) team. In an era where team communication and patient safety are paramount, standardized tools have been deemed critical to safe, efficient practice. In some cases – perhaps most notably in the surgical safety checklist (SSC) – these tools have been elevated as the key to safe patient care. However, an in-depth study suggests that the way the SSC is used by OR staff may explain its variation in efficacy. We explore the role and impact of the surgeon leader in a team environment deeply shaped by pre-existing hierarchies, professional boundaries, and institutionally mandated standardized safety practices.

Methods: Using a constructivist grounded theory approach, OR staff members (surgeons, anesthetists, nurses and perfusionists) were purposively sampled. Semi-structured interviews explored participant experiences working in the OR, focusing on the role and impact of the surgeon as leader. Data collection and analysis were iterative, where each interview was analysed and discussed in relation to the others. This process identified emerging topics and themes which were explored in subsequent interviews.

Results: Preliminary results suggest that surgeon leaders play a key role in fostering optimal team performance by shaping team psychological safety and facilitating a shared mental model. While standardized tools may guide team communication at particular moments, the behaviours of surgeon leaders were viewed as critical to team communication and cohesion. “I think that the surgeon role is almost like a skipper. You know, of a ship…It's really important when things go bad. Let's say.. there's major bleeding or something unexpected happens. That's when really the skipper needs to lead the group. If the skipper is in panic, everyone is gonna be in panic” (S2). Early findings also indicate that surgeon leadership can promote a balance between control and trust that allows each member of the team to function at their best, "in the operating room, obviously everybody is there with the same goal in mind, to get the patient through the OR safely and efficiently… But everybody has their own expertise, their own role and their own kind of sub goals.. I think [surgeon leadership] really does impact how the team functions" (N2).

Conclusion: While standardized safety tools play an important role in ensuring safe practice, we require a deepened understanding of how the surgeon leader shapes team factors throughout the operation.

S131

Ergonomic Assessment of Muscle Force and Fatigue with the Use of Laparoscopic Energy Devices: A Kinesiology Study

Hee Ju Sohn 1; Junkyung Song2; Jaebum Park2; Jin-Young Jang3; Yoo Shin Choi1; 1Department Of Surgery, Chung-Ang University College of Medicine, South Korea; 2Department of Physical Education, Seoul National University, South Korea; 3Department of Surgery and Cancer Research Institute, Seoul National University College of Medicine, South Korea

Background: With advantages of laparoscopic surgery and improvements of equipment and surgical environment has brought an in widespread increase in various surgical fields. Physical characteristics are different among laparoscopic energy devices, such as handle size, force required for use and the degree of muscle fatigue generated after its use. The aim of this study is to measure the force required to use laparoscopic energy devices and compare the change of accuracy and EMG profile after muscle fatigue.

Methods: Four different laparoscopic energy devices were tested on 20 surgeons, currently performing laparoscopic surgery. The experiment was set up in a kinematic laboratory and measured the EMG profile (10 individual muscles in the arm) and accuracy when using the devices with a motion capture system. A detailed analysis comparing the type of energy device and between men and women was investigated. Furthermore, a questionnaire of the subjective aspects of ergonomics in each devices was performed.

Results: 10-point scale survey was done on subjective comfort and complaints in use of each devices. Significant differences in scores of weight, grip strength, handle size and comfort, finger demand force and location of the trigger was noted. In device 4, there was difference in the score of handle size and force required for activation between men and women. (p < 0.01) In the kinematic experiment, after artificial fatigue occurred, there was significant difference in the amount of device tip error which is thought to reflect accuracy. (p < 0.01) In EMG profile, the median frequency patterns were different among devices, which means some devices were more vulnerable to muscle fatigue. (p < 0.01).

Conclusion: We identified that when fatigue occurs, there is objective functional impairment and subjective complaints in the use of different laparoscopic energy devices. Instruments that accept improvements in ergonomic aspects considering variables such as basic grip power and hand size should come out, and surgeons should also participate in the experiment and actively intervene.

S132

Long-Term Quality of life After Endoscopic Vacuum-Assisted Closure Therapy in Gastrointestinal Leaks

Lucas Fair, MD 1; Meghana Vankina2; Titus McGowan3; Rashmeen Rana3; Gerald Ogola, PhD3; Marc Ward, MD1; Steven Leeds, MD1; 1Baylor University Medical Center; 2Texas A&M College of Medicine; 3Baylor Scott and White Research Institute

Introduction: Endoscopic vacuum-assisted closure (EVAC) therapy has become a viable option for the treatment of gastrointestinal leaks without the need for additional surgery. Although EVAC therapy has been shown to be successful, the long-term quality of life (QoL) following this treatment is unknown. The short-form 36 (SF-36) health survey has proven to be a reliable tool when assessing both physical and mental health. The purpose of this study was to use the SF-36 questionnaire to quantify the long term QoL after EVAC therapy for the treatment of gastrointestinal leaks.

Methods: An institutional board-approved prospectively maintained database was retrospectively reviewed to identify patients who underwent EVAC therapy for treatment of upper and lower gastrointestinal leaks between June 2012 and May 2022. Patients were then contacted by telephone and sent the SF-36 survey electronically. QoL domains were derived from the 36-survey items. Continuous variables were presented as mean and standard deviation, while categorical variables as frequency and percentage.

Results: A total of 110 patients who underwent EVAC therapy for a gastrointestinal leak were identified. Ninety-five patients were alive, and 15 were deceased. A total of 33 patients (28 female; 11 male) with a mean age of 55.3 years completed the survey (35% response rate). The mean follow up was 5.1 years from the sentinel leak operation. The majority of patients underwent EVAC therapy for upper gastrointestinal leaks (n = 29, 88%). The most common sentinel operation type was bariatric surgery (n = 21) with the most common operation being a gastric sleeve (n = 14). EVAC therapy patients scored highest in the domains of emotional well-being (mean = 73.3 ± 21.3), role limitations due to emotional problems (mean = 70.7 ± 41.5), and social functioning (mean = 70.1 ± 29.6). Scores were substantially lower in the domain of energy/fatigue (mean = 46.1 ± 21.3). All other SF-36 QoL domain scores were as follows: physical functioning (mean = 69.7 ± 29.7); pain (mean = 60.6 ± 28.6); role limitations due to physical health problems (mean = 54.5 ± 45.3); general health (mean = 53.2 ± 23.9).

Conclusions: In the long term, patients who underwent EVAC therapy for gastrointestinal leaks score highest in the QoL domains pertaining to emotional health, social functioning, and physical functioning.

S133

Evolution of Laparoscopic Surgery in a Sub-Saharan African country: A 30-Year Literature Review in Senegal

Abdourahmane Ndong, MD, MPH, MSc 1; Adja C. Diallo, MD, MPH, MSc1; Armaun D. Rouhi, BA2; Jacques N. Tendeng1; William Yi, MD, MSEd, FACS2; Mohamed L. Diao1; Noel N. Williams, MD2; Mamadou Cisse, MD1; Kristoffel R. Dumon, MD, FACS2; Ibrahima Konaté1; 1Department of Surgery, Saint-Louis Regional Hospital, Gaston Berger University, Senegal; 2Division of Surgical Education, University of Pennsylvania Perelman School of Medicine

Introduction: The advent of laparoscopy has significantly reduced the morbidity associated with the majority of abdominal surgeries. In Senegal, the first studies evaluating this technique were published in the 1980s. Since then, there has been limited investigation into the feasibility of laparoscopic surgery in this low-resource setting. The objective of this systematic review is to assess the evolution of laparoscopy in Senegal.

Methodology: A search of PubMed and Google Scholar was carried out without limit of publication date. The keywords used were "senegal" AND "laparoscop*". Duplicates were removed, and remaining articles were assessed for selection criteria. We included all articles about laparoscopy published in Senegal. All references of included articles were manually screened to find relevant articles. The archives of the journal of Senegalese Surgical Association (Journal Africain de Chirurgie) was also screened for relevant articles. Conferences abstracts, letters, reviews and case reports were excluded.

The parameters studied in each included article were the place and year of study, average age, sex ratio, assessed indications and results. The combined prevalence of postoperative death and overall morbidity was estimated.

Results: 41 studies published between 1984 and 2021 met selection criteria. 95.1% (n = 39) studies were conducted in the capital city of Dakar. The average age of patients was 33 years (range 4.7–63). The sex ratio was 0.33. The main indications for laparoscopy according to the studies were: benign digestive pathology in 11 studies (26.8%), abdominal emergencies in 9 studies (22%), biliary pathology in 5 studies (12.2%), benign gynecological pathology in 6 studies (14.6%), malignant gynecological pathology in 2 studies (4.9%), diagnostic laparoscopy to evaluate non-surgical abdominal pathology in 2 studies (4.9%), parietal surgery in 2 studies (4.9%) and testicular pathology in 1 study (2.4%). 4 studies (9.8%) reported multiple indications for laparoscopy. Overall mortality was estimated at 0.9% (95% CI: 0.6–1.3) and overall morbidity for all complications was estimated at 5% (95% CI: 3.4–6.9).

Conclusion: This systematic review showed a predominance of the practice of laparoscopy in the capital in Dakar with favorable outcomes. This technique should be popularized in the different regions of the country and its indications expanded.

S134

The Role of Teleproctoring in Creating a State-of-the-art Locally-staffed Minimally Invasive Surgical Center in Rural Uganda

Catherine Tsai 1; Joseph O Damoi2; Umut Sarpel1; Linda Zhang1; Michael Marin1; Daniel Herron1; 1Mount Sinai Medical Center; 2Kyabirwa Surgical Center

Introduction: Approximately 5 billion people worldwide lack access to safe and affordable surgical care. In Uganda, there are 0.3 surgeons per 100,000 people compared to 55 for the same population in the US. Only 2.9% of current surgical needs in Uganda are met. In 2019, the Mount Sinai Department of Surgery in New York collaborated with locals in rural Uganda to construct the Kyabirwa Surgical Center (KSC) with the goal of creating an independent, replicable, self-sustaining surgical system in Uganda. To facilitate the introduction of minimally invasive surgery we developed a focused training program using a combination of in-person training supplemented by supervised teleproctoring with state-of-the-art teleconferencing technology. We present the results of our first year of MIS teleproctoring cases.

Methods: Mount Sinai Department of Surgery worked jointly with local Ugandan staff to create a modern, 2-OR ambulatory surgery center in the rural province of Jinja. An independent solar-powered battery system ensured continuous power availability. A 7-mile long underground fiber optic cable link was installed to provide stable high-speed internet connectivity for teleproctoring. The local Ugandan general surgeon (JOD) underwent a mini-fellowship in laparoscopic surgery and then trained extensively using the SAGES FLS program. The training process culminated in an intensive 1-week in-person session during which Mount Sinai surgeons proctored the Ugandan OR team on increasingly advanced cases. Subsequently, from October 2021 to September 2022, the Ugandan surgeon (JOD) performed laparoscopic cases with teleproctoring provided by surgeons from Mount Sinai. Audiovisual feeds from the KSC operating room as well as the laparoscopic video were streamed to New York, and live feedback was provided for all teleproctoring sessions.

Results: Nineteen laparoscopic operations were performed at KSC with live remote teleproctoring. All operations were outpatient and included 14 appendectomies and 5 cholecystectomies. Teleproctoring was provided from port placement until completion of the case. Surgeon feedback was provided regarding patient positioning, port placement, surgical technique, use of instruments, and critical steps of the operation. All cases were successfully completed without any intra-operative complications. Post-operatively, field medical workers visited patients in their homes to collect follow-up information. There were no complications reported in the short-term follow-up.

Conclusion: Teleproctoring can be safely implemented to assist local surgeons in previously underserved areas to provide advanced laparoscopic surgical care to the local patient population.

Fig. 1 The Kyabirwa Surgical Center, with solar panels visible over the roof

figure as

Fig. 2 Teleproctoring screen showing laparoscopic image and OR view

figure at

S135

Improving Pain Management Following Laparoscopic Surgery: A Randomized Quality Improvement Study

Jasmine Dhal; Krishna Asuri; Rashmi Ramachandran; Omprakash Prajapati; Virinder Bansal; AIIMS, New Delhi

Background: Postoperative pain and return to routine activity are the major determinants of patient satisfaction. However, pain remains a significant problem owing to lack of a standardized perioperative pain management regimen. Despite several existing recommendations on multimodal analgesic regimens, no studies suggesting a uniform and standardized pain management regimen exist, allowing variability and inconsistency in the perioperative pain management. This study was conducted to develop a standardized pain management regimen through a quality improvement (QI) study and test its efficacy through a randomized trial.

Methods: This study was conducted between 31st May 2019 and 31st December 2021, and included patients undergoing laparoscopic cholecystectomy or laparoscopic inguinal hernia repair. Phase 1 comprised of an observational study for the assessment of baseline postoperative pain and a root-cause analysis leading to development of a standardized pain management regimen, followed by the requisite number of Plan-Do-Study-Act (PDSA) cycle(s) until the aim of the QI study was achieved, i.e., postoperative pain scores on Visual Analogue Scale (VAS) ≤ 3 at every time interval (0 h, 1 h, 2 h, 4 h, 6 h, 12 h, at discharge, 1 week, 6 weeks and 3 months), in at least 70% of patients. Phase 2 included a randomized trial comparing the efficacy between existing pain management regimens and the standardized pain management regimen.

Results: In Phase 1, twenty patients were initially observed for postoperative pain scores, which were noted to be rising till the second postoperative hour (median pain score = 4), with suboptimal pain control till 6 h postoperatively, in majority of patients. The first PDSA cycle, comprising 20 patients, sufficed to achieve the aim of the quality improvement study, and the regimen was standardized. Median pain scores were found to be significantly lower (p < 0.05), from 1 h onwards till 3 months postoperatively. In Phase 2, forty patients were randomized, 20 in each arm, and baseline characteristics were comparable. The number of patients with acceptable pain (VAS ≤ 3) at 1 h, was significantly (p = 0.038) higher in the QI arm, correlating with significantly lower median pain scores at 0 h and 1 h, in the QI arm (p = 0.01). However, pain scores at other intervals, adverse effects and return to preoperative activity, were comparable in both arms.

Conclusions: A standardized perioperative pain management regimen may significantly benefit early postoperative pain following laparoscopic surgery, which needs to be verified in large-scale studies.

S136

Here comes the Sun: Understanding Industry’s Payments to General Surgeons

Rizwan Ahmed, MD; Jonathan Schoen, MD; Kweku Hazel, MD; Akshay Chauhan, MD; University of Colorado

Purpose: The Sunshine Act has reported eight years of industry payments to physicians. Collaboration between industry and physicians has helped advanced the field of minimally invasive and endoscopic surgery. In order to bring new technologies from bench to bedside appropriate understanding and training of surgeons is essential. Industry utilizes various methods to help promote understanding their devices. The purpose of this study is to ascertain payments to general surgeons from industry and compare them to various surgical specialties utilizing the most recent Sunshine Act database.

Methods: We analyzed payment trends using Physician Payment Sunshine Act (PPSA) data (January 2021-December 2021) to assess industry payments made to surgeons of various surgical specialties over four years published in the Open-Payments-Program website.

Results: In 2021, 2,04 billion was paid to 533,056 physicians, $769,429,289 was paid to 82,367 surgeons; the median (interquartile range [IQR]) was $305 (92–1,610). General surgeons collectively received 52,079,364 paid to 18,092 general surgeons, their median (IQR) was $217 (68–1,118). The highest paid general surgeon received $3,694,343. The highest paid surgical specialty was orthopedic surgery that received $432,702,734 paid to 23,928 orthopedic surgeons, their median (IQR) was $571 (114–2,443). The lowest paid surgical specialty was pediatric surgery $457,786 paid to 367 pediatric surgeons, their median (IQR) was $81 (31–183). Per surgeon, the highest median (IQR) was paid to cardiothoracic surgeons $762 (171–2,651) who collectively received $49,786,631 paid to 3,735 cardiothoracic surgeons. Out of sixteen different surgical specialties, general surgeons received the third highest amount of total payments and represented 22% of the total aggregate payments made to surgeons. The analysis per surgeon demonstrated that general surgeons received the tenth highest payments of the sixteen different surgical specialties. In comparison, urologists who utilize minimally invasive devices as well received the fourth highest number of payments at $34,811,589 and had a median (IQR) of $543 (166–1,324).

Conclusion: The PSSA data demonstrates that per surgeon, general surgeons received lower median payments in comparison to other surgical specialties. General surgeons were ranked at the tenth highest recipients of sixteen surgical specialties. As advances are made in the field of minimally invasive and endoscopic surgery, comparing this field's payments to other surgical specialties is beneficial to understand the amount of monetary exchange across all specialties nationally.

S137

Outcomes of an Enhanced Recovery after Surgery (ERAS) Program to Limit Perioperative Opioid Use in Outpatient Minimally Invasive GI and Hernia Surgeries

Norbert Hootsmans, MD, MPH 1; Sara Parmiter, MD1; Kevin Connors, MD1; Shivani Bajpai, MD1; Elise Snyder, MD1; Justin J Turcotte, PhD, MBA1; Shyam S Jayaraman, MD2; H. Reza Zahiri, DO3; 1Anne Arundel Medical Center, Annapolis, MD; 2Frederick Health Hospital, Frederick, MD; 3Doctors Community Medical Center, Lanham, MD

Introduction: Perioperative pain management is important for patient satisfaction while returning to homeostasis in the safest and most efficient way possible. Studies show that patients do not desire or require as many opioids as once thought. The benefits of ERAS pathways extend beyond enhancement of patients' perioperative experience, and include reducing opioid prescriptions in the face of the ongoing nationwide opioid crisis and evidence of prescription opioids as a contributor.

Methods: We performed a multisite, retrospective cohort study of patients undergoing same day minimally invasive surgery (MIS) procedures for GI and hernia disease using a minimal-opioid ERAS protocol at two community hospitals between January 2020 and May 2022. Included procedures were elective laparoscopic cholecystectomy (LC), laparoscopic appendectomy for acute appendicitis without perforation (LA), and minimally invasive (laparoscopic and robotic) inguinal and ventral hernia repair or abdominal wall reconstruction (AWR). Primary outcomes of postoperative opioid use were compared using one-way ANOVA and chi-square tests.

Results: A total of 509 patients were included, undergoing procedures of MIS hernia repair/AWR (52.5%), laparoscopic cholecystectomy (43.6%), and laparoscopic appendectomy (7.9%). Overall 23.5% of patients received narcotics in the PACU, with a significantly reduced rate of 13.4% in the MIS hernia/AWR group compared to 33.3% and 32.5% for the LC and LA groups, respectively (P < 0.001). Only 9.4% of patients received opioid prescriptions at discharge, with no significant differences between groups. Among the patients receiving a prescription at discharge, there was a significant difference between groups in morphine milligram equivalents (MME) prescribed (25.0 ± 0.0 in the LA group, 65.0 ± 41.4 in the LC group, 100.6 ± 46.2 in the MIS hernia/AWR group; P = 0.015). Nine percent of patients called with pain management concerns postoperatively. An ASA score ≥ 3 was the only risk factor with significantly increased odds for postoperative opioid prescription (OR 2.084; P = 0.014).

Conclusions: This study demonstrates that an opioid-sparing ERAS program can effectively minimize perioperative opioids for patients undergoing multiple outpatient MIS GI and hernia surgery procedures, and suggests generalizability across a diverse range of surgeries. Therefore, the use of ERAS may safely and effectively expand beyond inpatient MIS and open surgeries that target reduced length of stay to also minimize opioids for outpatient surgeries. Additional benefits of ERAS include resident education for pain management of surgical patients and a collaborative approach to patient care by the perioperative team.

S138

A Dedicated Feeding Tube Clinic Improves Nutritional Outcomes

Kevin Choy, MD; Danielle Abbitt, MD; Amber Moyer, MD; John T Moore, MD; Krzysztof J Wikiel, MD; Teresa S Jones, MD; Thomas N Robinson, MD; Edward L Jones, MD; University of Colorado

Objective: Optimizing nutrition is essential for recovery after major surgery or severe illness. Feeding tubes (FT) can be placed in the stomach (gastrostomy) or small bowel (jejunostomy) in patients limited by enteral nutrition or hydration by mouth. Given the myriad locations in which these procedures are performed (radiology, intensive care unit, endoscopy suite, etc.), routine follow-up is challenging. We established a clinic dedicated to managing FT patients and reduced ED visits by nearly 50%. The purpose of this study was to evaluate the impact of a FT clinic on nutrition. We hypothesized that enrollment in FT clinic would result in improved nutritional outcomes.

Methods: Retrospective review of patients who underwent feeding tube placement from 1/2010–1/2020 at a tertiary-care VA. Baseline demographics and body mass index (BMI) were recorded. Serum albumin was recorded within 1 month of tube placement and compared to the result within 1 month of tube removal, death, or the end of study period. Participation in FT clinic required at least 2 visits. Indications for FT placement and duration were measured. Patients were excluded if BMI or albumin levels were not appropriately recorded.

Results: Seventy patients underwent FT placement during the study period; 23 (25%) were excluded as above. Sixteen (23%) patients were seen in FT clinic (FTC), mean age 62.8 and 16 (100%) male. Fifty-four (77%) were managed outside of FTC (nFTC), mean age 64.7 and 53 (98%) male. There was no difference in age, gender or indication for FT. Mean albumin increased 0.4 ± 2.6 g/dL in the FTC group versus -0.04 ± 0.8 g/dL in the nFTC group (p = 0.047). There was a trend toward increased BMI lost in the nFTC group (-1.6 ± 3.6 kg/m2 vs. -0.1 ± 2.6 kg/m2, p = 0.13). The FTC group maintained their tubes significantly longer (77.4 vs. nFTC 19.0 days, p < 0.0001) although there was no significant difference in mortality (50% vs. 43%, p = 0.78). nFTC patients had higher head and neck cancer proportion (64.8% vs 43.8%, p = 0.035).

Conclusion: Patients managed in a dedicated feeding tube clinic experienced 10 × greater improvement in serum albumin than those who had no routine follow-up after placement. They also maintained their feedings tubes nearly 2 months longer with a trend toward a stable BMI versus a 1.6 g/m2 BMI loss in the routine follow-up group. To optimize nutrition and reduce weight loss, patients who require feeding tubes should be enrolled in a dedicated feeding tube clinic.

S139

Impact of Enhanced Recovery Pathway on Patient Reported Outcomes After Abdominal Surgery: A Systematic Review

Charbel El-Kefraoui 1; Andrew Miller1; Uyen Do1; Araz Kouyoumdjian, MD1; David Cui, MD2; Lawrence Lee, MD1; Liane S Feldman, MD1; Julio F Fiore Jr, PhD1; 1McGill University; 2University of Ottawa

Introduction: The objective of this study was to appraise the evidence regarding the impact of enhanced recovery pathways (ERPs) on patient-reported outcomes (PROs) after abdominal surgery. Previous literature supports that standardizing perioperative care using ERPs reduces hospital length of stay and 30-day complications. However, these measures of recovery may not reflect the perspective of patients who are the main stakeholders in the recovery process. While ERPs are increasingly used in clinical practice, their impact on PROs remains uncertain.

Methods and Procedures: In this systematic review, eight databases (Medline, Embase, Biosis, PsyncINFO, Cochrane, CINAHL, Web of Science, and Scopus) were searched for randomized controlled trials (RCTs) addressing the impact of ERPs versus traditional care on PROs (i.e., health reports coming directly from the patient via questionnaires) after abdominal surgery. We focused on distinct periods of recovery: early (within 7 days postoperatively) and late (beyond 8 days). Risk of bias was assessed using Cochrane's RoB 2.0. Results were appraised descriptively as heterogeneity hindered meta-analysis. Quality of evidence across different PRO health domains, surgical specialties, and periods of recovery was evaluated using GRADE.

Results: Thirty-three RCTs were identified [colorectal (n = 13), hepato-pancreatico-biliary (HPB) (n = 8), upper gastrointestinal (n = 7) and urological surgery (n = 5)]. Most trials had 'high' (n = 15) or 'some concerns' (n = 15) risk of bias. Evidence of 'very low' to 'moderate' certainty supports that ERPs reduce pain in the early recovery period across all specialties. Evidence of 'very low' certainty supports improvement in early overall health status (i.e., quality of life) across all specialties. After HPB and genitourinary procedures, 'very low' certainty of evidence support improvements in overall health status in the late postoperative period, while no benefit was observed in colorectal and upper gastrointestinal procedures. 'Very low' certainty of evidence supports that ERPs improve early fatigue and mental health after colorectal surgery with no benefit found in HPB, upper gastrointestinal, and genitourinary surgeries. Evidence of 'very low' certainty did not support that ERP improved early or late physical function in any surgical specialty.

Conclusions: This review supports that ERPs may have a positive impact on patient-reported pain, fatigue, mental health, and overall health status after abdominal surgery, particularly in the early postoperative period; however, data were largely derived from low-quality trials. Although these findings contribute important knowledge to inform evidence-based ERP implementation, there remains a great need to improve the quality of PRO assessment and reporting in ERP trials.

S140

Development of a Benign Biliary Dashboard to Assess Cost and Quality Metrics

Doreen Chang; Philip A Fong; Wendy Webster; John Rollman; Melissa Pressley; Garth S Herbert; Scott F Gallagher; Christopher R Watters; Sabino Zani; Jin S Yoo; Suresh Agarwal; Allan D Kirk; Peter J Allen; Keri A Seymour; Duke University Department of Surgery

Background: Biliary surgery includes numerous procedures and spans several surgical subspecialties. Thus, surgical treatment of benign biliary disease is an ideal cohort for a pilot study measuring quality outcomes within a healthcare system.

Methods: A privacy protected dashboard was developed to include patients > 18 years who underwent cholecystectomy for presumed benign biliary disease between July 2018 and June 2022. The health system includes a tertiary academic hospital, two regional community hospitals, and two ambulatory surgery centers. The Common Procedural Terminology (CPT) codes, payor mix, operative time, cost, outcomes, and healthcare utilization were included. 5,074 biliary procedures were performed by 68 surgeons in 11 surgical divisions. Exclusion included 76 biliary anastomoses, 418 cases that used unlisted CPT codes for procedures on the stomach, pancreas, or liver, and 21 operations that included multiple surgeons.

Results: 4,559 cholecystectomies performed by 57 surgeons in 5 divisions became the default setting on the dashboard. After applying additional filters for technique, the individual surgeon is compared to a selected division for volume (Fig. 1), intraoperative data, cost, and outcomes. 2,381 cholecystectomies were ambulatory procedures while 2,178 were performed during hospital admission. Of all procedures, 93% were performed laparoscopically and 3% were performed using the robotic platform while open cholecystectomy (or converted to open) represented 3% of cases.

Individual surgeon data was analyzed with thresholds of 10 laparoscopic cholecystectomies (CPT 47,562), 5 laparoscopic cholecystectomies with cholangiogram (CPT 47,563), and one open cholecystectomy (CPT 47,600). Patient age, body mass index, assistant training level, type of procedure, surgical approach, and day of week impacted operative time. Cholangiograms were performed in 12% of cases and use either iodinated contrast or indocyanine green. Cost was lowest at ambulatory surgery centers yet only 8.1% of elective procedures were performed at these facilities. The median waste was less than $10 per case. The return to emergency department rate and indications are shown in Fig. 2.

Conclusion: Transparency around surgeon level data aligns healthcare delivery with quality care. The data generated from clinical dashboards will guide strategies to standardize care, optimize efficiency, provide cost savings, and improve outcomes for benign biliary operations. Future analyses will include more complex cases and specialty-specific procedures to measure and improve quality.

Fig. 1 Surgical volume of an individual surgeon compared to peers

figure au

Fig. 2 Rate and indication for emergency department visits for an individual surgeon compared to deidentified peers

figure av

S142

Early Postoperative Telehealth Visit Protocol Implementation Reduces Emergency Department Utilization Following Benign Foregut Procedures

Tarik K Yuce, MD, MS; Patrick J Sweigert, MD; Roukaya T Hassanein, MD; Theresa N Wang, MD; Melissa Himes, APRNCNP; Kelly R Haisely; Kyle A Perry, MD; The Ohio State University, Wexner Medical Center

Introduction: Emergency department (ED) visits and readmissions following benign foregut surgery represent a burden on patients and the health care system. The use of early postoperative telehealth visits may reduce these encounters. The objective of this study was to identify differences in ED visits and readmissions before and after implementation of an early postoperative telehealth visit protocol for benign foregut procedures. We hypothesized that utilization of early postoperative telehealth visits would be associated with reduced post-operative ED and hospital utilization.

Methods: An early postoperative telehealth protocol was initiated in 2020 at an academic medical center to provide a video conference within the first postoperative week. An advanced practitioner conducted the visit, and each patient was subsequently seen by their surgeon four weeks later. Consecutive elective benign foregut operations including fundoplication, Linx, paraesophageal hernia repair, and Heller myotomy performed between 2018 and 2022 were included. Patient data were collected and maintained in an IRB approved database to retrospectively compare outcomes between patient groups before and after routine telehealth visit implementation. Outcomes of included 30-day readmission and ED visits without readmission. Bivariate analyses were performed using Chi-squared testing for categorical variables. The association between telehealth visits and outcomes were evaluated using multivariable logistic regression.

Results: 616 patients underwent benign foregut procedures during the study period. 310 (50.3%) were performed prior to the implementation of telehealth visits and 306 (49.7%) were after. 241 patients in the telehealth visit group (78.8%) completed their telehealth visit. A total of 39 patients (6.3%) were readmitted while 34 patients (5.5%) had ED visits without readmission within the first 30 days. The most common cause of readmissions and ED visits included pain (n = 18, 25%) and nausea/vomiting (n = 12, 16%). There was no difference in readmission rate before or after telehealth implementation (6.1% versus 6.5%; OR 0.93, 95% CI 0.49–1.78, p = 0.71); however, there was a significant reduction in ED visits without admission (7.4% vs 3.5%; OR 0.46, 95% CI 0.22–0.94, p = 0.03). The telehealth cohort had significantly lower ED visits for pain (31% vs 16.7%, p = 0.03) and nausea/vomiting (23.8% vs 6.7%, p = 0.01).

Discussion: Early telehealth follow-up was associated with a significant decrease in ED visits following benign foregut procedures. The majority of this was attributable to a reduction in ED visits for pain, nausea, and vomiting. These results provide a possible avenue for improving quality and cost-effectiveness within this patient population.

S143

Aspiring for Better: Comprehensive Stakeholder Perspectives on Reducing Post-operative Pulmonary Aspiration Events

Viviane Leite Abud, MD 1; Kyle Bata, MS1; Kelsey Boyette, RN2; Katelyn Burto, MSN, RN, CMSRN2; Lisa Casanova-Sidoti, DNP, RN, CNOR, RNF2; Anjelica Cromartie, PAC2; Kathryn Eckert, NP, MSN, BSN2; Meghann Griffin, SLP2; Alexandra Huechteman, PAC1; Erin Kinney, CQS, RN2; Danielle Koffenberger, MSPM1; Janet Kukreja, MD1; Laura Leonard, MD1; Randall Meacham, MD1; Katie Moller, PA1; Patrick Murphy, PA1; Kristen Oase, PA1; Sarah Tevis, MD1; Katie Thompson, NP1; Jeniann Yi, MD, MSCS1; Ethan Cumbler, MD, FHM, FACP1; 1Department of Surgery, University of Colorado; 2University of Colorado Hospital

Introduction: Respiratory failure after surgery (Patient Safety Indicator 11) is often precipitated by an aspiration event, however there has been little progress identifying effective measures to prevent postoperative aspiration or improve its outcomes. The purpose of this study is to identify high-yield opportunities for intervention to reduce adverse outcomes from post-operative aspiration events at a quaternary academic medical center.

Methods: Following root cause analyses of post-operative aspiration events, we developed a current-state process map, conducted interviews with best-practice units within the hospital and nationally, and performed a 360-degree stakeholder evaluation including patients, nursing, and clinicians to assess attitudes, beliefs, and perceptions of improvement opportunities. An interdisciplinary taskforce conducted two 3-h rapid improvement events to analyze the data collected and identify interventions to reduce post-operative aspiration.

Results: Qualitative interviews with 14 inpatients following surgery revealed that a significant proportion reported having symptoms that increase risk for aspiration yet did not notify their team. While some could identify behaviors to reduce the risk of aspiration, the majority stated they had not received education on aspiration. Insights from 62 surgical nursing staff revealed need for additional education in bedside swallow, risk assessment, and preventive measures. Lack of clarity of signs/symptoms for which surgical team should be notified was reported, along with a desire for novel patient education materials, a visible signal for at-risk patients, and a nurse-driven pathway for aspiration. Insights from 103 surgical clinicians found the majority believe we could do better at preventing aspiration events. Only half felt confident in their own ability to identify high risk patients. They identified education, screening for high-risk patients, and creation of a nurse-driven pathway as priorities. Additional opportunities included better adherence to the aspiration precaution bundle, intrateam communication regarding risk, and early recognition of aspiration risk symptoms.

Using defined process improvement methodology, rapid improvement events identified four domains for intervention to reduce post-operative aspiration: 1. Improving institutional Order Sets. 2. Creation of a nurse-driven pathway. 3. Implementing three-level screening for aspiration risk involving Pre-Procedural Services, bedside nurse, and speech pathologist, followed by clear communication of risk. 4. Creation of an education strategy for patients, nursing staff and providers.

Conclusion: This initiative illustrates a rigorous quality improvement approach to reducing post-operative complications by seeking input from diverse stakeholders and allowing insights from this process to inform the decisions made by a multidisciplinary taskforce guided by quality and process improvement staff.

S144

Outpatient Surgery Postoperative Ambulation and Emergency Department Utilization

Danielle Abbitt, MD; Kevin Choy, MD; Jake Cotton, MD; Edward L Jones, MD; Teresa S Jones, MD; Thomas N Robinson, MD; University of Colorado

Introduction: Ambulation is an important indicator for wellness and quality of life. A major health event, such as surgery, can derail this ability, and return to preoperative walking ability is a marker for recovery. Self-reported walking measurements by patients is subject to bias, thus wearable technology such as activity monitors have risen in popularity. We evaluated postoperative ambulation using an accelerometer in outpatient surgery.

Methods: Retrospective review of patients undergoing outpatient abdominal surgeries from 11/2016–7/2019 at a Veteran Affairs Medical Center. Patients wore an accelerometer preoperatively and postoperatively to measure their ambulation (steps/day). Outcome measures were readmissions and Emergency Department (ED) utilization. Postoperative ambulation was defined as daily percentages of their preoperative baseline. Patients without preoperative baseline data, > 3 missing days or missing days prior to reaching baseline were excluded.

Results: One-hundred-six patients underwent outpatient abdominal surgery. Twenty-two patients excluded as above. Patients stratified into adult (18–64 years, 44 patients, 52%) and geriatric(≥ 65 years, 40 patients, 48%) cohorts (Table 1). Adults who failed to meet their preoperative baseline by postoperative day 7 had higher ED utilization; 4(24%) vs 1(4%), p = 0.043 (Fig. 1). Geriatric patients who failed to meet their baseline trended toward increased ED utilization; 5(19%) vs 1(7%), p = 0.31.

Table 1 Results for adult and geriatric cohorts

 

Adult

Geriatric

 

Preoperative ambulation, avg(range)

7318

(969–15,953)

5751

(1925–16,901)

p = 0.019*

Inguinal hernia repair

Ventral hernia repair

Cholecystectomy

32(72%)

3(7%)

9(21%)

34(85%)

2(5%)

4(10%)

p = 0.37

 ≥ 100% baseline by POD7

 < 100% baseline by POD7

27(61%)

17(39%)

14(35%)

26(65%)

p = 0.016*

ED visits:

 ≥ 100% baseline by POD7

 < 100% baseline by POD7

1(4%)

4(24%)

1(7%)

5(19%)

p = 0.043*,0.31

Readmission

 ≥ 100% baseline by POD7

 < 100% baseline by POD7

0

1(4%)

1(7%)

2(8%)

 

Discussion: Patients aged 18–64 who fail to return to their preoperative daily step count within one week of outpatient abdominal surgery are 6 × more likely to be seen in the ED than those who have achieved their baseline (Fig. 2). Postoperative ambulation predicts ED utilization after outpatient surgery. More study is needed to provide patients and surgeons with guidelines for normal recovery.

figure aw
figure ax

S145

Expert Laparoscopist Performance on Virtual Reality Simulation Tasks With and Without Haptic Features

Margaret Siu, MD; Kaitlin Debbink, MD; Amanda Duda, BS; Georgios Orthopoulos, MD, PhD; John R Romanelli, MD; Jacqueline Wu, MD; Neal E Seymour, MD; University of Massachusetts Chan Medical School—Baystate

Introduction: Virtual reality (VR) laparoscopic simulators with force feedback features that reproduce a haptic "sense of touch" when instruments interact with virtual objects may enhance the ability to perform simulated laparoscopic tasks. Our objective was to determine if expert laparoscopists' skills characteristics differ using identical VR task software platforms delivered via haptic versus nonhaptic laparoscopic interfaces.

Methods and Procedures: Five expert laparoscopists (> 100 advanced laparoscopic procedures) performed seven basic skills tasks on two laparoscopic simulators, one with and one without haptic features (LAP Mentor III and LAP Mentor Express). Tasks were identical on the two platforms, consisting of 2-handed instrument navigation, retraction and exposure, cutting, electrosurgery, and complex object positioning. At least 12 iterations were performed for each task on each simulator, alternating platforms at default difficulty settings. Metrics included time, economy of movement, completed task elements, and errors trended for each task. Iteration quartile means were determined and compared for the two platforms using paired t-tests. Progressive change in performance for the final 3 iterations were determined by repeated measures ANOVA.

Results: No ongoing change was noted for any metric for the last 3 iterations of any task except left hand economy of movement for the nonhaptic complex object positioning task, which showed continued incremental improvement. Although there were no significant differences between platforms for the final 2 quartiles for most metrics, there were some exceptions. Avoidance of over-stretch error on retraction and cutting task were significantly better with haptics for all iteration quartiles (p < 0.03). Economy of instrument movement was significantly better with haptics for both right and left hands on clip application (p < 0.01), and better for right hand on complex object positioning (p < 0.05). Accuracy was better with haptics for retraction and cutting task (p < 0.05) and for clip application task (p < 0.05).

Conclusion: Results favored some performance advantage for accuracy, efficient instrument motion, and avoidance of excessive traction force on selected tasks performed on a VR simulator platform with force feedback-enabled haptic features compared to the same tasks performed on a VR platform without force feedback. This suggests expert laparoscopic surgeons interpreted machine-generated haptic cues appropriately and that this was associated with better compliance with VR task performance requirements. However, this was observed for the minority of measured performance items. While this may not demonstrate an advantage for skills acquisition, it warrants additional study.

S146

An Interprofessional Approach to Assessing Musculoskeletal Pain and Intraoperative Ergonomics in General Surgery Residents

Aleezay Haider; Hamza Hanif; Terryn M Dyche; Noah V Monagle; Andrea Patterson; Lauren Eberle; Patricia C Siegel; Jasmeet Paul; Alissa A Greenbaum; University of New Mexico

Introduction: Work-related musculoskeletal injuries are common in general surgeons resulting in chronic pain, lost work and premature retirement. However formal ergonomic curriculums in surgical training programs are rare. The aims of this study were to determine the prevalence of musculoskeletal pain in general surgery residents and to assess their intraoperative postural risk using validated ergonomic tools in collaboration with occupational therapy (OT) students.

Methods: General surgery residents at a single safety-net academic hospital completed an anonymous survey regarding their musculoskeletal pain and ergonomic practices. Four OT students captured intraoperative photos of trainees performing abdominal open and laparoscopic surgery over a 4-week period, which were also reviewed by an occupational therapy professor. Biomechanics were assessed using the Rapid Entire Body Assessment (REBA) and the Rapid Upper Limb Assessment (RULA) which are validated graded tools that measure ergonomic efficiency. The REBA and RULA scoring systems assess overall body and upper limb ergonomics by measuring the severity of postural risk, with higher scores representing a more unfavorable posture. Calculated scores were then correlated with recommended need for ergonomic change.

Results: The survey received 37/44 (84%) responses. All 37 respondents reported some degree of pain during or after surgery, most commonly neck pain (75%), followed by shoulder (61%) and foot pain (53%). Approximately half of the respondents reported requiring pain-relief medication (49%) with 62% never seeking definitive treatment such as OT or physical therapy. Most respondents (61%) felt their pain was exacerbated by laparoscopic surgery, though 50% of trainees also experienced pain with open surgery. Most residents (66%) felt some degree of pressure to continue performing surgery regardless of their pain. Ergonomic breaks directed by faculty during long operations were reported to occur less than 11% of the time. A total of 11 intra-operative observations were made by OT students of surgical trainees with a mean RULA score of 6.1, with the 'arm and wrist' sub-category mean score of 5.2. The mean REBA score was 7.3 with the 'neck, trunk and leg' sub-category mean score of 4.1. All category scores demonstrated suboptimal posture that required further investigation and recommendations for prompt change.

Conclusion: Musculoskeletal symptoms and intraoperative ergonomic dysfunction are exceedingly prevalent amongst general surgery residents, without formal workplace measures for management or prevention. This needs assessment will be used to create an ergonomics initiative for the surgery residency in collaboration with the university OT school.

S147

Effectiveness of Immersive Virtual Reality on Pneumothorax and Chest Tube Management Knowledge Acquisition Among Novice Surgical Residents: A Randomized Controlled Trial

William Yi, MD; Daniel Weber, BE; Dineth Meegoda; Jacqueline Li; Madeline Maurer; Jennifer Guo; Armaun Rouhi, BA; Caoimhe Duffy, MD; Josh Atkins, MD; Noel Williams, MD; Kristoffel Dumon, MD; University of Pennsylvania

Background: There is limited literature on immersive virtual reality (IVR) simulation in general surgery education. The aim of this randomized controlled study was to determine the effectiveness of an IVR application in teaching clinical knowledge related to pneumothorax and chest tube drain management to surgical interns.

Methods: Forty-eight incoming surgical interns were randomized into receiving a 45-min pneumothorax and chest tube drain management didactic in IVR or in a classroom setting. Participants were evaluated on clinical knowledge relating to pneumothorax and chest tube management prior to and immediately after intervention. Participant opinions on the effectiveness of the IVR or control intervention in achieving the educational objectives laid out prior to the intervention were collected. IVR participants were also surveyed regarding common motion sickness symptoms related to IVR.

Results: Prior to intervention, there was no significant difference in mean knowledge exam scores between IVR and control groups [3.00 ± 1.16 vs. 3.08 ± 1.21, p = 0.900]. After the educational interventions, both groups showed statistically significant improvements in mean knowledge exam scores (control: 3.08 ± 1.21 vs. 4.13 ± 0.99, p = 0.008; IVR: 3.00 ± 1.16 vs. 4.46 ± 1.10, p < 0.001). The effect size for IVR intervention was larger than that of the control (control: 0.94 ± 0.36, IVR: 1.29 ± 0.37). Participants of both the IVR and control intervention indicated their respective pneumothorax and chest tube management training was efficacious (4.09 ± 0.97 vs. 3.96 ± 0.75, p = 0.284). At least one motion sickness symptom was experienced by 19/22 (86%) IVR group participants, with dizziness reported most commonly (27%).

Conclusion: An IVR application is as effective as conventional in-classroom didactic and simulation at increasing clinical knowledge related to pneumothorax and chest tube management. IVR intervention had a larger effect on procedural and declarative knowledge acquisition than in-classroom teaching. Both groups found their respective interventions to be efficacious in achieving the stated educational goals. Further study is required to identify and limit causes of motion sickness with IVR, a frequent occurrence in this study.

S148

Hybrid Train-the-trainer Course for FUSE Certificates Brings Better Understanding of Surgical Energy and Develops Teaching Skills: A Feasibility Study

Akihiro Kondo, MD, PhD 1; Masato Tamate, MD, PhD2; Yuichi Nishihara, MD, PhD3; Yusuke Watanabe, MD, PhD4; 1Department of Gastroenterological Surgery, Faculty of Medicine, Kagawa University; 2Department of Obstetrics and Gynecology, Sapporo Medical University Hospital; 3Department of Gastrointestinal Surgery, Sassa General Hospital; 4Clinical Research and Medical Innovation Center, Institute of Health Science Innovation for Medical Care, Hokkaido University Hospital

Introduction: An increased number of well-trained expertise instructors who completed the "train-the-trainer (TTT)" course has been required to disseminate the safe use of surgical energy devices, which could be learned by the Fundamental Use of Surgical Energy (FUSE) program. We explored whether the hybrid TTT course is feasible, effective, and leads to improved teaching of surgical energy.

Methods: The hybrid TTT course, which was designed to train the FUSE certified personnel as instructors, consisted of three sessions spread over 4.5 h in total, and a one day in-person training, followed by a 100-min FUSE electrosurgery hands-on workshop in practice as an instructor. The course included modules on 1) the history and intellectual property of FUSE; 2) teaching tips; 3) extensive knowledge of major FUSE learning objectives; and 4) how to organize structured goal-oriented hands-on simulation. The trainees were asked to report on the levels of their understanding regarding various energy devices or adverse events, and their satisfaction levels before and after the course, respectively. Trainees and experienced FUSE instructors assessed trainees' presentation skills using the Presentation Skills Assessment Tools at the beginning of the in-person training and after the hands-on workshop.

Results: Out of 18 participants, 17 (general surgeons 10 [59%)], GYN 5 [29%], thoracic surgeon 1 [6%], ENT 1 [6%]) completed the TTT course, which we conducted thrice. Any network issues were observed during the virtual sessions. Almost all participants (94%) were satisfied with the course. Self-confidence in knowledge about the fundamentals of electrosurgery, and mechanisms and prevention of adverse events improved significantly (pre 33% vs. post 94%, p < 0.01). Significant improvement of the understanding occurred in monopolar instruments (pre 28% vs. post 94%, p < 0.01) and bipolar/advanced bipolar devices (pre 33% vs. post 94%, p < 0.01). The presentation skills index of both self and peer assessments improved after the workshop (self-assessment pre 44 vs. post 56, p < 0.01; peer assessment pre 39 vs. post 68, p < 0.01). Peer assessments after the course were significantly higher than self-assessment (p < 0.01) for each participant, although initial peer assessments were lower than self-assessment.

Conclusion: The hybrid TTT course can bring FUSE certified personnel an extensive understanding of surgical energy devices and improve their presentation skills. This could help disseminate the culture of FUSE by building trainees' self-confidence as instructors.

S149

Validity and Reliability Evidence Support Task-Specific Metrics

Alexis Desir, MD; Emile Farah, MD; Shruti Hegde, MD; Carla Holcomb, MD; Daniel J Scott, MD; Ganesh Sankaranarayanan, PhD; University of Texas Southwestern Medical Center.

Introduction: Laparoscopic hiatal hernia repair is a difficult procedure and surgeons may benefit from additional training opportunities. Our goal is to develop a virtual reality simulator for this purpose. However, data supporting task-specific metrics are lacking. The purpose of this IRB-approved study was to assess discriminant validity and reliability evidence for task-specific metrics for the fundoplication phase of the procedure, as compared to global performance ratings.

Methods: Participants were stratified into three groups: Novice (PGY1-2's), Intermediates (PGY3-5's) and Experts (fellows and faculty) and performed a standardized exercise by creating a Nissen fundoplication on a porcine stomach in a laparoscopic box trainer. Video recordings were independently assessed by two raters using global ratings (OSATS) and task-specific metrics for wrap creation, position, securing, and length. Bimanual and suturing skills were also assessed. Total scores for both metrics were computed and analyzed using the Kruskal–Wallis test. Mann–Whitney U test with Benjamini–Hochberg correction were used to evaluate between-group differences. Intraclass correlation coefficient (ICC) was used to assess interrater reliability (IRR). Spearman's rank correlation coefficient was computed to assess correlation between global and task-specific scores.

Results: Participants (n = 29) included novices (n = 11), intermediates (n = 12), and experts (n = 6). IRR was high for both global (ICC = 0.82, p < 0.001) and task-specific (ICC = 0.83, p < 0.001) metrics. There was a strong positive correlation between the global and task-specific scores (rs = 0.86, p < 0.001). Significant between-group differences (Fig. 1) were detected for both global (χ2 = 15.38, p < 0.001, df = 2) and task-specific (χ2 = 11.38, p = 0.003, df = 2) scores. Post hoc analysis showed novices performed poorer compared to intermediates and experts for both global (novices vs. intermediates (p = 0.001), vs. experts (p = 0.004)) and task-specific (novices vs. intermediates (p = 0.01), vs experts (p = 0.01)) scores. No significant differences were found between intermediates and experts using either global (p = 0.5) or task-specific (p = 0.3) scores.

Conclusion: Using an ex-vivo fundoplication model, this study documented high IRR and significant discriminant validity evidence in support of video-based assessment using task-specific metrics, with similar findings for global ratings. These data are encouraging for the use of task-specific metrics in the creation of a virtual reality simulator.

figure ay

S150

Qualitative Analysis of Expert Bariatric Surgeons

Karan Grover, MD, PharmD; Shravan Sarvepalli, MD; Daniel Praise Mowoh; Leena Khaitan; Mujjahid Abbas; University Hospitals/Case Western

Background: The safe and effective performance of a robotic roux-en-y gastric bypass (RRNYGB) requires the application of a complex body of knowledge and skills. This qualitative study aims to:

  • Define the tasks, subtasks, decision points, and pitfalls in a RRNYGB

  • Create a framework upon which training and objective evaluation of a RRNYGB can be based.

Methods: Hierarchical and cognitive task analyses for a RRNYGB were performed using semi-structured interviews of hernia experts to describe the thoughts and behaviors that exemplify optimal performance. Verbal data was recorded, transcribed verbatim, supplemented with published literature, coded and thematically analyzed by two independent reviewers.

Results: A conceptual framework was synthesized based on 4 literary sources and interviews of 3 bariatric surgery experts. Subject matter experts have practiced a median of 12 years and have completed a median of 400 RRNYGB. After inductive analysis, 50 subtasks, 20 potential errors and 8 decision points were identified and categorized into 8 major procedural steps. (Table 1) Of note, this is an ongoing study and results will be updated to include the findings from additional expert interviews by the date of the conference.

Conclusion: This study defines the key tasks, decisions, and cognitive behaviors that are essential to perform the RRNYGB safely and effectively. This framework has the potential to serve as the basis for training novices.

Table 1

Procedural steps

Subtasks

Decision points

Potential errors

Room Setup

7

0

1

Patient Positioning

4

0

1

Trocar Placement

8

0

2

Gastric Pouch Formation

8

0

4

GJ Anastomosis

10

5

3

JJ Anastomosis

5

1

4

Closure of Mesenteric Defects

5

1

3

Leak Test/EGD

3

1

2

Total

50

8

20

S151

What Influences General Surgery Residents’ Prospective Entrustment and Operative Efficiency in Robotic Inguinal Hernia Repairs

Michael P Meara, MD, MBA, FACS; Heidi Pieper, MHA; Ingrid Woelfel, MD; Theresa Wang, MD; David B Renton, MD, FACS; Xiaodong Chen, PhD; The Ohio State University Wexner Medical Center

Introduction: Teaching residents robotic-assisted inguinal hernia repair (RIHR) is an increasingly common extension of contemporary surgical training. Thus, we sought to investigate what variables would influence operative efficiency assessed by operative time (OT) and resident prospective entrustment in RIHR cases.

Methods: We prospectively collected 68 resident RIHR operative performance evaluations with an augmented validated surgical entrustable professional activities (SEPAs) instrument. Outpatient RIHR cases performed by 11 general surgery residents during 2020–2022 were included. The overall OT of matched cases was extracted from hospital billing; matched procedural task-specific OT was obtained from Intuitive Data Recorder (IDR). Statistical analysis was performed using Pearson correlation and one-way ANOVA.

Results: The evaluation instrument reliably assessed residents' RIHR performance (Cronbach's α = 0.93); residents' prospective entrustment strongly correlated with overall guidance provided by attending (r = 0.86, p < 0.0001), operative plan and judgment (r = 0.85, p < 0.0001) and other evaluative items (Table 1). The overall OT was significantly associated with the resident's team management (r = 0.28, p = 0.022). Procedural task-specific OT was significantly associated with residents' task-specific skill (r = − 0.25, p = 0.023). On average, RIHR cases with prospective entrustment level 5 (Resident can teach junior) showed the shortest task-specific OT (seconds): Creation of Peritoneal Incision (186), Peritoneal Flap Exploration & Exposure (312), Reduction of Hernia Sac (146), Mesh Placement & Attachment (172), and Closure of Peritoneum (422). Entrustment level 3 (Reactive guidance needed) was the turning point of all four RIHR procedural task-specific OT (Fig. 1).

Conclusions: Our findings suggest that the three primary variables contributing to resident prospective entrustment are attending surgeon guidance, resident operative plan and judgment, and resident technical skill. Resident performance also impacted RIHR operative efficiency, which influenced the attending surgeon's determination of prospective entrustment. Future studies with a larger sample size are needed to validate the findings further.

S152

Demographics and Practice Patterns Among Minimally Invasive Surgery Fellowship Graduates

Mohammad Noubani 1; Caroline Smolkin2; Jie Yang2; Shanu Kothari3; Aurora Pryor2; 1UNC at Chapel Hill; 2Stony Brook University School of Medicine; 3PRISMA Health

Introduction: Previous reports show over 85% of general surgery residents choose to pursue fellowship training after completing residency, with numbers increasing every year. Also, it is widely reported that gender and cultural disparities continue to affect many specialties, particularly surgery. In this study, we evaluated demographics, practice patterns, and possible gender discrepancies among graduating MIS fellows.

Methods and Procedures: MIS fellows were surveyed and 169 results were received from fellows who completed training in the years: 2010, 2015–2019. Surveys collected were used to create a descriptive analysis of the trends in MIS fellowship training. The variables analyzed included ethnicity/culture, practice patterns (e.g. colorectal, advanced GI surgery, bariatric surgery, flexible sigmoidoscopy, foregut, HPB), and job finding measures. Loglinear regression model was performed to assess gender trend variation over training years.

Results: Fellows’ self-reported gender showed 64.7% male, 30% female, and 2.4% prefer not to say. The cohort of participants was described as 45.3% white, 5.3% African American, and 6.5% Hispanic or Latino (see Table 1 for complete racial/ethnic background breakdown). Further, results showed 87.1% of fellows work in MIS surgery with 91.8% reporting their fellowship experience facilitated their ability to find a job. Different practice patterns of the graduating fellows are described in Table 1. Moreover, the number of female fellows increased from 29 to 41% (Fig. 1). However, loglinear regressions showed no significant increase over years for percent of female MIS fellows [p-value = 0.0810, Relative risk = 1.1994 (95%CI: 0.9778, 1.4711)].

Conclusion: Overall, there is good evidence to support that fellowship training facilitates future career advancements. Further, MIS fellows have differential practice patterns. However, the disparities in culture and gender that are present in the wide scope of medicine are also clearly present in MIS fellowship training. This emphasizes that fellowship recruitment must reflect our commitment to improving diversity and bridging the gap in gender representation.

figure az

S153

Innate Skills in Robotic Surgery: A Cohort Study of General Surgery Residents’ Performance

Andres A Abreu, MD; Benjamin Rail, BS; Emile Farah, MD; Juan Tellez, BS; Ganesh Sankaranarayanan, PhD; Daniel J Scott, MD; Herbert J 3rd Zeh, MD; Patricio M Polanco, MD; University of Texas Southwestern Medical Center

Background: The increasing utilization of robotic surgery demands residency programs to advance their curricula and train the next generation of surgeons on this new platform. This surgical technique also requires residents to develop a new set of skills. There is a lack of evidence regarding the impact of innate robotic skills abilities on subsequent robotic surgical training. The purpose of this study is to determine the extent that innate abilities assessed on first exposure to the VR robotic simulator before robotic training, predict future performance in a proficiency-based curriculum.

Methods: Over the course of two academic years, 33 general surgery PGY-1 s underwent robotic surgery bootcamp at University of Texas Southwestern a week prior to initiating their intern year. During bootcamp, subjects were asked to complete three previously validated VR tasks on the SimNow® platform (ring-rollercoaster-2, bid-dipper needle-driver, and around-the-world needle-driving). Subsequently, their performance metrics (Score, Time, and Economy of motion [EOM]) were extracted retrospectively from their Intuitive® learning accounts. These metrics were followed during their residency training curriculum until they reached proficiency benchmarks previously established by experts' consensus. Outcomes assessed were scores at proficiency, attempts to proficiency, and time to proficiency. Spearman's rho and independent two-sample t-test were used; median (IQR) was reported.?

Results: Of 33 residents that achieved proficiency in the VR robotic curriculum, 16 (48%) completed the three VR bootcamp tasks requested and correspond to study subjects. The median score at bootcamp was 13.16 (IQR: 8.66 – 26.33). The EOM at bootcamp was 567.54 cm (IQR: 514.71– 678.47). The score at bootcamp did not show a significant correlation with any of the outcome parameters (p > 0.05). EOM at bootcamp showed a significant correlation with both, number of attempts to achieve proficiency (R2 = 0.706; p < 0.01) and time to proficiency (R2 = 0.662; p < 0.01). Residents with average EOM below the median required significantly fewer attempts (p < 0.05) and time to achieve proficiency (p < 0.05) compared to those with average EOM above or equal to the median.?

Conclusion: The acquisition of robotic surgery skills represents a new challenge for surgical residents and programs. Those with an innate ability to perform tasks with better coordination of movements within the space of the surgeon console (economy of motion) may be able to acquire robotic surgery skills at a faster rate. Our future efforts aim to explore the impact of innate skills in operative performance throughout residency.

figure ba

S154

Operative Dictations in Surgical Trainees: An Underutilized Educational Resource?

Britta J Han, MD, MS; Julie Clanahan, MD; Cameron Casson, MD; Arnab Majumder, MD; Bradley S Kushner, MD; Jeffrey A Blatnik, MD; Washington University in Saint Louis

Introduction: Operative notes are imperative to patient care and used for billing, quality assurance, and medical-legal conflicts. However, quality of operative notes vary, with many lacking critical details. Unfortunately, no standardized training exists. This pilot study sought to determine resident ability to dictate a comprehensive operative note and determine need for a formal curriculum.

Methods: Four laparoscopic ventral hernia simulations were organized for 38 surgical residents between post-graduate years 1–4. Residents completed a pre-survey about prior operative dictation experience. Afterwards, residents completed an operative dictation. Notes were graded using a rubric.

Results: Thirty-five residents completed the pre-survey (Table 1) and 38 residents submitted a dictation. Out of 33 points, mean score was 18.9 ± 5.4 (Junior: 17.9 ± 5.4; Senior: 20.9 ± 4.8). Total mean scores did not significantly differ between juniors and seniors (p = 0.10). Seniors scored higher on presenting relevant patient history and operative note headers (p = 0.04). Senior and junior residents scored similarly on ability to present operative details (p = 0.29).

Conclusion: These results suggest standard surgical training may not provide enough teaching and feedback on operative dictations. A formal curriculum could bolster trainees' ability to complete an effective operative note.

Table 1

Question

Category

Number (%)

PGY Level

Junior

Senior

13 (65.7)

12 (34.3)

Dictations completed residency (#)

0—4

5—25

26—50

 > 50

9 (25.7)

20 (57.1)

6 (17.1)

0 (0)

Effectiveness of your dictations for communicating operative steps

Not at all

Somewhat

Moderately

Very

Extremely

5 (14.3)

15 (42.9)

9 (25.7)

6 (17.1)

0 (0)

Comfort level with performing dictations without templates

Not at all

Somewhat

Moderately

Very

Extremely

21 (60.0)

12 (34.3)

2 (5.7)

0 (0)

0 (0)

My dictations need improvement

Strongly agree

Somewhat agree

Neutral

Somewhat disagree

Strongly disagree

24 (68.6)

9 (25.7)

2 (5.7)

0 (0)

0 (0)

I want further training in dictations

Strongly Agree

Somewhat agree

Neutral

Somewhat disagree

Strongly disagree

20 (57.1)

11 (31.4)

2 (5.7)

1 (2.9)

1 (2.9)

S155

Preparing for and Passing the Fundamentals of Laparoscopic Surgery (FLS) Exam Improves General Surgery Resident Operative Performance and Autonomy

Rana M Higgins, MD; Mia S Turbati, BS; Matthew I Goldblatt, MD; Medical College of Wisconsin

Background: The American Board of Surgery made the Fundamentals of Laparoscopic Surgery (FLS) exam a prerequisite for board certification in 2009. Some residency programs have questioned the need for a continued FLS testing mandate given limited evidence that supports the impact of FLS on intraoperative skills. The Society for Improving Medical Professional Learning (SIMPL) app is a tool to evaluate resident intraoperative performance. We hypothesized that general surgery resident operative performance would improve immediately after preparing for the FLS exam.

Methods: The national public FLS data registry contains the trainee name, national provider identification number, and exam date. After IRB approval, this dataset was matched with SIMPL resident evaluations from 2015–2021 and de-identified. Basic laparoscopic procedures, appendectomy and cholecystectomy, were included. SIMPL evaluations are scored in three categories: supervision required (Zwisch scale 1–4, 1 = show and tell and 4 = supervision only), performance (scale 1–5, 1 = exceptional and 5 = unprepared), and case complexity (scale 1–3, 1 = easiest and 3 = hardest). Residents that had at least one evaluation both three months before, and three months following, their FLS exam date, were included. Statistical analyses compared pre and post-FLS exam resident average operative evaluation scores.

Results: There were a total of 76 general surgery residents, and 573 resident SIMPL evaluations (71% cholecystectomy and 29% appendectomy), included in this study. The average post-graduate year (PGY) level of residents was 3.63. Residents required less supervision in laparoscopic cases performed before and after the FLS exam (2.84 vs. 3.03, respectively, p = 0.007). Residents received improved performance scores on cases before and after the FLS exam (2.70 vs. 2.43, respectively, p = 0.001). Case complexity did not differ before versus after the FLS exam (2.13 vs. 2.18, respectively, p = 0.202). PGY level significantly predicted evaluation scores with a moderate correlation. A sub analysis grouped by PGY level revealed a significant improvement before and after the FLS exam in supervision among PGY-2 residents (2.33 vs. 2.58, respectively, p = 0.04) and performance among PGY-4 residents (2.67 vs 2.04, respectively, p < 0.001).

Conclusions: Preparation for, and passing, the FLS exam improves resident intraoperative laparoscopic performance. After passing the FLS exam, general surgery residents exhibit greater operative independence and perform at a higher skill level. Preparing for and passing the FLS exam earlier in residency improves intraoperative independence. We recommend taking the exam in the first two years of residency to enhance the laparoscopic experience for the remainder of training. The FLS exam continues to be relevant in general surgery training.

S156

Sex Differences in Self-Efficacy for Laparoscopic Procedures: Confirmed at the Case Level

Rachel Jensen, MD 1; Ananya Anand, MD1; LaDonna Kearse, MD2; James R Korndorffer Jr., MD, MHPE1; 1Stanford University; 2Howard University

Background: Studies suggest that there are key differences in operative experience based on a trainee's sex. A large-scale self-efficacy survey of general surgery residents after the 2020 American Board of Surgery In-Training Examination found that female sex was associated with decreased perceptions of self-efficacy for graduating PGY5 residents in all 4 laparoscopic procedures included on the survey (cholecystectomy, appendectomy, right hemicolectomy, and diagnostic laparoscopy). We sought to determine whether these differences were reflected at the case level when considering operative performance and supervision using an operative assessment tool (SIMPL OR).

Methods: Supervision and performance data reported through the SIMPL OR platform for the same laparoscopic procedures were aggregated for PGY5 residents in 2020, representing the same cohort of individuals who completed the self-efficacy survey. Independent t-tests and multiple linear regression were used to determine the relationship between trainee sex and supervision/performance ratings.

Results: For aggregate laparoscopic cases (n = 2708), male residents rated their performance higher than females (3.57 vs. 3.26, p < 0.001, 1 = critical deficiency, 5 = exceptional performance) and reported less supervision (3.15 vs. 2.85, p < 0.001, 1 = show and tell, 4 = supervision only); similar findings were seen when looking at attending reports. Attending sex did not significantly predict supervision or performance, while case complexity and female sex of trainee significantly affected both supervision and performance (p < 0.001); see Table 1.

Discussion: Female residents perceive themselves to be less self-efficacious at core laparoscopic procedures compared to their male colleagues. Comparison to more case-specific data confirms that female residents receive more supervision and lower performance ratings. This creates a cycle in which female residents receive less operative autonomy, preventing the opportunity to establish self-efficacy. Further research should identify opportunities to break this cycle and consider gender identity beyond the male/female construct.

Table 1 Multiple linear regression model for supervision and performance

 

Factor

Beta Coefficient (SE)

P-value

Supervision

Case complexity

-0.176 (0.024)

 < 0.001

 

Trainee sex

-0.309 (0.033)

 < 0.001

 

Attending sex

0.044 (0.027)

0.102

Performance

Case complexity

-0.093 (0.024)

 < 0.001

 

Trainee sex

-0.317 (0.034)

 < 0.001

 

Attending sex

0.039 (0.027)

0.151

S157

Major Adverse Cardiac Events After Elective Bariatric Surgery: Can Machine Learning Improve Prediction of this Serious Incident?

Gustavo Romero-Velez, MD1; Jerry Dang, MD, PhD1; Juan Barajas-Gamboa, MD2; Terrence Lee, PhD2; Andrew T Strong, MD 1; Stavros Stefanopoulos, MD1; Lorenzo Braghieri, MD1; Agam Bansal, MD1; Salvador Navarrete, MD1; Ricard Corcelles, MD1; John Rodriguez, MD2; Maan Fares, MD1; Matthew Kroh1; 1Cleveland Clinic; 2Cleveland Clinic Abu Dhabi

Introduction: Machine learning (ML) is an emerging type of artificial intelligence developed with the potential to predict and improve clinical outcomes such as adverse events, based on hidden pattern recognition embedded in the data. Major adverse cardiac events (MACE) after bariatric surgery have been reported with an incidence of 0.1% but are associated with significant morbidity and mortality. MACE after bariatric surgery has been analyzed using traditional statistical methods including logistic regression, however, studies reporting ML for MACE prediction in bariatric surgery remain limited. As such, the objective of this study was to evaluate and compare MACE prediction models in bariatric surgery using traditional statistical methods and ML.

Methods: This was a retrospective study of the MBSAQIP database, including patients from 2015 to 2019. A binary-outcome MACE prediction model was generated using three different modeling methods: (1) main-effects-only logistic regression, (2) neural network with a single 48 tanh-node hidden layer, and (3) XGBoost model with a max depth of 3. The same set of predictor variables and random split of the total data (80/20 training/validation) were used to train and validate each model. Overall performance was compared based on the area under the receiver operating curve (AUC).

Results: A total of 755,506 patients were included, of which 0.1% experienced a MACE. Of the total sample, 79.6% were female, 47.8% had hypertension, 26.2% had diabetes, 23.7% had hyperlipidemia, 8.4% used tobacco, 1.9% had previous PCI, 1.2% had a history of MI, 1.1% had previous cardiac surgery, and 0.6% had renal insufficiency. There were baseline differences between the patients that experienced MACE including older ager (54.0 vs 44.5, p < 0.001), male predominance (36% vs 20%, p < 0.001), higher BMI (46.7 vs 45.3, p < 0.001), prevalence of diabetes (48% vs 26%, p < 0.001), prevalence of hypertension (79% vs 47%, p < 0.001), previous PCI (12% vs 2%, p < 0.001), previous cardiac surgery (9% vs 1%, p < 0.001). The AUC for the three different MACE prediction models was: 0.778 for logistic regression, 0.765 for neural network and 0.772 for XGBoost. While the AUC appeared to be similar, the risk prediction histogram for neural network shifted in a smoother fashion.

Conclusion: ML techniques to predict MACE in bariatric surgery performed similarly to the logistic regression model. The ML models developed with data from MBSAQIP achieved good discriminant function in predicting MACE. ML can help surgeons for patient selection and to identify individuals who may be at elevated risk for MACE after bariatric surgery.

S158

Shifting Paradigms: Protocol Implementation to Reduce Length of Stay for Bariatric Surgery Following the Pandemic at the University of Nebraska Medical Center

Cyril Kamya, MD; Kyle Bavitz, MS; Corrigan L Mcbride, MD; University of Nebraska Medical Centre

The aim of this study was to evaluate the effectiveness of our new protocols targeted at reducing length of stay (LOS) for our bariatric surgery patients during the pandemic. Secondary outcomes included comparisons of readmission and complications compared to baseline data. With the increased adoption of ERAS after surgery, same day (SDD) and early next day discharge (NDD) for bariatric surgery is becoming more of a reality. Some series have demonstrated better patient outcomes for next day discharges. Khorgami et al. found that patients discharged on POD1 after LRYGB had fewer complications than patients discharged later (2.6% vs 4.1%, respectively; p < 0.001).

Methods: Our database was analyzed identifying patients who underwent LRYGB and LVSG from April-Nov/21. Mean LOS and complication rates including re-admission in this baseline group were documented. This was compared to a cohort who underwent the surgeries between Dec/21-Feb/22 and were managed within our new protocols for SDD and NDD.

Criteria for SDD and NDD were the MBSAQIP low acuity criteria.

SDD and NDD protocols included bedding patients in Phase 2 recovery for their post-operative stay, minimizing sedation after surgery, early discharge teaching, labs before discharge (CBC, BMP). Family involvement/presence was required.

Statistical Analysis: Fisher exact tests (due to small numbers in some categories) were used to compare the proportions between the two groups for complications. Two-sided tests of equivalence with P < 0.05 were considered statistically significant. Two sample T-test was used to compare mean length of stay between our groups.

Results:

  • 195 patients who underwent bariatric surgery in the baseline group and 87 patients in the early discharge cohort were included. (No significant differences were identified in baseline characteristics).

  • Statistically significant decrease in mean LOS comparing baseline group (34.5 h) and next day PACU discharges (25 h). P = 0.004. (See Fig. 1)

  • No increase in complication rate from the early discharge cohort against baseline group. (P = 0.014). (See Fig. 2)

figure bb

Figure 1

figure bc

Figure 2

Conclusions: SDD and NDD in carefully selected bariatric surgery patients is feasible with good outcomes. With ERAS protocols as a foundation and a multidisciplinary approach, this can be achieved in spite of pressures placed on bariatric units by the pandemic.

References

  • Khorgami et al. Fast track bariatric surgery: safety of discharge on the first post operative day after bariatric surgery. Surg Obes Relat Dis 2017; 13: 273–80

S159

Ventricular Repolarization Dispersion Changes after Bariatric Surgery: Short-term ECG Changes Associated with Improved Diastolic Function

Michal Schafer 1; Fredric Pieracci, MD2; Johannes von Alvensleben, MD1; Alexander Morton2; 1University of Colorado; 2Denver Health

figure bd

Introduction: Bariatric surgery is known to have an immediate therapeutic effect on excess weight loss, insulin resistance, and cardiometabolic health. Importantly, bariatric surgery has been shown to induce reverse bi-ventricular remodeling, and it substantially improves myocardial strain and diastolic function. However, cardiac imaging and its post-processing are not readily available in the outpatient setting or in safety-net hospitals and require a highly trained team of providers. Recent insights from ECG machine learning studies identified T-wave characteristics associated with clinical myocardial diastolic dysfunction. In this study, we evaluated the effect of bariatric surgery using the post-processing of standard 12-lead ECGs in order to evaluate more efficiently and effectively the effect of bariatric surgery on cardiovascular health.

Methods and Procedures: Patients (n = 43) undergoing bariatric surgery with pre-operative ECG and ECG within 1 month from surgery obtained as a part of primary care follow-up were evaluated. Standard 12-lead ECGs were digitized and pre-processed using a machine learning algorithm to describe specific T-wave features of the patient population. Precordial ECG leads V4 to V6 and lead I sensitive to repolarization measurements were considered for the analysis. T-wave down slope described by the interval between T-wave peak to T-wave end (TpTe) (Figure) was measured using automatic custom in-house software. TpTe interval is an established and validated marker of abnormal myocardial repolarization (higher value = larger dispersion) and is associated with echocardiographic markers of diastolic dysfunction. Pre- and post- T-wave measurement datasets were analyzed using specific pair-wise comparison tests as dictated by the variable distribution.

Results: Pre- vs post-surgery pair-wise comparison is graphically depicted in Figure. In comparison to pre-surgery, patients post- surgery had significantly decreased TpTe interval in lead-I (70 ± 13 vs 64 ± 10 ms, P = 0.023) and in lead-V5 (72 ± 13 vs 65 ± 11 ms, P = 0.041). In total of 38 lead-I and 36 lead-V5 specific paired ECG T-wave signals were considered for analysis (5 lead-I and 7 lead-V5 ECGs were removed from analysis due to noise or poor quality). There were no significant differences in TpTe interval in leads V4 and V6.

Conclusions: Patients post-bariatric surgery are exhibiting signs of improved myocardial repolarization. These changes were observed only within 1 month from surgery indicative of rapid myocardial remodeling and improvement in ventricular diastolic function. Our future studies will focus on the comparison of ECG changes with echocardiographic images and with standard markers of insulin resistance and metabolism.

S160

Preliminary Experience of Endoscopic Sleeve Gastroplasty Performed in Deep Sedation: Is It Feasible? A Single Center Retrospective Study

Elisa Reitano, MD 1; Pietro Riva, MD1; Maria Vannucci, MD2; Mathieu Zappaterra, MD3; Michel Vix, MD1; Didier Mutter, MD, PhD, FACS, FRSM1; Jacques Marescaux, MD, FACS, HonFRCS, Hon, FASA, Hon, APSA1; Silvana Perretta, MD, PhD1; 1IRCAD France; 2University of Turin, Italy; 3Nouvel Hôpital Civil, CHRU-Strasbourg, Strasbourg, France

Introduction: Endoscopic sleeve gastroplasty (ESG) is an emerging bariatric procedure currently performed under general anesthesia with orotracheal intubation (OTI). Despite the lack of clear guidelines, several studies showed the feasibility of several advanced endoscopic procedures under deep sedation (DS), without significant modification of patient's outcomes and adverse events rate. The aim of this study is to compare our preliminary experience of ESG in deep sedation with a matched controlled group of ESG patients under OTI.

Methods and procedures: From December 2016 and January 2021 fifty patients who underwent primary endo-sleeve gastrectomy (ESG) or ESG revision under DS, and fifty patients who underwent primary ESG under OTI, were retrospectively matched and analyzed. Demographic data, comorbidities, weight, BMI, operative times, intubation rate, administration of post-operative drugs for nausea and vomiting, hospital stay, readmissions, complications and 3 months weight loss were compared. Univariate and multivariate analysis were performed to find a correlation between the type of anesthesia and the pre-clinical and clinical variables. A subgroup analysis of patients undergoing primary ESG was performed too.

Results: Among the 50 patients in deep sedation, 21 patients (42%) underwent primary ESG, while 29 patients (58%) underwent ESG revision with resuturing due to loss of gastric tubulisation. No DS patient required intubation. Multivariate analysis correlated DS with younger age (p = 0.006), lower preoperative BMI (p = 0.002), and lower operative time (p ≤ 0.001). Forty-two DS patients (84%) and 10 (20%) OTI patients were treated in outpatient's care (p ≤ 0.001).

Considering only the primary ESG group, DS patients were younger (p = 0.006), mostly females (p = 0.001) with a lower preoperative BMI (p = 0.0027),showed lower operative times (p = 0.003), and a higher rate of outpatient's care (p = 0.047). No differences in terms of ASA score (p = 0.487), Mallampati score (p = 0.325) and% weight loss (p = 0.599) at 3 months follow-up were found between SD and OTI patients. There were no differences in the number of sutures used between the groups (p = 0.616). DS patients required less postoperative drugs such as Ketoprofen (p = 0.012) Paracetamol (p = 0.005), Tramadol (p ≤ 0.001), and antiemetics (p = 0.006). There were no rehospitalization in both DS and OTI group.

Conclusions: Endoscopic suturing under deep sedation is feasible in selected patients. This practice allows to increase the outpatient's care and reduce the use of postoperative drugs while granting the same results in terms of postoperative weight loss.

S161

Application of an Individualized Metabolic Surgery Score: Bariatric Procedure Selection Based on Diabetes Severity

Wissam Ghusn, MD 1; Karim Al Annan, MD1; Donna Maria Abboud, MD1; Pearl Ma, MD2; Benjamin Clapp, MD3; Konstantinos Spaniolas, MD4; Robert A. Vierkant1; Andres Acosta, MD, PhD1; Omar Ghanem, MD1; 1Mayo Clinic; 2Fresno Surgery School of Medicine; 3El Paso Bariatric Surgery; 4Stony Brook Medicine

Introduction: Patients with severe obesity [body-mass index (BMI) > 50 kg/m2] present with a higher prevalence of type-2-diabetes mellitus (T2DM). Roux-en-Y gastric bypass (RYGB) and sleeve gastrectomy (SG) are effective in enhancing T2DM remission. Aminian et al. constructed an individualized metabolic surgery score (IMS) which aims to guide procedural selection (RYGB vs SG) according to long-term glycemic control with ≥ 5 years of follow-up. Due to the severity of this disease, patients with severe obesity may have variable T2DM remission rates in response to different bariatric surgeries. In this study, we aim to apply this constructed model to patients with BM1 > 50 kg/m2 and T2DM having undergone bariatric surgery.

Methods: We performed a systematic review of electronic medical records of patients from the Mayo Clinic. We included all patients who had a BMI ≥ 50 kg/m2, T2DM, and had either RYGB or SG between 2008–2017 with at least 5 years of post-surgical follow-up. Associations of variables with remission status were assessed using t-tests and Fisher exact tests as appropriate. The discriminatory capability of IMS score on remission was examined using areas under the ROC curve.

Results: We included 78 (84%) RYGB patients (age 52.5 ± 11.81 years, BMI 56.1 ± 5.8 kg/m2, 69% female) and 15 (16%) SG patients (51.2 ± 9.2 years, BMI 59.2 ± 5.8 kg/m2, 60% female) with no baseline demographic differences. In our cohort, 50% of RYGB had T2DM remission compared to 27% for SG. When divided by IMS categories, remission rates for RYGB compared to SG were 82% vs 50% in mild (0–25), 56% vs 30% in intermediate (25–95), and 14% vs 0% in severe (> 95) IMS scores. Lower HbA1C values at baseline, shorter duration of T2DM prior to surgery, lower number of T2DM medications at baseline, no insulin use at baseline, lower IMS score, and mild-intermediate IMS score categories were associated with a higher remission rate (p < 0.001). Logistic regression models including the IMS point system (i.e., not categorized) revealed a ROC of 0.81 for predicting the remission status of our patients (Fig. 1). A subset analysis for RYGB and SG patients had ROC of 0.83 and 0.75, respectively.

Conclusion: RYGB appears to be more effective than SG in terms of T2DM remission in patients with severe obesity in all IMS categories despite our limited sample size, especially with regards to the severe category.

Fig. 1 ROC curves for diabetes prediction models for both surgeries (A), RYGB (B), and SG (C)

figure be

S162

The Decreasing Cost of Robotic Bariatric Surgery

Meagan Read, MD1; Emily Grimsley 1; Haroon Janjua1; Ricardo Pietrobon2; Paul Kuo1; Salvatore Docimo1; 1University of South Florida; 2SPORE Data

Introduction: As the surgical robot is assumed to be a fixed, indirect cost, we hypothesized that increased volume of robotic procedures will decrease overall costs per patient undergoing bariatric surgery over time. The model of minimally invasive bariatric surgery with either robotic or laparoscopic approach was chosen.

Methods and Procedures: Patients who underwent an elective, initial gastric Roux-en-Y bypass (bypass) or gastric sleeve (sleeve) for morbid obesity were selected from Florida AHCA (Agency for Health Care Administration) database from 2017 – 2021. Inflation-adjusted total, operative, and non-operative costs per patients were collected. Cost-over-time ($/patient/year) and change in cost-over-time were calculated for open (bypass only), laparoscopic, and robotic cases. Linear regression using cost as the dependent variable generated predictive parameters. Density plots utilizing area under the curve demonstrated cost overlap.

Results: Among 73 hospitals, 7,685 bypasses (181 open, 3,360 laparoscopic, 4,144 robotic) and 30,600 sleeves (21,285 laparoscopic, 9,315 robotic) were included. Total robotic costs were approximately 1.5-fold higher (p < 0.001) than laparoscopic for both procedures. For bypass, the laparoscopic approach demonstrated the lowest total and operative average cost ($15,310 / $6,379, respectively) compared to open or robotic approaches ($17,360 / $8,921 open vs $20,527 / $10,196 robot, respectively). Minimally-invasive approaches to sleeves demonstrate the same pattern, with the average laparoscopic overall cost significantly less than the robotic approach ($9,967 vs $14,919). However, when evaluating cost-over time, robotic costs are actually decreasing significantly while laparoscopic costs have consistently demonstrated a slight increase over the study period ($-184 vs $51/patient/year, p < 0.001).In contrast, while the robotic approach to bypass demonstrates an average increasing cost-over-time compared to laparoscopy ($88 vs $-570, respectively), the most recent year shows a divergence in this trend with a large decrease in both total and operative costs ($-1438, $-477, respectively).

There is a greater similarity in overall total cost between the minimally invasive approaches for gastric bypass than for sleeve (74 vs 56%, respectively). This difference is likely due to the narrow range of total costs for the evaluated laparoscopic sleeve cases, as well as the large overlap in non-operative costs between robotic and laparoscopic bypasses (90%).

Conclusions: Robotic bariatric surgery total and operative costs are decreasing at a higher rate than laparoscopic costs, a pattern not seen in other specialty operations utilizing robotic technology.

figure bf

S163

Safety of Bariatric Surgery Among Patients with Advanced CKD: a Nationwide Retrospective Cohort Study

Yung Lee, MD; Lea Tessier, MD; Audrey Jong, BHSc; Adelia Padoan, BHSc; Yasith Samarasinghe, BHSc; Tyler McKechnie, MD; Aristithes Doumouras, MD, MPH; Dennis Hong, MD, MSc; McMaster University

Introduction: Bariatric surgery is a procedure commonly performed to treat patients with obesity. As chronic kidney disease (CKD) has higher prevalence with obesity, there is growing need to understand the effects of CKD on bariatric surgical outcomes for the purposes of guiding patient-specific treatment plans. This study aims to determine on a national scale the short-term safety and healthcare utilization for performing bariatric surgery on severe CKD or end stage renal disease (ESRD) patients.

Methods: Data was obtained retrospectively from the National Inpatient Sample (NIS) database. Patients diagnosed with severe obesity who underwent bariatric surgery from 2015 to 2019 were included and divided into the following 3 cohorts based on diagnosis: non-CKD, CKD (stages 3 and 4) and ESRD. Outcomes analysed include perioperative outcomes (mortality, post-operative ICU admission, cause-specific complications) and healthcare utilization (length of stay and admission costs). Univariate and multivariate regression was utilized to determine the association between perioperative outcomes to CKD status. Confounders such as patient characteristics, disease characteristics, and hospital characteristics were used in the adjusted analysis.

Results: A total of 140,758 bariatric procedures documented in the NIS database were included (98.6% non-CKD, 1.1% CKD, 0.4% ESRD). No significant difference was found for in-hospital mortality for stage 3 and 4 CKD patients (aOR: 0.76 [0.22–2.63]) and ESRD patients (aOR: 2.16 [0.39–11.89]) relative to non-CKD patients. No significant difference was also observed for in-hospital complications for CKD patients (aOR: 1.15 [0.94–1.42]) and ESRD patients (aOR: 1.04 [0.71–1.52]) relative to non-CKD patients. Compared to non-CKD patients, CKD patients had an increased odds of post-operative ICU admission (aOR: 4.21 [3.29–5.39]). Mean length of stay was slightly longer for CKD (aOR: 0.14 [0.04–0.23]) and ESRD patients (aOR: 0.27 [0.10–0.43]) compared to non-CKD patients, however this difference was not clinically meaningful. ESRD patient demonstrated a slight increase in admission cost relative to non-CKD patients (aOR: $1982.65 [$677.07-$3288.22]).

Conclusion: The present study demonstrates short-term safety of bariatric surgery for patients with a pre-existing diagnosis of CKD or ESRD. While these populations may have a slightly higher length of stay and hospital costs, their diagnosis should not be contraindicative for this procedure as these differences are not clinically meaningful. Future studies should investigate long-term outcomes of bariatric surgery for CKD and ESRD patients.

S164

Evaluating the Spanish Readability of American Society for Metabolic and Bariatric Surgery Centers of Excellence websites

Theo Sher, BA 1; Joseph Sujka, MD2; Rahul Mhaskar, MPH, PhD1; Christopher DuCoin, MD, MPH, FACS2; 1University of South Florida, Morsani College of Medicine; 2Department of Surgery, University of South Florida

Introduction: We sought to evaluate accredited Bariatric Centers of Excellence (COE) for Spanish readability via their websites to determine ease of access of information for Spanish speakers. To evaluate readability, we implemented a survey tool based on five categories of Spanish visibility that we assigned to each website.

Methods: Between May 24th and August 18th of 2022, the websites of 103 COE accredited by American Society for Metabolic & Bariatric Surgery (ASMBS) and the American College of Surgeons (ACS) were evaluated and assigned one of the following Spanish Visibility Categories: "Full Spanish translation available," "Spanish material available from homepage," "1-click access to language services from homepage via Spanish link," "1-click access to language services from homepage via English link," or "Language services difficult to find."

The United States was divided into 4 regions: West, Midwest, Northeast, and South. Each COE was assigned to its respective region. A regional breakdown of Spanish visibility was calculated by dividing each category count by the number of institutions in each region.

County populations over age 5 and county populations over age 5 that "speak Spanish at home" and "speak English less than very well" were obtained from the US Census Bureau's 2009–2013 American Community Survey and appended to the dataset. Differences in the distribution of these populations across the Spanish Visibility Categories were investigated using the Mann–Whitney U test.

Results: Only 25% of websites were translatable to Spanish, and a regional discrepancy was found with 61% translatable in the West, 19% in Northeast, 19% in Midwest, and 15% in South. The median county population which "speaks Spanish at home" is higher where websites were translatable to Spanish (median 131,575, range 37,545, 3,653,910) compared to where websites were not translatable (median 108,823, range 8825, 1,313,020) (p = 0.039). Similarly, the median county population which "speaks English less than very well" is higher where websites were translatable (median 64,905, range 15,905, 1,677,980) compared to where websites were not translatable (median 45,843, range 2480, 656,030) (p = 0.029).

Conclusion: Healthcare disparities continue to be an ongoing struggle. This study points out that Spanish readability for ASMBS ACS COE websites should be improved regardless of regional or county differences in Spanish speaking populations. We believe it would be valuable for Bariatric COE websites to have standards for readability of Spanish and other languages.

S165

Do All Roads Lead to Rome?: A Retrospective Analysis on Surgical Technique in Roux-en-Y Gastric Bypass

Alexander H Vu, MD1; Jessica Chiang, MD 1; Chau Hoang, MD1; Yunzhi Qian, MPH2; Nilufar Tursunova, MD1; Jaein Nha, BA1; George Ferzli, MD, FACS1; 1New York University Langone Health, Department of General Surgery; 2Department of Nutrition, University of North Carolina at Chapel Hill

Background: New York University (NYU) Langone Health system is a designated MBSAQIP bariatric center, with 10 bariatric surgeons using different surgical techniques. We present a retrospective analysis that compares individual surgeon technique in Roux-en-Y gastric bypass (RNYGB) and that identifies potential associations with perioperative morbidity and mortality.

Methods: All adult patients who underwent Roux-en-Y gastric bypass between 2017–2021 at the NYU Langone Health campuses were evaluated via electronic medical records and MBSAQIP 30-day follow up data. We also surveyed all 10 practicing bariatric surgeons (Tables 1–5) to analyze the relationship between their techniques and total adverse outcomes (including bleeding, surgical site infections (SSI), any other morbidity eg nausea, vomiting, urinary tract infection, postoperative pneumonia, deep vein thrombosis and pulmonary embolism etc., mortality, readmission or reoperation). Bleeding, SSI, mortality, readmission, and reoperation were specifically sub-analyzed via Pearson chi-square test.

Results: Data from 3,815 adult patients were collected. Among them 54 (7.59%) out of 711 patients who underwent RNYGB encountered an adverse outcome (Tables 1–5).

We found that certain surgeon practices e.g. using flat positioning (vs. Trendelenburg), Covidien* staplers (vs. Intuitive*, Medtronic*, Ethicon*), gold stapler load (vs. blue, white, purple, and tan), sewing the enterostomy (vs. staple), routine pre-op EGD, and not proceeding with RNYGB if hiatal hernia present preoperatively have statistically significant lower total adverse outcome rates and lower bleeding rates compared to other surgical techniques.

We also found that a laparoscopic approach (vs. robotic), uni-directional stapler (vs. bi-directional), performing JJ anastomosis first (rather than GJ first), and dividing the mesentery (vs not dividing the mesentery) lowered the total adverse outcome rates. There was no statistically significant difference in SSI, readmission, reoperation, or mortality amongst the surgical techniques.

Conclusions: We found that certain surgical practices in Roux-en-Y gastric bypass within our bariatric surgery group had significant effects on the rates of total adverse outcomes and bleeding but ultimately had no effect on SSI, readmission, reoperation, or mortality rates. Our findings warrant further investigation into the aforementioned techniques via multivariate regression models.

Keywords: gastric bypass, Roux-en-Y, surgical technique, weight-loss surgery, bariatric surgery, perioperative management.

*These are trademarked companies.

figure bg
figure bh
figure bi
figure bj
figure bk
figure bl
figure bm
figure bn
figure bo
figure bp

S166

Weight Loss and Clinical Outcomes Following Primary Versus Secondary Roux-en-Y Gastric Bypass: A Multi-Institutional Experience

Brigitte Anderson, BS1; Bryan Robins, MD2; James A Fraser, MD 1; Luke Swaszek, MD3; Caroline Sanicola2; Neil King3; Aurora Pryor, MD2; Konstantinos Spaniolas, MD2; Renee Tholey, MD1; Francesco Palazzo, MD1; Alec Beekley, MD1; Talar Tatarian, MD1; 1Department of Surgery, Thomas Jefferson University Hospital; 2Department of Surgery, Stony Brook University Hospital; 3Department of Surgery, Temple University Hospital

Introduction: Revisional bariatric surgery is increasing for weight recurrence and return of co-morbidities. Herein, we compare weight loss and clinical outcomes following primary Roux-en-Y Gastric Bypass (P-RYGB), adjustable gastric banding to RYGB (B-RYGB), and sleeve gastrectomy to RYGB (S-RYGB) to determine if primary and secondary RYGB offer comparable benefits.

Methods: Participating institutions' EMRs and MBSAQIP databases were used to identify adult patients who underwent P-/B-/S-RYGB (2013–2019) with minimum one-year follow-up. Weight loss and clinical outcomes were assessed (30 days, 1-/5-years). Multivariable model controlled for year, institution, patient/procedure characteristics, and excess body weight (EBW).

Results: 768 patients underwent RYGB: P-RYGB n = 581 [75.7%]; B-RYGB n = 106 [13.7%]; S-RYGB n = 81 [10.5%]. Secondary RYGB procedures increased in recent years. The most common indications for B-RYGB and S-RYGB were weight recurrence/nonresponse (59.8%) and GERD (65.4%), respectively. Mean time from index operation to B-RYGB or S-RYGB were 8.9 and 3.9 years, respectively. After adjusting for EBW, 1 year %TWL and %EWL were greater after P-RYGB (56.7%, 30.4%) versus B-RYGB (49.4%, 26.2%) or S-RYGB (37%, 15.6%). Overall comorbidity resolution was comparable. Secondary RYGB group had longer adjusted mean length of stay (OR 1.17, p = 0.071) and higher risk of pre-discharge complications or 30-day reoperation (Table 1).

Conclusion: Primary RYGB offers superior short-term weight loss outcomes compared to secondary RYGB, with decreased risk of 30-day reoperation.

figure bq

S167

Sexual Dysfunction in Women Undergoing Bariatric Surgery

Rachel Tran, BS 1; Fernando Moreno-Garcia, BS1; K Paige Mihalsky, MD2; Valori Kruse, BA3; Kathy Cummings, RN3; Caitlin Stenberg, APRN, CNP2; Fernando Mier Giraud, MD2; Lieschen H Quiroz, MD3; Laura E Fischer, MD, MS2; 1University of Oklahoma College of Medicine; 2Department of Surgery, University of Oklahoma Health Science Center; 3Department of Obstetrics and Gynecology, University of Oklahoma Health Science Center

Introduction: Obesity has been linked to sexual dysfunction. Studies suggest weight loss may be associated with improvements in sexual dysfunction, however more studies are needed to examine this relationship. Bariatric surgery results in significant, sustainable weight loss and provides an opportunity to study weight loss-related improvement in sexual function in women.

Methods: This is an interim analysis of an ongoing, prospective, single-institution observational study looking at urinary incontinence and sexual dysfunction in women enrolled in a bariatric surgery program. Participants completed the Pelvic Organ Prolapse/Urinary Incontinence Sexual Questionnaire (PISQ-12) at program enrollment, pre-operatively, and 3, 6 and 12-months post-operatively. Demographic data including BMI was collected. Chi-Square test was used for categorical data and student's t-test for continuous data. Univariate analysis was used to examine risk factors for sexual dysfunction. P-values < 0.05 were considered significant.

Results: 108 patients enrolled in the study and 60% have completed surgery. Mean age was 45.6 ± 2.0 years and BMI was 49.8 ± 10.0. At 1-year, mean BMI was 31.1 ± 6.5 (35.1 ± 9.5% total body weight loss, %TBWL). Women reported decreasing sexual activity over time with 67.6% at enrollment and 35.7% at 1-year (p = 0.013). Sexual dysfunction improved by 6 months (NS). PISQ-12 score was not associated with BMI (p = 0.889) or %TBWL (p = 0.598) in. Sexual activity (p < 0.0001) and sexual dysfunction score (p < 0.0001) were both associated with increasing age.

Conclusion: In this population, sexual dysfunction scores improved over time, although decreasing sexual activity after bariatric surgery limited statistical power. Older age is associated with decreased sexual activity and increased sexual dysfunction. These data suggest that sexual dysfunction in women undergoing bariatric surgery is influenced by additional factors beyond weight loss alone.

Table 1 Sexual dysfunction in women in a bariatric surgery program

 

Baseline

Pre-operative

3-months

6-months

1-year

Surveys Completed/Total Enrollment

73/108

19/43

15/29

15/26

10/27

BMI

49.8 ± 10.0

44.3 ± 6.0

37.2 ± 5.5

33.6 ± 5.7

31.1 ± 6.5

%TBWL

–-

6.6 ± 4.0%

21.8 ± 4.7%

29.5 ± 6.0%

35.1 ± 9.5%

Sexually Active

67.6%

46.3%

50.0%

53.6%

35.7%

PISQ-12 Score

36.0 ± 6.0

35.5 ± 5.2

34.7 ± 6.0

37.5 ± 7.1

39.0 ± 4.5

S168

The Impact of the COVID-19 Pandemic on Sleeve Gastrectomy (SG) Cost: A 10-Year Trend at a Major Academic Institution

Danny Mou, MD, MPH1; Edward Lin, DO, MBA1; Scott Davis, MD1; Mathu Kumarasamy, BS2; Zachary Grunewald, PhD, MS, RDN, LD1; Kristin Cooper, MBA3; Katherine Fay, MD1; Carrie P Hall, MD1; Jennifer Bibler, BA3; John F Sweeney, MD1; Gregory J Esper, MD, MBA4; Elizabeth M Hechenbleikner, MD 1; 1Emory University Department of Surgery; 2Emory Healthcare Performance Analytics; 3Emory Healthcare Financial Services; 4Emory Healthcare

Introduction: The COVID-19 pandemic has significantly impacted metabolic and bariatric surgery (MBS) practices due to large-scale cancellations along with shortages in nursing staff, supplies, and space. Specifically, rising hospital-level contract labor costs have made delivery of care more expensive. These national trends likely affect the cost of SG, the most common weight loss surgery performed in the U.S. We analyzed SG hospital-level direct variable costs, hypothesizing that there have been significant increases in the post-COVID pandemic era.

Methods: Hospital cost-accounting software (MicroStrategy, Tysons, VA) was retrospectively reviewed for total average direct variable costs per SG at a large academic hospital from 2012 to 2022. No SG cases were performed from 3/11/2022 – 5/4/2022. Actual costs were obtained, not insurance charge estimates or hospital projections.

Total average direct variable SG costs components are categorized as follows: 1) Labor and benefits (costs of lab technicians, nurses), 2) Implants, 3) Drug costs during hospitalization, and 4) Medical/surgical supplies (non-implant related operating room items). The pre-COVID period (2018 to first half of 2020) and post-COVID period (second half of 2020 to first quarter of 2022) costs were compared via student's t-test.

Results: A total of 2,507 patients were included with an average length of stay (LOS) at 1.75 days. No difference in LOS was noted pre-COVID (1.65 days) vs. post-COVID (1.62 days; p = 0.90). Quarterly total average direct variable costs and its components from 2012 to 2022 are shown (Fig. 1). Pre-COVID costs differed significantly from post-COVID costs for labor and benefits cost ($3,032 pre-COVID to $3,801 post-COVID, p = 0.0) and drug cost ($1,521 pre-COVID to $807 post-COVID, p = 0.03; Fig. 2).

Conclusions: In the post-COVID pandemic period, there was an overall 1.3-fold increase in labor and benefits costs for SG, with a 1.6-fold increase noted in the last year. Deferred SGs with a salaried workforce in addition to pandemic-related increases in contract labor rates have likely led to higher cost per SG. In contrast, a significant decrease in drug costs per SG was noted which may be related to institutional efforts to operationalize and support an enhanced recovery protocol (ERP) for MBS patients starting May 2020 right when elective cases resumed. Sustained SG cost escalations without a concomitant increase in reimbursement is unsustainable. Potential solutions include deploying resources to quality improvement interventions like ERPs and shifting MBS cases to the outpatient setting to mitigate hospital-level cost constraints.

Figure 1

figure br

Figure 2

figure bs

S169

New Kid on the Block: Pre-operative Erector Spinae Plane Block Should be Considered a Viable Option for Laparoscopic Abdominal Surgeries

Fang Ting Wan 1; Shuen Ern Chin1; Ryan Gwee2; Yvette Chong3; Angie Au-yong4; Abbey Matthews4; Ma Wai Wai Zaw4; Sui-An Lie4; Leonard Loh4; Darius Aw3; Cheryl Chong3; Leonard Ho3; Jia Lin Ng3; Sharmini Sivarajah3; Winson Tan3; Fung Joon Foo3; Frederick Koh, MBBS, MRCS, MMED, FRCS3; 1Lee Kong Chian School of Medicine, Nanyang Technological University; 2Yong Loo Lin School of Medicine; 3Colorectal Service, Department of General Surgery, Sengkang General Hospital; 4Department of Anaesthesiology, Sengkang General Hospital

Introduction: The Erector Spinae Plane (ESP) block is a recent development in the field of regional anaesthesia and has been increasingly explored for abdominal surgeries to reduce opioid use and improve pain control. Colorectal cancer is the commonest cancer in multi-ethnic Singapore and requires surgery for curative treatment. ESP is a promising alternative in colorectal surgeries, but few studies have evaluated its efficacy in such surgeries. Therefore, this study aims to evaluate the use of ESP blocks in laparoscopic colorectal surgeries to establish its safety and efficacy in this field.

Methods: A prospective two-armed interventional cohort study comparing T8-T10 ESP blocks with conventional multimodal intravenous analgesia for laparoscopic colectomies was conducted in a single institution in Singapore. The decision for doing an ESP block versus conventional multimodal intravenous analgesia was made by a consensus between the attending surgeon and anesthesiologist. Single-shot bilateral ESP blocks were performed between the T8-T10 levels pre-induction by a trained consultant anaesthesiologist, under ultrasound guidance. Outcomes measured were total intra-operative opioid consumption, post-operative pain control and patient outcome. Post-operative pain control was measured by pain score, analgesia use, and amount of opioids consumed. Patient outcome was determined by presence of ileus. Univariate analyses were performed using Chi-squared and Mann–Whitney-U test as appropriate and multivariate analyses were performed using multinomial logistic regression and multiple linear regression, with p < 0.05 considered as statistically significant.

Results: A total of 146 patients were included, of which 30 patients received an ESP block. Overall, the ESP group had a significantly lower median opioid usage both intra-operatively and post-operatively (p < 0.05). Fewer patients required patient-controlled analgesia and rescue analgesia post-operatively for pain control (p < 0.001) amongst the ESP group. Pain scores were similar and post-operative ileus was absent in both groups. Multivariate analysis found that the ESP block had an independent effect on reducing intra-opioid consumption (p < 0.05). Multivariate analysis of post-operative opioid use and pain scores did not yield statistically significant results.

Conclusions: The ESP block was an effective alternative regional anaesthesia for colorectal surgery that reduced intra-operative and post-operative opioid use while attaining satisfactory pain control. This is important, considering the prevalence of colorectal cancer in Singapore and prevalence of opioid side effects, such as ileus and PONV, which affect recovery after colorectal surgery. Pre-operative ESP blocks should be considered a viable option for laparoscopic abdominal surgery and may also be considered post-operatively in the future as an alternative to post-operative opioid analgesia.

S170

Increasing Access to Pathology Services in Low- and Middle-Income Countries Through Innovative Use of Telepathology

Krsna K Kothari, BS1; Joseph Okello Damoi, MBChB, MMED, FCS2; Nebras Zeizafoun, MD3; Penninah Asiimwe, BS2; Katie Glerum, BA, MPH1; Moses Binoga Bakaleke, MBChB2; Angellica Giibwa, MBChB2; Melissa Umphlett, MD3; Jerome D Waye, MD1; Michael L Marin, MD1; Linda Zhang, MD 1; 1Department of Surgery, Mount Sinai Health System, New York, New York, USA; 2Kyabirwa Surgical Center, Global Surgical Initiatives Inc., Kyabirwa, Jinja, Uganda; 3Department of Pathology, Mount Sinai Health System, New York, New York, USA

Introduction: In many low- and middle-income countries (LMICs), surgical care and management is limited by access to pathology services. In Uganda, the pathologist to population ratio is 1/1,000,000. In response to this, the Kyabirwa Surgical Center in Jinja, Uganda created a telepathology service in collaboration with an academic institution in New York City. This study demonstrates the feasibility of implementing a telepathology model to supplement the critical pathology needs of a low-income country.

Methods: This was a retrospective, single-center study of an ambulatory surgery center with pathology capability using virtual microscopy. The microscope was controlled by the remote pathologist, who reviewed histology images transmitted across the network in real time. Demographics, clinical history, surgeon's preliminary diagnosis, and pathologist's final conclusion were collected from the pathology reports and center's electronic medical record.

Results: Nikon's NIS Element Software was used as the telemicroscopy platform to view and interact with the digitized light microscope images. Internet connectivity was established by an underground fiber optic cable. The lab technician and pathologist were able to use the software after a two-hour tutorial session. Pathology sessions were held approximately once every two weeks reviewing an average of 4 patients in a one hour session, with each patient's sample consisting of up to 21 slides. The average time from the collection of tissue to the final pathology report was 18 days. Between April 2021 to June 2022, tissue samples of 113 patients were examined. This telepathology program was able to confirm the surgeon's clinical suspicion of cancer in 65% (n = 64) of tissue samples. Of the cases not tagged as suspicious for malignancy by the surgeon, 12% (n = 6) had malignant features detected by the pathologist. The most common malignancies on histology were squamous cell carcinoma of the esophagus, invasive ductal carcinoma of the breast, and colorectal adenocarcinoma.

Conclusion: With increasing availability of video conference platforms and network connections, a telepathology model can be easily implemented in countries with poor access to pathology services. Telepathology is an emerging field that can be used by surgeons in LMICs to confirm clinical suspicion and histological diagnosis of malignancies to assure appropriate treatment for patients.

Figures

Fig. 1 Schema of Telepathology Process at Kyabirwa Surgical Center

figure bt

Fig. 2 Zoom Conference of Pathology Meeting using Nikon NIS Element Software with coordinator, lab technician, pathologist, and Mount Sinai pathology and surgical residents

figure bu

Fig. 3 Control Panel in Closer View of the NIS Element Software

figure bv

S171

Study of Effectiveness Intraoperative Indocyanine Green Angiography for the Identification of the Parathyroid Glands During Total Thyroidectomy

Roman Parfentiev, PhD; Viktor Grubnyk, PhD; Yuriy Grubnik, Professor; Volodymyr Grubnik, MD; Vadym Ilyashenko, PhD; Vladislav Slepov; Odessa National Medical University

Postoperative hypocalcemia is a common complication of thyroidectomy. This problem is most often associated with accidental devascularisation or excision of the parathyroid glands (PG).

Aim was study near-infrared (NIR) fluorescent imaging with intraoperative parathyroid gland indocyanine green (ICG) angiography to help identify and preserve parathyroid glands during total thyroidectomy in order to avoid postoperative hypocalcemia.

Methods: For period from 2017 to 2019 years, 58 patients in Odessa regional hospital were underwent total thyroidectomy. Indications for surgery were multinodular goiter (42), thyroid cancer (11) and Graves' disease (5). By randomization all patients were divided into two groups: in the first group, 28 patients underwent standard total thyroidectomy, in the second group 30 patients underwent NIR-assisted total thyroidectomy with ICG angiography. Parathyroid autofluorescence was detected using a near infrared/indocyanine green endoscopic system (Karl Storz, Germany). Serum calcium and parathyroid hormone (PTH) levels were compared between the two groups of patients in 1, 7 -15 days after surgery and then 3, 6 months later.

Results: In the first group, on based of a visual assessment of the PG, autotransplantation the PG were conduct in only 4 cases (in 3 cases—one gland, in one case—2 glands). In the second group, autotransplantation was performed in 11 patients (in 8 cases—one gland, in 2 cases—two, in one case—3). The transient postoperative hypocalcemia was observed in 5 patients of the first group (17,86%) and in the 2 patients of second group (6,67%) on 5–10 postoperative days. In the first group 1 patient at 3 months after surgery had permanent hypocalcemia.

Conclusion: This technique is safe and provides improved detecting and assessment of the perfusion of the PG. The need for autotransplantation of the PG can be determined more objectively using ICG imaging than simple visualization.

S172

Novel Endoscopic Management of Gastroenterological Anastomosis Leakage by Injecting Gel-Forming Solutions: A Large Animal Experimental Study

Yusuke Watanabe, MD, PhD 1; Keiko Yamamoto, MD, PhD2; Zijian Yang, MD3; Haruna Tsuchibora, BSc4; Masakazu Fujii, MD5; Shinya Kosuge, MD5; Shoko Ono, MD, PhD2; Takayuki Kurokawa, PhD6; Naoki Sakamoto, MD, PhD7; 1Clinical Research and Medical Innovation Center, Institute of Health Science Innovation for Medical Care, Hokkaido University Hospital; 2Division of Endoscopy, Hokkaido University Hospital; 3Department of Advanced Transdisciplinary Sciences, Faculty of Advanced Life Science, Hokkaido University; 4Division of Soft Matter, Graduate School of Life Science, Hokkaido University; 5Department of Gastroenterological Surgery II, Graduate School of Medicine, Hokkaido University; 6Department of Advanced Transdisciplinary Sciences, Hokkaido University Faculty of Advanced Life Science; 7Department of Gastroenterology and Hepatology, Graduate School of Medicine, Hokkaido University

Background: Anastomotic leakage (AL) after gastrointestinal surgery remains a challenging complication requiring surgical or non-surgical treatment. While various reliable and effective endoscopic therapeutic techniques, including stent placement, endoscopic suturing, and endoscopic internal drainage or vacuum therapy are available, no definitive expert consensus or evidence-based interventions exist. Despite the advantages of endoscopic intervention, related complications, such as bowel perforation and stent migration, remain. As an alternative endoscopic therapeutic option for AL, we developed a novel endoscopic submucosal injection method using gel-forming mixed solutions intended to close an anastomotic leak. We also sought to evaluate outcomes and tissue healing after the intervention.

Materials and Methods: An AL was created on porcine anterior gastric walls by cutting a mechanical stapling line. After one day, it was treated by injecting the gel-forming solutions. Gel formation occurs through a chemical reaction between calcium carbonate- and chitosan-based solutions. The formed gel is resistant to both acids and alkalis, usually lasting about a week within the gastrointestinal wall. After the endoscopic/laparoscopic confirmation of the AL, a calcium carbonate-based solution was endoscopically injected into the submucosal layer around the leakage site in a circular manner, followed by a chitosan-based solution. The defect was physically closed by pressure from the forming gel. The therapeutic intervention was completed after the endoscopic confirmation of defect closure. Thereafter, outcomes were recorded and histopathological effectiveness was evaluated.

Results: Four consecutive pigs with an AL underwent this intervention. Each endoscopic injection was completed in less than 10 min. No significant complications were observed for three weeks after the intervention. All AL sites were macroscopically healed. Histopathologic findings at three weeks showed that the wall defect was filled with collagen fibers that had grown around the site of the muscle layer tear. A part of the formed gel remained in the surrounding submucosa with a mild foreign body reaction. No tissue necrosis was observed.

Conclusion: This pre-clinical study demonstrated that the therapeutic injection method for gastroenterological AL using gel-forming solutions could be an alternative endoscopic treatment for AL, especially in patients with severe conditions or comorbidities. The optimal targets of this treatment would be AL without poor blood flow or intense hypertrophic scar lesions.

figure bw

S173

The Impact of Preoperative Telemedicine on Satisfaction with Care Among Bariatric Surgical Patients

Ruyan Zhang, BA; Paige L Martinez, MS; Colline Prassad, BSN; Eric T Volckmann, MD; Ellen H Morrow, MD, MS; Jennwood Chen, MD; Natalie A Turner, RN, BSN; Anna Ibele, MD; University of Utah School of Medicine

Introduction: Telemedicine was widely utilized during the COVID-19 pandemic, and was valuable for maintaining patient access to surgical care when in-person visits were limited. For patients with work constraints or long travel times, telehealth may continue to be a valuable tool for access to surgical care. One criticism of telehealth is that it may lead to inferior patient satisfaction and quality of care. We sought to compare whether visit type (telemedicine vs. office visits) impacted patient satisfaction and surgical outcomes among patients going through preoperative bariatric surgical consultation during the COVID-19 pandemic.

Methods: Patients who had bariatric preoperative appointments at our tertiary academic medical center from mid-March 2020- March 2022 were invited to complete post-visit Press Ganey surveys designed to measure satisfaction with care. Following surgery, patients received invitations to complete Hospital Consumer Assessment of Healthcare Providers and Systems (HCAHPS) surveys to measure satisfaction with care during surgical admission. We reviewed survey responses, comparing patients who engaged in preoperative care exclusively via telehealth, those with exclusively traditional in-person visits, and patients who received both types of preoperative visits.

Results: Sixty-five patients completed preoperative Press Ganey surveys. Satisfaction scores were similar across all groups. For the domains of Access to Care, Satisfaction with Care Provider, and Overall Assessment of Practice, median scores were 100 for all groups. Fifty patients completed HCAHPS following their hospitalization for bariatric surgery. While these demonstrated more variation, all three groups scored their satisfaction with physician communication at the highest level. Five patients experienced complications requiring intervention, reoperation, or readmission during the study period- all of these occurred in the office visit only group, suggesting that telehealth visits did not compromise patient compliance or impact complication risk.

Conclusion: Our study suggests that quality of care and patient satisfaction is comparable for telemedicine and in-person visits without adversely affecting 30-day postoperative complication rates. Given these findings, physicians, hospitals, and payers should continue to advocate for telehealth to improve bariatric surgical access.

figure bx

S174

Comparative Study of Video Recorders in Laparoscopic Surgery

Valentina Duran Espinoza, MD 1; Mariana Miguieles Schilling, MD1; Maria Ines Gaete Dañobeitía, MD1; Javier Ignacio Vela Ulloa, MD1; Felipe Silva Peña1; Cristian Jarry Trujillo, MD, MSc1; Julian Varas Cohen, MD, MSc1; Pablo Achurra Tirado, MD1; Martín Inzunza Agüero, MD2; 1Experimental Surgery and Simulation Center, Department of Digestive Surgery, Pontificia Universidad Católica de Chile; 2Departament of Digestive Surgery, Pontificia Universidad Catolica de Chile

Introduction: Laparoscopic surgery is the approach of choice for multiple procedures. Likewise, video recording of these surgeries has become widespread. Currently, the market offers medical recording devices (MRD) with an approximate cost of 5,000 USD and alternative non-medical recording devices (NMRD) with a cost ranging from 120–200 USD. So far, no comparative studies between the available devices have been found. We aim to compare the perception of the quality of videos recorded by MRD and NMRD in a group of surgeons and surgical residents.

Methods: Cross-sectional study. An elective laparoscopic cholecystectomy was recorded with four video recording devices simultaneously: three NMRD(AverMedia 60fps, Elgato 30fps, Hauppauge 30fps) and one MRD (MediCapture 20fps). An online survey was designed with Likert-type questions comparing recordings in a blind, random fashion. The following aspects were assessed: definition of anatomical structures(DA), fluidity and speed of movements(FS), similarity with the screen view in the Operating Room(ORsim), and overall quality of the video recording(OQ). Surgeons and surgical residents(SR) who routinely perform laparoscopic surgery were included. A sample size of 34 participants was calculated to achieve 80% power, and 5% alpha, with large effect size. Descriptive and nonparametric analytical statistics were applied. Results were analyzed using JMP-15.

Results: Forty complete surveys were collected, 80%(n = 32) were responded by surgeons and 20%(n = 8) by RS. Regarding DA, MRD had the worst outcome. Medians were compared between NMRDs Elgato, AverMedia, and Hauppauge obtained a median score of 4.7[2–5],4.4[2–5],4.5[2–5] respectively; while MRD obtained 4[3–5]. When analyzing FS, NMRDs obtained a median of 4.0[3–5],4[2–5],4[2–5] while the tested MRD obtained a 3[1–5] score. Regarding ORsim, the NMRD obtained 3.8[3–5],3.7[2–5],3.8[2–5] versus 2.7[1–5] for the MRD. In regards to OQ, 4.5[3–5],4.4[3–5],4.4[3–5] were obtained for NMRD. Statistically significant differences were found between NMRDs and MRD's score 3.1[2–4]. In a comparative screen, 70% chose NMRD Hauppauge as the highest quality device, and 78% chose Medicapture as the poorest. This difference was statistically significant.

Conclusions: For recording laparoscopic procedures, NMRDs had a better overall score than the MRD Medicapture, with Hauppauge being the best evaluated. NMRDs could be considered as a less expensive and better quality alternative for video recording of laparoscopic surgeries.

S175

Augmented Reality Visualization and Guidance for Laparoscopic Procedures: Two-Center Clinical Translation Experience

Trong Nguyen, PhD1; Xinyang Liu, PhD1; Will L Plishker, PhD2; David A Geller, MD3; Timothy D Kane, MD1; Raj Shekhar, PhD 1; 1Children's National Hospital; 2IGI Medical Technologies; 3University of Pittsburgh Medical Center

Introduction: The use of laparoscopic ultrasound (LUS) during laparoscopic procedures means that the surgeon must mentally integrate multimodality images, a task that is inefficient, subjective and variable with expertise. We developed a novel augmented reality (AR) visualization technology that substitutes mental integration with explicit image fusion. By attaching electromagnetic (EM) tracking sensors to the imaging devices, the technology fuses live LUS with live laparoscopic video (Fig. 1). For ablations, we further track the needle and overlay its path on the AR view for precise needle placement. Following laboratory and animal validation experiments, this study reports on early clinical translation of the technology.

figure by

Methods and Procedures: With FDA and IRB approvals, we performed 11 human cases, 5 at Children's National Hospital (CNH) and 6 at University of Pittsburgh Medical Center (UPMC) between September 2020 and October 2021. The CNH cases were laparoscopic cholecystectomies in children. At CNH, we used Storz vision system and BK FlexFocus 700 ultrasound. The 6 UPMC cases involved liver ablations using Stryker vision system, BK R5000 ultrasound, and Solero microwave ablation system. All cases were preceded by AR system calibration performed a day in advance. After calibration, the laparoscope, the LUS probe, and tracking fixtures were sterilized. They were reassembled at the start of the procedure. The AR visualization was performed for up to 5 min prior to starting the actual surgery. Live LUS, laparoscopic video and AR feeds were recorded.

Results: All 5 cases at CNH were successful and correctly registered AR visualization of the surgical field was produced (Fig. 2). The feedback from the operating surgeon was positive, who noted that the AR technology allowed clear visualization of the confluence of the cystic duct and the common bile duct and could reduce bile duct injuries. At UPMC, 3 out of 6 cases were successful (Fig. 3). Various technical and operational factors (inadequate device preparations, improper patient position relative to EM field generator, and patient's knee implant interfering with EM tracking) prevented successful AR visualization in the other 3 cases. The feedback from the operating surgeon was that while the AR capability could be helpful, particularly to less-experienced surgeons, further system simplification was needed.

figure bz
figure ca

Conclusion: The study confirmed technical and clinical feasibility of the described laparoscopic AR visualization and guidance technology and helped identify areas of improvement for its continued development. When used routinely, the technology could make laparoscopic procedures more precise, faster, and safer.

S176

Bringing Artificial Intelligence to the Operating Room: Edge Computing for Real Time Surgical Phase Recognition

Sarah Choksi, MD, MPH 1; Skyler Szot, BS2; Chengbo Zang, BS2; Kaan Yarali, BS2; Yuqing Cao, BS2; Zixuan Xiang, BS2; Daniel P Bitner, MD1; Zoran Kostic, PhD2; Filippo Filicori, MD1; 1Lenox Hill Hospital, Northwell Health, Department of Surgery, Intraoperative Performance Analytics Laboratory; 2Columbia Unviersity, Department of Electrical Engineering

Background: Automation of surgical phase recognition is a key effort in the development of Computer Vision (CV) algorithms, for both workflow optimization and video based assessment. CV is a form of Artificial Intelligence (AI) that allows interpretation of images through a deep learning based algorithm. The improvements in Graphic Processing Unit (GPU) based computing allow researchers to apply these algorithms for recognition of content in videos in real-time. Edge computing, where data is collected, analyzed, and acted upon in close proximity to the collection source, can help meet the demands of workflow optimization by providing algorithm application in real-time. We applied a previously developed surgical phase recognition algorithm by our group to 10 Robotic Inguinal Hernia Repairs (RIHR) to obtain phase predictions during the procedure.

Methods: Our phase recognition algorithm was developed with 211 videos of RIHR originally annotated into 14 surgical phases (adhesiolysis, blurry, catheter insertion, mesh placement, mesh positioning, out of body, peritoneal closure, peritoneal scoring, positioning suture, preperitoneal dissection, primary hernia repair, reduction of hernia, stationary idle and transitionary idle). Using these videos, a deep learning model with the Resnet50 backbone was trained and validated in an 80:20 split to automatically recognize surgical phases as previously described by our group. This was transferred to our GPU, Nvidia® Jetson Xavier™ NX edge computing device.

Results: This model was tested on 10 inguinal hernia repairs from 4 surgeons in real-time as shown in Fig. 1. The model was improved with post-recording processing methods such as phase merging into 7 final phases (peritoneal scoring, mesh placement, preperitoneal dissection, reduction of hernia, out of body, peritoneal closure, and transitionary idle) and averaging of frames. Predictions were made once every second with a latency of ~ 250 ms. Figure 2 shows real-time inference during RIHR as the case is occurring. The accuracy of the real-time predictions was determined by comparing the outputs with annotations performed by a senior surgery resident, which was considered to be the ground truth (Fig. 3). Accuracy of the real-time predictions ranged from 58.9% to 76.9% with an average accuracy of 67.3%.

Conclusion: A real-time phase prediction of RIHR using a CV deep learning model was successfully implemented with an average accuracy of 67.3%. This real-time CV phase segmentation system can be useful for monitoring surgical progress and be integrated into software to provide hospital workflow optimization.

figure cb
figure cc
figure cd

S177

One-year Review of Real-time Artificial Intelligence (AI)-aided Endoscopy Performance

Shuen Ern Chin 1; Fang Ting Wan1; Jasmine Ladlad2; Fung Joon Foo, FRCS2; Frederick Koh, MBBS, MRCS, MMed, FRCS2; 1Lee Kong Chian School of Medicine, Nanyang Technological University; 2Colorectal Service, Department of General Surgery, Sengkang General Hospital, SingHealth

Background: Colonoscopies have long been the gold standard for detection of premalignant neoplastic lesions of the colon, thereby reducing preventable deaths. Our previous study trialed a real-time Artificial Intelligence (AI)-aided colonoscopy over a period of three months and found significant improvements in collective and individual endoscopist's adenoma detection rates compared to baseline polypectomy rates. As an expansion, this study evaluates the one-year performance of AI-aided colonoscopy in the same institution.

Methods: A prospective cohort study was conducted in a single institution in Singapore. The AI software used in the study was GI Genius™ Intelligent Endoscopy Module, US-DG-2000309 © 2021 Medtronic. Between July 2021 and June 2022, polypectomy rates in non-AI-aided colonoscopies and AI-aided colonoscopies were calculated and compared. Statistical analysis was performed via a Chi-squared test, with p < 0.05 considered as statistically significant. Some of the AI-aided colonoscopies were recorded and four independent authors video-reviewed these recordings. A "hit" was defined as a sustained detection of an area by the AI. If a polypectomy was performed for a "hit", its histology was reviewed. Additional calculations for polyp detection rate, adenoma detection rate, and adenoma detection per colonoscopy (ADPC; defined as the mean number of adenomas detected per scope which detected adenomas) were performed. Cost analysis was also performed to determine if subscription to the AI program was cost effective for the institution.

Results: 2433 AI-aided colonoscopies were performed between July 2021 and June 2022, and compared against 1770 non-AI-aided colonoscopies performed in the same time period. AI-aided colonoscopies yielded significantly higher rates of polypectomies (33.6%) as compared with non-AI-aided colonoscopies (28.4%) (p < 0.001). Amongst the AI-aided colonoscopies, 1050 were recorded and reviewed, and a final count of 843 were included for additional analysis. The polypectomy to "hit" ratio was 57.4%. Polyp detection rate was 45.6% and adenoma detection rate was 32.4%. The ADPC was 2.08. Among all the polyps that reviewed for histology, 25 polyps (3.13%) were detected to be sessile serrated adenomas. Cost analysis of the two endoscopy suites where the AI program was used showed that the increase in polypectomy rates in AI-aided colonoscopes led to an increase in revenue which covered the subscription cost with an excess of USD 20,000.

Conclusions: AI-aided colonoscopy is a cost effective means of improving polyp and adenoma detection rate and may help advance colorectal cancer screening in Singapore by improving the quality of colonoscopy.

S178

Artificial Intelligence During Colonoscopy: The First Thousand Procedures

Miroslaw Szura, Prof, MD, PhD, FACS1; Tomasz Gach, MD, PhD, FACS 1; Zofia Orzeszko, MD2; Pawel Bogacki, MD1; Jan M Krzak, MD, FACS3; Beata Markowska, MD1; Maciej Szura, MD4; Maciej Matyja, MD, PhD1; 1Jagiellonian University; 2Andrzej Frycz Modrzewski Krakow University; 3Sygehus Sønderjylland; 4The University Hospital in Krakow

Colonoscopy, as one of the few studies in medicine, has precisely defined quality criteria. The parameters of the colonic clearing for the study, the caecal intubation rate, adenoma detection rate (ADR) and advanced adenoma detection rate (AADR) were precisely determined for this purpose. It is not known whether they are high enough and whether they can be raised even more. The introduction of the next generation of endoscopes with 4 K resolution and computer-aided diagnostics based on artificial intelligence may be the next step to improve the quality of tests performed.

The aim of the study was to retrospectively analyze the results of colonoscopy performed with or without the support of artificial intelligence with 4 K endoscopes.

Material and Method: 2,000 thousand patients undergoing colonoscopy were included in the study. Group I consisted of 1000 patients with AI, group II—1000 patients without AI. Both groups were examined with endoscopes with 4 K image resolution. The analysis was based on a review of database records. Patient age, sex, colon cleansing for study, cecal intubation rates, Adenoma detection rate (ADR) and Advanced adenoma detection rate (AADR), polyp detection rate (PDR) were assessed in each group. The total number of polyps in each patient was also assessed.

Results: Both groups were comparable in relation to age (mean age in group I—62.4, II—63.2), sex, bowel cleansing (I-91.5% vs II-92.1%) and the frequency of cecum intubation (I -95.5% vs II-94.9%). Adenoma detection rate (ADR) and Advanced adenoma detection rate (AADR) were respectively for group I and II (29.7% vs 28.9%) and (6.91% vs 7.13%). Also, the polyp detection rate (PDR) was similar in both groups (46.7% vs 44.9%). The total number of polyps in the patient in Group I was 2.7 in Group I and 1.9 in Group II (p < 0.05).

Conclusions: The introduction of artificial intelligence in colonoscopy did not increase the detection of adenomas. On the other hand, the number of polyps detected in patients increased. Prospective randomized trials should elucidate the role of artificial intelligence in colonoscopy.

S179

SAGES Consensus Recommendation on Surgical Video Data Structure, Use and Exploration (for Research in Artificial Intelligence, Clinical Quality Improvement, and Surgical Education)

Jennifer A Eckhoff, MD 1; Mark Talamini2; Elan Witkowski1; Danail Stoyanov3; Keno Maerz4; Maria Altieri5; Carla Pugh6; Mehran Anvari7; Nicolas Padoy8; Pietro Mascagni8; Lena Maier-Hein4; Amin Madani9; Daniel A Hashimoto1; Martin Wagner10; Yutong Ban1; Thomas Ward1; Paresh Shaw11; Speidel Stefanie12; Filippo Filicori2; Guy Rosman, PhD1; Ozanan R Meireles, MD, FACS1; 1Surgical Artificial Intelligence and Innovation Laboratory, MGH; 2Northwell Health; 3University College London; 4German Cancer Research Center; 5Penn Medicine; 6Stanford University; 7McMaster University; 8IHU, Straßbourg; 9Bentley University; 10University of Heidelberg; 11NYU Langone Health; 12National Center for Tumor Diseases

Background: A vast amount of imaging data is generated daily in the operating room from each procedure. This data holds tremendous value for surgical research, clinical outcome assessment, quality control, and education. The data lifecycle is influenced by different facets of the data and its contexts, such as data structure and its interplay with the acquisition, storage, and sharing; the data use, through preprocessing and annotation, data exploration; for research, education, and clinical use, and data governance, covering all policies and regulations associated with the data. Therefore, there is a universal need among all stakeholders involved in surgical data science to establish standardized frameworks for encompassing all aspects of the data lifecycle to ensure the quality and purpose of the data.

Methods: Working groups of 48 representatives from surgery, computer science, and industry were built focusing on: (1) Data Structure, (2) Data Use, (3) Data Governance, and (4) Data Exploration. The results from panel discussions in the working groups were evaluated in a two-round Delphi process. Results are shown in Fig. 1.

Results: A consensus among experts was reached, conceptualizing and structuring each domain to generate concise guidelines for using surgical imaging data. We identified the key stakeholders influencing the critical principles associated with the data lifecycle and incorporated their viewpoints as we created the proposed guidelines. Standardization of the data structure should include the format and quality, data sources, documentation, and metadata, as well as accounting for bias within the data. Guidelines for the data use should be comprehensive, easily understandable, and applicable to all use cases and stakeholders. Data governance should be accessible to all stakeholders and account for legal and ethical considerations around the data. To promote scientific exploration of the data, the data acquisition and processing should reflect data diversity and be adaptable to future applications and utilizations of the data. Moreover, most participants agreed that surgical video data is essential for research, clinical applications, and education among the surgical community.

Conclusion: This consensus framework presents an essential step towards generating standardized and diverse surgical imaging databanks, accounting for all stakeholders involved in data generation and use throughout its lifecycle. Additional explorations addressing the innate characteristics of video data and the individual aspects of the data lifecycle remain to be done. Finally, this consensus-based framework lays the foundation for quality control in imaging data-driven research, education, and clinical quality improvement.

figure ce

S180

Development of Real-Time, Intraoperative Artificial Intelligence to Assess Critical View of Safety in Laparoscopic Cholecystectomy

Masahiro Kawamura 1; Yuichi Endo1; Atsuro Fujinaga1; Misako Kimura2; Aika Ejima2; Yusuke Matsunobu2; Hiroki Orimoto1; Shota Amano1; Takahide Kawasaki1; Yoko Kawano1; Takashi Masuda1; Teijiro Hirashita1; Tatsushi Tokuyasu2; Masafumi Inomata1; 1Department of Gastroenterological and Pediatric Surgery, Faculty of Medicine, Oita University; 2Faculty of Information Engineering, Department of Information and Systems Engineering, Fukuoka Institute of Technology

Background: The Critical View of Safety (CVS), proposed in 1995, is commonly advocated to prevent bile duct injuries (BDI) in laparoscopic cholecystectomy (LC). Although LC has become more widespread and the number of cases has increased, the incidence of BDI has remained stable. This is probably due to inconsistent application of CVS, poor understanding of CVS, and unreliable reporting. Objective CVS evaluation using artificial intelligence (AI) could help in quality audits and help reduce BDI. Development of deep neural network model to assess the criteria defining the CVS was previously reported as an objective CVS evaluation. The overall accuracy was 71.9% in postoperative evaluation for still images. The aim of this study is to develop a learning model that intraoperatively assesses the criteria of CVS during LC.

Method: 21 LC videos performed at our institution were retrieved and video segments of the hepatocystic triangle dissection were edited and converted to image frame at 30 fps. Still images from LC videos were annotated with the criteria of CVS's archivement, proposed by Sanford and Strasburg. Annotation for the criteria of CVS's archivement was labelled on each images, where the CVS could be seen from anterior or posterior view. We excluded the images from the training dataset, which the field of view was outside the CVS region of interest or images in which gauzes or forceps obstructed the field of view. A deep neuralnetwork model to assess each of three criteria of CVS's achievement was trained with 15 of the 21 annotation datasets and was evaluated with the remaining datasets. The overall accuracy of the training model was examined.

Results: A total of 5480 images from 21 LC videos were annotated. 4823 images from 15 LC videos were used to train the model, and 657 images from 6 LC videos were used to evaluate the model. The training model assessed the criteria of CVS's archivement with the overall accuracy of 71.7%.

Conclusion: The present study is the first to develop real-time, intraoperative AI to assess CVS. The results suggest the feasibility of intraoperative use of AI in the future. We will ensure the quality and quantity of annotation data and optimize the training parameters to improve the accuracy of CVS evaluation.

S181

Comprehension of fundamental Knowledge About Pediatric Endoscopic Surgery: A Cross-Sectional Study in Japan

Shinichiro Yokoyama 1; Yo Kurashima2; Satoshi Ieiri3; Atsuyuki Yamataka4; Hrioomi Okuyama5; Hiroo Uchida6; Tetsuya Ishimaru7; Satoshi Hirano8; 1Department of Pediatric Surgery, Hokkaido Medical Center for Child Health and Rehabilitation, Sapporo, Japan; 2Clinical Simulation Center, Hokkaido University Graduate School of Medicine, Sapporo, Japan; 3Department of Pediatric Surgery, Research Field in Medicine and Health Sciences, Medical and Dental Sciences Area, Research and Education Assembly, Kagoshima University, Kagoshima, Japan; 4Department of Pediatric General and Urogenital Surgery, Juntendo University School of Medicine, Tokyo, Japan; 5Department of Pediatric Surgery, Osaka University Graduate School of Medicine, Osaka, Japan; 6Department of Pediatric Surgery, Nagoya University Graduate School of Medicine, Nagoya, Japan; 7Division of Pediatric Surgery, Saitama Children's Medical Center, Saitama, Japan; 8Department of Gastroenterological Surgery II, Hokkaido University Faculty of Medicine, Sapporo, Japan

Background: Pediatric endoscopic surgery has become the alternative to conventional techniques with the development of medical equipment. However, there is no formal, standardized curriculum for pediatric endoscopic surgery and its requirement remains elusive. The purpose of this study is to determine the baseline knowledge of pediatric surgeons that is considered essential for pediatric endoscopic surgery.

Methods: Sixteen web-based multiple-choice questions based on the fundamental cognitive knowledge of pediatric endoscopic surgery were administered. The questions were created based on the fields covered by the Fundamentals of Laparoscopic Surgery TM (FLS) certification examination blueprints and eight specifics diseases of pediatric surgery relating to inguinal hernia, appendicitis, Hirschsprung disease, splenectomy, esophageal atresia, pyloroplasty, fundoplication and diaphragmatic hernia. Pediatric surgeons and pediatric surgical trainees participated in this study voluntarily. The results are expressed as mean and standard deviation unless otherwise stated. Student's t-test was used to analyze differences between the groups.

Results: A total of 122 surgeons participated through the Japanese Society of Pediatric Surgeons. The response rate was 95% (122/128). The total mean examination score of all participants was 79.4% (77.3 – 81.4%). There were no significant differences in total scores between the board-certified pediatric surgeons without an endoscopic surgical skill qualification and the non-board-certified pediatric surgeons (80.4% vs. 77.1%, p = 0.12). The endoscopic surgical skill qualified surgeons had significantly higher percentages of correct responses in specific subjects than board-certified pediatric surgeons and surgeons without pediatric board certification (94.3% vs. 82.9% vs. 77.5%, p = 0.0002). The FLS original subjects' scores were not significantly different among them. There was no significant difference in total score among the different PGY groups (p = 0.27). The mean score of surgeons who had experienced more than 200 cases of endoscopic surgery, including adult cases, was 83.2% (80.4 – 85.9%).

Conclusions: In this study, there was a significant difference in the degree of understanding of pediatric endoscopic surgical fundamental knowledge among the participants that are actively involved in pediatric surgery. It is important to note that fundamental knowledge in pediatric endoscopic surgery could also be learned from endoscopic surgery in adults, and that even those with many years of pediatric surgical experience may not have sufficient knowledge of endoscopic surgery. In order to perform surgery on patients safely, developing structured training referring to the validated training program effectively and maintaining the need for assessments for pediatric surgeons are essential.

S182

Outcomes after Peroral Endoscopic Myotomy for Zenker’s Diverticula (Z-POEM) and Correlation with Impedance Planimetry (EndoFLIP)

Simon Y Che, MD 1; Kristine Kutcha, MA1; Julia R Amundson, MD, MPH2; Stephanie Joseph, MD, MPH3; Shun Ishii4; Vanessa N VanDruff, MD2; Christopher J Zimmermann, MD1; Herbert M Hedberg, MD1; Michael B Ujiki1; 1NorthShore University HealthSystem; 2University of Chicago; 3Wayne State University; 4Kyorin University School of Medicine

Introduction: Zenker’s diverticulum is a false pulsion diverticulum of the cervical esophagus. It's found in the older adults and manifests with dysphagia. The purpose of this study is to describe our experience with Z-POEM and to detail our intraoperative FLIP measurements.

Methods and Procedures: We performed a single institution retrospective review of patients undergoing Z-POEM in a prospective database between 2014 and 2022. Distensibility index (DI) is measured by FLIP before and after myotomy. The primary outcome was clinical success. Secondary outcomes included technical failure, adverse events and quality of life assed by the reflux severity index (RSI) and dysphagia score. Statistical analysis was performed using the paired t-test and significance set at p < 0.05.

Results: 54 patients underwent Z-POEM with 30 FLIP measurements completed. We achieved technical success and clinical success in 54/54 patients and 46/54 patients (85%), respectively. Three patients (6%) experienced complications: two contained leaks and one post-operative pneumonia. Three patients with residual dysphagia underwent subsequent diverticulopexy, all of which had diverticula > 4 cm. Following myotomy, DI increased by 1.7 ± 1.2 mm2/mmHg (3.5 ± 1.2 to 5.0 ± 1.6, p < 0.001). In those with good clinical success, change in DI was 1.5 ± 1.1 mm2/mmHg. Significant improvement was found in RSI, but not dysphagia score (Table 1).

Conclusion: Z-POEM is a safe and feasible for treatment of Zenker’s Diverticulum without any cases of intraoperative abandonment. Large diverticula (> 4 cm) should be considered for concurrent diverticulopexy. A DI change of at least 1.0–2.0 mm2/mmHg may be appropriate to ensure complete myotomy.

Table 1 Quality of Life for Z-POEM patients

 

Preop

3 weeks

6 months

1 year

2 years

5 years

 

N = 32

N = 21

N = 15

N = 11

N = 8

N = 6

RSI (Mean ± SD)

19.2 ± 9.2

10.0 ± 8.6

6.0 ± 5.1

5.1 ± 4.8

2.7 ± 4.1

0.7 ± 1.0

p-value from preop

 

0.0003

0.0002

0.0006

 < 0.0001

0.0003

Dysphagia score (Mean ± SD)

1.9 ± 0.8

2.0 ± 1.1

1.3 ± 0.6

1.3 ± 0.6

1.1 ± 0.4

1.0 ± 0.0

p-value from preop

 

0.4293

0.0961

0.0805

0.1747

0.2080

S183

Achalasia Symptom Duration is Not Associated with Peroral Endoscopic Myotomy (POEM) Failure

Christina S Boutros, DO 1; Saher-Zahara Khan, MD1; Jamie R Benson, MD1; Megan Z Chiu, MD1; Joshua Lyons, MD1; Daniel A Hashimoto, MD2; Jeffrey M Marks, MD, FACS1; 1Department of Surgery, University Hospitals Cleveland Medical Center, Case Western Reserve Universit; 2Department of Surgery, University of Pennsylvania Hospital

figure cf

Introduction: Peroral endoscopic myotomy (POEM) is a well established technique to address achalasia, however little is known about the efficacy of POEM for patients with long standing achalasia. We hypothesize that patients with long standing achalasia will be more recalcitrant to POEM than patients with symptoms for a short duration.

Methods: We performed a retrospective analysis of patients with achalasia who received a POEM at a single tertiary care academic institution from 2012 – 2022. Patients were included if they had a POEM and a documented diagnosis of achalasia. Patients were excluded if they were missing data on diagnosis of achalasia, symptom duration, or missing pre- and post-operative Eckart score. Patients were grouped into cohorts based on symptom duration: < 1 year, 1–3 years, 4–10 years, > 10 years. POEM failure was defined as need for repeat intervention, symptom recurrence, and a high post-operative Eckart score. Univariate analysis was conducted for demographic, preoperative, and post-operative outcomes. Multivariate logistic regression was used to analyze the association between symptom duration and response to POEM.

Results: During the study period, 132 patients met inclusion criteria. 44 patients had symptoms for < 1 year, 42 patients had symptoms from 1–3 years, 26 patients had symptoms from 4- 10 years, and 20 patients had symptoms > 10 years. Patient age at surgery, sex, BMI, Charleston-Deyo Comorbidity Index, and number of patients with diabetes with and without end organ complications, connective tissue diseases, and ulcer disease did not differ amongst cohorts. Pre-operative Eckart scores were between 4 – 5 across groups (p = 0.31). Patients that had symptoms for greater than 10 years had significantly more endoscopic interventions prior to their POEM (30% vs, 60% p = 0.002). Post procedure Eckart scores ranged from 0.5 – 1.2 for all cohorts with no significant differences (p = 0.15). Patients in all cohorts experienced the same number of symptoms post POEM (p = 0.2). Symptom recurrence, need for repeat endoscopic intervention, repeat surgical intervention, or repeat POEM also did not vary across cohorts. Having symptoms of achalasia > 10 years did not increase the odds of having a higher post-operative Eckart score, worse dysphagia, regurgitation, or weight loss on multivariate logistical regression.

Conclusions: This data suggests that longer symptom duration is not associated with increased rates of POEM failure. This is promising as clinicians should not exclude patients for POEM eligibility based on duration of symptoms alone.

S184

Predictors of Reintervention After Peroral Endoscopic Myotomy

Saher-Zahra Khan, MD; Jamie Benson, MD; Christina Boutros, DO; Joshua Lyons, MD; Daniel Hashimoto, MD, MS; Jeffrey M Marks, MD, FACS, FASGE; University Hospitals Cleveland Medical Center, Department of Surgery

Several studies have assessed outcomes of patients undergoing peroral endoscopic myotomy (POEM) after previous endoscopic intervention of botulinum injection or pneumatic dilatation. These have had various results regarding rates of reintervention and repeat myotomy. We hypothesize that patients with previous endoscopic intervention(s) are more likely to require repeat intervention than treatment-naïve patients.

This is a retrospective cohort study of patients that underwent POEM for achalasia at a single tertiary care center by a single surgeon between 2011 and 2022. Patients were excluded if they had previous myotomy (POEM or Heller). The remaining patients were stratified into treatment-naïve patients (TN), those with single previous endoscopic intervention (SEI), and those with multiple previous endoscopic interventions (MEI). Primary outcome was need for repeat endoscopy, manometry or pH testing, intervention, or repeat myotomy. Multivariate logistic regression using preoperative and postoperative factors was completed to assess odds of needing repeat endoscopy for testing or intervention.

A total of 165 patients were included in the analysis, 90 TN, 36 SEI, and 39 MEI. Patients in the MEI group had significantly higher Charlson-Deyo Index (p = 0.018). There was a significant difference in surgeon experience (defined as distribution of patients among quartile of POEM performed) with 69% of MEI patients falling into the third and fourth quartiles of cases compared to 41% of TN and 53% of SEI. There were no other significant differences in demographics. There was a significant difference in severity of preoperative regurgitation (p = 0.008), however there was no difference in total Eckardt score (p = 0.28). There was no difference in the proportion of patients that required repeat endoscopy for testing or intervention (39% TN, 61% SEI, 49% MEI, p = 0.072) or repeat myotomy (3% TN, 6% SEI, 3% MEI, p = 0.74). In the logistic regression analysis, preoperative Eckardt (OR 1.52, [1.14, 12.03], p = 0.004) and postoperative Eckardt scores (OR 1.61, [1.08, 2.42], p = 0.021) were significantly associated with need for repeat endoscopy. There was no association among the SEI or MEI groups compared to the TN group.

There were no increased odds of needing repeat endoscopy for testing or intervention depending on endoscopic treatment with botulinum injection or dilatation prior to POEM, implying similar efficacy. However, there was an increased chance of having repeat procedure with higher Eckardt scores both preoperatively and postoperatively, which is clinically warranted if persisting or recurrent symptoms occur after POEM.

S185

An Inanimate Line Figure Tracing Model to Teach Endoscopic Scope Tip Control Skills Required for Endoscopic Submucosal Dissection (ESD)

Neil Mitra, MD, MBA 1; Yanni Hedjar, MD1; Giorgio Guiulfo, MD1; Poppy Addison, MD1; David Wang, MD1; Michael Ma, MD1; Hunsani Anagammana, MD1; Timothy Pistell, BA, JD2; Richard L Whelan, MD1; 1Division of Colon and Rectal Surgery, Department of Surgery, Lenox Hill Hospital, Northwell Health, New York, NY 10,028.; 2UMDNJ Medical School, Newark NJ, USA

Introduction: ESD, the gold standard for en bloc removal of large sessile colon polyps, requires unique skills that most endoscopists do not possess. In snare polypectomy the scope tip is held still while the snare is advanced, the polyp encircled, and the snare closed. In ESD, the polyp's mucosal/submucosal attachments are separately and systematically incised/detached; to cut tissue the scope tip with "hot" needle knife extended must be moving. Learning to finely control the moving scope tip in the clinical setting is a challenge. This preliminary study assessed an inanimate model designed to teach fine scope tip control by tasking trainees to trace line figures using an endoscopic "pen".

Methods: A hollow plastic tube with a 5 × 6 cm "window" cutout serves as the colon; a sponge with a central slit is the "anus". An index card with a line figure on it (circle, S, figure of 8, or 7 line grid) is placed over in the window. A sclerotherapy catheter with a marker tip inserted in its end serves as pen (ink is methylene blue). After scope insertion the figure is traced after which the paper is exchanged for a new figure. The tracings are timed and videoed; each session consists of 8 tracings (4 figures × 2). The variables tracked were: time, number of pen strokes, deviations from line (> 2 mm), and tracing gaps. The mean number of sessions to reach a plateau for each figure and parameter was determined.

Results: Eight trainees each completed a minimum of 18 sessions (range 18–74, mean 32). The aggregate tracing time (mean for 8 tracings) for each session was determined; the mean number of sessions required to reach a stable (shortest) time line for the 8 trainees was 10 (range 5–19). The average time per tracing (after learning curve) was 26.2 s (range 17.25–35.3 s). The mean number of sessions required for trainees to trace the figure of 8 with only 1 deviation > 2 mm from line was 22.5 (range 7–55). Resident participants (6) felt the training improved their overall scope skills (6/6), scope insertion skills (4/6), and their biopsy and polypectomy skills (4/6). The 1 attending trainee reported that his EMR/ESD skills clearly improved.

Conclusion: These preliminary results suggest that this inanimate model holds promise as a means of improving scope tip control; the clinical impact of training needs to be determined. Repetitive figure tracing may enhance endoscopic skills.

S186

Flying Endoscopists in the Arctic. Initiatives for Quality Assurance of Endoscopies in Greenland

Jan Krzak, MD, FACS1; Lise Rasmussen, MD 2; Steen Erik Holm, MD2; Miroslaw Szura, Professor, MD, PhD, FACS3; Simon Bernth-Andersen, MD2; 1South Jutland Hospital, Aabenraa, Denmark; 2Queen Ingrid's Hospital, National Hospital of Greenland, Nuuk; 3St. John Grande Hospital, Cracow, Poland

Background: "Surgical Coastal Expeditions" (SCE) have been organized in Greenland since the eighties. They serve a goal of providing small coastal hospitals with specialist services such as endoscopies (SCEE), by deploying a specialist surgeon, endoscopy staff and necessary equipment to the hospital for a short period of time (of approximately one week). The purpose of this program is to increase accessibility as well as reduce costs associated with patient transport to the central hospital in Nuuk (e.g. Sisimiut 325 km, Qaqortoq 480 km). SCTE accounted for 22.3% of all 1656 GI endoscopies in Greenland in 2021.

Method: Retrospective review of documentation from SCE and patient records with identification of ESGE quality indicators, such as colon cleansing (CC), cecal intubation rate (CIR), polyp detection rate (PDR), adenoma and advanced adenoma detection rate (ADR, AADR) to investigate current status and establish a quality assurance program for SCEE in Greenland.

Results: During two SCE (8 working days) to Sisimiut and Qaqortoq, 89 GI endoscopies were performed on 77 patients. 60 patients underwent colonoscopy (32 men/28 women), with a median age of 61 years (24–80 years). CIR was achieved in 91.7%. In 8 colonoscopies (13.3%) the bowel cleansing was assessed as poor or insufficient, resulting in two interrupted examinations. PDR, ADR and AADR were respectively 43.3%, 35% and 11.7%.

As an unexpected but desirable benefit, reduction of the environmental footprint of gastrointestinal (GI) endoscopies recommended by The European Society of Gastrointestinal Endoscopy (ESGE) was also achieved.

Conclusion: The results show a satisfactory level of CIR, PDR, ADR and AADR, but clearly demonstrated the need to develop better bowel cleansing procedures that which will take into account local diet and meal habits. A need for improved patient communication was also identified. Documentation from SCE and patient records did not allow identifying and assessing all indicators recommended by ESGE, which is why this review is only considered as a pilot project. It only gave us a snapshot of colonoscopies in Greenland, and clearly shows the necessity of continued initiatives to ensure that quality of provided service meets current standards and ESGE recommendations.

The 2 SCE reduced the environmental footprint of GI endoscopies by reducing airborne patient transport from 77 patients (61,830 km) to 8 healthcare professionals (6440 km).

S187

Preoperative Manometric Data Can Predict Symptom Improvement After Per Oral Endoscopic Myotomy for Treatment of Achalasia

Joshua Lyons, MD; Christina Boutros, MD; Saher-Zahra Khan, MD; Jamie Benson; Daniel Hashimoto, MD; Jeffrey Marks; University Hospitals

Background: While per oral endoscopic myotomy (POEM) has been shown to be efficacious in the treatment of achalasia, it can be difficult to predict who will have a robust and durable response. Historically, high lower esophageal sphincter pressures have been shown to predict a worse response to endoscopic therapies such as botox therapy. This study was designed to evaluate if modern preoperative manometric data could predict a response to therapy after POEM.

Methods: This was a retrospective study of 144 patients who underwent a POEM at a single institution by a single surgeon over an 8-year period (2014–2022) who had high resolution manometry performed preoperatively and had an Eckardt symptom score performed both preoperatively and postoperatively. The achalasia type and integrated relaxation pressures (IRP) were then tested for potential correlation with need for any further achalasia interventions postoperatively as well as the degree of Eckardt score reduction using univariate analysis.

Results: The achalasia type on preoperatively manometry was not predictive of need for further interventions or degree of Eckardt score reduction (p = 0.74 and 0.44 respectively). A higher IRP was not predictive of need for further interventions however it was predictive of a higher reduction in postoperative Eckardt scores. (p = 0.03) as shown by a nonzero regression slope (Fig. 1).

Conclusions: In this study, achalasia type was not a predictive factor in need for further interventions or degree of symptom relief. While IRP was not predictive of need for further interventions, a higher IRP did predict better symptomatic relief postoperatively. This result is opposite that of endoscopic treatment modalities. Therefore, patients with higher IRP on high resolution manometry would likely benefit from myotomy which provides significant symptomatic relief postoperatively.

figure cg

S188

Endoscopic Removal Gastric and Jejunal Box Cutter Blades Using a Pediatric Colonoscope: Reemphasizing the Role of Surgeon Endoscopists

Amir Bashiri, MD, MSc 1; Robert D Fanelli, MD, MHA, FACS, FASGE2; 1The Robert Packer Hospital Guthrie Clinic Residency Program in General Surgery; 2Department of Surgery, Division of Minimally Invasive Surgery and Surgical Endoscopy, The Guthrie Clinic

Background: While foreign body (FB) ingestion overall is an uncommon event, sharp FB ingestion requires immediate attention.

Case Report: A 37-year-old female with extensive psychiatric and surgical histories presented after ingesting 5 box cutter blades. Initial esophagogastroduodenoscopy (EGD) performed by gastroenterology showed 5 blades embedded in the gastric cardia. The surgical team was consulted for assistance in retrieval. Computed tomography imaging showed a cluster of blades in the stomach and a single blade in the small bowel (Fig. 1). As the patient was a poor surgical candidate, an endoscopic approach was attempted. Four blades found in the gastric cardia were removed using EGD through an overtube (Fig. 2A–C). A fifth razor blade was identified in the jejunum, about 30 cm from the ligament of Treitz, which was removed using a pediatric colonoscope through an overtube (Fig. 2D–I). Partial endoscopic pyloromyotomy was performed to facilitate retrieval (Fig. 2F–G). Foreign body ingestion is common among patients with psychiatric conditions.

Conclusion: Here, a successful retrieval of a foreign body within the jejunum in a poor surgical candidate is described via a safe, readily available, and cost-effective approach. This case emphasizes the pivotal role of the surgeon as an innovator and specialist in endoscopy, and the importance of training surgical residents in endoscopy.

Fig. 1 Computed tomography imaging of the box cutter razor blades within the stomach and the jejunum

figure ch

Fig. 2 A-C, Retrieval of 4 box cutter razor blades from the gastric cardia. D-F, Single box cutter razor blade within the proximal jejunum about 30 cm from the ligament of Treitz. G, Endoscopic pyloromyotomy allowing the passage of the box cutter razor blade through the pylorus. H, Pyloromyotomy site. I, Jejunal box cutter razor blade retrieved successfully

figure ci

S189

Surgeon-Performed Pediatric ERCP at Academic Center: 10 Year Data

Maria Jose Donado; M. Carolina Jimenez, MD; Stephano Bonitto; Jonathan King; Robert F Cubas, MD; R. Joshua Bowles, MD; Jose M Martinez, MD; Department of Surgery, Division of Laparoendoscopic Surgery, University of Miami Miller School of Medicine, Jackson Memorial Hospital

Introduction: The use of endoscopic retrograde cholangiopancreatography (ERCP) in pediatric patients has increased over time as a diagnostic and therapeutic strategy. Procedure-related complications rates have traditionally been reported between 5 and 7%, with post-ERCP pancreatitis as the most common one. Previous studies have demonstrated that ERCP is a safe procedure in the pediatric population. However, there is a paucity of evidence on the outcomes of pediatric ERCP performed by surgeons.

Methods: Retrospective review of patients under the age of 19, who underwent ERCP between August 1, 2012 and August 1, 2022, at an academic center by a fellowship-trained surgeon. Demographic data, indications, procedure data and outcomes during admission were reviewed. The studied population included liver transplant and trauma patients.

Results: A total of 134 ERCP were performed during the study period, in patients between 2 months and 18 years of age. Almost all cases (99.9%) were performed under general anesthesia and 75% underwent any form of therapeutic intervention (e.g. sphincterotomy, stone removal, stent placement or removal or stricture dilation). The most common indication for ERCP was choledocholithiasis (58%), followed by transplant-related complications (6%) and recurrent pancreatitis (6%). Four cases (2.9%) were aborted due to unsuccessful cannulation. 7 patients (5.2%) developed a procedure-related complication during or after ERCP. Complications included mild post-ERCP pancreatitis (3.7%) and ampullary bleeding (1.5%) after sphincterotomy that resolved with epinephrine injection and balloon pressure.

Conclusion: We demonstrate that ERCP is a safe procedure with high success rate when performed in the pediatric population by a fellowship-trained surgeon.

S190

Does the Submucosa Play a Role in the Pathophysiology of Achalasia?

Timothy J Morley, MD; Matthew F Mikulski, MD, MSHCT; Alicja Zalewski, MD; David J Desilets, MD, PhD; John R Romanelli, MD; UMass Chan Medical School—Baystate

Introduction: The therapeutic benefit of Per-Oral Endoscopic Myotomy (POEM) in the treatment of achalasia and other neuromuscular esophageal dysmotility disorders has been centered around the creation of a longitudinal myotomy to relieve the obstruction and restore esophageal outflow. While it has been thought that the therapeutic benefit is derived from the myotomy alone, it is unknown if the submucosa contributes to the pathophysiology of achalasia. The purpose of this study is to determine whether creation of the submucosal tunnel (SMT) alone contributes positive physiologic changes as measured by EndoFLIP.

Methods: This is a retrospective review of consecutive POEM cases performed by a single surgeon-endoscopist team at a high-volume tertiary referral center from June 1, 2011 to September 1, 2022 in which EndoFlip was utilized. Patients with diagnoses of achalasia or esophagogastric junction outflow obstruction (EGJOO) were included and grouped by EndoFLIP into those with measurements (diameter, distensibility index (DI)) before creation of the SMT and after myotomy (Group 1) and those that also included a third intermediate measurement after SMT creation but prior to myotomy (Group 2). Demographics, POEM outcomes, and EndoFLIP data were analyzed using descriptive statistics and Mann–Whitney U tests.

Results: Sixty-six patients met inclusion criteria, of whom 57(86.4%) had achalasia, 32(48.5%) were female, and median pre-POEM Eckart score was 7[IQR:6–9]. There were no differences between pre-POEM, post-POEM, or pre/post change in Eckardt score between groups, or differences in myotomy length or perioperative length of stay. Group 2 had a larger overall increase in diameter from pre-SMT to post-myotomy (median 5.6 cm[IQR:4.25–6.3] vs. 4.6 cm[IQR:3.55–5.6], p = 0.029) compared to Group 1, with SMT creation comprising 2.15 cm[IQR:1.75–3.275] of Group 2's total increase in diameter. There was no difference between Groups in DI, though 1[IQR:0.5–1.2] of the overall 3.35[IQR:2.4–3.975] increase in Group 2's DI was attributed to SMT creation alone.

Conclusions: Measuring the creation of the SMT and myotomy as separate entities can guide intraoperative management to obtain a larger change in esophageal diameter, though without a difference in DI. Although not statistically significant, notable changes in diameter and DI occur with SMT creation alone, implying that submucosal fibrosis may play a role in the pathophysiology of achalasia.

S191

Multicenter Phase II Trial of Transanal Total Mesorectal Excision (TME) for Rectal Cancer: Preliminary Results

Patricia Sylla, MD 1; Dana Sands, MD2; Alison Ricardo1; Antoinette Bonaccorso1; John Marks3; Mariana Berho2; Alex Polydorides1; Mark Whiteford4; Justin Maykel5; Karim Alavi5; Karen Zaghiyan6; Sami Chadi7; Sherief Shawki, MD8; Scott Steele, MD9; Alessio Pigazzi, MD, PhD10; Matt Albert, MD11; Teresa DeBeche11; Steve Wexner, MD2; 1Mount Sinai Hospital; 2Cleveland Clinic Florida; 3Lankenau Hospital; 4Providence Hospital; 5University of Massachusetts; 6Cedar Sinai Hospital; 7University of Toronto; 8Mayo Clinic; 9Cleveland Clinic Ohio; 10Weil Cornell Medical College; 11Orlando Advent Health

Background: taTME combines abdominal and transanal dissection to facilitate sphincter preservation in patients with low rectal tumors. The safety of taTME has not been reported in large phase II/III trials. We report early results from a North American prospective multicenter phase II trial (NCT03144765).

Methods: 100 patients with stage I-III rectal adenocarcinoma located ≤ 10 cm from the anal verge, eligible for sphincter preservation with negative predicted circumferential margin were enrolled across 11 taTME centers. Primary and secondary endpoints were TME quality with central blinded review of specimen photographs, 30 and 90-day perioperative outcomes, and stoma closure rate, respectively.

Results: Between September 2017 and April 2022, 70 males and 30 females with median age of 55 (range 25–86) years and BMI 27.5 (range 18–45.7) underwent taTME for tumors located a median 5.75 (range 2–9) cm from the AV. Tumors were stage I, II and III in 35%, 23%, and 42% of patients respectively; 71% were treated with neoadjuvant therapy. taTME was performed with laparoscopic or robotic assistance with mucosectomy or partial intersphincteric resection performed in 41%, transanal specimen extraction in 52%, and handsewn anastomoses in 54% of patients. A loop ileostomy was constructed in all cases. Intraoperative complications occurred in 8% including 5 organ injuries, 1 suspected CO2 embolism, bleeding and ischemic conduit requiring conversion to robotic TME in 1 and to open surgery in 2 patients. Median length of stay was 5 (range 2–74) days with 30-day complications in 46 patients graded Clavien Dindo I/II (72%) and CDIII (28%). Between 30 and 90 days postoperatively, 10 patients developed additional CDI/II (70%) and CDIII (2%) complications with one non cancer-related mortality. Anastomotic complications occurred in 18%, including 10 early leaks (< 30 days) and 8 delayed leaks/sinus/strictures (> 30 days). Interventions ranging from dilatation to anastomotic revision were performed in 14 cases, with a 98% stoma closure rate achieved a median of 5 (range 1.4–23.1) months post-taTME. TME grading was complete or near complete in 90% with a median of 19 (range 3–68) lymph nodes. Complete pathological response was achieved in 32% with positive CRM or distal margins in 2 cases.

Conclusions: taTME was demonstrated to be safe when performed in carefully selected patients at expert centers. Sphincter salvage was achieved for tumors ≤ 6 cm with a 2% open conversion and 98% stoma closure rate. Mid-term oncologic and functional results will further evaluate the oncologic safety and functional impact of taTME.

S192

The Laparoscopy In Biliary Exploration Research and Training Initiative (LIBERTI) Trial: Simulator-Based Training for Laparoscopic Management of Choledocholithiasis

Vanessa N VanDruff, MD 1; B Fernando Santos, MD, FACS2; Michael Cutshall, MD3; Brian R Davis, MD, FACS, FASGE3; Roxann A Lerma, MD4; Edward D Auyang, MD, MS, FACS4; Abigail J Uryga5; Julia R Amundson, MD, MPH1; Stephanie Joseph, MD, MPH6; Kristine Kuchta, MS5; Michael McCormack, MD5; H Mason Hedberg, MD5; Casey R Lamb, MD2; Danielle Abbitt, MD7; Edward L Jones, MD, MS, FACS7; Wendy Li, MD8; Eugene P Ceppa, MD, FACS8; Michael B Ujiki, MD, FACS5; 1University of Chicago; 2Dartmouth-Hitchcock Medical Center; 3Texas Tech University Health Science Center El Paso; 4University of New Mexico; 5NorthShore University HealthSystem; 6Wayne State University; 7University of Colorado Anschutz Medical Campus; 8Indiana University Health Hospital

Background: Laparoscopic cholecystectomy with common bile duct exploration (LCBDE) is equivalent in safety and efficacy to endoscopic retrograde cholangiopancreatography (ERCP) plus laparoscopic cholecystectomy (LC) while decreasing the number of procedures, length of stay (LOS), and costs associated with choledocholithiasis management. Despite these advantages LCBDE is infrequently utilized, in part due to limited exposure to LCBDE in current surgical training. We hypothesized that instituting formal, simulation-based training in LCBDE would result in increased utilization of LCBDE and improve patient outcomes across participating institutions.

Methods: Data was obtained from an on-going multi-center study in which simulator-based transcystic LCBDE training curricula were instituted for surgery attendings and residents. A 2-year retrospective review of LCBDE utilization prior to LCBDE training was compared to utilization up to 2 years after initiation of LCBDE training. Patient outcomes and cost analysis were analyzed between LCBDE and ERCP + LC groups using chi square, t-tests, and Wilcoxon rank tests.

Results: To date, results included a total of 32 attendings and 35 residents trained in LCBDE since January 2021. Initial LCBDE utilization rate ranged from 0.74% to 4.5%, and increased among all institutions after training, ranging from 9.3% to 44.7% of cases. Of 100 choledocholithiasis patients analyzed (SURGERY n = 48 versus ERCP n = 52) 63 patients received LCBDE. There were 23 (36.5%) LCBDE failures, of whom 16 proceeded with ERCP after LCBDE attempt, and 7 who had spontaneous passage of CBD stones without further intervention. Patients who received LCBDE were younger than the ERCP + LC group (age 39 ± 15 versus 49 ± 22, p = 0.041). No statistically significant differences were found when comparing sex, medical history, or ASA score between groups. Overall, the LCBDE group was associated with shorter median LOS (3 days versus 4 days, p < 0.0001), and lower mean charges ($64,356 ± $9,481 versus $73,448 ± $14,203 p = 0.020) when compared to the ERCP + LC group. No statistically significant differences were found in readmission rates between LCBDE (n = 2, 4.2%) versus ERCP + LC (n = 4, 7.7%, p = 0.679), or in post-procedure complication rates between groups.

Conclusion: Simulator-based training in LCBDE results in higher LCBDE utilization rates, shorter LOS, and lower costs, with equivalent efficacy and safety to ERCP plus cholecystectomy. Therefore, implementation of LCBDE training is strongly recommended to optimize healthcare utilization and management of patients with choledocholithiasis.

S194

Bariatric Surgery Alters Mitochondrial Function in Gut Mucosa

Robert C Ross, MD, MPH; Elizabeth C Heintz; Elizabeth R Zunica, PhD; Robbie L Townsend; Amanda E Spence; Philip R Schauer, MD; John P Kirwan, PhD; Christopher L Axelrod, MEd; Vance L Albaugh, MD, PhD; Pennington Biomedical Research Center

The obesity pandemic has led to worsening disease burden worldwide, including diabetes, cardiovascular disease, and cancer. Although bariatric surgery is the most effective and durable obesity treatment, its mechanisms underlying long-term weight loss maintenance remain unclear. Mitochondria, the primary sites of energy metabolism, are reduced in tissues of patients with obesity and diabetes and increasing mitochondrial function is associated with healthier outcomes. Thus, mitochondrial function was examined in response to bariatric surgery in mice to test whether intestinal mitochondrial function is altered postoperatively.

Methods: Mice were randomized into surgery and control groups. Interposition of a 2 cm segment of distal ileum into the proximal bowel 0.5 cm distal to the ligament of Treitz was performed in mice. After a 4-week recovery period, mucosal small bowel scrapings (duodenum, jejunum, ileum, and ileal interposition) were prepared and assayed for NADH- and succinate-linked oxidative phosphorylation (OXPHOS) and electron transfer capacity as markers of mitochondrial function.

Results: Mitochondrial respiration in control animals was highest in the duodenum, followed by the jejunum, with the lowest rates occurring in the ileum. Interestingly, interposed ileum exhibited similar respiration to the native ileum in controls, but the remaining segments (duodenum, jejunum, and distal ileum) all exhibited significantly elevated OXPHOS capacity.

Conclusions: Ileal interposition increases capacity for intestinal mitochondrial respiration in mice. This functional augmentation throughout the small bowel with preserved respiration in the interposed ileal segment suggests a global mucosal response to segment specific nutrition signals in the distal bowel. This may be an important mechanism responsible for the substantial long-term weight loss success seen in patients undergoing bariatric surgery. Future studies are focusing on quantifying gene and protein expression underlying these mitochondrial changes, with specific emphasis on identifying the changes associated with bariatric surgery.

figure cj

S195

Development of Pancreatic Acinar Cell Metaplasia After Long-Term Use of Vonoprazan

Rodrigo L Edelmuth, MD 1; Maria Cristina Riascos, MD2; Hala Al Asadi, MD2; Jacques A Greenberg, MD2; Ileana Miranda, DVM, MSc2; Carl V Crawford, MD2; Felice H Schnoll-Sussman, MD2; Philip Katz, MD2; Brendan M Finnerty, MD2; Thomas J Fahey III, MD2; Rasa Zarnegar, MD2; 1Hospital Israelita Albert Einstein; 2Weill Cornell Medicine

Introduction: Vonoprazan is a new, first-in class oral acid-suppressing agent that was launched in December 2014 in Japan and received FDA approval in 2022. It reversibly inhibits gastric acid secretion by competing with the K + on the luminal surface of the parietal cell (potassium-competitive acid blockers or P-CABs). They rapidly reach high plasma concentrations and are associated with a fast onset of action as well as longer acid inhibition time, and are less influenced by other drugs and food. Vonoprazan has been on the market for a short time and there are many clinical trials to support its clinical application. However, medical experience and comprehensive clinical data is still limited to if and how gastric histology is altered due to therapy.

Methods: A prospective randomized 12-week trial with 30 Wistar rats was conducted. At six weeks of age, rats were randomly assigned to one of 5 groups: (1) saline as negative control group, (2) oral omeprazole (40 mg/kg), as positive control group, (3) oral omeprazole (40 mg/kg) for 4 weeks, followed by 8 weeks off omeprazole, (4) oral vonoprazan (4 mg/kg), as positive control group and (5) oral vonoprazan (4 mg/kg) for 4 weeks, followed by 8 weeks off vonoprazan. Each group consisted of 6 animals. The goal of this study was to assess the presence of gastrointestinal morphologic abnormalities upon long-term administration of omeprazole and vonoprazan.

Results: Important histologic findings were seen in the stomach of animals that received oral vonoprazan. These are non-inflammatory alterations characterized by loss of parietal (oxyntic) cells and chief (zymogen) cell metaplasia into pancreatic acinar cell metaplasia (PACM). No significant abnormalities were identified in any other hepatobiliary or gastrointestinal tissue.

Discussion: PCAM has been reported in gastric mucosa, at the esophagogastric junction, at the distal esophagus, and in Barrett esophagus. However, the pathogenesis of this entity is still unclear. Whereas some authors have suggested that PCAM is an acquired process representing a metaplastic change in association with chronic gastritis, autoimmune gastritis, chronic use of proton pump inhibitors or NSAIDs, others have raised the possibility of PCAM being congenital in nature.

Conclusions: Our results suggest that the duration of vonoprazan administration at a dose of 4 mg/kg plays an important role in the development of PCAM and parietal cell loss in the gastric mucosa over time. Further studies to determine the implications of this are warranted.

figure ck

S196

Current Insight: The impact of Gut Microbiota on Liver Regeneration after Open and Laparoscopic Left Partial Hepatectomy

Constantinos S Mammas, MD, MSc, PhD, SURGEON; Surgical Laboratory Prof. C.Tountas Athens HELLAS-Program of Excellence 2014–16

Objectives: The liver regeneration in the remnant liver of a porcine experimental model after Open (OLPH) and Laparoscopic Left Partial Hepatectomy (LLPH) is compared.

Materials and Methods: Two pairs of double-series of twenty-nine liver tissues taken from twenty nine porcine livers, after OLPH or LLPH(PD0) and on the 7th postoperative day(PD7), were randomly allocated into two groups: the OLPH (n1 = 19 × 2) and the LLPH (n2 = 10 × 2).The liver tissue sections in each group, had been prepared and stained with PCNA, Gaspase-III and anti-Ubiquitin and anti-VEGF stain. A PCNA based proliferative index (LCPI), a Gaspase-III based (Liver Cell Apoptotic Index (LCAI), and an Ubiquitin based intracellular stress index (LISI) were prospectively estimated by microscopic visualization of the aforementioned tissue stained samples. A VEGF based angiogenesis index (LCANGI) was estimated by microscopic visualization (MV) of a sample of twenty eight tissue sections (n1 = 8 × 2, n2 = 6 × 2) retrospectively. A PAS-stain Candida Species RL lobule colonization index (CRLLCI) of a sample of twenty eight tissue sections (n1 = 7 × 2, n2 = 7 × 2) estimated by MV. LCPI, LCAI, LISI, LCANGI and CRLLCI compared between OLPH and LLPH on PD0 and on PD7.

Results: There was not any difference in the mean weights of the remnant livers and in the post-surgical complications rates between OLPH and LLPH on PD7. Statistical comparisons of LISI resulted in a statistically significant difference, favoring the OLPH group on PD0 (p < 0.1).LCPI was statistically significantly higher after OLPH than LLPHon PD7 (p = 0.002). LCANGI OLPH was statistically significantly higher than LCANGILLPH, on PD7 (p = 0.028).No difference was found in the apoptotic level between OLPH and LLPH on PD0 (0.926) and on PD7 (p > 0.313). Multiple linear regression models used to investigate the effect of technique (OLPH vs LLPH) and time (day 0 vs day 7) on the PCNA parameter taking into account the possible influence of the Gaspase, Ubiquitin and VEGF parameters. Initially, apart from the technique and time, only the GASPASE and UBIQUITIN parameters were examined in a sample of 54 individuals with available measurements, while the VEGF parameter was also examined in a smaller sample of 27 individuals. In all cases the effect of Gaspase, Ubiquitin and VEGF parameters was not statistically significant (p-value > 0.05). CRLLCI was significantly higher in the OLPH in comparison to the LLPH on PD0 (p = 0.158) an on PD7 (p = 0.006).

Conclusions: The Liver regenerative process after OLPH seems significantly delayed in comparison to LLPH probably because of the significantly higher level of gut microbial translocation to the remnant liver.

figure cl

S197

Long-Term Potassium-Competitive Acid Blockers Administration Causes Microbiota Changes in Rats

Rodrigo L Edelmuth, MD; Maria Cristina Riascos, MD; Hala Al Asadi, MD; Jacques A Greenberg, MD; Ileana Miranda, DVM, MSc; Carl V Crawford, MD; Felice H Schnoll-Sussman, MD; Philip O Katz, MD; Brendan M Finnerty, MD; Thomas J Fahey III, MD; Rasa Zarnegar, MD; Weill Cornell Medical College

Introduction: Potassium-competitive acid blockers (P-CABs) are novel drugs that bind reversibly to K + ions and block the H + , K + ATPase enzyme, thus preventing acid secretion. The most known available P-CAB is vonoprazan, marketed in Japan in early 2015 and approved by the FDA in the beginning of 2022. When compared to proton pump inhibitors (PPIs), P-CABs have a faster onset of action, longer duration, and more consistent and more potent acid suppression. Vonoprazan-based Helicobacter pylori treatment is highly effective in eradicating the target bacteria and several studies have demonstrated that a vonoprazan-antibiotic regimen affects microbiome. However, the impact of vonoprazan alone on gut microbiota is unclear.

Methods: We conducted a prospective randomized 12-week experimental trial with 18 Wistar rats. At six weeks of age, rats were randomly assigned to one of 3 groups: (1) saline as negative control group, (2) oral vonoprazan (4 mg/kg), as positive control group and (3) oral vonoprazan (4 mg/kg) for 4 weeks, followed by 8 weeks off vonoprazan. Each group consisted of 6 animals. The goal of this study was to assess gut microbiome changes upon long-term administration of vonoprazan.

Results: When compared to the control group, the animals that received vonoprazan for 12 weeks had an important increase in both Firmicutes and Deferribacteres phyla (14% vs. 35.5% and 1.1% vs. 2.5%, respectively). A decrease in the Proteobacteria phylum (24.4% vs. 8.8%) was seen in the vonoprazan group, mainly because of Parasutterella excrementihominis decrease (23.7 vs. 4.2%). Despite the overall decrease in the Proteobacteria phylum, there was an important increase in Escherichia coli presence (0.1% to 4.5%). Vonoprazan also increased the following species: Lactobacillus reuteri, Lactobacillus murinus, Prevotella sp, and Lactobacillus intestinalis.

Discussion: Numerous studies in humans and animal models have established associations between alteration in the microbiota composition and a wide range of diseases, such as inflammatory bowel disease, rheumatoid arthritis, obesity, diabetes, metabolic syndrome, and an increased risk of enteric infections caused by Clostridioides difficile, Salmonella spp., Shigella spp and Campylobacter spp.

Conclusions: Our study showed that gut dysbiosis was caused by vonoprazan. To our knowledge this is the first study that demonstrated that vonoprazan alone induces microbiome changes.

figure cm
figure cn
figure co

S198

Efficacy of Anti-EGFR (Epidermal Growth Factor Receptor) versus Anti-VEGF (Vascular Endothelial Growth Factor) Therapy for WT RAS mCRC: A Real-World Experience from a Single Institute

Shih-Wei Chiang, MD 1; Feng-Fan Chiang, PhD2; 1Chiayi Branch, Taichung Veterans General Hospital; 2Taichung Veterans General Hospital

Background: Results from top prospective clinical trials comparing anti-epidermal growth factor receptor (EGFR) therapy and anti-vascular endothelial growth factor (VEGF) therapy plus chemotherapy for WT(wild-type) RAS metastatic colorectal cancer (mCRC) yielded differences. This study establishes real-world experiences and facing challenges while treating patients with metastatic colorectal cancer (mCRC).

Method: This is a retrospective study that identified first-line mCRC patients comparing anti-EGFR plus chemotherapy versus anti-VEGF plus chemotherapy; data were subsequently updated using recent congress presentations. This study estimated the overall survival (OS) treatment effect of first-line chemotherapy plus anti-EGFR or anti-VEGF in patients with RAS WT mCRC. Secondary end-points were progression-free survival (PFS), objective response rate (ORR), resection management, and OS according to performed interventions in the first encounter (synchronous resection, upfront resection of primary tumor, systemic treatment).

Results: A total of one hundred and fourteen patients with RAS WT mCRC were included from Jun 01, 2018, to Jun 30, 2021. Median overall survival was 27 months (22.8- 31.3) in the anti-EGFR plus chemotherapy and 23.6 months (18.6- 28.5 months) in the anti-VEGF plus chemotherapy (p = 0.22). Median progression-free survival was 13.6 months (10.9- 16.4) in the anti-EGFR plus chemotherapy and 12.4 months (10.1- 14.7) in the anti-VEGF plus chemotherapy (p = 0.94). There was no significant difference in the primary and secondary endpoints, it revealed similarities between treatments. It showed relative inferior OS and PFS while the primary tumor was located in the right side colon than the left side colon. Better treatment benefits were observed in right-side primary tumor patients receiving first-line chemotherapy with anti-VEGF than anti-EGFR. In the sub-group analysis, it showed superior OS in the synchronous resection group, patients receiving metastasectomy in the upfront resection group, and receiving both primary and metastases resection in the systemic treatment group.

Conclusions: This study supported similar benefits for first-line anti-EGFR plus chemotherapy versus anti-VEGF plus chemotherapy with respect to OS, PFS, and ORR in patients with RAS WT mCRC. It showed better response and survival when using anti-VEGF in right-side tumor patients. Surgery still has an important role in the mCRC patients taking advantage of systemic treatments. Further retrospective data is awaited and analyzed.

S199

Characterisation of Low Cost Meshes to Assess Suitability for Hernia Repair Under UK Technology Appraisal Regulations: Effects of Ethylene Oxide Sterilisation

Alessandra Grillo, Dr1; Zargham Hyder, Dr 2; Vivek Mudera Mudera, Dr1; Alvena Kureshi, Dr1; 1University of London; 2Homerton University Hospital, NHS Trust

Herniation is a highly prevalent condition in the developing world, with an estimated incidence of up to 30% in Africa [1]. The scarcity of resources led surgeons to repurpose commonly available low-cost meshes (LCM) for use as surgical mesh for repairing hernias. Previous studies of surgical implantation of LCM in humans have shown encouraging post-operative outcomes, comparable to the commercially available products[2]. This frugal innovation represents a cost-effective alternative to expensive commercial meshes, although regulatory barriers, such as sterilisation process, have prevented its use in the developed world. Grillo et al. [3] previously characterised the ultrastructure, mechanical properties and in vitro biocompatibility of two UK-sourced low-cost meshes, highlighting human dermal fibroblasts attachment and proliferation over the fibres of the two meshes. This present study has assessed the effects of EO sterilisation, a method approved for use of medical devices in EU and UK, on ultrastructure, mechanical properties and biocompatibility of the two LCM. Uniaxial tensile tests showed a significant difference between non-sterilised and EO-sterilised meshes, with the latter having a twofold lower break stress (p < 0.0001), yet remaining in the range of the native tissue mechanical properties, contrarily to the over-engineered CM. In addition, break strain decreased with EO sterilisation for both meshes (p < 0.05 for LCM1 and p < 0.0001 for LCM2), whilst Young's modulus increased significantly for LCM2 with EO sterilisation (p < 0.001). Cytotoxicity and biocompatibility tests did not reveal any significant difference between control and EO sterilised meshes, confirming the safety of use of these LCM in vitro. These findings provide crucial information for the adoption of this frugal innovation beyond the developing world, opening the possibility of a cost-effective solution for high-income countries.

S204

Health-Related Quality Of Life After Laparoscopic Versus Open Distal Pancreatectomy: A Prospective Observational Study

Giovanni Guarneri; Chiara Limongi; Alessia Vallorani; Alice W Licinio; Martina Abati; Massimo Falconi; Nicolò Pecorelli; San Raffaele Hospital, Milan, Italy

Background: Limited data comparing health-related quality of life (HRQOL) after laparoscopic (LDP) versus open distal pancreatectomy (ODP) are available. The aim of this study was to assess the impact of minimally invasive approach on postoperative HRQOL after DP using the Patient-Reported Outcomes Measurement Information System (PROMIS), a validated tool developed by the NIH to address the lack of generalizable and universal measure of patient-reported outcomes.

Methods: Data from consecutive patients who underwent DP (2020–2022) enrolled in a prospective clinical trial(NCT04431076) were reviewed. Patients completed PROMIS-29 questionnaires preoperatively, at 15, 30, 90 days after surgery. Linear regression analysis adjusting for confounders including preoperative PROMIS scores, age, gender, ASA score, diagnosis, multivisceral resection, clinically relevant postoperative pancreatic fistula (CR-POPF) was used to estimate between-group differences in PROMIS domains T scores.

Results: Overall, 217 patients underwent DP (median age 65 years, female 54%, ASA score ≥ 3 28%, pancreatic cancer 39%, multivisceral reresection 10%, CR-POPF 31%, median LOS 6 days). LDP was performed in 126 patients, ODP in 91 patients.

At 15 postoperative days, the proportions of patients who recovered or improved preoperative physical function, social roles, fatigue, pain interference, sleep disturbance, anxiety, depression scores were 14%, 27%, 36%, 42%, 58%, 75%, 80%, respectively. At adjusted analysis, LDP was associated with higher physical function scores (mean difference 4.4, p = 0.001) compared to ODP, while other HRQOL domains' scores were similar.

At 30 postoperative days, the proportion of patients who recovered or improved preoperative scores was 34% for physical function, 39% for social roles, 48% for fatigue, 51%, for pain interference, 68% for sleep disturbance, 78% for anxiety, 80% for depression. At multivariate analysis, LDP was associated with higher physical function (mean difference 4.7, p = 0.001) and social roles (mean difference 4.6, p = 0.004), lower anxiety (mean difference -3.6, p = 0.016) and fatigue (mean difference -2.5, p = 0.048) scores compared to ODP.

At 90 postoperative days, the proportion of patients who recovered or improved preoperative scores was 71% for physical function, 64% for social roles, 68% for fatigue, 75% for pain interference, 78% for sleep disturbance, 90% for anxiety, 87% for depression. No significant differences in HRQOL PROMIS domains were found between LDP and ODP patients.

Conclusions: According to PROMIS-29, minimally invasive surgery for DP resulted in improved physical and social functioning and reduced anxiety and fatigue up to 30 days after surgery compared to the open approach, while no difference was found for other HRQOL domains and in the longer-term.

S205

Effect of Gender Discordance on Surgical Outcomes in Predominantly Female Patient Surgeries in NYS

Caroline O Smolkin, BS 1; Xiaoyue Zhang, MS1; Ila Sethi, BA1; Adrian Torres, BS1; Jie Yang, PhD1; Jeri A Ramilo2; Aurora Pryor, MD, MBA, FACS, FASMBS1; 1Stony Brook University Hospital; 2New York Institute of Technology

Introduction: Preliminary evidence demonstrates female surgeons often have improved post-operative outcomes compared to their male colleagues, especially with female patients, despite being underrepresented in surgical specialties. This study aimed to identify the effect of patient-surgeon gender discordance on surgical outcomes in three surgical specialties with high female patient populations: bariatric, foregut, and colorectal surgery.

Methods and Procedures: This is a retrospective database study using the New York State (NYS) SPARCS database and the first study evaluating outcomes based on surgeon/patient concordance in NYS. Bariatric, foregut, and colorectal surgery cases from 2013–2017 were identified. Logistic regression models and generalized linear regression models were used to compare outcome factors among four cohorts: gender concordance/female patient (CC/FP), gender discordance/female patient (DC/FP), gender concordance/male patient (CC/MP), gender discordance/male patient (DC/MP). Gender discordance is defined as patient/surgeon with opposite genders. Gender concordance is defined as patient/surgeon with same gender.

Results: BARIATRIC: FP and MP had significantly different 30-day readmission (p-value = 0.0494) and in-hospital complications (p-value < 0.0001) based on gender concordance or discordance. CC/FP had significant lower risk of 30-day readmission (OR = 0.78, 95%CI 0.68–0.90) but significant higher risk of developing complications (OR = 1.33, 95%CI 1.24–1.43) compared with DC/FP. CC/MP had non-significant higher risk of 30-day readmission (OR = 1.07, 95%CI 0.81–1.41) but significant lower risk of developing complications (OR = 0.77, 95%CI 0.66–0.90) compared with DC/MP.

FOREGUT: Difference between gender concordance and discordance was significant between FP and MP on LOS (p-value = 0.0030), 30-day readmission (p-value = 0.0081), and 30-day ED visit (p-value = 0.0075). CC/FP had significant less LOS (ratio = 0.94, 95%CI 0.91–0.97), lower risk of 30-day readmission (OR = 0.75, 95%CI 0.60–0.94), and lower risk of 30-day ED visit (OR = 0.76, 95%CI 0.65–0.88) compared to DC/FP. MP had the opposite trend in risk, although non-significant.

COLORECTAL: The difference between gender concordance and discordance was non-significant within FP or MP group as to any outcomes. FP and MP had significantly different 30-day readmissions based on patient-surgeon gender concordance (OR 0.91vs1.10, p-value = 0.0379).

Conclusion(s): Overall, female bariatric patients show benefit with gender concordance with their surgeon in certain parameters, but also show benefits with male surgeons. Female foregut patients benefit from gender concordance with their surgeon. Although non-significant, male foregut patients benefit from the gender discordance (i.e., having female surgeon). Female colorectal patients show benefit from gender concordance. This emphasizes the need for surgeons to be conscious of care provided to opposite gender patients and underscores the necessity for more female representation amongst surgeons in high female patient population fields.

S206

Effect of Neuromuscular Blockade Reversal on Post-operative Urinary Retention Following Inguinal Herniorrhaphy

Alexa Jarman, MD; Lucy Ching Chau, MD; Ryan Soheim, MD; Michael Dix, MD; Kellie McFarlin, MD; Cletus Stanton, MD; Henry Ford Hospital

Introduction: Within the United States, reversal of nondepolarizing neuromuscular blockade is routinely accomplished by anticholinesterases such as neostigmine combined with glycopyrrolate or edrophonium combined with atropine. Sugammadex, a selective antagonist for rocuronium, is commonly utilized due to rapid onset of action and decreased side effect profile. Sugammadex was previously associated with reduced rates of post operative urinary retention (POUR) compared to anticholinesterases. This study aims to define the risk of POUR following inguinal herniorrhaphy in those that received sugammadex compared to anticholinesterases in a single tertiary referral center.

Methods: All adults undergoing inguinal herniorrhaphy from 1/1/2019 to 7/1/2022 with at least 30 days of follow up were identified. Inclusion criteria include those who received aminosteroid non-depolarizing muscle relaxants rocuronium or edrophonium and were reversed with an anticholinesterase or sugammadex. Patients who did not receive a neuromuscular blocking agent were excluded. The 30-day incidence of new onset POUR was identified through chart review. 1 to 2 propensity score matched models were fitted to evaluate the treatment of effect of sugammadex vs. anticholinesterase on POUR adjusting for patient comorbidities, ASA class, wound class, operative laterality, urgency of case, and open vs. minimally invasive repair.

Results: 3345 patients were included in this study with 1101 receiving sugammadex and 2244 receiving anticholinesterase for neuromuscular blockade reversal. The 30-day rate of new POUR was 2.8%, 1.4% in the sugammadex and 4.4% in the anticholinesterase groups respectively. 61% of the cohort received elective herniorrhaphy, 73.6% were unilateral repairs, 0.75% were repeat herniorrhaphy on the same side, 3.2% required inpatient stay post-operatively with average length of stay of 1.5 days [IQR: 1–2]. After propensity score matching, patients receiving sugammadex had significantly lower risk of POUR compared to anticholinesterase overall (OR: 0.340, P < 0.001, 95% C.I. = 0.198–0.585), in open cases (OR: 0.296, P = 0.013, 95% C.I. = 0.113–0.775), minimally invasive cases (OR: 0.36, P = 0.002, 95% C.I. = 0.188–0.693), unilateral repairs (OR: 0.371, P = 0.001, 95% C.I. = 0.203–0.681), bilateral repairs (OR: 0.25, P = 0.025, 95% C.I. = 0.074–0.838), elective cases (OR: 0.329, P < 0.001, 95% C.I. = 0.185–0.584), and clean cases (OR: 0.312, P < 0.001, 95% C.I. = 0.176–0.553).

Conclusions: The incidence of 30-day new onset post-operative urinary retention in our cohort was 2.8%. Sugammadex for neuromuscular blockade was associated with significantly lower risk of 30-day new onset POUR compared to anticholinesterase overall and when stratifying by operative modality, laterality, and wound class.

figure cp

S207

Urinary Incontinence in Women Undergoing Bariatric Surgery

K Paige Mihalsky, MD; Rachel Tran; Fernando Moreno-Garcia; Valori Kruse; Kathy Cummings, RN; Caitlin Stenberg, APRN, CNP; Fernando Mier Giraud, MD, FACS; Lieschen H Quiroz, MD, FACOG; Laura E Fischer, MD, MS, FACS, FASMBS; University of Oklahoma HSC

Introduction: Obesity is a known risk factor for urinary incontinence (UI). As bariatric surgery can result in significant and sustainable weight loss, many chronic diseases closely linked to obesity have likewise shown improvement after surgical weight loss. We propose that bariatric surgery may significantly improve obesity-related UI symptoms as well as improve quality of life. Our aim was to observe UI symptoms in women in a bariatric program.

Methods and Procedures: This is an interim analysis of an ongoing, prospective, single-institution observational study looking at UI and sexual dysfunction in women enrolled in a bariatric surgery program. Participants completed the Pelvic Floor Distress Inventory (PFDI-20), International Consultation on Incontinence Questionnaire – Urinary Incontinence Short Form (ICIQ-UI-SF), King's Health Questionnaire (KHQ), and Patient Global Impression of Improvement (PGI-I). Questionnaires were administered upon enrollment, pre-operatively, and at 3, 6, and 12 months post-operatively. Demographic data was collected at each interval and analyzed with descriptive statistics. Chi square test was used to examine categorical data and Student's t-test for continuous data. Univariate and multivariable analyses were used to identify risk factors for UI symptoms. P-values < 0.05 were considered significant.

Results: At interim analysis, 108 patients had enrolled in the study and 60% had progressed to surgery. We analyzed the following surveys: enrollment (n = 108), pre-operative (n = 43), 3-month (n = 29), 6-month (n = 26), and 1-year (n = 27). Mean BMI decreased from 49.8 to 31.1 at 1-year (35.1% total body weight loss (%TBWL)). All surveys showed significant improvement in UI symptoms over time (Fig. 1). Overall, urinary incontinence symptoms (PDFI-20) are correlated with BMI at time of survey and %TBWL (p = 0.03, p = 0.019) with the urinary leakage component (UDI-6) having the strongest relationship (p < 0.0001). Additionally, perception of symptom improvement with surgery (PGI-I) improved over time (3-month p = 0.0289, 6-month p = 0.0024, 12-month p = 0.0035). Quality of life related to UI symptoms (KHQ) significantly improved after surgery (p = 0.0047 3-month, p = 0.0042 6-month, p = 0.0165 1-year), with general health perception having the strongest correlation with %TBWL (p = 0.025).

Conclusions: Although the relationship is complex and likely depends on many factors, weight loss after bariatric surgery is associated with improvement in UI symptoms. Additionally, there is a significant improvement in perception of UI symptoms and in quality of life related to UI. Bariatric surgery can play a role in the long-term treatment of urinary incontinence in women with obesity to improve not only physical symptoms but also quality of life.

figure cq

S208

What Happens After Discharge? A Two + Year Follow-up After Laparoscopic Subtotal Cholecystectomy

Sunjay S Kumar, MD 1; Erin Briggs2; Talar Tatarian, MD1; Renee M Tholey, MD1; Karen A Chojnacki, MD1; Francesco Palazzo, MD1; 1Thomas Jefferson University Hospital; 2Sidney Kimmel Medical College

Background: Laparoscopic subtotal cholecystectomy is increasingly utilized for the difficult gallbladder to avoid bile duct injury. Both reconstituting and fenestrating operative approaches have been described but little is known about the medium-term outcomes or whether a specific technique should be favored.

Methods: The electronic medical record was queried for all cholecystectomies performed by our surgical group (2018–2020) using a search methodology that employed CPT® codes, selected key words, and detailed chart review to confirm subtotal cholecystectomy was performed. Outcomes assessed included bile duct injury, bile leak, post-operative endoscopic retrograde cholangiopancreatography (ERCP), discharge from hospital with drain, return to operating room (OR), and return to emergency department (ED) in < 30 days. A telephone survey was used to assess whether patients received treatment for a post-operative problem outside of our health system. Subgroup analysis based on technique was performed. Demographic/outcomes data were compared using the student's T test and Mann Whitney U test on categorical and continuous variables.

Results: 28 patients met inclusion criteria. The median follow-up period was 32.7 months (IQR 1.01–37.49). There were no bile duct injuries or returns to the OR. 21% of patients required a post-operative ERCP and 50% were discharged home with a drain. Two patients returned to the ED 30 days after operation. When comparing groups based on technique utilized, postoperative bile leaks were found to be statistically more prevalent in the fenestrating group (38% vs 0%, Table 1).

Conclusion: In this single institution analysis, laparoscopic subtotal cholecystectomy results in satisfactory outcomes well past 24 months independent of whether a reconstituting or fenestrating approach is utilized, except for bile leaks which favors reconstituting subtotal cholecystectomy.

figure cr

S209

“FLIP Don't Lie”: Using Impedance Planimetry to Define the End of a Peroral Endoscopic Myotomy

Monica Polcz, MD, MS; Dau Ku, MS; Gregory Scarola, MS; Paul Colavita, MD, FACS; Atrium Carolinas Medical Center

Introduction: Peroral endoscopic myotomy (POEM) is a minimally invasive endoscopic option for management of achalasia. It is the authors' practice to use impedance planimetry (EndoFLIP) during the procedure. The objective of this study is to evaluate our clinical outcomes after POEM and final length of gastric myotomy.

Methods: 34 consecutive patients undergoing POEM with EndoFLIP were included in this study. EndoFLIP measurements, including esophagogastric junction distensibility index (DI), diameter (Dmin), and cross-sectional area (CSA) were recorded at 30-mL and 40-mL balloon-fill pre-, intra-, and post-myotomy. The author's practice is to measure EndoFLIP values after initial 1 cm gastric myotomy and repeat every 0.5-1 cm in an effort to limit the gastric myotomy. The myotomy was considered complete when DI ≥ 3.4mm2/mmHg. Postoperative Eckardt score (ES) was determined at initial postoperative visit and most recent follow-up visit (ES2). Linear regression was used to estimate the association between post-myotomy EndoFLIP measurements, postoperative Eckardt score, and change in Eckardt score.

Results: Final EGJ-DI > 3.4 mm2/mmHg was achieved in 33 (97%) patients. In this group, final length of gastric myotomy measured 1.5 cm in 8 (24.2%) patients, 2 cm in 22 (66.7%) patients, and 2.5 cm in 3 (9.1%) patients. Preoperative ES ranged from 2–12 (median 7). Postoperative ES = 0 in 19 (57.6%) patients, ES = 1 in 10 (30.3%) patients, and ES = 2 in 4 (12.1%) patients at initial follow-up. 22 patients had longer-term follow up at a mean of 11 months postoperatively, with ES2 = 0 in 11 (50%), ES2 = 1 in 7 (31.8%), ES2 = 2 in 3 (13.6%), and ES2 = 7 in 1 (4.5%) patient. No significant associations were demonstrated between post-myotomy EndoFLIP parameters and postoperative Eckardt score or change in Eckardt score. Applying previously reported thresholds of DI, CSA, and Dmin to this patient population, there was no statistical significance with clinical result.

Discussion: Normal EGJ-DI was reached in 97% of patients at a gastric myotomy length of 1.5–2.5 cm. Using EndoFLIP to define a completed myotomy at DI of 3.4mm2/mmHg yielded variable lengths of gastric myotomy. Postoperative Eckardt score ≤ 2 in all patients at short term follow-up and 95% at long term follow-up, and was not predicted by other EndoFLIP measurements in this cohort.

S210

Disparities in Access to Minimally Invasive Surgery for Inflammatory Bowel Disease and Outcome by Insurance Status

Yung Lee, MD 1; Lauren Andrew, MD2; Sarah Hill, MD3; Kevin An, MD4; Louisa Chatroux, MD5; Dennis Hong, MD1; Angela Kuhnen, MD6; 1McMaster University; 2University of Calgary; 3Harvard Medical School; 4University of Toronto; 5Brigham and Women's Hospital; 6Lahey Hospital

Introduction: Minimally invasive surgery (MIS) has been widely utilized in colorectal diseases since the early twenty-first century. In the era of MIS being the preferred and safe approach for treating colorectal cancer or diverticular disease, the adoption of MIS has been slower for treating patients with inflammatory bowel disease (IBD) due to its technical challenge. The present study aims to assess the disparities in use of MIS for patients with IBD using a large national database.

Methods: This was a population-based, retrospective observational study utilizing the US National Inpatient Sample (NIS) database from October 1st, 2015, to December 31st, 2019. ICD-10 codes were utilized to identify adult patients with a primary diagnosis of IBD (Crohn's disease or ulcerative colitis) who underwent surgery for their disease. Patients were stratified by insurance status, either private insurance or Medicaid. Primary outcome was access to MIS as their index operation. Secondary outcomes were in-hospital mortality, complications, length of stay, and total admission cost. Univariate and multivariate regression was utilized to determine the association between our outcomes to insurance status. Confounders such as patient characteristics, disease characteristics, and hospital characteristics were used in the adjusted analysis.

Results: The analysis of NIS database revealed a total of 7,866 private insurance and 1,689 Medicaid patients with IBD who underwent surgery. Univariate analysis demonstrated Medicaid patients having less access to MIS, higher in-hospital mortality, complications, length of stay, and admission costs. After adjusting for possible confounders, Medicaid patients have 15% less odds of receiving MIS for their IBD than Private Insurance patients (OR 0.85, 95%CI 0.74–0.97, P = 0.017). There was no difference in rate of in-hospital mortality or overall complications. The Medicaid group had 36% higher odds of genitourinary complications from their index procedure (OR 1.36, 95%CI 1.08–1.71, P = 0.009). In addition, length of stay was longer by 1.76 days (95%CI 1.12–2.41, P < 0.001) and the total cost of admission was higher by $5,043 USD (95%CI 2539–7547, P < 0.001) in the Medicaid group. The independent predictors of more likely receiving MIS were elective operation, under 40 years old, female sex, highest income quartile, diagnosis of ulcerative colitis, and being in a teaching hospital.

Conclusions: Patients with Medicaid are less likely to receive MIS, have longer length of stay, and incur higher costs for the surgical management of their IBD. The results warrant further investigations into improving access to advances in surgical technology and better chronic disease management for this unique patient population.

S211

Comparing the Safety of Revisional Bariatric Procedures for Weight Recurrence and Inadequate Weight Loss After Sleeve Gastrectomy: An Analysis of the MBSAQIP Database

Jerry T Dang, MD, PhD 1; Juan S Barajas-Gamboa, MD2; Gustavo Romero-Velez, MD1; Hadika Mubashir1; Matthew Allemang1; Salvador Navarrete, MD1; John Rodriguez, MD2; Ricard Corcelles, MD, PhD1; Matthew Kroh, MD1; 1Cleveland Clinic Ohio; 2Cleveland Clinic Abu Dhabi

Introduction: There is still controversy concerning the safety and feasibility of the revisional bariatric procedures after sleeve gastrectomy (SG). Revision from SG is becoming increasingly common, due to weight recurrence, inadequate weight loss and gastroesophageal reflux disease (GERD). Current evidence comparing revision from SG to Roux-en-Y gastric bypass (RYGB), single anastomosis duodeno-ileal bypass with sleeve (SADI-S), or biliopancreatic diversion with duodenal switch (BPD-DS) remains limited. As such, the objective of this study was to compare the 30-day rate of serious complications and mortality between RYGB, SADI-S and BPD-DS for weight recurrence and inadequate weight loss after SG.

Methods and Procedures: Patients undergoing revisions from SG to RYGB, SADI-S and BPD-DS were identified from the 2020 MBSAQIP database. Only patients undergoing revision for weight-related indications were included. A multivariable logistic regression analysis was performed between groups to determine if RYGB, SADI-S and BPD-DS were an independent predictors of 30-day serious complications or mortality.

Results: A total of 2,888 patients were included in this study, 2,195 (76.0%) patients underwent RYGB, 268 (9.3%) SADI-S and 425 (14.7%) BPD-DS. The mean age at the time of surgery was 44.8 years without significant difference between groups (p = 0.547). The mean body mass index (BMI) prior to surgery was lower in the RYGB cohort (43.1 RYGB vs 45.5 SADI-S vs 45.6 BPD-DS kg/m2, p < 0.001). Indications for surgery included weight recurrence (63.5%) and inadequate weight loss (36.5%). There were significantly higher rates of diabetes in the SADI-S and BPD-DS cohorts (17.9% SADI-S vs 17.4% BPD-DS vs 12.8% RYGB, p < 0.001). There were also significantly higher rates of GERD in the RYGB cohort (47.7% RYGB vs 35.8% SADI-S vs 28.0% BPD-DS, p < 0.001). SADI-S had slightly longer operative times (155.5 SADI-S vs 149.8 BPD-DS vs 143.4 RYGB minutes, p < 0.001). Procedures were similar in terms of serious complications (p = 0.467). SADI-S was associated with a higher rate of leaks (2.2% SADI-S vs 1.4% BPD-DS vs 0.5% RYGB, p < 0.001). There were no statistical differences between cohorts for reoperations (p = 0.366), readmissions (p = 0.999) and deaths (p = 0.854) within 30 days. On multivariable analysis, revisional procedure was not independently predictive of serious complications when adjusting for age, sex, BMI, comorbidities, and operative time.

Conclusions: RYGB represents 76.0% of SG revisional procedures for weight-related reasons. Our study demonstrates that revision from SG to RYGB, SADI-S and BPD-DS are equally safe, with similar 30-day complication and mortality rates, although SADI-S had slightly longer operative times and higher leak rates.

S212

Comparison of Outcomes of Robot Assisted Laparoscopic and Laparoscopic Heller Myotomy with Dor Fundoplication

Megan McCaul, MD1; Naden Kreitz, MS1; Mark Ward, MD1; Madhuri S Mulekar, PhD2; William Richards, MD1; 1USA Health University Hospital; 2University of South Alabama, Department of Mathematics

Introduction: Robot assisted laparoscopic surgery has greater magnification and the benefit of three-dimensional views during surgery when compared to traditional laparoscopic surgery. In addition, the endowrist allows greater degrees of freedom and range of motion compared to laparoscopic instruments. The surgeon can adjust hand to instrument ratios, and tremor reduction enhances fingertip control in the robotic instruments. A retrospective study of the outcomes of robot assisted laparoscopic Heller myotomy with Dor fundoplication cases were compared to laparoscopic procedures to evaluate patient outcomes.

Methods: Seventy-two patients who underwent robot assisted laparoscopic or laparoscopic Heller myotomy with Dor fundoplication with a single surgeon at the University of South Alabama from March 2014 to July 2022 were included in this IRB approved study. The key variables obtained were patient demographics, operative duration, postoperative length of stay, intra-operative perforation, need for postoperative dilation, need for revision surgery, pre- and post-op GERD HQRL questionnaires.

Results: The robotic group had no intraoperative perforation (0/46) which was significantly lower than laparoscopic group (3/26 11.54%) (p = 0.0436). The robotic group did have significantly longer mean operative duration, 179 min vs 121 min (p < 0.0001). Also, the operative duration decreased with experience in the robotic group. When comparing hospital length of stay the median days to discharge was significantly higher in the laparoscopic group at 2 days compared to 1 day for the robotic group (p = 0.0245). Both groups had significantly reduced GERD HQRL post operatively. The laparoscopic group decreased from a mean of 42.2 to 20.87 (p < 0.0001) and the robotic group decreased from a mean of 28.07 to 10.66 (p < 0.0001).

Conclusions: Robotic technology has allowed for safe performance of dissection with significantly less intraoperative perforations during Heller myotomy. This procedure has continued to become a more efficient and has resulted in shorter hospital stays. Patient symptom outcomes have been excellent and better than laparoscopic. Robotic Heller myotomy with Dor fundoplication is our procedure of choice for patients with achalasia and results in lower rates of intraoperative perforation, better outcomes and lower length of stay compared to traditional laparoscopy.

S213

Tailoring the Wrap: Intraoperative Functional Lumen Imaging Probe (FLIP) During Hiatal Hernia Repair

Hemasat Alkhatib, MD; Ali Kara, MD; AJ Haas, MD, MMSc; Clara Lai, MBBS; Sergio Bardaro, MD, FACS, FASMBS; Amelia Dorsey, MD; Kevin El-Hayek, MD; The Metrohealth System

Introduction: The introduction of the Functional Lumen Imaging Probe (FLIP) has provided objective, real-time feedback on the geometric variations with each component of a hiatal hernia repair (HHR). The utility of this technology in altering intraoperative decision making has not been reported. Herein, we report a single-center series of intraoperative FLIP during HHR.

Method: A retrospective review of electronic medical records between 2020–2022 was conducted and all patients undergoing non-recurrent HHR with FLIP were queried. Patient and hernia characteristics, intraoperative FLIP values and changes in decision making, as well as early post operative outcomes were reported. Both diameter and distensibility index (DI) were measured at 40 ml and 50 ml balloon inflation after hiatal dissection, after cruroplasty, and after fundoplication when indicated.

Results: Thirty-two patients met inclusion criteria. Mean age was 62 ± 14 years and mean BMI was 28 ± 6 kg/m2. The majority (53%) were type I hiatal hernias. The largest drop in DI occurred after cruroplasty, with minimal change seen after fundoplication (mean DI of 4.26 ± 2.03 after completion of HH dissection, vs 2.71 ± 1.16 after cruroplasty and 2.37 ± 1.08 after fundoplication when performed). In 13 (41%) of cases, FLIP values directly impacted intraoperative decision-making. Fundoplication was deferred in 4/13 (31%) patients, the wrap was loosened in 6/13 (46%); the type of fundoplication was altered to achieve adequate anti-reflux values in 2/13 (15%) patients, and in 1/13 (8%) the wrap was tightened. At 30 day follow up, 20/32 (62%) of patients had resolution of symptoms, while 12/32 (38%) of patients reported gastrointestinal symptoms such as bloating (n = 3), dysphagia (n = 8), or some reflux (n = 9).

Conclusion: FLIP measurements can be used intraoperatively to guide decision making and alter management plan based on objective values. Long-term outcomes and further prospective studies are required to better delineate the value of this technology.

S214

Using Prucalopride for prevention of Post-operative Ileus in Gastrointestinal Surgery: A Systematic Review and Meta-analysis of Randomized Controlled Trials

Gaurav Talwar 1; Sahil Sharma1; Shuling Yang2; Jigish Khamar2; Tyler McKechnie1; Aristithes Doumouras3; Dennis Hong3; Cagla Eskicioglu3; 1Department of Surgery, Division of General Surgery, McMaster University; 2Michael G. DeGroote School of Medicine, McMaster University; 3Department of Surgery, Division of General Surgery, McMaster University, St. Joseph's Healthcare

Introduction: Post-operative ileus is a difficult to treat consequence of abdominal surgery that contributes to patient morbidity and prolonged hospitalization. Prucalopride, a highly selective 5-hydroxytryptamine (5HT4) receptor agonist, is a novel pharmaceutical proposed to enhance bowel motility through prokinetic and anti-inflammatory mechanisms. This review aims to elucidate whether the use of prucalopride compared to placebo is associated with preventing prolonged postoperative ileus in patients undergoing gastrointestinal surgery.

Methods and procedures: OVID, CENTRAL and EMBASE were searched as of July 2022 to identify randomized controlled trials comparing outcomes of prucalopride versus placebo for pharmacologic prevention of post-operative ileus in adult patients undergoing surgery of the gastrointestinal tract. The primary outcomes were time to defecation, time to flatus and length of stay. The secondary outcomes were post-operative complications scored by Clavien-Dindo classification, medication related adverse events, nasogastric tube reinsertion, and overall costs. A pairwise meta-analysis was performed using an inverse variance random effects model.

Results: From 143 citations, three randomized controlled trials with 139 patients treated with prucalopride (33% female) and 140 patients treated with placebo (39% female) were included. In the prucalopride group, 74 patients received 2 mg 2 h pre-operatively and 2 mg daily for 6 days, 55 patients received 2 mg daily for 7 days post operatively, and 10 patients received 2 mg 16 h and 2 h pre-operatively. Patient underwent a variety of upper gastrointestinal and pancreatic surgeries, well matched amongst groups. Patients treated with prucalopride had a decreased time to defecation (mean difference -1.49 days, 95% CI -2.44-(-)0.53, p < 0.002, I2 = 61%) but no significant difference in length of stay (mean difference -3.48 days, 95% CI -8.52–1.56, p = 0.18, I2 = 91%). Time to flatus was reported by two studies with both reporting significant reduction with prucalopride. One study studied post-operative costs and found no difference. Post-operative complications and adverse events could not be meta-analyzed due to heterogeneity, yet data from individual studies suggested no significant differences between the two treatment groups.

Conclusions: Prucalopride may be an appropriate pharmacological adjunct to enhance return of bowel function in patients undergoing gastrointestinal surgery. Larger prospective randomized controlled trials assessing clinically relevant outcomes and associated costs are needed before routine use of this agent.

Fig. 1 Time to Stool in days. Random effect meta-analysis comparing treatment with prucalopride versus placebo

figure cs

Fig. 2 Postoperative length of stay in days. Random effect meta-analysis comparing treatment with prucalopride versus placebo

figure ct

S215

Can Robotic Approach Reduce Wound Morbidity in High-Risk Patients? A Propensity Score Analysis of Open Versus Robotic Retromuscular Ventral Hernia Repair Utilizing the ACHQC Database

Jeffrey G Gaskins, MD 1; Li-Ching Huang, PhD2; Sean C O'Connor, MD1; 1Mountain Area Health Education Center, Inc.; 2Vanderbilt University Medical Center

Background: Retromuscular sublay (RMS) technique for ventral hernias has gained popularity due to lower risk of recurrence, wound morbidity, and mesh related complications1. Compared to traditional open RMS, robotic approaches have been shown to decrease hospital stay2, but most studies have failed to show a reduction in wound morbidity. Utilizing the Abdominal Core Health Quality Collaborative (ACHQC) database, this study sought to determine the effect of robotic approach on wound morbidity, while specifically focusing on a high-risk population.

Methods and Procedures: A retrospective review of all elective robotic and open RMS repairs in the ACHQC database was performed. Patients deemed to be high-risk for wound complications were included: adult patients with body mass index (BMI) greater than 35 and who were either current smokers or diabetics. A propensity score match was then done to balance covariates between the two comparison groups. Primary outcomes were surgical site occurrences (SSO), surgical site infections (SSI), and surgical site occurrences requiring procedural intervention (SSOPI) at 30-day follow-up.

Results: A total of 917 patients met inclusion criteria, 664 (72.4%) underwent Open RMS repair, and 253 (27.6%) underwent Robotic RMS repair. After propensity score matching, 212 patients matched for each approach. There was no difference in overall SSO (22% for Open vs 23% for Robotic, p = 0.91). Open repair was associated with higher rates of SSI (5% vs 1%, p = 0.03) and SSOPI (12% vs 3%, p < 0. 001). As seen in previous studies, there was a higher rate of seroma associated with Robotic RMS repair (85% vs 45%, p < 0.001) in patients that developed an SSO.

Conclusions: In this analysis of the ACHQC database, a Robotic approach was associated with decreased rates of SSI and SSOPI in obese patients who were either current smokers or diabetics. A robotic approach would eliminate the need for intervention in 1 of 11 patients in this cohort. A minimally invasive approach to RMS should be considered in this patient population in order to decrease wound morbidity and the associated physical and economic costs.

References

1. Holihan JL, et al. Mesh Location in Open Ventral Hernia Repair: A Systematic Review and Network Meta-analysis. World J Surg. 2016;40(1):89-99. doi: 10.1007/s00268-015-3252-9

2. Carbonell AM, et al. Reducing Length of Stay Using a Robotic-assisted Approach for Retromuscular Ventral Hernia Repair: A Comparative Analysis from the Americas Hernia Society Quality Collaborative. Ann Surg. 2018;267(2):210-217. doi: 10.1097/SLA.0000000000002244

S216

Endoscopic Sleeve Gastroplasty (ESG) as a Novel and Efficient Option to Treat Obesity: A Case Series Report of 65 Patients

Ruben Neris, MD 1; Abel Bello, MD, FACS, FASMBS2; 1MIB surgery, WRHE; 2MIB surgery

Background and Aims: Obesity has become a pandemic, which has worsened for the last 50 years. It is the second most common cause of preventable death after smoking. The growing burden of obesity as a chronic disease necessitates a multifaceted approach to management. Bariatric surgery is the most successful treatment for obesity. However, many patients avoid surgery due to its perceived invasive nature and fear of complications. Endoscopic sleeve gastroplasty (ESG) has then emerged as a seemingly less invasive option. We performed this study to evaluate the efficacy and safety of ESG in the treatment of obesity.

Methods: This was a large case series report study. Cases were based on participants who underwent ESG in a single, private medical center. All procedures were performed by the same surgeon/endoscopist. Inclusion criteria were patients >18 years old, BMI of >30kg/m2, who underwent an ESG between August 2020 and July 2021. Patients were then divided into 3 groups based on the time post-procedure (<2 months, 2-3 months, 4-10 months). The primary outcomes were % total body weight loss (TBWL), total weight loss, % BMI loss and moderate-adverse events (AEs).

Results: We reported 65 cases (86% female) with a BMI of 33±3 kg/m2 that underwent an ESG. Out of the 65 cases documented on the study, 24 composed the group 1 (<2 months post-procedure), 26 and 15 the group 2 and 3 (2-3 months and >4 months, respectively). In the group 1, the data reported a mean total weight loss of 19 (pounds), a % total body weight loss (TBWL) of 10.75% and a total BMI loss of 3.37. In the group 2, the mean total weight loss reported was 25 (pounds), a % total body weight loss (TBWL) of 13.3% and the total BMI loss of 3.8. Ultimately, the group 3 had a mean total weight loss of 27 (pounds), a % total body weight loss (TBWL) of 14.3% and a total BMI loss of 4.6. The overall moderate-adverse event (AEs) was 1.53%. No fatal or severe adverse events were reported.

Conclusion: Our findings suggest that ESG is a considerable option for weight loss in obese patients. According to the data analyzed, there is significant weight loss as early as a month post-procedure, which can be increased by half in the next months. Therefore, this becomes important in obese patients that desire a safe, non-invasive procedure for weight loss.

S217

Influence of Race on Bariatric Surgery Choice

Meagan Read, MD 1; Emily Grimsley1; Haroon Janjua1; Ricardo Pietrobon2; Paul Kuo1; Salvatore Docimo1; 1University of South Florida; 2SPORE Data

Introduction: We hypothesized that minorities would be more likely to receive a gastric sleeve than a Roux-en-Y bypass (bypass), when body mass index (BMI) and comorbidities were held constant. Inequality in healthcare can be seen in access to care, as well as in the choice of treatments patients are offered.

Methods and Procedures: Patients who underwent an elective, initial bypass or gastric sleeve were selected from ASMBS (American Society for Metabolic and Bariatric Surgery) database from 2017 – 2020. Propensity score matching was utilized to estimate differences in procedure choice using a 1:1 matching algorithm according to BMI group, age, sex, and comorbidities including GERD, diabetes, and smoking status. Density plots utilizing area under the curve demonstrated cost overlap.

Results: There were 669,374 cases (483,708 sleeve, 185,666 bypass) included. Overall, a significantly higher percentage of White patients received a gastric bypass rather than sleeve (29 vs 71%, respectively) compared with the higher utilization of sleeve in Black patients (22 vs 78%) (p<0.001). Overall, White patients more often underwent bypass (71%), while Black patients more often underwent sleeve (78%). After propensity matching to control for differences in comorbidities, Black patients were still more likely to undergo sleeve than their White and Hispanic counterparts with the same BMI. Interestingly, Black patients were more likely to receive a robotic surgery than patients of other races. Hispanic patients were least likely to receive robotic surgery. While statistically significant, there was minimal difference in 30-day follow-up between the cohorts (95.2 – 96.3%, p<0.001). Black patients did, however, have a higher rate of readmissions within 30 days, but there was not a large difference in interventions or re-operations between races.

Conclusions: While bariatric surgery has become more prevalent, the opportunity to receive the best and appropriate surgery for the patient derives from many factors. After controlling for comorbidities and BMI differences, minority patients appear to continue to undergo sleeves at a higher BMI than their white counterparts. As robotic technology increases, there remains a disproportional use of this technology, which continues to be limited to certain populations.

figure cu

S218

Automatic Surgical Phase Recognition with Artificial Intelligence

Sarah Choksi, MD, MPH 1; Daniel Bitner, MD1; Chengbo Zang, BS2; Yuqing Cao, BS2; Zixuan Xiang, BS2; Kaan Yarali, BS2; Skyler Szot, BS2; Zoran Kostic, PhD2; Filippo Filicori, MD1; 1Lenox Hill Hospital, Northwell Health Department of Surgery Intraoperative Performance Analytics Laboratory; 2Columbia University Department of Electrical Engineering

Background: Automated phase recognition can provide easy storage and access capabilities for surgical video files, optimization of the surgical workflow and enhanced video based assessment. Computer Vision (CV) is a form of Artificial Intelligence (AI) that allows interpretation of images through a deep learning algorithm. Our aim was to develop and train a CV model that is able to recognize surgical phases with acceptable accuracy and latency for one of the most commonly performed surgical operations: Inguinal Hernia Repairs (IHR).

Methods: Two hundred and eleven IHR videos performed by 9 different surgeons were obtained from a large multicenter tertiary hospital network. The surgical videos were downsampled to 1 frame per second and annotated into 14 surgical phases (adhesiolysis, blurry, catheter insertion, mesh placement, mesh positioning, out of body, peritoneal closure, peritoneal scoring, positioning suture, preperitoneal dissection, primary hernia repair, reduction of hernia, stationary idle and transitionary idle).

Results: After several architectures were considered, a ResNet-50 model was selected for image classification tasks. ResNet-50 is a convolutional neural network containing approximately 23 million trainable parameters, occupying 90 MB in memory. Using the annotated videos, the ResNet-50 model was trained to automatically classify surgical phases. Thirteen different combinations of hyperparameters, training splits, and data augmentation were investigated. The best performing model achieved a weighted F1-score of 0.8011. A TMRNet model with best predicted F1 score of 0.82467 was also considered, however, ResNet-50 was chosen due to its ability to predict phases without temporal association allowing future implementations in real time.

Conclusions: The ResNet-50 model training for inguinal hernia repair phase recognition shows its ability to predict surgical phases with good accuracy. Further optimization of the model is needed to improve inference outcomes. Real time implementation of this model in the operating room would allow same time decisions to be made by surgical staff that could optimize workflow.

figure cv

S219

Avoiding Parathyroid Injury During TOETVA: An Experience from a Low Resource Single Center in Rural India

Narendra V Lohokare, MD, DNB, FACS; Aditi N Lohokare, MBA; Siddhakala Hospital

Introduction: The advantage of enhanced vision in Trans Oral Endoscopic Thyroidectomy Vestibular Approach (TOETVA) has enabled the surgeons to reduce incidence of parathyroid injury.

In our series of 42 TOETVA surgeries we tried to find ways to avoid parathyroid injuries by use of advanced modalities available with newer optics:

Indigo Cyanine Green (ICG) angiography & Autofluorescence

Materials & Methods: In 42 patients operated from November 2019 to April 2022 advanced modalities were used to identify Parathyroids.

Initially superior Parathyroids were identified and confirmed using the Autofluorescence.

While ICG angiography was used to check the vascular anatomy and perfusion status.

Parathyroids were graded from 0 to 2 with respect to their viability & perfusion status.

This method of Identifying Parathyroids was very useful in our hospital which is a low resource center in Rural India

Results: Advanced modalities helped in correctly identifying & confirming the parathyroids with more accuracy there by avoiding injury and ultimately hypoparathyroidism.

In our series, out of 42 patients Autofluorescence was used first to identify & confirm the Parathyroids.

ICG Angiography was later used to identify its vascular anatomy & perfusion status and at a later stage to confirm the perfusion & Viability.

The use in the later stage helped in deciding the requirement of auto transplantation.

Measurement of PTH levels intraoperative helped in postoperative planning for treatment of hypocalcemia.

The incidence of permanent or transient hypoparathyroidism was very less (< 1%, < 3%) in our series.

Conclusion: Along with the advantage of better vision of endoscopy, parathyroid visualization & confirmation has become more reliable with newer modalities thus bettering the preservation of parathyroids during Endoscopic thyroid surgery (TOETVA).

S220

Outcomes following Laparoscopic Transperitoneal Adrenalectomy: Experience of a Decade from a Tertiary Care Centre

Krishna Asuri; Deepti Singh; Keshav Aggarwal; Raghunandan Dixit; Virinder Bansal, Prof; Omprakash Prajapati; Subodh Kumar; Mahesh Misra; AIIMS, New Delhi

Introduction: Minimally invasive adrenelectomy is the standard of care for adrenal lesion. We describe the outcomes of laparoscopic transperitoneal adrenelectomy (LTA) done in a single surgical unit of a tertiary care hospital over one decade.

Materials and Methods: This was a retrospective analysis of a prospectively collected database of patients undergoing LTA between January 2011 and May 2021. The demographic profile, details of clinical workup and laboratory parameters were recorded in a pre-structured proforma. Functional workup and anatomical imaging were performed for all the patients. Patients were taken up for surgery after adequate preoperative optimisation with anti-hypertensives, steroids and fluids, as indicated. Outcomes including conversions, hospital stay, wound infection were analysed.

Results: A total of 173 adrenelectomies were performed out of which 158 were LTA ( right 55 (34.8%), left 88 (55.7%) and bilateral 15 (9.5%)). The most common indication was pheochromocytoma (54%) followed by non functional adenomas. The mean tumor size was 5.5 cm. The mean operative time was 80 min (range 30-174 min). The mean estimated blood loss was 50 ml and the duration of hospital stay was 2.8 days. Conversion to open procedure was required in 15 patients. The most common post operative complication was bleeding seen in 6 (3.3%) patients which required a re-exploration. There was no 30-day mortality. The follow-up was 15 months (range; 6-72 months. None of the patients had recurrence of the primary disease during the follow up period.

Conclusion: Preoperative evaluation, optimisation and accurate selection of patients is crucial for successful laparoscopic adrenalectomy resulting in good outcomes with minimal post operative morbididty.

S221

Minimally Invasive Versus Open Pancreatectomy for Nonfunctioning Pancreatic Neuroendocrine Tumors: A Systematic Review and Meta-Analysis

Amanda Cyntia Lima Fonseca Rodrigues; Fatemeh Shojaeian, MD, MPH; Theethawat Thanawiboonchai, MD; Kelly Lafaro, MD, MPH; Gina Adrales, MD, MPH; The Johns Hopkins Hospitals, The Johns Hopkins University School of Medicine

Introduction: Minimally invasive surgery (MIS) has been suggested to reduce adverse postoperative outcomes compared to open surgery (OS) in pancreatic neuroendocrine neoplasms. However, safety and efficacy of MIS for non-functioning pancreatic neuroendocrine tumors (NF-PNETs) still have limited evidence. This meta-analysis aimed to compare MIS and OS for NF-PNETs.

Methods and Procedures: A systematic review and meta-analysis was conducted on minimally invasive compared with open pancreatectomy for nonfunctioning pancreatic neuroendocrine tumors. MEDLINE, EMBASE, and Cochrane databases were screened until September, 2022. Out of 94 results, 10 studies were selected for full review, and 3 articles were considered in the final investigation. The disease-free survival (DFS) was assessed as the primary outcome and postoperative complications (POC) were the second outcomes.

Results: Three prospective studies with a total of 1093 patients were included. In Hwang et al., DFS rates (OS [87.3%/80.9%] vs MIS [94.1%/94.1%]), p = 0.001 were significantly higher in the MIS group than in the OS group. POC was lower at 52 (27.6) in MIS than in OS 73 (23.4). However, in Kim et al., there was no statistically significant difference found in DFS between OS and MIS group, p = 0.428, with a total time of 80 months. During this period, the MIS DFS was higher than the OS, however, from the 30th month onwards, the OS DFS dropped dramatically. In POC p = 0.65, approximately 50% of patients experienced complications, of whom 9 (8.1%). In Patelli et al., the DFS was p = 0.234 at 150 months and the POC was significantly lower in the MIS group (p = 0.005, grade of complication with highest estimated probability O vs 2).

Conclusion: The benefits of MIS compared to OS in both outcomes are promising and high. It is possible to state that MIS is safer and more effective than OS. Nonetheless, due to the scarcity of studies on this topic, further researches are encouraged to better assess the selected outcomes.

S222

Comparison of Effectiveness and Complications of Laparoscopic Transabdominal and Retroperitoneoscopic Adrenalectomy

Volodymyr Grubnik, MD; Yuriy Grubnik, Professor; Viktor Grubnyk; Vadym Ilyashenko, PhD; Roman Parfentiev, PhD; Odessa National Medical University

Laparoscopic adrenalectomy is the "gold standard" approach for benign adrenal framers. Majority of surgeons perform laparoscopic transabdominal adrenalectomies. Retroperitoneoscopic adrenalectomies in prone position by Waltz have some advantages for patients. Aim of the study was to compare effectiveness and number of complications of two methods: transabdominal and retroperitoneoscopic laparoscopic adrenalectomies.

Material and Methods: From 2000 to 2021 years 472 laparoscopic adrenalectomies were performed in our clinic. Age of the patients from 19 to 79 years, mean age 50,5 ± 10,2 years. There were 315 women, and 157 men. Indication for operation were follows: incidentaloma—32,5%, pheochromocytoma—30,2%, aldosteronoma—19%, corticectomy -10,3%, myelolipoma—3%, metastatic cancer – 5%. Size of tumors was from 1 to 10 cm.

Results: Transabdominal adrenalectomies were performed in 316 patients: right adrenal ecotones—206 patients, left adrenalectomies – 110 patients. Size of tumors were: 1-2 cm – 110 patients, 3-4 cm – 98 patients, 4-6 cm – 81and more than 6 cm in 27 patients. Complication's rate was 8,9% (28 patients), conversions were in 8 patients (2,6%). Retroperitoneoscopic adrenalectomies were performed in 156 patients. Size of tumors were: 1-2 cm in 96 patients. 3-4 cm – 39 patients, 4-6 cm – 18 patients, more than 6 cm in 3 patients. Complication rate was 12,2% (19 patients). Conversions to transabdominal approach was in 3 patients with tumor size more than 6 cm and in 4 patients with size of tumor 4-6 cm, conversions rate was 4,9%. In the patients with small tumors retroperitoneoscopic approach has some advantages: shorter curation of surgery, lower operative blood loss, diminished pain intensity, lower prevalence of shoulder – tip pain, shorter time to oral intake, shorter length of hospital stays.

Conclusions: Both approaches were equally safe. In the patients with small tumors retroperitoneoscopic adrenalectomies were superior to transabdominal approach in terms of shorter surgery duration, lower blood loss, lower postoperative pain, faster recovery, good cosmetic results. In the patients with large tumors transabdominal approach is much better with lower complications and conversions rate.

S223

Comparative Outcomes Following Adrenalectomy Performed by General Surgeons Versus Urologists: A National Vizient-Database Study

Ryan Lamm, MD1; Sunjay Kumar, MD 1; Costas Lallas, MD1; Dani Tresiman2; Talar Tatarian, MD1; Francesco Palazzo, MD1; 1Thomas Jefferson University Hospital; 2Vizient

Introduction: Adrenalectomy is a complex procedure performed by general and urologic surgeons. Specialty training may lead to different results in terms of patient care. There has been no national study comparing the surgical outcomes following this procedure between these specialties. We hypothesize that no difference in outcomes would be identified and targeted a large national database for the query.

Methods and Procedures: Adult patients undergoing adrenalectomy were identified in the Vizient® Clinical Data Base (2017-2020). The groups were divided based on specialty. Postoperative outcomes included length of stay, reoperation, readmission, mortality, surgical site infections, and total cost. Categorical and continuous variables were compared using Chi-square and Wilcoxon rank sum tests on. Additional risk-adjusted analysis was performed.

Results: 14,865 adrenalectomy patients were included in the study, 6,710 performed by general surgeons and 8,155 by urologists. Univariate analysis revealed statistically significant differences in length of stay, readmissions, surgical site infections, and total costs. However, when these results were risk-adjusted, analysis showed urologists had significantly higher risk of readmission (OR 1.97, p = 0.049), but showed no difference in mortality (OR 1.18, p = 0.236) and total cost (OR 0.434, p = 0.664) compared to general surgeons (Table 1).

Conclusion: At a national level, urologists performed more overall adrenalectomies and more open procedures. Based on our risk-adjusted results, subspecialty training does not seem to be linked to different outcomes in adrenal resections.

Table 1

 

Laparoscopic

Open

 

General (N = 4,436)

Urology (N = 4,203)

p-value

General (N = 2,274)

Urology (N = 3,952)

p-value

Length of Stay, Days (IQR)

2 (1–2)

2 (1–3)

 < 0.001

6 (4–9)

4.5 (3–7)

 < 0.001

Reoperation, N (%)

27 (0.6)

27 (0.6)

0.8

43 (1.9)

56 (1.4)

0.2

Readmission, N (%)

183 (4.1)

207 (4.9)

0.074

308 (14)

358 (9.1)

 < 0.001

Mortality, N (%)

9 (0.2)

10 (0.2)

0.7

44 (1.9)

53 (1.3)

0.068

Surgical Site Infection (SSI), N (%)

10 (0.2)

11 (0.3)

0.7

64 (2.8)

42 (1.1)

 < 0.001

Total Cost, US dollars (IQR)

$11,806 ($8,880-$16,472)

$13,904 ($10,206-$19,386)

 < 0.001

$25,042 ($16,803-$38,999)

$20,036 ($14,335-$31,329)

 < 0.001

S224

Outcomes of Adrenalectomy for Pheochromocytoma in the NSQIP Database

Minjae Kim, MD; Selim Gebran, MD; Dosuk Yoon, DO; Leaque Ahmed, MD, FACS; Hector DePaz, MD, FACS; Paritosh Suman, MD, FACS; Wyckoff Hospital

Introduction: Pheochromocytoma may rarely present as a hypertensive crisis with multisystem organ failure. Patients diagnosed with pheochromocytoma crises require intensive antihypertensive treatment and urgent adrenalectomy with expected poor prognosis.

Methods: Patients with pheochromocytoma treated surgically in the NSQIP (2013 to 2019) were selected based on combination of post-operative ICD and CPT codes. Patient characteristics and in-hospital as well as 30-day outcomes were compared between elective vs urgent/emergent surgery using univariable analyses.

Results: A total of 1,344 laparoscopic or open adrenalectomieswere identified in the seven-year study period. Of those one hundred (7.4%) were for a diagnosis of pheochromocytoma. Eleven patients underwent urgent/emergent adrenalectomy, and eighty-nine underwent elective adrenalectomy. The surgical approach was laparoscopic in 68% and open in 32%. Six patients had a severe complication (Clavien-Dindo IV/V) and four developed a myocardial infarction of whom one died within thirty days (p > 0.05). Nine patients received perioperative transfusion(s). Operative characteristics and thirty-day outcomes that were statistically different between elective and urgent/emergent settings were median operative time (2.2 vs 3.5 h, p = 0.002), median length of hospital stay (2 vs 12d, p < 0.001) and peri-operative transfusion (5.6% vs 36.4%, p = 0.008).

Conclusion: Overall complications post-adrenalectomy for pheochromocytoma were not more common in the urgent/emergent setting than in the elective setting. However, patients who received urgent/emergent adrenalectomy more often received a blood transfusion perioperatively and had a longer operative time and hospital length of stay.

S225

Thirty-Day Postoperative Outcomes Following Laparoscopic Adrenalectomy for Functional Adrenal Tumors

Raisa R Gao, DO; Gitonga Munene, MD; Saad Shebrain, MD; Western Michigan University Homer Stryker M.D. School of Medicine

Background: The study aims to evaluate demographic characteristics and 30-day outcomes after laparoscopic adrenalectomy for Functional adrenal tumors (FATs). FATs are rare, and if left untreated, there is a substantial risk of morbidity and mortality due to uncontrolled excess hormone secretion. The three most common FATs are cortisone-producing tumors (Cushing's syndrome, CUSH), primary hyperaldosteronism (Conn's syndrome, PACS), and pheochromocytoma (PHEO).

Methods: Patients who underwent adrenalectomy were selected from the ACS-NSQIP database (2015-2017). Patients with a diagnosis of FATs were identified. Preoperative demographics/medical comorbidities, and 30-day postoperative outcomes were analyzed using the chi-squared test, ANOVA, and Kruskal-Wallis.

Results: Of a total of 2410 patients who underwent laparoscopic adrenalectomy, 345 (14.3%) patients had FATs, of which PACS was present in 199/345 (57.7%) patients, CUSH in 110/345 (31.9%), and PHEO in 36/345 (10.4%) patients. Compared to PACS and PHEO groups, patients in the CUSH group were younger (47.8 [± 14.7], vs. 51.6 ± [11.7] vs 53.5 ± [16.1]), had a higher proportion of females (84.5% vs. 39.7% vs. 63.9%, p < 0.001), had a higher BMI (34.6 ± [7.7] vs 31.0 ± [6.7] vs. 26.0 ± [6.3]), had a higher proportion of DM (33.6% vs.19.6% vs. 11.1%, p = 0.004), and had a higher proportion of white ethnicity(71.8% vs. 47.7% vs. 61.1%, p < 0.001). Compared to CUSH and PHEO, the PACS group had a higher proportion of Black ethnicity (21.6% vs. 4.5% vs. 8.3%, p < 0.001) and a higher proportion of HTN requiring medication (95.5% vs. 69.1% vs. 77.8%, p < 0.001). Thirty-day postoperative outcomes: compared to CUSH and PACS, the PHEO group had a higher rate of serious morbidity (11.1% vs. 3.6% vs. 0.0%, p < 0.001), overall morbidity (13.9% vs. 4.5% vs. 1.0%, p < 0.001), and higher readmission rates (5.6% vs. 4.5% vs. 4.0%, p = 0.9). There were three deaths: 1 in the PHEO and 2 in the CUSH groups. No reported return to OR in any group. Operative time, in minutes, was longer in CUSH compared to PACS and PHEO (157 ± [81] vs. 121 ± [55] vs. 137 ± [70]). Median LOS was higher in CUSH and PHEO compared PACS (2 [1, 2.25] vs. 1 [1,2], p = 0.014) and (1.5 [1, 3] vs. 1 [1,2], p < 0.001).

Conclusion: Functional adrenal tumors show distinct variations in patient demographics. While almost all patients with PACS have HTN, patients with CUSH have higher BMI and diabetes. Patients with pheochromocytomas have higher overall and serious morbidity, especially cardiovascular events. Both CUSH and PHEO have longer LOS. Preoperative optimization is essential.

S226

Do Advances in Technology Translate to Improved Outcomes? Comparing Robotic Bariatric Surgery Outcomes Over Two Time Intervals Utilizing the MBSAQIP Database

Alisa Coker1; Raul Sebastian2; Jacob Tatum 1; Jorge Cornejo3; Alba Zevallos2; Christina Li4; Michael Schweitzer1; Gina Adrales1; 1Johns Hopkins Medicine; 2Carroll County Memorial Hospital; 3Mayo Clinic Jacksonville; 4LifeBridge Health

Introduction: This study aims to compare outcomes and utilization of robotics in bariatric procedures across two recent time intervals, chosen because they correspond to drastic changes in technology utilization – namely, a new platform and a new stapling device. Outcomes of robotic roux-en-y gastric bypass (rRYGB) and robotic sleeve gastrectomy (rSG) across this changing landscape have not been well studied, despite increasing popularity.

Methods and Procedures: The MBSAQIP database was analyzed over an early (2015-2016) and late (2019-2020) time interval. Patients who underwent rSG and rRYGB were identified, and the cohorts were matched for 26 preoperative characteristics using Propensity Score Matching analysis. We then compared 30-day outcomes and bariatric-specific complications between the early and late time frames for rSG and rRYGB.

Results: 49,442 rSG were identified: 13,526 cases in the early time frame and 35,916 in the late time frame. The matched cohorts were 13,526 for the two groups. 30-day outcomes showed that in the late time frame, rSG was associated with lower rates of pulmonary complications (0.1% vs 0.3%, p < 0.001), readmissions (2.5% vs 3.6%, p < 0.001), interventions (0.6% vs 1.4%, p < 0.001), reoperations (0.7% vs 1.0%, p = 0.024), length of stay (1.36 + 1.01 days vs 1.76 + 1.79 days, p < 0.001), operative time (92.47 + 41.70 min vs102.76 + 45.67 min p < 0.001), staple line leaks (0.2% vs 0.4%, p = 0.001) and strictures (0.0% vs 0.2%, p < 0.001). Similarly, 21,933 rRYGB were found: 6,514 cases were identified in the early time frame and 15,419 in the late time frame. The matched cohorts were 6,513 for the two groups. 30-day outcomes revealed that the late time fame rRYGB was associated with lower rates of pulmonary complications (0.1% vs 0.3%, p = 0.012), readmissions (6.3% vs 7.2%, p = 0.050), interventions (2.0% vs 3.1%, p < 0.001), length of stay (1.69 + 1.46 days vs 2.13 + 2.12 days p < 0.001), postoperative bleeding (0.4% vs 0.7%, p = 0.001), stricture (0.4% vs 0.8%, p < 0.001) and anastomotic ulcer (0.2% vs 0.4%, p = 0.013).

Conclusion: Compared to early robotic bariatric surgery outcomes, a significant reduction in pulmonary complications, readmissions, reoperations, interventions and length of stay were seen in 2019-20 after rSG and rRYGB. Potential contributing factors include increased surgical experience and advances in the robotic platform. A significant recent reduction in staple line leaks with faster operative times associated with rSG suggests that stapling technology has had a positive impact on patient outcomes.

S227

Learning Curves of Robotic Extended Totally Extraperitoneal (ETEP) Hernia Repair Among Two Surgeons at a High-Volume Community Hospital: A Cumulative Sum Analysis

Katie L Korneffel, MD 1; Wendy Nuzzo, MS2; Madelyn Krebs, BS3; C. Micha Belden, PhD2; Lindsee McPhail, MD4; Sean O'Connor, MD4; 1Mountain Area Health Education Center (MAHEC); 2MAHEC Department of Research; 3UNC School of Medicine; 4HCA Healthcare—Mission Hospital

Introduction: Robotic extended totally extraperitoneal hernia (eTEP) repair is a novel technique for minimally invasive ventral hernia repair with retromuscular placement of mesh. This study aimed to evaluate the learning curve for robotic eTEP hernia repair using a risk-adjusted cumulative sum (RA-CUSUM) analysis for two general surgeons—one with dedicated fellowship training and experience in robotic eTEP technique (surgeon 2), and another surgeon without fellowship training and experience (surgeon 1). Understanding the expected learning curve can guide expectations for acquiring more advanced surgical skills by community-based general surgeons.

Methods and Procedures: We conducted a retrospective analysis of 98 patients undergoing robotic eTEP hernia repair from July 2020 to February 2022 for two general surgeons. A RA-CUSUM method was applied to the overall operating time in minutes (OR time) for the two surgeons, adjusting for transversus abdominis release (TAR).

Results: Figures 1 (surgeon 1) and 2 (surgeon 2) illustrate the three phases in the RA-CUSUM graphs of OR time. For surgeon 1, the cases for each phase were determined: phase 1 (1 to 12), phase 2 (13 to 24), and phase 3 (25 to 51). For surgeon 2, the cases for each phase were determined: phase 1 (1 to 8), phase 2 (9 to 32), and phase 3 (33 to 47). A significant (p = 0.017) difference existed for the OR times between phases 1 (262 ± 69) and 3 (192 ± 63.0) for surgeon 1. For surgeon 2, there were no significant differences between the OR times when comparing any of the phases. OR time in comparison to the risk-adjusted value began to decrease after case 12 for surgeon 1 and case 8 for surgeon 2.

Conclusions: The initial learning curve for surgeon 1 reached its plateau after 12 cases, shorter than comparable studies. This was likely due to the surgeon's intentional focus on learning this technique through courses, proctoring, and active mentorship. The flat learning curve seen in surgeon 2's initial series illustrates the value of experience gained during fellowship training. Our data supports that, given the right resources and support, a short learning curve for eTEP is attainable for a community-based surgeon without prior training in the technique.

figure cw
figure cx

S228

Not Deep Enough: Modeling the Effect of Shallow Placement of the DaVinci XI “bariatric” Long Troca on the mUscular Abdominal Wall

Fred N Qafiti, MD, MSBE; Jessica Buicko, MD; Florida Atlantic University

Introduction: The DaVinci Xi Robotic Surgical System (Xi) long cannula (Intuitive Surgical Company, Sunnyvale, CA) provides five additional centimeters of distal length compared to the standard Xi trocar. The extra length allows the cannula to traverse prohibitively thick body wall tissue. This is perceived as a benign adjunct to robotic surgery and is often necessary in robotic bariatrics. Our aims are to quantitatively model the limits of the long trocar in preserving a rotational centerpoint of motion (RCM) at the muscular abdominal wall. This essential tenet of robotic surgery is violated with shallow placement of the long trocar, which leads to unchecked, unnoticed blunt widening of port sites by the robotic arm, increasing hernia risk.

Methods: We begin with an exploration of the schematic of the Xi robotic arm as patented by Intuitive (U.S. Patent #5931832). We characterize the parallelogram-based arm joint motion that allows for the RCM. We trigonometrically model the resultant lateral displacement of the abdominal wall at the trocar site with respect to three measurable parameters: vertical trocar shallowness (distance from the true RCM to the abdominal wall), instrument tip depth and instrument tip lateral motion from the port's midline orientation. Reasonable ranges of surgical use are conservatively estimated within the confines of the Xi system's range of motion. MATLAB r2022a (Mathworks, Natick, MA) was used for modeling.

Results: The rigid parallelogram movement structure of the Xi preserves the RCM at the thick black marker printed on every Xi cannula. By limitation of design, both long and standard trocars must have this marker at the exact same distance from their proximal end. Proper long trocar use requires this marker to be placed at the muscular abdominal wall which is infrequently achievable in bariatric patients despite the distal canula entering the peritoneal cavity with ease. The value ranges of our model parameters (presuming a reasonable maximum orientation angle of 45° from midline) are: trocar shallowness [1 cm, 7 cm]; instrument tip depth [0 cm, 20 cm]; instrument tip lateral movement [0.0 cm, 14.1 cm]. Abdominal wall displacement increased proportionally as each instrument tip parameter reached its maximum deviation from the orthogonal midline as described in the plot figure. Maximal wall displacement at maximal shallowness was approximately 7.0 cm.

Conclusions: Robotic surgery revolutionizes modern operation, particularly within bariatrics. However, the current Xi arm design disallows a true bariatric trocar to be used safely without compromising the RCM, thereby risking hernia development.

figure cy
figure cz

S229

What is the Educational Value and Clinical Utility of Artificial Intelligence for Intraoperative and Postoperative Video Analysis? A Survey of Surgeons and Trainees

Mohamed Saif Hameed, MBBS, DNB 1; Simon Laplante, MD1; Caterina Masino, MA1; Muhammad U Khalid, BMSc2; Haochi Zhang, BASc3; Sergey Protserov, BSc3; Jaryd Hunter, BSc, BFA3; Pouria Mashouri, MSc3; Andras B Fecso, MD, PhD1; Michael Brudno, PhD3; Amin Madani, MD, PhD1; 1Surgical Artificial Intelligence Research Academy, University Health Network, Toronto, ON, Canada.; 2Temerty Faculty of Medicine, University of Toronto, ON, Canada; 3UHN Data Group, University Health Network, Toronto, ON, Canada

Introduction: Surgical complications often occur due to lapses in judgment and decision-making. Advances in artificial intelligence (AI) have made it possible to train algorithms that identify anatomy and interpret the surgical field. These algorithms can potentially be used for both intraoperative decision-support and postoperative video analysis and feedback. Despite the very early success of proof-of-concept algorithms, it remains unknown whether this innovation meets the needs of end-users or how best to deploy it. This study aims to assess the educational value, usability, and design preferences of adapting AI for surgical video analysis in both intraoperative and postoperative settings.

Methods and Procedures: A web-accessible, device-agnostic software was developed to provide AI inference both in real-time intraoperatively (synchronous mode), or by uploading surgical videos postoperatively for analysis (asynchronous mode). For this use-case, GoNoGoNet was used—an AI algorithm that identifies safe (Go) and dangerous (No-Go) zones of dissection during laparoscopic cholecystectomy (Fig. 1). Surgeons and surgical trainees were invited to participate and experience the AI algorithm in either setting, and complete a web-based 5-point Likert scale survey to evaluate the value, usability and design of the platform.

Fig. 1 GoNoGoNet during a laparoscopic cholecystectomy. Green = Go (safe) zone; Red = No-Go (dangerous) zone

figure da

Results: Nineteen participants (11 surgeons and 8 trainees) evaluated GoNoGoNet (intraoperatively, n = 9; postoperatively, n = 9; both, n = 1). Most either agreed or strongly agreed that the AI tool is an effective adjunct to surgical training (80%; neutral = 10%), effective for providing either real-time feedback (65%; neutral = 20%) or postoperative feedback (70%; neutral = 25%), and capable of improving surgeon confidence (65%; neutral = 30%). Only 35% (neutral = 55%) and 55% (neutral = 45%) believe that the tool is effective in improving intraoperative decisions and performance, or beneficial for patient care, respectively. Overall, 40% (neutral = 40%) reported they would use this platform consistently if it were available. Most either agreed or strongly agreed that the platform was easy to use (80%; neutral = 15%) and has acceptable resolution (65%; neutral = 25%), while 20% (neutral = 35%) reported that it disrupted the OR workflow, and 15% (neutral = 20%) reported significant time lag. All respondents reported that such a system should be available "on-demand" to turn on/off at their discretion.

Conclusions: Most end-users found AI to be a useful adjunct for surgical training and effective for providing support and feedback to surgeons both during and after surgery. The usability findings from this study should be used to inform future design implementation into surgical workflows and educational curricula.

S230

Design and Evaluation for a Soft Intro-abdominal Wireless Laparoscope

Hui Liu1; Berat Aktar1; Ning Li2; Gregory Mancini1; Jindong Tan 1; 1University of Tennessee; 2Azure medical innovation

Introduction: In single-incision laparoscopic surgery (SILS), magnetic anchoring and guidance system (MAGS) is promising to prevent clutter in the surgical workspace and provide a larger vision field. Existing camera designs rely on rigid structure and sliding motion, which may cause stress concentration and tissue damage on the curved abdominal wall. In this work, we proposed a wireless MAGS consisting of soft material and wheel structure design. The camera can passively bend and adapt to the curved tissue surface to relieve stress concentration. The wheel structure transfers sliding motion to rolling motion when the camera tilts and translates. The evaluation shows the new design provides safer interaction.

Methods and Procedures: In Fig. 1, the camera is anchored and steered by a manipulator through magnetic coupling. The actuator and camera contain two magnet pairs and the exploded views are shown in Fig. 2. The camera is made of silicon rubber. Attracted by magnetic force, it passively bends along the curved abdominal wall. The magnetic actuation and wheeled mechanism are shown in Fig. 3. Two internal permanent magnets (IPMs) locate on each side of the camera. Supported by two bearings, the PCB board and IPMs can rotate relative to the shell. Two external permanent magnets (EPMs) in the actuator are aligned with IPMs and driven by a micromotor.

figure db
figure dc
figure dd

Results and Discussion: The simulation results of the stress distribution on curved tissue are shown in Fig. 4. The maximum stress drops from 0.03Mpa to 0.009Mpa by applying soft material. In Fig. 5, our camera has 80 mm in length and 17 mm in diameter. The soft tubes and inner soft shaft are made with Dragon skin 20 and Smooth-Sil 945 respectively. The camera board is built around a Texas Instruments cc2541 MCU. Analog video and sensing data are sent out through wireless communication. The motor (1016M009SR, FAULHABER) inside the stator can generate 1Nm torque. The locomotion ability is tested with a URe5 and an abdomen simulator.

figure de
figure df

Conclusions: This work proposed the design and evaluation of a soft wireless camera system in SILS. The camera is driven by pure magnetic coupling. Composed of soft material, the camera can passively adapt to the curved abdominal wall. The simulation and experiments showed the stress concentration on tissue is relieved apparently. The wheeled design allows the camera to roll along the tissue instead of sliding, which provides a safer interaction. In the future, closed-loop control will be further applied to realize precise and stable control.

S231

Feasibility of On-demand Robotics in Colorectal Surgery. First Cases

Dieter Hahnloser, Prof, Dr, Med; Fabian Grass, PD, Dr, Med; University Hospital Lausanne

Introduction: The key benefits of robotics are improved precision and control, thanks to fully articulated robotic instruments and enhanced, stable endoscope control. However, colorectal procedures also require large movements such as medialization of the colon where a robotic platform is not always needed. We present the world's first experience in colorectal surgery with a new open platform of on-demand robotics.

Methods and Procedures: Standard laparoscopic 3-D camera, insufflator, trocars and energy devices, available in all hospitals performing laparoscopic surgery, are used in combination with the Dexter System from Distalmotion SA, which includes two robotic instrument arms, one robotic endoscope arm and a sterile surgeon console. We present the first 8 colorectal cases of robotic assisted ventral mesh rectopexy (n = 2), oncologic right colectomies (n = 5) and transverse colectomy (n = 1) using the Dexter System.

Results: All surgeries could be conducted as planned using 4 standard laparoscopy 10/12 mm trocars (3 robotic and one assistant).The two ventral mesh rectopexies were fully robotic, requiring no switch form standard laparoscopy to robotic assistance. Patients were discharged at day 2, with no morbidity at 90 days. The robotic platform was used for central vascular ligation (CVL) in all 6 oncologic colectomies, whereas medialization of the colon and transection was performed with standard laparoscopy. Intracorporal anastomosis was performed in the last 2/6 cases (stapling by standard laparoscopy and suturing of the defect with robotic assistance). Patients were discharged at a median of 6 days (4-9), with two postoperative ileus (treated conservatively), no anastomotic leakage nor other morbidity. Median procedure time was 158 min (range [121 – 212] minutes) for the entire operation, and the switch between laparoscopy and robotics was performed in a median of 25 s.

Conclusions: On-demand robotics is feasible and combines the best of two worlds: Robotics where precision and enhanced dexterity are required and standard laparoscopy where it is at its best. The surgeon remains scrubbed-in at all times, allowing a switch between robotics and laparoscopy within seconds.

S232

Laparoscopic Gastrectomy 3D versus 2D in Distal Gastric Cancer: A Systematic Review and Meta-Analysis

Amanda Cyntia Lima Fonseca Rodrigues; Fatemeh Shojaeian, MD, MPH; Theethawat Thanawiboonchai, MD; Jonathan Greer, MD; Gina Adrales, MD, MPH; The Johns Hopkins Hospitals, The Johns Hopkins University School of Medicine

Introduction: Technological advancements in resolution and stereoscopic depth, three-dimensional (3D) laparoscopy has been widely used. This meta-analysis aims to address the efficacy and effects of the applicability of 3D gastrectomy in patients with distal gastric cancer (DGC) compared to two-dimensional (2D) laparoscopy.

Methods and Procedures: A systematic review and meta-analysis was conducted on laparoscopic gastrectomy 3D compared with 2D in distal gastric cancer. MEDLINE, EMBASE, and Cochrane databases were screened until September, 2022 using the keywords "3D laparoscopic", "2D laparoscopic" and "distal gastric cancer". Out of 88 results, after deduplication, 14 studies were selected for full review, and 3 articles were considered in the final investigation. The total operating time was assessed as the primary endpoint and postoperative complications were the secondary endpoints.

Results: Three prospective studies with a total of 382 patients were included, 187 underwent 3D laparoscopy and 195 underwent 2D laparoscopy. Kang et al. obtained significantly shorter operating time in the 3D group (115.6 + 34.0 min) compared to the 2D group (129.4 + 38.5 min), p = 0.012. Postoperative complications in 2D were 13 (14.4%) and in 3D were 5 (5.6%), p = 0.828. Li et al. had a mean operating time for the 3D laparoscopic group of 181.03-36.76 min, and for the 2D laparoscopic group it was 191.47 + 47.19 min, p = 0.171 and postoperative complications were 1/60 (3D) and 4/64 (2D), p = 0.195. Lee et al. showed that the total operative time in 3D groups was significantly shorter than the 2D group (122 [106.5-161] versus 101 [77.75-125.5] minutes, p = 0.001 and the rates of postoperative complications in the 3D groups were significantly smaller than the 2D (24.4% versus 7.9%, respectively, p = 0.045.

Conclusion: The studies have brought encouraging and safe results in the use of the 3D technique compared to 2D, with a positive impact on the reduction of surgery time and postoperative complications. However, we strongly recommend further long-term studies, especially RCTs, to best establish faster operation time and postoperative complications between 3D versus 2D laparoscopic techniques in DGC.

S233

Automated Identification of the Critical View in Hiatal Hernia Repair Using Deep Learning

Alexis Desir, MD1; Shruti Hegde, MD 1; Babak Namazi, PhD1; Marc Ward, MD2; Steven G Leeds, MD2; Carla Holcomb, MD1; Daniel Scott, MD1; Ganesh Sankaranarayanan, PhD1; 1University of Texas Southwestern Medical Center; 2Baylor University Medical Center

figure dg

Introduction: Hiatal hernia repair is a complex procedure and inadequate technique can lead to high recurrence rates. We have previously proposed the establishment of a critical view for assessment of adequate intra-abdominal esophageal length with a non-obstructed view of the retroesophageal window. The goal of this work is to develop an artificial intelligence model to automatically identify this critical view from expert hiatal hernia repair videos.

Methods: Laparoscopic and robotic hiatal hernia repair videos from UT Southwestern and Baylor University Medical Center were collected and 83 intraoperative images were extracted after complete esophageal mobilization and prior to the crural repair. The images were annotated by two surgeons for satisfying the following two critical view conditions: adequate esophageal length of at least 3 cm assessed visually (C1) and a clear visualization of the confluence/base of crura (C2). Cohen's kappa was used to assess the quality of the annotations. To enable real-time multi-label classification, we used MobileNetV2, a convolutional neural network model, which was preloaded with weights trained on more than 14 million images from Imagenet and then trained using transfer learning. Keras k-fold cross-validation and random image augmentation were applied to maximize the possible learning on this small data set. Due to class imbalance and multi label classification, categorical accuracy and F2 macro scores were chosen as metrics to assess the performance of the model.

Results: Of 83 images, 51 (61%) satisfied both conditions and 5 (6%) didn't satisfy either of them. C1 was present more than C2 (84% vs 71%). Cohen's kappa for C1 and C2 was 0.73 and 0.832 respectively, indicating good agreement. The neural network detected "C1 but not C2", "C2 but not C1", "C1 and C2", and "not C1 or C2" with 92% categorical accuracy and 84% F2 metric. Figure 1 shows correctly predicted results for all categories.

Conclusion: We developed a multi-label transfer learning based convolutional neural network able to detect and classify any combination of these two criteria of the critical view in hiatal hernia repair videos simultaneously. Future work to collect and annotate additional videos will improve the performance and robustness of our model.

S234

Laparoscopic Surgeons Transitioning to Robotic-assisted Inguinal Hernia Repair: A Prospective Analysis of Efficiency

William Hope, MD, FACS 1; James Bittner, MD, FACS2; Rana Pullatt, MD, FACS, FASMBS, DABOM3; Jose Erbella, MD, FACS4; William Newcomb, MD5; Steven Thies, MD, FACS6; Juan-Carlos Verdeja, MD, FACS7; 1New Hanover Regional Medical Center; 2University of Connecticut School of Medicine; 3Medical University of South Carolina; 4Manatee Memorial Hospital; 5Riverside Hospital; 6CaroMont Regional Medical Center; 7Baptist Health South Florida

Background and Purpose: Literature reports of surgeon efficiency when transitioning from laparoscopic to robotic-assisted (RA) inguinal hernia repair (IHR) are retrospective or describe single-surgeon experience. The purpose of this study is to provide a prospective, multi-surgeon assessment of surgeon efficiency when transitioning to RA-IHR.

Methods: Prospective, post-market, observational pilot study of procedure (skin to skin) time as a measurement of efficiency, time to establish critical view of the myopectineal orifice (MPO), and rate of intraoperative adverse events (AEs) for experienced laparoscopic general surgeons (≥ 300 laparoscopic IHRs) performing their first 75 RA-IHRs (no prior RA surgery experience or no RA cases one year prior to study). Mean outcomes with 95% confidence intervals (95% CI) describe surgeons' collective bilateral and collective unilateral early, mid-, and late-phase cases; each surgeon contributed 25 consecutive cases with each phase.

Results: Four surgeons (four medical centers) consented to enroll in the study. For bilateral repairs (n = 87; early phase (n = 27), mid-phase (n = 35), late-phase (n = 25)), mean skin-to-skin times at each phase were, respectively: 112.4 min [95% CI: 97.2, 127.5], 92.8 min [95% CI: 80.6, 105.0], and 80.6 min [95% CI: 68.9, 92.3]. Procedure time efficiency improved during late-phase procedures. Efficiency in establishment of the MPO critical view for bilateral cases improved during the mid-phase procedures: early (50.5 min [95% CI: 41.2, 59.9]); mid-phase (36.7 min [95% CI: 32.5, 40.9]), and late-phase (33.0 min [95% CI: 27.9, 38.1]). For unilateral repairs (n = 212; early (n = 72), mid-phase (n = 65), late-phase (n = 75)), overall procedure efficiency improved during mid-phase procedures. Mean skin-to-skin times at each phase were, respectively: 78.5 min [95% CI: 71.7, 85.3], 64.5 min [95% CI: 57.8, 71.1], and 67.7 [95% CI: 60.4, 75.0]. For unilateral cases, mean phase-to-phase dissection time to MPO did not differ: 19.9 min [95% CI: 17.0, 22.7], 16.3 min [95% CI: 13.6, 19.0], and 16.0 min [95% CI: 13.5, 18.5]. One (1.15%) intraoperative AE was reported in the bilateral group: a trocar injury to the bowel was treated with intraoperative stapling; the repair was completed without conversion. No AEs were reported for unilateral cases. One patient (0.5%) in the unilateral group had severe scarring medially from a previous prostatectomy; the procedure was converted to open.

Conclusions: Experienced laparoscopic surgeons improved efficiency in bilateral RA-IHR between 51–75 cases and in unilateral RA-IHR between 26–50 cases. Time to MPO does not appear to effect differences across phases in skin-to-skin times for unilateral procedures.

S235

Stray Energy Injury During Robotic Versus Laparoscopic Inguinal Hernia Repair: A Randomized Controlled Trial

Krzysztof J Wikiel, MD; Paul M Montero, MD; Teresa S Jones, MD; Thomas N Robinson, MD; Edward L Jones, MD; University of Colorado/RMR Denver VA

Background: Stray energy transfer from monopolar instruments during laparoscopic surgery is a recognized cause of potentially catastrophic complications. Thermal injury to port sites during laparoscopic cholecystectomy has been documented by our group. There are limited data on stray energy injuries in robotic surgery. We sought to characterize stray energy injury in the form of superficial burns to the skin surrounding laparoscopic and robotic trocars. Our hypothesis was that the camera port site would demonstrate the most stray energy injury, and that there would be no difference between robotic or laparoscopic technique.

Methods: We conducted a prospective, randomized controlled trial of patients undergoing elective unilateral inguinal hernia repair at a Veterans Affairs Medical Center. Surgery was performed via transabdominal preperitoneal approach either laparoscopic-assisted (TAPP) or robotic-assisted (rTAPP). A monopolar scissor was used to deliver energy at 30W coagulation mode (ForceTriad, Covidien, Boulder, CO) for all cases. At completion of the procedure, skin biopsies were taken from the camera port, assistant grasper port, and the energized working instrument port. A picrosirius red stain was utilized to identify thermal injury by a blinded pathologist.

Results: Over half (54%, 59/109) of all samples demonstrated thermal injury to the skin at one of the port sites. In the TAPP group 49% (25/51) of samples showed thermal injury vs. 58% (34/58) in the rTAPP group (p = 0.548). The camera port was the most frequently involved with 68% (13/19) of rTAPP samples showing injury vs. 47% (8/17) in the TAPP group (p = 0.503). There was no difference in the rate of injury at the working port site (rTAPP 58%, 10/19 vs. TAPP 47%, 8/17; p = 0.991) or the assistant port site (rTAPP 58%, 11/19 vs. TAPP 53%, 10/19; p = 0.873).

Conclusions: Stray energy causes thermal injury to the skin at port sites in 54% of robotic and laparoscopic inguinal hernia repairs. There is no difference in stray energy transfer between the laparoscopic or robotic platform. This is the first study to confirm in vivo transfer of stray energy during robotic surgical procedures and more study is needed to determine the clinical significance of these thermal injuries.

S236

Clinical Outcomes of Elective Robotic vs Laparoscopic Surgery for Colon Cancer Utilizing a Large National Database

Vanita Ahuja, MD, MPH, MBA1,2; Lucero G Paredes, MD 1,3; Melissa F Perkal, MD1,2; Joseph T King Jr, MD2,4; 1Department of Surgery, Yale University School of Medicine, New Haven, Connecticut; 2VA Connecticut Healthcare System, US Department of Veterans Affairs, West Haven, Connecticut; 3National Clinician Scholars Program, Yale School of Medicine, New Haven, Connecticut; 4Department of Neurosurgery, Yale School of Medicine, New Haven, Connecticut

Introduction: Robotic surgery for colon cancer resection has become increasingly common. Prior studies have shown comparable outcomes across laparoscopic and robotic approaches; however, these have been mostly limited to single-year or single-center cohorts. This study investigates robotic vs laparoscopic colectomy, focusing on differences in clinical outcomes across several years.

Methods: We analyzed data from ACS NSQIP for patients who underwent elective robotic or laparoscopic colon cancer surgery from 2012-2020. Inverse probability weighting with regression adjustment (IPWRA) incorporating demographics, year, operative factors, BMI, smoking, steroid use, and comorbidities was used to adjust for differences in patients in order to examine the relationship between surgical approach and 30-day mortality. Secondary outcomes included complications, return to the operating room (OR), post-operative length of stay (LOS), and operative time.

Results: From 2012-2020, 83,941 patients underwent elective robotic or laparoscopic colectomy procedures for colon cancer. The mean age was 65 years old, 48% female, 65% White, 55% ASA3, and 30% BMI ≥ 30. Robotic approach steadily increased from 0.02% in 2012 to 19.1% in 2020 (Fig. 1). Patients who underwent a robotic procedure were younger, more likely to be male and non-Hispanic white, with higher BMI, and fewer medical comorbidities. Overall in the robotic surgery group, there were 772 (0.92%) deaths 14,159 (16.9%) patients with complications, and 3,219 (3.8%) patients who returned to OR. Mean (SD) LOS was 5.1 (4.4) days and mean (SD) operative time was 194 (96) minutes. After adjustment, there were no differences between robotic and laparoscopic surgery for 30-day mortality (0.80% vs 0.94% respectively, P = 0.236) or complications (16.4% vs 16.9% respectively, P = 0.318). Return to OR rates were higher in robotic surgery patients (5.1% vs 3.6%, P < 0.001). LOS was slightly lower in robotic surgery patients (5.1 vs 5.4 days, P < 0.001), and operative time was longer (246 min vs 184 min, P < 0.001).

Conclusions: Robotic approach for elective colon cancer resection has similar outcomes to its laparoscopic counterpart. There were no differences in peri-operative mortality or rate of complications in comparison to laparoscopic approach, with robotic cases associated with increased rate of return to OR, however shorter length of stay overall. Further investigation is imperative to better understand the potential impact of technological advancement such as robotic surgery on patient outcomes.

figure dh

S237

Clinical Outcomes and Kinematic Data in Robotic Cholecystectomies

Daniel Bitner 1; Sarah Choksi, MD1; Katherine Carsky, MD1; Poppy Addison, MD1; Robert Andrews, MD1; Rebecca Kowalski1; Edward Yatco1; Rachel Webman, MD1; Shane Dawson1; Valery Dronsky1; Andrew Yee, PhD2; Anthony Jarc, PhD2; Filippo Filicori, MD2; 1Lenox Hill Hospital; 2Intuitive Surgical, Inc

Background: Recent reports that link surgical skill with improved clinical outcomes can be extended using quantitative analysis of intraoperative technique given the growing access to surgical video. Beyond manual, subjective analysis of the video, robotic procedures can be further broken down into Objective Performance Indicators (OPIs) in which kinematics (e.g., how and where the instrument moves) and events data (e.g., how often energy is activated and attention distracted from the robotic console) are recorded. One device used for this type of measurement is the Intuitive Data Recorder (IDR, Sunnyvale, CA). Herein we explore the relationship between intraoperative OPIs and postoperative recovery, hypothesizing that more efficient dissection intraoperatively would be associated with higher postoperative well-being scores.

Methods: 13 robotic cholecystectomies between March and August 2022 were routinely recorded with the IDR at a large tertiary referral center. OPIs output was made available after video processing and then linked to clinical cases for chart review. Two phases of robotic cholecystectomies were reviewed: dissection of Calot's triangle and dissection of the gallbladder from the liver. Postoperative recovery was measured with the SF-36 well-being survey at POD5-7. Univariate analysis was conducted for preliminary review using linear regression. A p < 0.05 was considered significant.

Results: In our preliminary analysis, certain OPIs were associated with early postoperative well-being. During the phase Dissection of Calot's Triangle, total volume of movement was inversely related to SF-36 scores (R -0.749, R2 0.561, p 0.013). During the phase Dissection of Gallbladder from the liver, the rate of camera movement was positively associated with SF-36 scores (R 0.683, R2 0.467, p 0.029). Instrument roll velocity of the console arm was inversely associated with postoperative SF-36 scores (R -0.973, R2 0.947, p 0.005).

Conclusions: In our preliminary analysis, and for the first time, several OPIs have been correlated with early postoperative well-being scores. The relationship of camera movement during the dissection of the gallbladder off the liver and improved postoperative recovery may fit with prior studies in which camera movements have correlated with increased operative experience. Further investigation across additional OPIs and additional tasks may build on these results to explore how more efficient intraoperative technique might allow for improved postoperative recovery.

S238

Subtotal Cholecystectomy Versus Total Cholecystectomy in Complicated Cholecystitis

Yuriy Grubnik, Professor; Volodymyr Grubnik, MDViktor Grubnyk, PhD; Vadym Ilyashenko, PhD; Odessa National Medical University

Introduction: In severe cholecystitis laparoscopic cholecystectomy (LC) can be technically difficult with a high risk of duct and arterial injury. To prevent injury, conversion to open cholecystectomy (OC) is usually made. Another solution is performing of laparoscopic subtotal cholecystectomy (LSTC). Aim of the study was to study the safety and complications of laparoscopic subtotal cholecystectomy (LSTC) compared to conversion to OC for technically difficult cholecystitis.

Methods: A retrospective review of 8764 LC performed from 2005 to 2020 in single center was done. In the 1st period, from 2005 to 2008, 2720 LC were performed. In technical difficulties conversion to OC was done. In the 2nd period, from 2009 to 2018, 4468 LC were performed. In technical difficulties LSTC was done.

Results: During the 1st period, there were 127 patients (4.7%) with technically difficult LC whom conversion to OC was done. Complication rate among these patients was 23%, bile duct injuries were detected in 3 patients (2.4%), mortality was 1.6%. During the 2nd period, there were 134 patients with technically difficult LC, thus LSTC was performed for 108 patients of them. Therefore, conversion to OC was made for the remaining 22 patients (0.6%). There were no bile duct injuries and no mortality in these patients. Complication rate after LSTC was 8.5%. Quality of life was assessed 6–48 months after surgery in 109 patients whom conversion to OC was done, and in 92 patients whom LSTC was done. Quality of life was better in patients after LSTC.

Conclusion: LSTC is good alternative to conversion to OC in complicated cases. It is good solution to prevent bile duct injury and save a principle of minimally invasive procedure in technically difficult LC, especially in high-risk patients.

S239

Percutaneous Transhepatic Choledochoscopy for Mirizzi Syndrome: A Minimally Invasive Option for a Complex Surgical Problem?

Camilo Barragan, MD, MSc; Sebastian Mafeld; Eran Shlomovitz, MD; UHN—Toronto General Hospital

Purpose: To describe our experience with a Percutaneous transhepatic choledochoscopy (PTCC) as a minimally invasive, treatment option for patients with Mirizzi Syndrome (MS).

Material and methods: Institutional ethics review board approved a retrospective cohort study of patients with MS treated during the years 2013-2022 in a single center. A total of 62 patients were found diagnosed with this condition. We selected those who had percutaneous management of this pathology and data was collected including demographics, preoperative diagnosis, intraoperative findings, procedural time, type of intervention, type of anesthesia provided, duration of the treatment, and postoperative data as well as post-procedure outcomes. 11 patients were managed with the percutaneous approach. Data was collected from Electronic Patient Records (EPR) and EPIC system with the diagnosis of Mirizzi syndrome. Of those, 11 patients were managed percutaneously.

Results: 25 stone removal procedures in total were performed in the 11 patients. 2 of them had total stone removal. 3 of them remained with large stones after percutaneous treatment and 6 patients had small stone fragments after the attempt to remove them. All of them had subsequent procedures (percutaneous lithotripsy, percutaneous retrieval) with clearance of stones after a few more sessions. The average number of procedures needed to complete stone removal was 2.2 (range 1-4). Two patients had self-limiting complications after treatment; one of them developed a liver abscess after the 3rd session of PTCC managed percutaneously with complete recovery. No mortality at 30 days related to the procedure occurred.

Conclusion: Percutaneous cholangioscopy provides a safe and feasible management option for patients with Mirizzi Syndrome and is of great value in those patients that are not candidates for surgery or endoscopic treatment, also for patients that would require complex reconstruction with good technique and clinical outcomes.

S240

The Impact of Interval Cholecystectomy Timing after Percutaneous Cholecystostomy on Post-Operative Adverse Outcomes

Mohammad Noubani, MD 1; Elizabeth McCarthy, MD2; Samuel Stanley2; Jie Yang, PhD2; Konstatinos Spaniolas, MD2; Aurora Pryor, MD2; 1University of North Carolina Health System; 2Stony Brook University School of Medicine

Objective: This study aims to explore the impact of timing between percutaneous cholecystostomy tube (PCT) placement and interval cholecystectomy (IC) on postoperative outcomes.

Methods: A retrospective database analysis of patients with a history of PCT followed by IC was conducted using the New York State (NYS) SPARCs database between 2005 and 2015. Patients with IC timing ranged between > 1 week and < 2 years. These patients were further divided into quartiles using 4-time intervals; including1-5 weeks (Q1), 5-8 weeks (Q2), 8-12 weeks(Q3), and > 12 weeks(Q4). The study's primary outcome was hospital length of stay (LOS). Secondary outcomes included discharge to home, 30-day readmission, 30-day ED visit, and 90-day reoperation. Multivariable regression models were used to compare patients across the four time intervals after adjusting for possible confounding factors.

Results: A total of 1,038 patients with a history of PCT followed by IC between > 1 week and < 2 years were included in the final analysis. The median time to IC was 7.7 weeks. Q2 and Q3 both have a median LOS of 3 days, while Q1 and Q4 had a significantly longer LOS with both having a median LOS of 5 days (p < 0.0001). Patients from racial and ethnic minorities (e.g., African American and Hispanics) were more likely to get their IC at a later time interval, particularly > 12 weeks (p < 0.05). Further, Black, non-Hispanic patients had a significantly higher median LOS than White, non-Hispanic patients (6 days vs 4 days, p < 0.0001). Upon multivariable regression modeling, patients had significantly shorter LOS if they had their IC in Q2 (Ratio, 0.76, 95%, 0.67-0.87, p < 0.0001), and Q3 (Ratio 0.75, 95% CI, 065-0.86, p < 0.0001) compared to those who got their IC in Q4. Similar findings exist when comparing patients getting their IC during Q2 and Q3 to those receiving treatment during Q1. No differences in LOS between patients with IC between Q1 and Q4 were found. No significant differences were observed for discharge status, 30-day readmission, 30-day ED visits, and 90-day reoperation across the 4 different time intervals.

Conclusion: Patients undergoing IC after receiving a PCT have a decreased LOS between 5-12 weeks. Patients from racial minorities are more likely to have increased LOS and IC at later time intervals.

figure di

S241

A Study to Develop an Objective Scoring System to Grade the Difficulty of Laparoscopic Cholecystectomy

Virinder Bansal, Prof; Kruthika S; Krishna Asuri, Prof; Omprakash Prajapati; Sanjeet Rai; Rashmi Ramachandran; AIIMS, New Delhi

Aims and Objectives: To develop and validate a scoring system to grade the difficulty of laparoscopic cholecystectomy using both preoperative and intraoperative parameters.

Materials and Methods: This study was conducted in 2 phases. Phase 1 was a retrospective study that included all the patients who underwent LC between January 2017 and December 2019. Intraoperative findings of these cases were assessed by the operating surgeon retrospectively and intraoperative difficulty of each surgery was graded according to modified Nassar's subjective scoring system. All the preoperative and intra operative variables were compared against the given Nassar's score and significant variables contributing to the difficulty, after multivariate regression analysis were noted. These parameters were given a weightage score using Schneeweiss scoring system. Phase 2 was a prospective study that included patients who underwent LC between July 2020 and April 2021. Both the pre-operative and intraoperative scores were validated against modified Nassar's score, on a prospective cohort. ROC curve was applied and cut offs were calculated to categorize the level of difficulty into easy, moderately difficult and difficult LCs using Kruskal Wallis test and Spearman's correlation.

Results: On final multivariate analysis preoperative factors which were found to be significant predictor's of difficulty included ASA grade 3, history of acute cholecystitis, history of pancreatitis, drainage procedure done for post pancreatitis, ERCP done prior to surgery, and gallbladder wall thickness > 4 mm on imaging. Intraoperative parameter's included use of Palmer's point for pneumoperitoneum, use of additional ports, presence of visceral adhesions, requirement of decompression of gallbladder, difficult Calot's triangle, impacted stone, bile/stone spillage/ drain placement/ port site enlargement and presence of cholecysto-enteric fistula (intra operative). The final score range for preoperative parameters was 0-7 and for intra operative parameters was 0-15.

In phase 2 after validation against modified Nassar's score, an intraoperative score of > 8 was equivalent to difficult, < 2 was easy, 2-8 was moderately difficult laparoscopic cholecystectomy. The diagnostic accuracy of intraoperative score was found to be statistically superior to preoperative score (p value < 0.05).

Conclusion: The scoring system developed and validated in this study has shown an excellent correlation with difficulty of cholecystectomy. This scoring system which incorporates both preoperative and intraoperative parameter's will not only help in grading the difficulty of laparoscopic cholecystectomy objectively but also provide a standard reporting scale for reporting of laparoscopic cholecystectomy.

S242

Can Artificial Intelligence Provide Decision-Support to Avoid High-Risk Behaviors During Laparoscopic Cholecystectomy? A Postoperative Video Analysis

Muhammad U Khalid, BMSc 1; Simon Laplante, MD, FRCSC1; Adnan Alseidi, MD, EdM, FACS2; Shiva Jayaraman, MD, MESc, FRCSC3; Amin Madani, MD, PhD, FRCSC1; 1Surgical Artificial Intelligence Research Academy, University Health Network, Toronto, Ontario, Canada; 2Department of Surgery, University of California, San Francisco, California, USA; 3Department of Surgery, St Joseph's Health Centre, Toronto, Ontario, Canada

Introduction: Major bile duct injuries (BDIs) are a significant source of morbidity amongst patients undergoing laparoscopic cholecystectomy (LC). Recent advancements in artificial intelligence (AI) have the potential to prevent these devastating injuries by providing real-time intraoperative decision-support. GoNoGoNet is an AI algorithm that has been developed and validated to identify safe ("Go") and dangerous ("No-Go") zones of dissection during LC. Yet, it is unclear how this algorithm would perform during LCs where a BDI occurred. This study aims to evaluate GoNoGoNet's ability to predict Go/No-Go zones during LCs with BDIs, and assess how these predictions relate to the injury.

Methods and Procedures: LC videos with known BDIs were analyzed using GoNoGoNet. For each video, reference frames were selected from each minute prior to the injury. To minimize algorithmic bias, frames were selected if they included an adequately retracted view of the inferior edge of the gallbladder, hepatocystic triangle, and liver. All tool-tissue interactions, including the final act that caused the BDI, were then characterized in relation to the Go/No-Go zones of the reference frame within the same minute of dissection. Summary statistics are displayed as (%) and median [interquartile range].

Results: Fifteen LC videos with known BDIs were obtained, of which 4 were excluded due to poor recording quality. Time from the beginning of hepatocystic triangle dissection to the point of injury was at a median of 1:35 [0:39-14:55] minutes. Videos had Parkland severity scores of 2 (9%), 3 (36%), 4 (45%), and 5 (9%), and included Strasberg C (9%), D (36%), E1 (36%), and E2 (18%) type injuries. Interactions were located in the Go, No-Go, and in-between zones for 65%, 28%, and 7% of the time respectively for sharp dissections, and 44%, 50%, and 7% of the time respectively for blunt dissections. For the interaction that caused the BDI, 4 (36%) were in the No-Go zone, 1 (9%) was in-between, and 6 (55%) were in the Go zone. Qualitative assessment of incorrect GoNoGoNet predictions suggested that at least 2 (33%) were biased by surgical instruments.

Conclusion: AI has the potential to identify dissection in planes that may lead to BDIs during LC. Nevertheless, lack of such detection does not necessarily imply that subsequent dissection will be safe. Future work should focus on designing AI for the operating room to support surgeons by serving as a second opinion to aid in avoidance of high risk behaviors.

figure dj

S243

Pharmaceutical Utilization as Measure of Comorbidity Resolution After Bariatric Surgery: A Population Based Cohort Study

Ted Wu, MD, MSc, FRCSC; Wenjing He, MD, PhD; Krista Hardy, MD, MSc, FRCSC, FACS; Ashley Vergis, MD, MMEd, FRCSC, FACS; University of Manitoba

Introduction: Obesity is a chronic and progressive disease associated with significant morbidity, mortality, and health care costs. Bariatric surgery is the most effective intervention for sustainable weight loss and resolution of obesity-related comorbidities. Studies looking at comorbidity resolution largely rely on individual self-reported outcomes and review of electronic records. We present a population-based study using administrative data of prescription medication dispensed before and after bariatric surgery as a measure of comorbidity resolution.

Methods and Procedures: All patients enrolled in the Center for Metabolic and Bariatric Surgery who underwent either gastric bypass or sleeve gastrectomy between 2012-2017 in Manitoba were included. Demographic information, follow up and prescription utilization data were collected through the Manitoba Center for Health Policy for five years pre and post surgery.

Results: A total of 1184 patients were included.

figure dk

Conclusion: This is the first Canadian study utilizing a provincial wide prescription database to measure long term comorbidity resolution after bariatric surgery. The use of administrative data eliminates potential biases and inaccuracies in follow up and self-reported outcomes. Consistent with the literature, prescriptions for the treatment of metabolic syndrome were all significantly decreased and sustained at both 3 and 5 years, while neuropsychiatric and thyroid medications expectedly remained stable. Increase in post-operative PPI use is consistent with routine administration postoperatively to prevent marginal ulceration. Postoperative opioid use is variably reported in the literature, while we demonstrate a transient increase in opioid dispensation, this may not reflect true utilization by the patient. Further studies are needed to further delineate the effects of altered pharmaceutical utilization on patient quality of life and health care expenditures.

S244

Using the Distressed Communities Index to Predict Long-term Weight Recurrence After Bariatric Surgery

Alexandra Johns, MD, MPH; M. Siobhan Luce, MD, MPH; Victoria Lyo, MD, MTM; Mohammed Ali, MD; Shushmita Ahmed, MD; University of California, Davis

Background: The role of socioeconomic status (SES) on long-term postoperative weight recurrence remains unclear. Distressed Community Index (DCI) is an SES metric which uses indicators such as education, housing status, employment status, and poverty rates to create a composite score of community economic well-being. This study aims to evaluate the effect of DCI on post-bariatric weight recurrence.

Methods: Retrospective analysis of patients undergoing primary laparoscopic Roux-en-Y gastric bypass or sleeve gastrectomy was performed. Preoperative characteristics and postoperative weights were documented. Last recorded postoperative weight (LRPW) included last weight recorded in the electronic medical record, not limited to bariatric visits. Patients were stratified into low-tier (LT) and high-tier (HT) DCI groups based on zip code.

Results: Between 2015 and 2020, 431 patients underwent surgery and had LRPW > 3 years postop; 238 had LRPW > 5 years. Average follow-up in bariatric clinic was 1.78 ± 1.6 years. Average time to LRPW was 4.17 ± 1.95 years. The rate of bariatric clinic follow-up within the last year was similar between groups (13.9% LT-DCI vs 16.2% HT-DCI, p = 0.48). Percent excess weight loss (%EWL) between groups was similar at 1 year (61.2% LT-DCI vs 56.9% HT-DCI, p = 0.10) and 3-5 years postop (42.7% LT-DCI vs 40.5% HT-DCI, p = 0.66). LT-DCI patients had significantly greater weight loss at > 5 years postop on univariate analysis (45.1% LT-DCI vs 34.5% HT-DCI, p = 0.02); however, this was not significant on multivariate regression (controlling for factors such as age, preoperative body mass index (BMI), ASA status, and insurance type). There was no differcentage of patients who gained > 15% EWL between one and 3-5 years postop (48.1% LT-DCI vs 43.2% HT-DCI, p = 0.73) or between one and > 5 years postop (44.0% LT-DCI vs 45.0% HT-DCI, p = 0.93). Average %EWL increase for all patients was 16.4% between 1 and 3-5 years and 18% between 1 and > 5 years. The percentage of patients with Rates were similar for percentage of patients with BMI > 35 at 3-5 years (45.8% LT-DCI vs 53.5% HT-DCI, p = 0.36) and > 5 years (48.6% LT-DCI vs 54% HT-DCI, p = 0.59).

Conclusions: Our study showed similar weight loss and recurrence between DCI groups, despite the assumption that lowered tiered communities have fewer resources. These results are compelling and challenge prior studies showing SES effects weight recurrence. Therefore, further evaluation is needed in the role of composite SES scores such as DCI on long term weight loss and weight recurrence.

S245

High Social Vulnerability Index is an Independent Predictor for Subsequent Conversion to Gastric Bypass Within 5 Years: A Review of Patients Undergoing Sleeve Gastrectomy at a Bariatric Center of Excellence

Santosh Swaminathan, MD; Nicholas Druar, MD, MPH; Priscilla Lam, MD; Shohan Shetty, MD, FACS; Saint Mary's Hospital, Waterbury,CT

Introduction: Bariatric surgery is the only therapeutic modality proven to produce long-term weight loss and reduce associated comorbidity in morbidly obese patients. Sleeve gastrectomy accounts for majority of the annually expanding numbers of such procedures in the United States. However, following sleeve gastrectomy, a subset of patients do not achieve significant weight loss and/or can experience subsequent weight gain requiring a conversion to gastric bypass. We sought to understand the socioeconomic differences encompassed by Social Vulnerability Index (SVI) of patients undergoing sleeve gastrectomy and its correlation with conversion to a gastric bypass.

Methods: Retrospective review of a database of consecutive patients undergoing bariatric surgical procedures at a single community hospital from 2017-2021 performed. Patients who underwent sleeve gastrectomy and those who subsequently proceeded with conversion to gastric bypass within 5 years identified using appropriate CPT codes. Census tracts of the patients were identified using residential address. We utilized the SVI based on these census tracts to identify each patients' social vulnerability. We excluded patients without an address and those with an address out of state. Follow up interval was calculated from the date of sleeve gastrectomy to the date of their last clinical encounter with a bariatric surgeon as documented in their medical record.

Results: Of the 846 patients who underwent sleeve gastrectomy, 33 (4%) patients had a conversion from sleeve to gastric bypass. Patients undergoing conversion from sleeve to bypass were significantly younger at the time of their sleeve surgery (36 years vs 41 years, p < 0.05) and had a significantly higher median overall SVI (0.88 vs 0.78, p < 0.05) which remained significant and correlated with requirement for conversion to bypass on multivariate analysis (OR 15.4 95% CI 1.7-141.4, p < 0.05). In addition, analysis of the Kaplan-Meier curves showed that patients with a high SVI had a significantly longer time from sleeve to gastric bypass (38 months vs 29 months, log rank test p < 0.05, See Fig. 1)

Conclusions: Higher overall social vulnerability correlated with patients who required a gastric bypass following sleeve gastrectomy within 5 years. Identification of this marker suggests the need for greater involvement in the care of patients in marginalized groups to support effective and sustained weight loss following gastric sleeve and predict the potential need for future conversion to gastric bypass.

Fig. 1 Follow up (in months) of patients who underwent a sleeve gastrectomy and subsequent conversion to a gastric bypass stratifed by low and high SVI

figure dl

S246

Screening for GERD in Bariatric Surgery: The Predictive Value of the GERD-HRQL Questionnaire Score Compared with Preoperative EGD Findings

Michael Z Caposole, DO, MBS; Mohamed Aboueisha, MD; Jonathan K Allotey, MS; Erin E Coonan, BS; Benjamin G Crisp; John W Baker, MD, FACS; Shauna Levy, MD, MS, FACS, FASMBS, DABOM; Carlos Galvani, MD, FACS, FASMBS; Department of Surgery, Tulane University School of Medicine

Introduction: There is an ongoing debate on how to best identify patients with gastroesophageal reflux disease (GERD) before bariatric surgery. The value of routine preoperative esophagogastroduodenoscopy (EGD) is questioned, and patient reported symptoms are commonly used for screening. The goal of this study is to determine if patient reported symptoms using a validated questionnaire correlates with preoperative EGD findings.

Methods and Procedures: A prospective cohort study at a single institution was performed. Patients undergoing bariatric surgery between December 2020 and March 2022 were required to complete a preoperative GERD Health-Related Quality of Life (GERD-HRQL) questionnaire and undergo a mandatory EGD. Patients were stratified into two cohorts: (A.) symptomatic (score > 0) and (B.) asymptomatic (score = 0). Statistical analysis was conducted using two-sample T-test and Wilcoxon rank-sum test in SPSS version 26. The predictive value of the GERD-HRQL score was analyzed using Receiver Operating Characteristic (ROC) curves and Areas Under the Curve (AUC; AUC = 0.5 not predictive, 0.50.9 excellent prediction).

Results: There were 103 patients included (44 ± 11 years old, BMI of 46 ± 9 kg/m2). 65 (63%) patients reported symptoms of GERD ((A.) score of 13 ± 19). When scores were stratified by endoscopic-confirmed esophagitis, there was no difference in the number of patients with esophagitis (42% versus 29%, p = 0.2) or hiatal hernias (41.5% versus 45%, p = 0.9) found between cohort (A.) or (B.), respectively. ROC analysis showed the total GERD-HRQL score, heartburn score and regurgitation score were poor predictors of reflux esophagitis [AUC = 0.56 (95% CI, p = 0.2), AUC = 0.56 (95% CI, p = 0.2) and AUC = 0.53 (95% CI, p = 0.65)] and also poor predictors of hiatal hernia [AUC = 0.55 (95%CI, p = 0.4), AUC = 0.55 (95%CI, p = 0.4) and AUC = 0.51 (95%CI, p = 0.8)].

Conclusion: This study demonstrates using patient reported symptoms to guide use of preop EGD is unreliable since 1/3 of patients with esophagitis are asymptomatic. Therefore, the authors recommend routine use of preoperative EGD in all bariatric patients.

S247

Do All Roads Lead to Rome?: A Retrospective Analysis on Surgical Technique in Sleeve Gastrectomy

Alexander H Vu, MD 1; Jessica Chiang, MD1; Chau Hoang, MD1; Yunzhi Qian, MPH2; Nilufar Tursunova, MD1; Jaein Nha, BA1; George Ferzli, MD, FACS1; 1New York University Langone Health, Department of General Surgery; 2Department of Nutrition, University of North Carolina at Chapel Hill

Background: New York University (NYU) Langone Health system is a designated MBSAQIP bariatric center, with 10 bariatric surgeons using different surgical techniques. We present a retrospective analysis that compares individual surgeon technique in sleeve gastrectomy and that identifies potential associations with perioperative morbidity and mortality.

Methods: All adult patients who underwent sleeve gastrectomy between 2017-2021 at the NYU Langone Health campuses were evaluated via electronic medical records and MBSAQIP 30-day follow up data. We also surveyed all 10 practicing bariatric surgeons (Tables 1–5) to analyze the relationship between their techniques and total adverse outcomes (including bleeding, surgical site infections (SSI), any other morbidity eg nausea, vomiting, urinary tract infection, postoperative pneumonia, deep vein thrombosis and pulmonary embolism etc., mortality, readmission or reoperation). Bleeding, SSI, mortality, readmission, and reoperation were specifically sub-analyzed via Pearson chi-square test.

Results: Data from 3,815 adult patients were collected. Among them, 86 (2.77%) out of 3,104 patients who underwent sleeve gastrectomy encountered an adverse event (Tables 1–5). We found that certain surgeon practices e.g. laparoscopic approach (vs. robotic), use of 40-Fr size bougie (vs. 36-Fr), use of Ethicon* stapler (vs. Covidien*, Medtronic*, Intuitive*), use of buttressing (vs. no buttressing), no oversewing (vs. oversewing), use of hemostatic agent (vs. no hemostatic agent), having a staple line to pylorus of approximately 3-cm (vs. 4 to 6-cm), and no routine pre-op UGIS (vs. routine pre-op UGIS) have statistically significant lower total adverse outcome rates and lower bleeding rates.

Furthermore, use of green, gold, blue color staple cartridges (vs. black and purple) had lower adverse outcome rates but otherwise staple color had no specific effect on bleeding rates. Covidien* stapler (vs. Ethicon*, Medtronic*, and Intuitive*), no hemostatic agent use (vs. hemostatic agent use) and routine preoperative UGIS before bariatric surgery had statistically significant lower SSI rates. There was no statistically significant difference observed between the techniques in readmission, reoperation, or mortality.

Conclusions: We found that certain surgical practices in sleeve gastrectomy within our bariatric surgery group had significant effect on the rates of total adverse outcomes, bleeding, and surgical site infection, but ultimately no effect on rates of readmission, reoperation, or mortality. Our findings warrant further investigation into the aforementioned techniques via multivariate regression models.

Keywords: gastric bypass, gastric sleeve, surgical technique, perioperative management, weight-loss surgery, bariatric surgery

*These are trademarked companies.

figure dm
figure dn
figure do
figure dp
figure dq

S248

Application of Machine Learning to Predict Postoperative Gastrointestinal Bleed in Bariatric Surgery

Justin L Hsu, MD 1; Kevin A Chen, MD1; Logan R Butler, BS2; Anoosh Bahraini, MD1; Muneera R Kapadia, MD, MME1; Shawn M Gomez, EngScD3; Timothy M Farrell, MD, FACS1; 1University of North Carolina School of Medicine, Department of Surgery; 2University of North Carolina School of Medicine; 3University of North Carolina at Chapel Hill, Department of Biomedical Engineering

Introduction: Postoperative gastrointestinal bleeding (GIB) is a bariatric postoperative morbidity that leads to readmission, transfusion, intervention, or even reoperation. The recent rise in extended venous thromboembolism regimens as well as outpatient bariatric surgery may increase the risk of postoperative GIB or lead to delay in diagnosis. This study seeks to use machine learning (ML) to create a model that predicts postoperative GIB to aid surgeon decision making and improve patient counseling on their risk for postoperative bleeds.

Methods and Procedures: The Metabolic and Bariatric Surgery Accreditation and Quality Improvement Program (MBSAQIP) database was used to train and validate three types of ML methods: random forest (RF), gradient boosting (XGB), and deep neural networks (NN), and compare them with logistic regression (LR) regarding postoperative GIB. Data were split into a training and validation set, consisting of 80% and 20% of 2020 data respectively. The performance of the models was assessed using area under the receiver operating characteristic curve (AUROC) and compared with DeLong test. Variables with the strongest effect were identified using Shapley additive explanations (SHAP).

Results: 159,959 patients were included in the study. 632 patients were identified with postoperative GIB. The three ML methods, RF (AUROC 0.75; 95% CI, 0.75-0.755), XGB (AUROC 0.742; 95% CI, 0.739-0.744), and NN (AUROC 0.704; 95% CI, 0.702-0.707) all outperformed LR (AUROC 0.694; 95% CI, 0.691-0.697). Using DeLong testing, the difference between RF and LR was determined to be significant with p = 0.011. The best ML method, RF, was able to predict postoperative GIB with a specificity and sensitivity of 70.0% and 73.8% respectively. Type of bariatric surgery, therapeutic anticoagulation use, body mass index, history of pulmonary embolism, history of myocardial infarction, highest preoperative weight, weight closest to operation, the amount of pre-operative weight loss, chronic steroid use, preoperative creatinine, functional status, age, and gastroesophageal reflux disease were the most important features identified by ML.

Conclusion: We have developed a ML model that outperformed LR in predicting postoperative GIB. The bleeding risk calculator can be a helpful tool for both surgeons and patients undergoing bariatric procedures.

figure dr
figure ds
figure dt
figure du

S249

Randomized Single Blind Trial to Compare the Short Term Post-operative Outcome and Cost Analysis of Laparoscopic Guided TAP Block Versus Ultrasound Guided TAP Block in Patients Undergoing Bariatric Surgery

Deborshi Sharma, Prof; Sanjay Meena, Dr; Lady Hardinge Medical College

Introduction: Laparoscopic Bariatric surgery despite being minimally invasive can cause moderate to severe pain in the immediate postoperative period. Hence adequate pain management remains a major challenge. Transversus Abdominis Plane (TAP) block is a regional anaesthetic technique that targets the sensory nerve supply of the anterior-lateral abdominal wall.

Aims and Objectives:

Primary objective: Evaluate Laparoscopic guided TAP block and USG guided TAP block on post-operative analgesia in patients undergoing laparoscopic bariatric surgery.

Secondary objectives: Compare efficacy and cost effectiveness of Laparoscopic guided TAP block with USG guided TAP block in patients undergoing bariatric surgery.

Materials and Methods: Randomized Single blind study was undertaken in a teaching unit of tertiary care hospital. Sample size was calculated by (N) = 2(Zα + Z1-β)2σ2/δ2 which proposed 60 patients in each group. Redo/Revision surgeries along with patients developing complications were excluded and were randomized using random block allocation into Group I: Laparoscopic guided TAP block (LAP-TAP) & Group II: USG guided TAP block. (USG-TAP). 20 ml of 0.25% bupivacaine was injected bilaterally in both groups immediately after completion of bariatric surgery. SPSS v23 (IBM Corp.) was used for data analysis.

Results: Group I (n = 62 27F/4 M) & Group II (n = 60 21F/9 M) were comparable for mean age & BMI. Group I (3.58 ± 0.67) had significantly lower procedure time compared to Group II (12.47 ± 1.61) (p-Value < 0.001).

Time (In hrs) for 1st rescue analgesia in Group I: 7.07 ± 2.61 vs Group II: 7.21 ± 2.39 (p-Value 0.659), total rescue analgesic dose requirement in first 24 h Group I: 1.29 ± 0.53 vs Group II: 1.39 ± 0.50 (p-Value 0.487). VAS scores measured during rest and on movement 2 hourly till 24 h post-operative hours were statistically similar in both groups. Group II cost incurred were INR 4000 ($50) more on an average.

Conclusion: Laparoscopic TAP block is safe and effective approach for postoperative pain management and provides similar comparable analgesic effect as the USG-TAP block. Lap TAP is a surgeon delivered, easy to administer and significantly less time consuming procedure, which is even feasible in an operation theatre which lacks an ultrasound machine. Cost considered the use an USG machine is more among group II. Laparoscopic TAP is operator dependent like the USG guided block and require moderate skills for an effective block.

S250

Bite by Byte: Can Fitness Wearables Help Bariatric Patients Lose More Weight After Surgery?

Hannah M Hollandsworth, MD; Daniel Chung, BA; Estella Y Huang, MD; Nicole H Goldhaber, MD; Lorijane Robles; Bryan J Sandler, MD, FACS; Garth R Jacobsen, MD, FACS; Ryan C Broderick, MD, FACS; Eduardo Grunvald, MD; Santiago Horgan, MD, FACS; UCSD

Introduction: Multidisciplinary approaches to weight loss have been shown to improve outcomes in bariatric patients. Few studies have been performed assessing the utility and compliance of fitness tracking devices after bariatric surgery. We aim to determine whether use of an activity tracking device assists bariatric patients in improving post-operative weight loss behaviors.

Methods: A fitness wearable was offered to patients undergoing bariatric surgery from 2019-2022. Data collected included daily steps, frequency of use, and use of features such as diet log, weight log, and sleep tracking. Six to 12 months after surgery, a telephone survey was conducted to elucidate the impact of the device on the patient's post-operative weight loss efforts. Transcripts were blinded and coded by three separate reviewers to find common themes and conclusions. Sleeve gastrectomy (SG) patients receiving the fitness wearable (FW) were matched to a group of SG patients who did not receive one (non-FW), and total weight loss (%TWL) outcomes were compared between the groups.

Results: Thirty-seven patients met inclusion criteria, 20 of whom responded to our telephone survey. Three patients reported not using the device and were excluded. Fifteen underwent SG and the remaining two underwent gastric bypass (RYGB). Device data was able to be gathered for 10 patients. Patients walked an average of 4,659.9 steps per day while wearing the device. 90% used the weight log, 100% used the sleep tracking feature, and 20% used the diet log. Fifteen (88.2%) of users said it assisted their weight loss, would continue to use the device and recommend it to others undergoing bariatric surgery. All participants who used the device reported that it was user-friendly and easy to use. On average, patients used the device for 8.7 months (range 1 – 20) with 10 (58.8%) reporting that they still currently use a fitness wearable, either the one provided for them or a different one. Demographic data between FW and non-FW groups (age, sex, CCI, initial BMI, and surgery BMI) did not differ significantly. The FW group trended towards more %TWL at 3 months (24.0% vs 19.6%, p = 0.11), 6 months (36.5% vs 20.6%, p = 0.18), and 1 year (37.5% vs 24.7%, p = 0.068).

Conclusions: The use of an activity tracking device enhances a patient's post-bariatric surgery experience, serving to keep patients informed and motivated, and leading to improved activity and better weight loss outcomes.

figure dv

S251

Does the ViSiGi 3D® reduce staple loads and operative time? A randomized controlled trial comparing endoscopic vs. suction calibration device in laparoscopic sleeve gastrectomy

Danny Mou, MD, MPH; Edward Lin, DO, MBA; Scott Davis, MD; Victoria Delgado, RDN, CSOWM, LD; Zachary Grunewald, PhD, MS, RDN, LD; Jahnavi Srinivasan, MD; Jamil L Stetler, MD; Federico J Serrot, MD; Ankit Patel, MD; Omobolanle Oyefule, MD; Maggie Diller, MD; Katherine Fay, MD; Carrie P Hall, MD; Elizabeth M Hechenbleikner, MD; Emory University Department of Surgery

Introduction: The laparoscopic sleeve gastrectomy (LSG) is currently the most common bariatric surgery procedure performed in the United States. It is critical to ensure appropriate and consistent sleeve size and orientation. Various devices have been used to achieve this, including the weighted bougie, esophagogastroduodenoscopy (EGD), and suction calibration devices (SCDs) such as the ViSiGi 3D® (Boehringer Labs, Phoenixville, PA). SCDs employ suction to fixate the conduit during stapling and standardize sleeve orientation with prior reports showing that usage may decrease staple load firings and operative time. However, existing studies are limited to single-surgeon experience and retrospective data. We performed the first randomized controlled trial comparing a SCD against EGD. We hypothesize that use of SCDs results in fewer staple load firings.

Methods: This is a prospective, randomized, nonblinded, open-label study from a single MBSAQIP-accredited academic center. Medically cleared patients for LSG were randomized to either SCD or EGD calibration. Inclusion criteria include ≥ 18 years of age and surgeon agreement that LSG is clinically appropriate per consensus criteria. Patients with prior weight loss surgery or intraoperative hiatal hernia repair were excluded. A randomized block design was employed to control for BMI, gender, and race. Seven surgeons employed a standardized operative technique that included powered mechanical staplers, bioabsorbable staple line reinforcement, and intraoperative leak test via SCD or EGD. The primary endpoint was number of staple load firings and secondary endpoint was operative duration. Results were analyzed via t-test.

Results: A total of 107 LSG patients (88% female) met inclusion criteria and were enrolled, with an average age of 43 years and average BMI of 48 kg/m2. Of note, nine patients were excluded due to intraoperative hiatal hernia repair. Patients were randomized to receive either EGD (n = 56) or SCD (n = 51) calibration. No significant differences in baseline characteristics were identified in these two groups. The mean number of staple load firings for EGD and SCD calibration arms were 5.4 and 5.3, respectively (Table 1; p = 0.417). The mean operative times for EGD and SCD calibration groups were 96.5 min and 94.6 min, respectively (p = 0.776). There were no significant differences in 30-day post-operative complications.

Conclusions: In our cohort of LSG patients, there was no difference between the EGD and SCD calibration techniques in terms of the number of staple load firings and operative duration. Further studies are needed to compare the SCD against different techniques, hospital settings, and patient cohorts.

Table 1

figure dw

S252

Does Preoperative Esophageal Testing Predict Postoperative GERD Status in Sleeve Gastrectomy Patients?

R Sillcox, MD; S Khandelwal, MD; MK Bryant, MD; B Vierra, MD; RP Tatum, MD; RB Yates, MD; JY Chen, MD; University of Washington

Objectives: Vertical sleeve gastrectomy (VSG) is associated with rates of postoperative GERD as high as 84% and revision to RYGB as high as 13.3% 5-10 years post-VSG. The reasons are multifactorial; however half of revisions are due to intractable GERD. Our institution routinely performs preoperative pH studies, high-resolution manometry (HRM), and upper endoscopy (EGD) to aid in operative decision making. We hypothesize that abnormal DeMeester scores and ineffective esophageal motility (IEM) would lead to higher rates of postoperative GERD in VSG patients.

Methods: A single institution retrospective review was conducted of adult patients who underwent preoperative testing and VSG between 2015-2022. Patients that did not complete all three preoperative tests were excluded. Esophageal motility diagnoses were classified using Chicago Classification. Patients filled out a symptom questionnaire at the time of testing. Patients were counseled on risks of postoperative GERD with VSG and an operation was chosen with shared decision making. Postoperative GERD was defined by patient-reported symptoms and categorized as de novo, resolved, and unresolved GERD at 1 year follow up. We examined revision rates to RYGB between 2015-2022. Univariate logistic regression was used to examine the relationship between preoperative esophageal tests and postoperative GERD status.

Results: 291 patients underwent VSG. 230 (79%) were female with a median age of 46 (18-70) years old and a median BMI of 47 (33-77). 33 (11.3%) were diagnosed with IEM, 67 (23%) had an abnormal DeMeester score, and 16 (5.5%) had both. Preoperatively, symptomatic GERD was reported by 122 patients (41.9%); of those, 69 (56.6%) had resolution and 53 (43.4%) did not. Postoperatively, 27 of 291 patients (9.5%) developed de novo GERD. Median reduction in BMI was 11.8 (8.6-15.5). Revision to RYGB was performed in 5 patients (1.7%) for intractable GERD. Preoperative abnormal DeMeester score, IEM, a combination of both, and BMI reduction were not statistically significant predictors of postoperative GERD status (p = 0.46, p = 0.37, p = 0.49, p = 0.48).

Conclusion: This series represents the largest cohort with preoperative esophageal testing and long term follow up. Our hypothesis that abnormal pH studies and IEM would predict postoperative GERD status and improve patient selection for VSG was not supported. However, we do counsel patients with more substantial abnormal findings to consider RYGB over VSG which could explain our low VSG to RYGB revision rates and relatively low postoperative de novo GERD incidence.

S253

Decision Regret Up To 6 Years After Sleeve Gastrectomy

Theofano Zoumpou, MD 1; Aaron J Fleishman, MPH2; Daniel B Jones, MD, MS, FACS1; Christina C Wee, MD, MPH3; 1Department of Surgery, Rutgers, New Jersey Medical School, Newark, NJ, USA; 2Department of Surgery, Beth Israel Deaconess Medical Center, Harvard Medical School, Boston, MA, USA; 3Department of Medicine, Beth Israel Deaconess Medical Center, Harvard Medical School, Boston, MA, USA

Introduction: Sleeve Gastrectomy (SG) is one of the most popular types of weight loss surgery today but is neither risk-free nor universally effective. We previously demonstrated that 5% of patients who underwent Roux-en-Y Gastric Bypass (RYGB) and up to 20% of those who underwent gastric banding report overall regret 4 years postoperatively. This study aims to explore patients' attitudes towards their decision to have SG and examine their decision regret up to 6 years postoperatively.

Methods and Procedures: We surveyed 185 patients who were at least 6 months post-SG (response rate 30%). We used a modified version of the Decision Regret Scale developed by Brehaut et al. We converted responses to a 0–100 scale so that higher scores reflect greater regret; scores were summed and averaged so that an overall score > 50 reflects overall decision regret. We characterized patients who expressed having overall decision regret (score > 50) vs. those who did not (≤ 50). Demographic and preoperative clinical information was extracted from the online medical records.

Results: Of 185 SG patients, only 13 (7%) reported regret scores > 50 (i.e. high decision regret); 1 (0.5%) did not think they made the right decision; 3 (1.6%) regretted having this type of surgery very much; 4 (2.2%) would not undergo SG again. The mean time from SG to survey completion was 41 months (range: 6-76 months). Patients with regret scores > 50 were more likely to be further out from SG than those with scores < 50 (median time 61.8 months vs. 41.1 months). Unadjusted comparisons between the two groups revealed that patients with high regret scores had significantly lower mean weight loss (13.2% vs. 20.9% of initial weight), and reported less improvement in Quality-of-Life (QoL) including in physical (46.2% vs. 93.5% "somewhat" or "significantly" improved) and emotional health (30.8% vs. 80.4%), self-esteem (23.1% vs. 81.2%), body image (38.5% vs. 81.6%), sexual function (7.7% vs. 48.1%), work-life (7.7% vs. 55%), and social life (0% vs. 51.7%). The two groups did not differ in short-term complications, but those reporting overall regret were more likely to report GI complaints such as bloating (61.5% vs. 30.4%).

Conclusion(s): In our study, very few patients reported regret (7%) up to 6 years after SG in line with prior reports after RYGB. Those with regret reported poorer QoL.

S254

Predictors of Post-discharge Pain and Satisfaction with Pain Management After Laparoscopic Bariatric Surgery

Maxime Lapointe-Gagner, BSc 1; Shrieda Jain, BSc1; Naser Alali, MD2; Hiba Elhaj, MSc1; Anne-Sophie Poirier, BSc1; Pepa Kaneva, MSc1; Mohsen Alhashemi, MD2; Lawrence Lee, MD, PhD2; Liane S Feldman, MD2; Michel Gagner, MD3; Amin Andalib, MD2; Julio F Fiore Jr, PhD2; 1Steinberg-Bernstein Centre for Minimally Invasive Surgery and Innovation, McGill University Health Centre; 2Division of General Surgery, Department of Surgery, McGill University; 3Clinique Michel Gagner MD Inc

Introduction: The aim of this study was to evaluate the extent to which patient and surgical factors predict 7-day pain intensity, interference, and satisfaction with pain management after bariatric surgery. Acute pain management after bariatric surgery remains challenging as patient- and procedure-related factors may increase the risk of analgesia-related adverse events (e.g., marginal ulcers, opioid-use disorder). Therefore, there is a dire need to minimize these risks while ensuring satisfactory pain management. Identifying modifiable factors associated with patient-reported pain outcomes may ultimately improve quality of care after bariatric surgery.

Methods and Procedures: We analyzed data from a prospective database of patients undergoing laparoscopic bariatric surgery at two university-affiliated hospitals and one private clinic in Montreal. Preoperative assessments included demographic variables, Pain Catastrophizing Scale (scale range 0-52, higher = worse), Patient Activation Measure (low [1-2] vs. high activation [3-4]), pain expectation (0-10, higher = worse), and Patient-Reported Outcomes Measurement Information System-29 (PROMIS-29) anxiety (40.3-81.6, higher = worse) and depression scales (41.0-79.4, higher = worse). At 7 days post-discharge, assessments included PROMIS-29 pain intensity (0-10, higher = worse), pain interference (41.6-75.6, higher = worse), and satisfaction with pain management (dichotomized as high [10-9] vs. lower [80-0] satisfaction). Linear and logistic regression was used to assess the association of pain outcomes with potential predictors identified from literature and/or clinical expertise.

Results: 351 patients were recruited (mean age = 44, mean BMI = 45, 77% female, 71% sleeve gastrectomy) from September 2021 to April 2022. Discharge prescriptions included acetaminophen (100%) ± celecoxib (3.7%) around-the-clock and opioids as needed (99.7%). At 7 days post-discharge, median pain intensity was 2.5 (IQR 1-5), pain interference was 55.6 (IQR 52.0-61.2), and most patients had high satisfaction with pain management (75.7%). In multivariate analysis, higher preoperative pain expectation and anxiety were independently associated with 7-day pain intensity (expectation + 0.15 [95%CI + 0.05 to + 0.26]; anxiety + 0.04 [95%CI + 0.01 to + 0.07]) and pain interference (expectation + 0.45 [95%CI + 0.06 to + 0.83]; anxiety + 0.22 [95%CI + 0.11 to + 0.33]). Lower satisfaction with pain management was independently associated with younger age (OR 0.97 [95%CI 0.95 to 1.00]), non-White race (OR 1.92 [95%CI 1.03 to 3.55]), increased pain catastrophizing (OR 1.03 [95%CI 1.01 to 1.05]), and low patient activation (OR 0.53 [95%CI 0.29 to 0.95]).

Conclusion: This study supports that patient-related factors are important predictors of post-discharge pain outcomes after laparoscopic bariatric surgery. Our findings highlight the importance of addressing the needs of racial minorities and implementing interventions focused on preoperative anxiety, pain expectation, catastrophizing, and patient engagement to improve postoperative pain outcomes.

S255

Imaging in Complicated Cholecystitis is Poorly Predictive of Surgical and Pathological Findings

Daniel Horwitz, MD, MBA; Charlotte Hartwell, MD; Cristina Hajdu, MD; Bari Dane, MD; Cris Malino, MD, MPH; NYU Langone Health

Introduction: The diagnosis of gallbladder pathology requires a combination of history, physical, lab work, and imaging. While HIDA is the gold standard for diagnosing cholecystitis, it can delay diagnosis and is not universally available. While prior studies show high sensitivity/specificity of CT (94%/59%) and US (88%/80%), we hypothesize that the quicker, more available studies are inaccurate in complicated (hemorrhagic, gangrenous, perforated, emphysematous) cholecystitis. Preoperative workup is essential to management and timing particularly for comorbid patients as surgical management carries a higher risk of morbidity and mortality which necessitates an informed discussion.

Methods: Consecutive surgical case requests were queried over a two year periods from 2020-2022. A total of 988 case requests were identified. 205 charts were reviewed and after removing duplicates, cancelled cases, and incomplete data, 143 cholecystectomies were left for analysis. Data was collected on demographics, patient characteristics, and reports from imaging, operation and pathology. Complicated cholecystitis was defined as any evidence of hemorrhage, perforation, or necrosis on final pathology. Diagnoses were considered surgically concordant if the surgeons post-operative diagnosis matched final pathology or if a diagnosis of biliary colic or gallstone pancreatitis had a final pathology of chronic cholecystitis. Imaging was considered concordant if the final impression matched pathology or a diagnosis of cholelithiasis had a final pathology of chronic cholecystitis.

Results: 87/143 (60.1%) of the patients were female. The average BMI was 30.57. 87.7% of cholecystectomies were emergent, vs. interval/elective. 17/143 (11.8%) were found to have features consistent with complicated cholecystitis on post-operative pathology. There was surgical concordance in 85/143 (59.4%) of cases and in 9/17 (52.9%) of complicated cases. A total of 127/143 (88.8%) of patients underwent an ultrasound and there was imaging concordance in 66/143 (46.1%) of cases. 14/17 (82.4%) complicated cases underwent an ultrasound with 4/14 (28.5%) concordance. 44/143 (30.1%) of patients underwent CT evaluation with a concordance of 21/44 (47.7%). In complicated cases, 8/17 (47.0%) underwent a CT and there was concordance in 2/8 (25.0%) of cases. 26/143 (18.1%) of patients underwent MR evaluation. 3/17 (17.6%) of complicated cases underwent MR and there was concordance in 0/3 (0%) of cases.

Discussion: Complicated cholecystitis remains a diagnostic and technical challenge. Although the initial data from our single center review reveals the overall rate of complicated cholecystitis is low, it also suggests that imaging findings are less accurate than that of uncomplicated cholecystitis. Further analysis will be aimed at identifying imaging features to guide management.

S256

Does the Timing of Cholecystectomy After Percutaneous Cholecystostomy for acute Cholecystitis Impact Patient Outcomes?

Spyridon Giannopoulos, MD; Keith Makhecha, BS; Sathvik Madduri, BS; Felix Garcia, BS; Timothy C Baumgartner, BS; Dimitrios Stefanidis, MD, PhD; Department of Surgery, Indiana University School of Medicine, Indianapolis, IN, USA

Introduction: Percutaneous cholecystostomy (PC) represents an important treatment approach in patients with acute cholecystitis (AC) who are unfit for laparoscopic cholecystectomy (LC). However, the optimal timing to perform LC for definitive treatment after PC is unknown. This study aimed to investigate the effect of the interval between PC and LC on perioperative complications.

Methods and Procedures: This retrospective cohort study included adult patients diagnosed with AC who underwent PC followed by LC at a single academic center between 2016 and 2020. Patients with a history of hepatobiliary surgery, common bile duct stones, cirrhosis, active malignancy, or chronic immunosuppressive therapy were excluded. Additionally, baseline demographics, PC and LC hospitalization data, and perioperative outcomes following LC were collected. Linear and logistic regression models were used to analyze the impact of the interval between PC and LC on perioperative outcomes.

Results: One hundred twelve patients (60% male) with a mean age of 64.6 ± 1.4 (mean ± SD) years were diagnosed with AC (55% mild, 37% moderate, 9% severe), and underwent PC on average 1.8 days following diagnosis. The interval between PC and LC was 65 (interquartile range [IQR], 48-96.5) days and was not associated with procedure duration, estimated blood loss, or length of hospitalization (Table 1). The timing of LC also did not influence the 30 day readmission (OR 0.988, p = 0.602), reintervention (OR 0.994, p = 0.469), emergency department visits (OR 0.994, p = 0.370), biliary injury (OR 0.987, p = 0.679), leak (OR 0.99, p = 0.831), intraoperative events (OR 0.979, p = 0.164) or the rate of conversion to open (OR 0.989, p = 0.267). However, delayed LC after PC increased the intensive care unit (ICU) stay significantly (Coef 0.003, p < 0.001).

Conclusion: While the timing of LC after PC did not influence the perioperative outcomes longer intervals were associated with significantly increased length of ICU stay. Based on these findings, patients may benefit from early LC after percutaneous cholecystostomy for AC. Multi-institutional studies are necessary to confirm these results.

figure dx

S257

Laparoscopic Cholecystectomy is Safe and Associated with Better Outcomes When Compared to ERCP and No-Intervention in Elderly Patients Presenting with Acute Gallstone Pancreatitis

Irena Stefanova; Muhammad Tobbal; Darmarajah Veeramootoo; Henry De'Ath; Frimley Health NHS Foundation Trust

Introduction: With an ageing population, gallstone pancreatitis (GSP) is increasingly common in elderly patients and carries comparatively worse outcomes. Laparoscopic cholecystectomy (LC) is recognised as the best treatment for GSP, although in elderly and morbid populations an Endoscopic Retrograde Cholangio Pancreatogram with sphincterotomy (ERCP) is valued as a risk-reducing alternative. Evidence and guidelines specifically aimed at the management of GSP in the elderly is lacking, however; so, this study aimed to investigate and compare the outcomes of specific management strategies for biliary pancreatitis in the elderly.

Methods: A retrospective single centre study of all patients ≥ 65 years of age with first presentation of GSP undergoing either (1) LC only, (2) ERCP, (3) ERCP followed by LC or (4) no intervention. Patient demographics, disease specific parameters and outcomes including biliary related mortality and recurrence of biliary disease were compared.

Results: A total of 127 patients were included. The median age was 77 years (IQR 71-84), and 61 (48%) were male and 66 (52%) females. The majority (78%, n = 99) presented with mild pancreatitis, whilst 10% (n = 12) had severe pancreatitis based on modified Atlanta classification. 29% (n = 37) of patients were treated with ERCP only; 36% (n = 46) underwent LC alone; 9% (n = 11) had ERCP followed by LC and 26% (n = 33) received no intervention. Patients managed without intervention were significantly older (p < 0.001), with higher frailty scores (p < 0.001).

The LC only group had significantly lower post-procedure re-admission rates of 4.5% (n = 2) in comparison to 24% (n = 9) for ERCP (4 of whom had recurrent biliary pancreatitis), 37.5% (n = 3) for ERCP + LC and 32% (n = 8) for the non-intervention group (p = 0.01).

Biliary cause mortality was significantly higher in the non-intervention group (n = 7, 21%) when compared to the LC only, ERCP followed by LC and ERCP only cohorts (n = 0; n = 0; n = 3, 8% respectively, p < 0.001).

In all patients undergoing LC, there were no conversions to open, no bile leaks or common bile duct injuries. There were two cases of retained stones (3.5%) and 1 post-operative collection. In all individuals who underwent ERCP, there were no perforations or bleeds. There were two cases of post-ERCP pancreatitis (4%) and two subsequent admissions with cholangitis (4%).

Conclusion: In the setting of best practice, treatment for GSP must be tailored to each individual case, although a laparoscopic cholecystectomy represents the gold standard for the elderly patients. Biliary complications in the elderly lead to significant mortality and morbidity and timely intervention is therefore crucial.

S258

Comparison of clinical Outcomes Between Minimally Invasive (Laparoscopic and Robotic) and Open Extended Cholecystectomy

Hee Ju Sohn 1; Yoo Shin Choi1; Hongbeom Kim2; Mirang Lee2; Wooil Kwon2; Jin-Young Jang2; Jun Suh Lee3; Yoo-Seok Yoon2; Chang Sup Lee4; 1Department of Surgery, Chung-Ang University College of Medicine, Chung-Ang University, Korea; 2Department of Surgery, Seoul National University Hospital, Korea; 3Department of Surgery, Seoul National University Bundang Hospital, Korea; 4Department of Surgery, Seoul National University Boramae Medical Center, Korea

Background: Minimally invasive surgery (MIS), both laparoscopic and robotic surgery for gallbladder cancer (GBC) has rapidly increased recently, however, there is a lack of large multicenter studies of its safety and long-term outcome. This study was undertaken to determine the feasibility of MIS-extended cholecystectomy for GBC and compare it with conventional open surgery.

Methods: Patients diagnosed with clinically suspected GBC who underwent extended cholecystectomy (EC) from 2010 to 2020 in 3 large volume hepatobiliary and pancreatic surgery centers were studied. EC was defined as a wedge resection of liver bed including cholecystectomy and regional lymphadenectomy. Clinicopathologic data of O-EC and MIS-EC was analyzed and propensity score matching was performed to compare the short-term and long-term outcomes. Subgroup analysis of laparoscopic and robotic surgery was evaluated.

Results: A total of 377 patients were included, O-EC and MIS-EC group (laparoscopic EC: 40, robotic EC: 29) were 308 and 69 patients, respectively. Though MIS-EC group had a longer operative time (188.9 vs 238.1 min, p < 0.001), shorter length of hospital stay (9.0 vs7.2 days, p = 0.007), there was no difference in operative blood loss, complication rate, 30-day mortality rate. More lymph nodes were retrieved in O-EC (8.5 vs 7.1, p = 0.044) and there was no significant difference in 3-year overall survival. In subgroup analysis of MIS-EC, laparoscopic EC had longer operative time (264.4 vs 202.0 min, p = 0.001), however, other perioperative outcomes and 3-year survival outcomes were comparable.

Conclusion: MIS-EC is feasible with advantages of decreased length of stay and comparable survival to O-EC in GBC. Both laparoscopic and robotic EC had comparable perioperative and oncologic outcomes so it can be chosen according to the preference of the surgeon.

S259

Early Versus Delayed Laparoscopic Cholecystectomy After Endoscopic Retrograde Cholangiopancreatography For The Management Of Cholecystocholedocholithiasis

Warit Rungsrithananon, MD; Warakorn Jaseanchiun, MD; Mana Sirapat, MD; Setthasiri Pantanakul, MD; Panot Yimcharoen, MD; Bhumibol Adulyadej Hospital

Introduction: Approximately 15% of patients with gallstones have concomitant common bile duct stones, also known as cholecystocholedocholithiasis (CCL). The treatment for CCL may be achieved by a two-step-protocol, endoscopic retrograde cholangiopancreatography (ERCP) followed by laparoscopic cholecystectomy (LC). Optimal timing between two procedures remains controversial, particularly in setting with limited resources. Balancing between morbidity from early surgery and adverse events from delaying treatment is a major concern. This study aimed to compare operative outcomes and gallstone-related adverse events between early laparoscopic cholecystectomy (ELC) and delayed laparoscopic cholecystectomy (DLC) after ERCP in patients with CCL.

Methods: A retrospective cohort study was conducted on CCL patients who underwent ERCP followed by LC between January 2015 to December 2018 at our institute. ELC group was defined as LC carried out within 7 days after ERCP in the same admission. DLC was defined as LC carried out within 7 to 180 days after ERCP. Baseline characteristics included: age, gender, body mass index, and comorbidity. Treatment outcomes included: time-to-surgery (TTS), conversion, operative complication (gallbladder perforation, bile duct injury, bile leak, and surgical site infection), operative time, and length of stay. Adverse events prior to LC included: acute cholecystitis, recurrent choledocholithiasis, acute cholangitis, and biliary pancreatitis.

Results: A total of 60 cases were included in this study: 16 cases (27%) to the ELC group and 44 cases (73%) to the DLC group. Baseline characteristics were not significantly different between two groups. Mean TTS was 3 ± 2 days in ELC and 82 ± 37 days in DLC. There was no statistically significant difference in both treatment groups: conversion rate (12.5% vs 11.4% p = 0.903), gallbladder perforation (31.3% vs 11.4%, p = 0.68), bile duct injury (0% vs 6.8% p = 0.284), surgical site infection (6.3% vs 2.3% p = 0.448), operative time (156 vs 135 min, p = 0.235), and length of stay (11 vs 7 days, p = 0.055). In DLC group, 11.4% developed acute cholecystitis and 4.54% suffered from recurrent choledocholithiasis and cholangitis while waiting for the surgery.

Conclusion: ELC and DLC following ERCP are both safe for the management of CCL without significant difference in complication or conversion rate. However, early surgery is favorable in order to minimize adverse events prior to LC. This study is limited by its retrospective nature and a small sample size, therefore, more extensive large-scale studies are essential to further validate our results.

S260

ERCP Findings Provide Further Justification for a Surgery First Mindset in Choledocholithiasis

Gloria Sanin, MD; Gabriel E Cambronero, MD; Aravindh M Ganapathy, MD; Maggie E Bosley, MD; Allison M Perko, BS; Jake A Niebler, BS; Carl J Westcott, MD; Andrew M Nunn, MD; Preston R Miller, MD; Lucas P Neff, MD; Wake Forest Baptist Medical Center

Introduction: Using endoscopic retrograde cholangiopancreatography (ERCP) initially for choledocholithiasis management creates a two-procedure pathway of ERCP followed by laparoscopic cholecystectomy (LC). In contrast, a single-stage, "surgery first", approach consisting of LC with cholangiogram and possible laparoscopic common bile duct exploration (LCBDE) has a shorter length of stay with an equivalent safety profile. Despite this, nationwide referral patterns heavily favor ERCP first. The result is an undescribed proportion of patients undergoing ERCP prior to LC with CBD obstructions likely amenable to clearance via LCBDE. Preoperative ERCPs were reviewed in patients with a non-complicated presentation of choledocholithiasis to determine what proportion of patients may benefit from a surgery first approach that will decrease the instances in which patients are unnecessarily exposed to two separate anesthetic events.

Methods: We retrospectively reviewed 71 patients over the age of 18 who underwent ERCP first at a single center. Thirteen patients were excluded because of missing data, a diagnosis of cholangitis, or due to poor surgical candidacy. Endoscopic information collected and analyzed included ERCP indications, presence of stones and/or sludge, imaging, and stone size and number. Based on accepted criteria, stone size was categorized as small (0-4 mm), medium (5-7 mm), and large (≥ 8 mm).

Results: At the time of ERCP, 11(19%) patients had sludge only, 15 (26%) had sludge and stones, while 32 (55%) had stones only. Of those 47 with stones, 17 (36%) were small, 16 (34%) were medium, and 14 (30%) were large. Twenty-six patients (45%) had isolated choledocholithiasis, making it the most common primary indication for ERCP. This was followed by 25 (43%) with concomitant cholelithiasis and/or cholecystitis, gallstone pancreatitis 7 (12%). The diagnosis was suggested on ultrasound in 38 patients (65%), CT scan in 36 patients (63%), and MRCP in 23 patients (40%), with 21 (36%) undergoing multiple imaging modalities that included MRCP. Of all patients who underwent ERCP, 76% of patients had findings of sludge and/or stones that might have been amenable to basic LCBDE maneuvers and would have benefited from a surgery first approach.

Conclusion: Our findings suggest that the majority of patients who underwent preoperative ERCP for suspected choledocholithiasis had obstructions that are amenable to LCBDE. Implementing surgery first management for suspected choledocholithiasis can offer an efficient alternative to the more common pathway of ERCP followed by LC. Renewed efforts are needed to promote LCBDE implementation and disseminate effective training to general surgeons.

S261

Cost Effectiveness of Laparoscopic Common Bile Duct Exploration Using a Disposable Single-Use Choledoschoscope: Bringing an Old Concept Back To Life

Omar Bellorin-Marin, MD1; Julia Button, MD 1; Amy Holstrom, MD1; Subhash Krishnamoorthy, MD2; Stephen Oh, MD1; Beth Hochman, MD2; Cheguevara Afaneh, MD1; 1New York Presbyterian-Weill Cornell; 2New York Presbyterian-Columbia

Introduction: Laparoscopic common bile duct exploration followed by cholecystectomy has been demonstrated to be cost effective. The use of repurposed endoscopes has been limited by the risk of bacterial colonization of the bile duct. Lack of training and coexistence of ERCP have also prevented adoption of this method as a treatment option for choledocholithiasis.

Methods and Procedures: A retrospective study from a prospective collected database of patients eligible for laparoscopic common bile duct exploration at two large tertiary academic centers was performed from June 2021 to June 2022. Patient eligibility was identified following an algorithm for choledocholithiasis. Patients with a positive preoperative MRCP or intraoperative cholangiogram positive for small stones, less than 3 stones and favorable cystic duct anatomy were included for transcystic bile duct exploration. Exclusion criteria included the presence of cholangitis on admission, large stones, and 3 or more stones. A care pathway was designed to expedite patient admission and discharge following the surgical procedure. Four surgeons trained in minimally invasive surgery were selected to be involved in the study. A transcystic common bile duct exploration was performed using an FDA approved disposable single-use choledochoscope.

Results: A total of 22 patients underwent laparoscopic common bile duct exploration over the study period including 15 females and 7 males. The median age of presentation was 41.5 SD ± 19.6 years with a BMI of 30.5 SD ± 7.96; 54% of patients were ASA 3 or 4. Median procedure time was 136 SD ± 73.2 min. Intraoperative cholangiogram was performed on all patients with a successful rate of bile duct clearance after bile duct exploration of 72.7%. Median hospital length of stay was 2.55 SD ± 2.38 days. No intraoperative complications, 30 day complications or 90 day readmissions were reported. When compared to the ERCP followed by cholecystectomy pathway, there was a reduction of length of stay by 50% with an estimated potential savings of $126,363 of fixed costs.

Conclusions: A single stage pathway for choledocholithiasis resolution with a new single-use disposable choledochoscope is safe and effective with potential savings in length of stay.

S262

Delay for cholecystectomy after common bile duct clearance with ERCP is just running at recurrent biliary event.

Eric Bergeron, MD, MSc; Thibaut Maniere, MD; Theo Doyon; Etienne Desilets, MD; Charles-LeMoyne Hospital

Background: Gallstone disease will affect 15% of the adult population throughout life with a prevalence reaching 80% at the age of 90. Concomitant common bile duct stone (CBDS) occurs up to 30% of patients. Endoscopic retrograde cholangiopancreatography (ERCP) is the mainstay of management for removal of CBDS, as cholecystectomy for the prevention of recurrent biliary event (RBE). RBE occurs in up to 47% if cholecystectomy is not done. The goal of this study was to evaluate the timing of occurrence of RBE after common bile duct clearance with ERCP and associated outcomes.

Methods: The records of all patients who underwent ERCP for gallstone disease before cholecystectomy, in a single center from 2010 to 2022, were reviewed retrospectively. All RBE were identified. The intervals between ERCP with clearance, and RBE and cholecystectomy were analyzed. Actuarial incidence of recurrence was built. Patients with and without RBE were compared. Chi-square test, Student's T test and ANOVA were applied when appropriate.

Results: The study population is composed of 529 patients. Mean age was 58.0 (18-95). There were 221 RBE in 151 patients (28.5%), 45/151 (29.8%) having more than one episode. The most frequent RBE was acute cholecystitis (n = 104), 33/104 with complication (abscessed, perforated, gangrenous). Episodes associated with recurrent CBDS occurred in 95 cases, with cholangitis and pancreatitis in 22/95 (23.1%) and 8/95 (8.4%) cases respectively. Median time for first RBE was 54 days. Actuarial incidence of RBE was 2.5% at 7 days, 6.1% at 14 days, 15.3% at one month, 27.6% at 3 months, 37.7% at 6 months, and 53.3% at one year. Groups with or without RBE were comparable in terms of demographics. Distribution of ASA score was comparable. Patients with RBE had significant longer hospitalisation time (11.7 vs 6.4 days; p < 0.0001), longer operative time (66 vs 48 min; p < 0.0001), longer postoperative stay (2.9 vs 0.9 days; p < 0.0001), higher open surgery rate (7.9% vs 1.3%; p < 0.0001), and more complicated pathology (23.8% vs 5.8%; p < 0.0001) and cholecystitis (64.2% vs 25.2%; p < 0.0001) as final diagnoses.

Conclusions: In our study, RBE occurred in 28.5% of the subjects at a median time of 54 days. RBE occurred even within 7 days after common bile duct clearance with ERCP and incidence steadily increases thereafter. Cholecystectomy should be carried out as soon as possible after clearance in order to prevent RBE and adverse outcomes.

S263

Laparoscopic Cholecystectomy Critical View of Safety (LC-CVS): A Multi-national Validation Study of an Objective, Procedure-Specific Assessment Using Video-Based Assessment (VBA)

Gina Adrales, MD, MPH 1; Francesco Ardito, MD, PhD, FACS2; Pradeep Chowbey, MD, FACS, MNAMS, FRCS, FIMSA, FAIS3; Salvador Morales-Conde, MD4; Alberto Ferreres, MD, PhD, JD, MPH5; Chrys Hensman, FRCS, MS6; David Martin, MD7; Hanno Matthaei, MD8; Bruce Ramshaw, MD9; Keith J Roberts, PhD, FRCS10; Harald Schrem, MD11; Anil Sharma, FRCS, MD, MS, FICS12; Stephen Tabiri, MD, PhD, MEd13; Eric Vibert, MD14; Michael S Woods, MD, MMM15; 1Chief, Division of Minimally Invasive Surgery, Director, Minimally Invasive Surgical Training and Innovation Center (MISTIC), Johns Hopkins Hospital; 2Hepatobiliary Surgery Unit, Fondazione Policlinico Universitario Agostino Gemelli, IRCCS. Catholic University, Rome, Italy; 3Institute of Laparoscopic, Endoscopic and Bariatric Surgery, Max Super Specialty Hospital, Saket, New Delhi, India; 4Unit of Innovation in Minimally Invasive Surgery, University Hospital Virgen del Rocío. University of Sevilla (Spain); 5Department of Surgery, University of Buenos Aires; 6Monash University Department of Surgery & LapSurgery Australia; 7University of Minnesota; 8Department of Surgery, University Medical Center Bonn, Bonn, Germany; 9CQInsights PBC, Knoxville, TN and Medical Advisor, Caresyntax Corp.; 10Liver Transplant and HPB Surgery, University Hospitals Birmingham NHS Trust; 11General, Visceral and Transplant Surgery, Medical University Graz, Austria; 12Max Institute of Laparoscopic, Endoscopic & Bariatric Surgery, Max Healthcare, Saket, New Delhi; 13Consultant General Surgeon, University for Development Studies-School of Medicine and Health Sciences, Tamale Teaching Hospital; 14Centre Hépato-Biliaire, Paul Brousse Hospital, AP-HP, Villejuif, France; 15Caresyntax, Corp.

Introduction: The critical view of safety in laparoscopic cholecystectomy is one key to avoiding severe injuries to the biliary tree and a key element of training. This study was designed to preliminarily validate an objective, procedure-specific assessment (OPSA) to document safe vs. unsafe LC-CVS.

Methods and Procedures: Two board-certified LC surgeons developed an experientially-driven OPSA for videoscopic LC-CVS. Items are listed in Table 1, assessed on a three-point scale: 1 = Poor (unsafe); 2 = Adequate (safe); 3 = Excellent (safe—expert). A LC expert selected 36 cases from > 450 videos of varying difficulty. Videos were de-identified, randomized, and uploaded into a proprietary SaaS-based surgical data platform used for video-based assessment (VBA) to improve surgical quality and safety. Nine surgeons representing eight countries (mean practice duration: 17.1 years) were trained on the SaaS-based assessment and completed the LC-CVS assessment on all videos. Data were downloaded to a.cvs file for analysis. The primary analysis was total agreement — the ratio of raters that agreed on case safety — based upon each task being assessed as either safe (score ≥ 2) or unsafe (score < 2).

Results: 324 video reviews were analyzed.

figure dy

Table 1 LC-CVS OPSA. *Until division of the cystic artery and duct.

Total agreement between raters in concluding the procedure was safe based upon the LC-CVS OPSA was high, ranging from a low of 77% (infundibulum rotation), a high of 87% (cystic duct identification), and an average of 81%.

Conclusions: This large, multi-national study documents the LC-CVS OPSA has a high percentage of total agreement regarding safety, signifying and represents a valid measure of safe conduct in LC and may be used to help determine surgeon competency and the granting of autonomy.

S264

Complications of Percutaneous Endoscopic Biliary Lithectomy (PEBL): A Single Institution Experience

Antoinette Hu, MD; Colin G DeLong, MD; Randy S Haluck, MD; John J Knoedler, MD; Eric M Pauli, MD; Joshua S Winder, MD; Penn State Health Milton S. Hershey Medical Center

Background: High-risk patients presenting with complicated gallstone-related diseases are often managed with percutaneous cholecystostomy tube (PCT) drainage. For many, underlying disease precludes cholecystectomy and the PCT remains chronically. Percutaneous endoscopic biliary lithectomy (PEBL) is an emerging technique that offers stone clearance from the biliary system via drain tracts utilizing basket extraction, laser lithotripsy (LL), and/or electrohydraulic lithotripsy (EHL). Although PEBL avoids the morbidity of cholecystectomy or bile duct exploration, there is a dearth of literature on PEBL complications and their management.

Methods: An institutional review board approved, retrospective chart review was conducted for patients who underwent PEBL at a single, tertiary-care academic medical center from January 2020 to August 2022. Patient demographics, medical comorbidities, and indications for PEBL were reviewed. Interoperative and postoperative complications with subsequent management were subsequently evaluated.

Results: Twenty-eight patients who underwent a total of 53 PEBL procedures were included: mean age 68 ± 13 years, 60.7% male mean body mass index 30.6 ± 7.2 kg/m2, mean ASA 3, mean PCT duration of 211.8 ± 139.6 days. Most patients (67.9%, n = 19) were on therapeutic anticoagulation and/or antiplatelet agents. Patients required an average of 1.9 ± 0.9 PEBL procedures; 7.5% (n = 4) required LL and 37.7% (n = 20) required EHL. Average operative time was 90.1 ± 48.9 min. Complete stone clearance and PCT removal was achieved in 71.4% (n = 20) with the rest requiring a formal cholecystectomy (n = 2), continued PCT drainage (n = 4), an anticipated future PEBL (n = 1) or endoscopic retrograde cholangiopancreatography (n = 1). During a mean follow up duration of 58.7 ± 93.2 days were five (10.6%) complications: two intraoperative and three postoperative. One patient had gallbladder rupture during scout cholangiogram due to complete cystic duct obstruction, which was managed conservatively with PCT and ultimately open cholecystectomy. One patient had bleeding after scout cholangiogram was performed, which was initially managed conservatively but required postoperative cystic artery embolization. Postoperative complications included cholangitis that resolved with non-operative management, cellulitis at the drain site requiring antibiotics, and pancreatitis from drain obstruction of the pancreatic duct requiring drain repositioning under fluoroscopy within 24 h of the initial PEBL.

Conclusion: PEBL is a useful technique for biliary stone clearance in high-risk patients and negates the morbidity of operative interventions. Complications are not uncommon but the majority can be managed without urgent surgical intervention. A multidisciplinary care team is integral to the successful management of these patients and post-PEBL complications.

S265

Comparing Utilization Rates and Perioperative Outcomes of Intraoperative Cholangiography in Cholecystectomy

Ronald L Franzen1; Sandesh D Reddy1; David A Hsiou1; Laurynn Garcia 1; Eugene A Choi, MD2; Natasha S Becker, MD2; Christy Y Chai, MD2; 1Baylor College of Medicine; 2Michael E DeBakey Department of Surgery, Baylor College of Medicine; Michael E DeBakey VA Medical Center

Introduction: Intraoperative indocyanine green cholangiogram (ICGC) allows for visualization of biliary anatomy during cholecystectomy, potentially reducing the risk of bile duct injury. With its ease of use compared to intraoperative fluoroscopic cholangiogram (FC), the ICGC has gained significant interest in its role of delineating/confirming a "critical view" of safety prior to cystic duct/artery ligation. We examine the utilization of ICGC and FC during minimally invasive (MIS) cholecystectomies at our institution and clinical outcomes between patients who had cholangiography vs who did not.

Methods and Procedures: We conducted a retrospective chart review of patients who underwent MIS cholecystectomies as the index operation at a single academic institution from August 2020 to July 2022. We assessed rates of ICGC and FC by surgical approaches. In addition, we compared perioperative outcomes for patients who underwent ICGC and/or FC with those who did not. Model significance for surgical outcomes were determined using unpaired t-tests or Chi-squared tests for homogeneity.

Results: 192 consecutive patients (median age 61 yo, range 22-86) underwent MIS cholecystectomies between August 2020 to July 2022. Of those, 131 patients had an intraoperative cholangiogram (124 ICGC vs 9 FC). ICGC was performed more frequently in robotic cases (88.5% vs 45.4%). Six laparoscopic cases converted to open, 67% of which did not use cholangiography [Fig. 1]. Patients who underwent ICGC or FC trended toward higher rates of perioperative surgical complication (16.0% vs 9.8%, p = 0.25) and 30-day readmission rates (9.2% vs. 6.6%, p = 0.54) although they did not reach statistical significance. The same-day discharge rate and average length of stay for admitted patients were similar in both groups (40.5% vs. 44.3%, p = 0.62 and 3.5 days vs. 4.9 days, p = 0.24). No direct bile duct injury in the study group was reported.

Conclusion: While both ICGC and FC are used as an adjuvant to identify the biliary anatomy during cholecystectomies, their utility in improving clinical outcomes is unclear. However, increasing use of ICGC is noted compared to FC during MIS cholecystectomy (laparoscopic < robotic) likely due to its convenience and ease of use. We noted a higher trend for surgical complications and 30-day readmission rates in patients with cholangiograms, which could be due to selection bias in these patients having more difficult cholecystectomies and clinical presentations. We plan to investigate further the role and efficacy of ICGC in relation to preoperative and intraoperative variables.

figure dz

S266

Distinguishing Characteristics of Xanthogranulomatous Cholecystitis and Gallbladder Adenocarcinoma: A Persistent Diagnostic Dilemma

James J Reeves, MD; Estella Y Huang, MD; Ryan C Broderick, MD, FACS; Nicole Goldhaber, MD; Joaquin L Serra, MD; Julie An, MD; Kathryn Fowler, MD; Mojgan Hosseini, MD; Bryan J Sandler, MD, FACS; Garth R Jacobsen, MD, FACS; Santiago Horgan, MD, FACS; Bryan M Clary, MD, FACS; UCSD

Introduction: Xanthogranulomatous cholecystitis (XGC) is a rare variant of chronic cholecystitis which can resemble gallbladder adenocarcinoma (GAC) on pre-operative imaging. We examined our experience with each pathology to identify distinguishing characteristics that may guide patient counseling and surgical management.

Methods and Procedures: A retrospective review of all pathologically confirmed cases of XGC and GAC following cholecystectomy between 2015 – 2021 was performed. Clinical, biochemical, radiographic, and intra-operative features were compared. A statistical analysis was conducted utilizing Wilcoxon rank sum for continuous variables and Fisher's exact test for categorical variables.

Results: There were thirty-seven cases of XGC and twenty cases of GAC. Patients with GAC were older (mean 70.3 years vs 58.0, p = 0.01) and exclusively female (100% vs 45.9%, p < 0.0001). The primary presenting symptom was abdominal pain for both XGC (83.7%) and GAC (75.0%) and there were no significant differences between accompanying symptoms (nausea/vomiting, fevers, or jaundice). The mean maximum white blood cell count was elevated for XGC compared to GAC (16.4 vs 8.6 respectively, p = 0.044); however, there were no differences in the remainder of the biochemical profile, including bilirubin, liver transaminases, CEA, and CA 19-9 (mean maximum [SD] 70.6 [136.1] for GAC and 134.9 [366.9] for XGC, p = 0.67). A comparison among each major imaging modality is provided in Table 1. The presence of an intraluminal mass (61.1% vs 9.1%, p = 0.0001) and lymphadenopathy (18.8%. vs 0.0%, p = 0.045) were associated with malignancy, whereas gallstones (76.5% vs. 47.4%, p = 0.04) and gallbladder wall thickening (87.9% vs 38.9%, p = 0.0008) were more often present with chronic cholecystitis. There were no intraoperative findings that reliably distinguished between diagnoses, though cases of XGC more often had significant adhesions/inflammation (83.8% vs 55.0%, p = 0.03). Both XGC (45.9%) and GAC (50%) frequently had large aggregate stone burdens. Among cases of XGC with pre-operative concern for malignancy, 9/10 (90%) were able to be completed laparoscopically.

Conclusion(s): Clinical features that may favor benign chronic cholecystitis over gallbladder adenocarcinoma include younger age, male gender, current or prior leukocytosis, and the absence of an intraluminal mass or lymphadenopathy. Laparoscopic cholecystectomy is a safe surgical option for equivocal presentations. Intra-operative frozen section or intentional staging of more extensive procedures based upon final histopathology are valuable surgical strategies.

S267

Use of Targeted Education Resources to Improve Robotic Bariatric Surgery Training

Julie M Clanahan, MD; Michael M Awad, MD, PhD; Francesca Dimou, MD, MS; Washington University in St. Louis

Introduction: Despite the growth of robotic surgery within bariatrics, evidence for how to best train surgical residents for these cases is lacking. We developed a series of targeted educational resources to promote progression on the robotic bariatric learning curve. This study aimed to characterize the effect of these resources on resident participation in robotic bariatric procedures.

Methods & Procedures: Performance metrics from daVinci Surgical System were retrospectively reviewed for all sleeve gastrectomy (SG) and Roux-en-Y gastric bypass (RYGB) cases involving general surgery trainees and a single robotic bariatric surgeon at one institution. Annotated pictorial case guides (Image 1) and narrated operative videos were then developed for robotic SG/RYGB and disseminated to trainees. Percent active control time (%ACT)—amount of trainee console time spent in active system manipulations over total active time from both consoles—was the primary outcome measure following dissemination. Kruskal-Wallis and Mann-Whitney U statistical tests were applied. Free-text feedback was reviewed from faculty evaluations.

Results: From September 2020-July 2021, 50 cases comprised of 54% SG and 46% RYGB involving 14 unique trainees (PGY1-PGY5) were included. Baseline median %ACT for cases in aggregate was 11% (IQR 5-29%) for PGY1s, 18% (IQR 12-27%) for PGY3s, 35% (IQR 25-40%) for PGY4s, and 35% (IQR 29-49%) for PGY5s (p = 0.0016). From November 2021-May 2022 following dissemination, 29 cases comprised of 31% SG and 69% RYGB involving 8 unique trainees were included. Median %ACT was observed to increase with trainee level for cases in aggregate (38% [IQR 27-49%] PGY3s, 45% [IQR 39-54%] PGY4s, 55% [IQR 36-62%] PGY5s, p = 0.14). When compared by trainee level, median %ACT significantly increased across most groups following resource distribution (PGY3s 18% v 38%, p = 0.083; PGY4s 35% v 45%, p = 0.0005; PGY5s 35% v 55%, p = 0.0017) (Fig. 1). Feedback from participants highlighted high utility of the resources including their clear detail of operational set-up, steps, and critical portions. Many requested similar resources for other robotic procedures.

Conclusions: In this study, use of targeted educational resources promoted increases in trainee participation in robotic bariatric procedures with more time spent operating at the console instead of observing. In addition, trainees reported that such resources helped them to systematically prepare and progress through key operative steps. As educators continue to develop robotic curricula, efforts should include additional resource development for other sub-specialty procedures.

Image 1 Sample from RYGB pictorial case guide

figure ea

Fig. 1 Comparison of trainee participation before and after resources

figure eb

S268

Training for Excellence: Using a Multimodal Teleconferencing Platform to Coach Surgeons and Improve Intra-operative Performance

Golsa Shafa, BSc1; Parmiss Kiani, BSc 2; Caterina Masino, MA2; Allan Okrainec, MD, MHPE3; Jesse Pasternak, MD, MPH3; Adnan Alseidi, MD, MEd4; Amin Madani, MD, PhD2; 1Temerty Faculty of Medicine University of Toronto, Toronto, ON, Canada; 2Temerty Advanced Surgical Education and Simulation Centre, University Health Network, Toronto, ON, Canada; 3Division of General Surgery, University Health Network, Toronto, ON, Canada; 4Department of Surgery, University of California, San Francisco, California, USA

Introduction: Continuing Professional Development opportunities for lifelong learning are fundamental to the acquisition of surgical expertise. However, few opportunities exist for longitudinal and structured learning to support the educational needs of surgeons in practice. While peer-to-peer coaching has been proposed as a potential solution, there remains significant logistical constraints and a lack of evidence to support its effectiveness. The purpose of this study is to determine whether the use of remote videoconferencing for video-based coaching improves operative performance.

Methods: Early career surgeons (mentees) were asked to select a coach of their choice. Remote coaching and telestration was implemented using Zoom platform (Zoom Video Communications, San Jose, CA). This enabled the unrestricted sharing of radiological study images, surgical videos and annotation with free-hand drawing directly onto the radiological images and/or surgical field in order to articulate feedback. The coach evaluated mentee performance using the Intra-operative Performance Assessment Tool (IPAT), a validated assessment tool. All participants completed a 5-point Likert scale on the educational value of the coaching program.

Results: Eight surgeons were enrolled in the study (75% male, 25% female), six of which completed a total of two coaching sessions (baseline and 6-month) during the 12-month study period. Subspecialties included surgical oncology, endocrine surgery and hepatopancreatobiliary surgery. Mean age of the participants was 39 (± 3.3), with mean 5 (± 4.1) years in independent practice. Total IPAT scores increased significantly from the first session (mean 47.0 ± 1.9) to second session (mean 51.8 ± 2.1), p = 0.03. Specifically, subset analysis showed improvement only in the Advanced Cognitive Skills domain (33.2 ± 2.5 to 37.0 ± 2.4, p < 0.01), including items such as "tissue handling", "tactical modification" and "threat/risk perception". There was no improvement in the Psychomotor Skills category (4.0 to 4.2 ± 0.4, p = 0.2). Almost all participants either agreed or strongly agreed that the coaching program facilitated the acquisition of their surgical skills and decision-making, and improved their overall performance (coaches: 85%; mentees: 100%).

Conclusion: Remote peer-to-peer video-based surgical coaching seems to be feasible and has educational value using ubiquitous commercially-available teleconferencing platforms. Logistical issues with scheduling and finding cases aligned with learning objectives continues to be a main barrier to program adoption and widespread dissemination.

S269

Evidence Supporting Performance Measures of Laparoscopic Appendectomy Using a Novel Appendectomy Skills Assessment Tool and Low-Cost Training System

Christopher W Reynolds 1; Deborah M Rooney1; David Jeffcoach2; Melanie Barnard3; Mark J Snell4; Blessing Ngoin Ngam4; John Tanyi4; C. Yoonhee Ryder1; Kevin El-Hayek5; Grace J Kim1; 1University of Michigan Medical School; 2Soddo Christian Hospital; 3Southern Illinois University; 4Mbingo Baptist Hospital; 5MetroHealth

Background: Laparoscopic appendectomy is one of the most common surgical operations performed in the world with proven advantages over the open approach for patients. However, training in laparoscopy remains inaccessible for surgeons in many low- and middle-income countries (LMICs). We developed an assessment tool to support learning, practice, and assessment of laparoscopic skills in appendectomy to be used in ALL-SAFE, our low-cost laparoscopic training system.

Methods: We conducted a pilot study in July 2022 at sites in Ethiopia, Cameroon, and the United States to assess the evidence supporting the performance measures of the novel ALL-SAFE appendectomy skills assessment tool (APPY-VOP). Participants viewed a total of three videos, one each from novice, intermediate, and expert users performing a laparoscopic appendectomy using the ALL-SAFE training system. Participants then rated the three videos using the APPY-VOP across its three components: a checklist consisting of key tasks and avoidance of critical errors, a modified Objective Structured Assessment of Technical Skills (m-OSATS), and an overall competency rating. Analysis using the Kruskal-Wallis test was performed to determine: i) the capacity of the APPY-VOP to differentiate novice, intermediate, and expert performance and ii) inter-rater agreement among novice and expert raters.

Results: There were 12 participants including medical students, residents, and expert laparoscopic surgeons. The checklist summed scores increased from novice (M = 15.45) to intermediate (M = 19.67) and expert (M = 22.67) for all items with statistical significance reached in 6 of 13 individual items. The checklist (P < 0.001) and m-OSATS (P < 0.001) summed scores both differentiated novice from expert performance, regardless of the rater's level of expertise. The overall competency rating effectively discriminated between novice (M = 1.18), intermediate (M = 2.33), and expert (M = 3.00) performances (P < 0.001). Analysis of rating differences across novice (n = 12, M = 3.1) and expert (n = 3, M = 2.9) raters suggested no statistical difference between these groups.

Conclusions: All components of APPY-VOP discriminated novice, intermediate, and expert performance in laparoscopic appendectomy skills. Rating alignment among novice and expert judges suggests consistent evaluation, regardless of expertise. These results support the use of APPY-VOP among users of all skill levels and could support the implementation of a peer review system for ALL-SAFE. This could minimize the demands on faculty and local experts to increase scalability of the low-cost training system in low-resource settings. Future studies will focus on calibrating feedback to peers and exploring future operation modules for ALL-SAFE.

S270

The Impact of Virtual Interviews on the Fellowship Council Application and Main Match Process

Douglas J Cassidy, MD 1; Julie M Clanahan, MD1; Rohan Jeyarajah, MD2; L. Michael Brunt, MD1; 1Washington University in St. Louis; 2TCU School of Medicine and Methodist Richardson Medical Center

Background: In March 2020, the COVID pandemic led the Fellowship Council (FC) to transition rapidly from in-person to virtual interviews mid-interview cycle. As interviews have since remained virtual, we investigated the impact of in-person to virtual interviews on the FC application and main match process.

Methods: Five years (2018-2022) of deidentified FC applicant, program, and match rank data was used to assess differences between in-person (2018-2019) and virtual interview (2021-2022) cycles. The 2020 application year had both in-person and virtual interviews and was excluded from comparative analysis. Data are mean ± SD and statistical analysis was by one-way and two-way MANOVA.

Results: Over 5 years, there were 1493 applicants in the FC Main Match. Residents applied to an average of 30.4 ± 24.3 programs and ranked an average of 10.7 ± 9.7 programs with a 57% match rate and average rank position of 3.6 ± 3.3. Fellowship programs received an average of 64.9 ± 28.6 applications and ranked an average of 15.4 ± 8.8 applicants with a 95% match rate and average applicant rank position of 3.0 ± 3.4.

Applicants who interviewed virtually applied to a greater number of programs (32.7 vs. 27.0; p < 0.001) and ranked a greater number of programs (11.5 vs. 10.0; p = 0.004) but had no difference in match rates (58% vs. 55%, p = 0.291). Amongst matched applicants, there was a significant difference in average rank position (3.20 vs. 4.30, p < 0.001), favoring the in-person cohort. Likewise, fellowship programs had more applicants per program (69.2 vs. 57.8; p < 0.001) and ranked more applicants (17.4 vs. 13.3; p < 0.001) during the virtual interview cycles. There was no difference in match rates (93% vs. 96%, p = 0.178) and applicant rank position (3.09 vs. 2.93, p = 0.561) between in-person and virtual application cycles. Further analysis by fellowship program designation was also significant for an increase in the number of applicants (p < 0.001) and applicants ranked (p < 0.002) during the virtual interview cycle, but with no effect on program match rates (p = 0.415).

Conclusions: The transition to virtual interviews increased the number of applications for fellowship and applicants ranked by programs but did not impact match rates of either group. Rank match position declined for applicants but not for fellowship programs. These findings suggest that virtual interviews offer more opportunities for applicants and candidates for fellowship programs with only a slight decrement in fellow match rank position. These changes should be considered against the cost and convenience benefits of the virtual interview process as the FC guides future application cycles.

S271

Characterization of the Robotic Surgery Experience in Minimally Invasive Surgery Fellowships from 2010-2021

Nathan Haywood, MD; Joshua Scott, MD; Aimee Zhang, MD; Matthew Byler, MD; Peter Hallowell, MD; Bruce Schirmer, MD; University of Virginia

Introduction: Robotic surgery has experienced exponential growth in the past decade. Few studies have evaluated the impact of robotics within minimally invasive surgery (MIS) fellowship training programs. Trainees report a wide variety of robotic experience and subsequent impact on laparoscopic or open operative volume. The purpose of our study was to examine and characterize recent trends in robotic surgery within MIS fellowship training programs.

Methods: De-identified case log data from the Fellowship Council from 2010-2021 was evaluated. Percentage of operations performed with robot assistance over time was assessed and compared to the laparoscopic and open experience. Case logs were further stratified by operative category (ie. bariatric, hernia, hepatobiliary) and robotic experience over time was evaluated for each category. Programs were stratified by overall percent robot use and the experience over time within each quartile was evaluated.

Results: The number of MIS fellowship training programs with robotic platform increased from 46.8% (66/141) in 2010 to 88.2% (157/178) in 2021. Among programs offering robotic experience, the percent of cases performed per year with robotic assistance increased from 4.1% (1282/31372) to 23.4% (17941/76738). Among all MIS training programs, the percent of laparoscopic cases per year decreased from 74.2% (47708/64265) to 57% (50042/87842) from 2010 to 2021 while the percent of robotic cases increased from 2.0% (1282/64265) to 20.4% (17941/87842). Hernia and colorectal procedural categories had the largest increase in robot usage from 2010 to 2021 [hernia: 0.7% (62/9147) to 37.2% (4787/12870); colorectal 5% (213/4228) to 35% (1248/3564)]. Bariatric [1.8% (335/18445) to 20.5% (5801/28236)] and foregut [2.4% (156/6521) to 23.0% (3011/13111)] categories also demonstrated substantial increase (Figure A). When stratified by percentage of robot utilization, programs from 2010-11 in the > 95th percentile performed 63.3% (812/1282) of all robotic cases. Current (2020-21) programs in the > 95th percentile performed 28.2% (5062/17941) of robotic operations and programs in the > 50th percentile performed 91.7% (16458/17941) of all robotic cases (Figure B).

Conclusions: Robotic use in MIS fellowship training programs has grown substantially in the past decade. The experience by operative category differs and there remains an imbalance with the top 50% of busiest robotic programs performing over 90% of robot trainee cases. The robotic experience in MIS programs varies widely and trainees should examine program specific case logs closely to confirm parallel interests.

figure ec

S272

Residents Perform Better Technically, Have Less Stress And Workload, And Prefer Robotic To Laparoscopic Technique During Inanimate Simulation

Kristine Kuchta, MS; Sung Hoon Choi; Aram Rojas, MD; Syed Mehdi, MBBS, MPH; Melissa Ramirez-Braga; Mark Talamonti, MD, FACS; Melissa Hogg, MS, FACS; NorthShore University HealthSystem

Introduction: With the widespread adoption of minimally invasive surgery, there is a growing need for surgical residents to be trained by a procedure-specific curriculum. The current study aimed to evaluate the technical performance and feedback of surgical residents undergoing the robotic and laparoscopic hepaticojejunostomy (HJ) and gastrojejunostomy (GJ) biotissue modules.

Methods: The study was conducted in a simulation laboratory with laparoscopic equipment and da Vinci Surgical System. PGY-3 surgical residents participated in a robotic curriculum and were asked to perform the HJ and GJ drill laparoscopically following the first robotic anastomosis respectively. The drills were recorded and scored by two independent graders using the objective structured assessment of technical skills (OSATS). After completing each drill, all participants filled out the NASA Task Load Index (NASA-TLX), Borg Exertion Scale (10 points), and Edwards Arousal Rating Questionnaire. Statistical comparisons were made using the Wilcoxon rank-sum test.

Results: A total of 23 surgical residents participated in this study. 22 (95.7%) residents had already received fundamentals of laparoscopic surgery certification and 20 (86.9%) had experienced ≥ 50 laparoscopic surgeries. No resident had previous robotic virtual simulation experience, only 4 residents (16.4%) had robotic inanimate box training, and median number of robotic cases was 4 (0-30). In the HJ comparison of the six OSATS domains, the robotic system was superior in Gentleness (p = 0.0375) with a trend in Instrument Handling (p = 0.0501). In the GJ comparison, the robotic system was superior in Time and Motion (p = 0.0002), Instrument Handling (p = 0.0010), Flow of Operation (p = 0.0057), Tissue Exposure (p = 0.0188), and Summary (p = 0.0021). Participants answered significantly higher demand scores on all six facets of NASA-TLX for the HJ and GJ (p < 0.05). The Borg Level of Exertion was > 2 points higher for laparoscopic HJ and GJ (p < 0.0001). Residents rated more Nervousness and Anxiety for laparoscopic compared to robotic (p < 0.05) HJ and GJ. Additionally, when asked to score preference for robotic and laparoscopic approach in terms of technique and ergonomics, residents scored robot as better (laparoscopy worse) for both HJ and GJ in both domains.

Conclusions: Despite less experience in robotic training and surgery compared to laparoscopic, the robotic surgical system provided a more favorable environment for trainees with a less mental and physical burden for minimally invasive HJ and GJ curriculum.

figure ed

S273

Application of the Acquisition of Data for Outcome and Procedure Transfer (ADOPT) method to a hands-on course for teaching eTEP hernia repair to practicing surgeons

Mary Kate Bryant, MD, MSCR 1; Rachel Sillcox, MD1; Erin Schwarz, BA2; Jonathan Dort, MD, FACS3; Andrew S Wright, MD1; 1University of Washington; 2Society of American Gastrointestinal and Endoscopic Surgeons; 3University of Virginia School of Medicine/Inova Health System

Background: After completion of training, practicing surgeons rely on various learning instruments to expand their procedure armamentarium and improve their surgical technique. However, courses designed to accomplish such goals rely on short time blocks and vary in standardized teaching methods. SAGES developed the Acquisition of Data for Outcomes and Procedure Transfer (ADOPT) program as a method of longitudinal instruction utilizing standardized teaching techniques, mentorship, and webinars to cover additional techniques. This study examines the adoption of learned techniques and participant confidence before and after an ADOPT course, focused on extended-view totally extraperitoneal (eTEP) hernia repair.

Methods: A hands-on eTEP hernia repair course was conducted with enrollment capped at 10 participants. The pre-course survey collected demographics, training records, self-reported hernia case volume, and confidence related to skills specific to eTEP procedure. A three-month post-course survey determined the implementation of the learned procedure, case volume, and confidence with eTEP skills. A 5-point Likert scale (1 = not confident at all to 5 = completely confident) assessed confidence levels. Survey responses were summarized using descriptive statistics.

Results: Of the 10 participants, 10 (100%) completed the pre-course survey, and 6 (60%) completed the post-course survey. Course participants were a median age of 48.5y (36,56) with a median of 16y (2,23) in practice, mostly in the community setting (70%). After the course, 50% had performed an eTEP procedure within three months, and 100% reported considering this technique during surgical planning. The most reported barrier to implementation was the lack of appropriate patients for the technique. Participants reported higher confidence in eTEP-specific skills at three months post-course from pre-course levels (Fig. 1). The greatest change in confidence was seen for the following skills: accessing the retromuscular/extraperitoneal space for ventral hernia and recognizing when the linea alba has been violated, p < 0.05.

Conclusion: This study shows rapid incorporation of learned techniques can be achieved through the ADOPT format. Furthermore, through longitudinal mentorship and a structured hands-on course, the ADOPT course supports practicing surgeons to attain autonomy and confidence even when teaching a relatively technically challenging procedure, such as eTEP.

figure ee

S274

Development of FLS 2.0 Integrated Tasks: A Stepwise Approach

Maryam Wagner, PhD 1; Carlos Gomez, PhD1; Allan Okrainec, MD2; Neal Seymour, MD3; Melina M Vassiliou, MD1; 1McGill University; 2University of Toronto; 3University of Massachusetts Chan Medical School—Baystate

Introduction: The purpose of this study was to initiate the development of new, integrated tasks for the Fundamentals of Laparoscopic Surgery (FLS) 2.0 using a stepwise approach. SAGES is updating the FLS program in response to advances in MIS and education. FLS 2.0 is based on a framework of ten Entrustable Professional Activities (EPAs) and underlying competencies that together define the construct of the fundamentals of laparoscopy. It will include a new version of the current examination and additional, integrated assessments.

Methods: We employed a multi-method approach consisting of: 1) Administering a survey during multiple online retreats with subject-matter experts (SMEs) to identify competencies that should be assessed. The findings of this survey were cross-validated with the results of a SAGES-wide survey investigating the priorities for technical skills testing; 2) Interviewing 10 experts querying how they distinguish level of expertise across each skill; and 3) Engaging a diverse group of SMEs and education scientists in a retreat to analyze samples of various levels of performance on three of the current skills tasks. Retreat participants were provided with simple materials to develop prototypes for integrated tasks.

Results: Data were analyzed across all methods in steps 1 and 2 to identify 22 of the 67 competencies that were selected for skills testing. These competencies were further refined to identify a cluster of the most important competencies:

  • Operates laparoscope and camera effectively, maintaining focus, horizon, and directional control of scope

  • Applies appropriate force while maintaining traction/countertraction

  • Separates tissue using sharp, blunt, or energy-based instruments

  • Exposes and dissects correct tissue plane

  • Uses operative strategies and techniques to improve exposure

  • Performs effective laparoscopic interrupted suturing and knot tying without tension

  • Appraises given working space while suturing and adapts technique to avoid injury to adjacent structures

Several of the key performance markers and conceptually accurate distinguishing behaviours that will be used in measuring performance were identified (e.g., instrument handling). Three groups created initial integrated task prototypes mapped to these competencies.

Conclusions: These findings represent a rigorous, step-wise approach to the development of FLS 2.0 integrated tasks. The methods ensure that the essential aspects of current FLS manual skills testing are preserved, while also including the recently identified and highest priority competencies in a more integrated and authentic way. These changes align with feedback from stakeholders, and strive for a program that remains relevant and provides value to all users.

S275

Automatic Assessment of Performance in the FLS Trainer using Computer Vision

Aviad Sela Lazar1; Gideon Sroka, MD, MSc 2; Shlomi Laufer, PhD1; 1Technion—Israel Institute of Technology; 2Bnei Zion Medical Center

Introduction: Fundamentals of Laparoscopic Surgery (FLS) box trainer is a well-accepted method for training and evaluating laparoscopic skills. It mandates an observer that will measure and evaluate the trainee's performance. Measuring performance in the Peg Transfer task includes time and penalty for dropping pegs. This study aimed to assess whether computer vision (CV) may be used to automatically measure performance in the FLS box trainer.

Methods: Four groups of metrics were defined and measured automatically using CV. Validity was assessed by dividing participants to 3 groups of experience levels. Twenty-seven participants were recorded performing the PegTransfer task 2-4 times, amounting to 72 videos. Frames were sampled from the videos and labeled to create an image dataset. Using these images, we trained a deep neural network (YOLO) to detect the different objects in the video (pegs, graspers, triangles). We developed an algorithm that tracks the transfer of the triangles. The metric groups were: Time, Grasper Movement Speed, Path Efficiency, and Grasper Coordination. The performance was compared based on their last video (3 participants were excluded due to technical issues).

Results:

figure ef

Conclusions: The statistical analysis shows that the metrics can differentiate between the experts and novices performing the task. Thus they may provide a more detailed performance analysis than is currently used. Moreover, these objective metrics are calculated automatically based solely on the video camera of the FLS trainer. Thus, they provide the opportunity for independent training and assessment.

S276

Utilizing the LAP-VEGaS Guidelines to Assess for the Educational Quality of Online Laparoscopic Jejunostomy Training Videos

Jeffrey L Roberson, MD; Armaun Rouhi, BA; William Yi, MD; Amanda Bader, MD; Jon B Morris, MD; Noel Williams, MD; Kristofell Dumon, MD; The Hospital of the University of Pennsylvania

Introduction: Laparoscopic feeding jejunostomy is a common enteral access option for patients who cannot tolerate oral or gastric nutrition. However, there is a dearth of literature on the educational quality of training videos available publicly on YouTube®. The LAParoscopic surgery Video Educational GuidelineS (LAP-VEGaS) video assessment tool, released in 2020, has been developed to ensure that teaching videos are of appropriate quality. Consisting of nine different items, each category is scored from zero (not presented in the video) to two (discussed extensively). In order to determine the overall educational value, this study applies the LAP-VEGaS tool to currently available laparoscopic jejunostomy videos.

Methods: A retrospective review of YouTube® videos was conducted for "laparoscopic jejunostomy". Results were sorted by relevance and the first 50 videos were gathered for consideration of inclusion. Videos with patient education or concomitant procedures were excluded. Included videos were categorized as physician-produced, academic center-produced, commercial-produced, or professional society produced and rated by two independent investigators using LAP-VEGaS video assessment tool (0-18). One-way ANOVA with Bonferroni correction was used to evaluate differences in LAP-VEGaS scores between video categories and date of publication relative to 2020. Spearman's correlation test was performed to measure association between scores and length, number of views and likes.

Results: 27 unique videos met selection criteria. The average LAP-VEGaS score was 8.69 ± 4.76 with a median of 9.00 (range 2-17). Inter-rater reliability was 0.77. Academic and physician video walkthroughs did not demonstrate a significant difference in mean scores (9.92 ± 4.31 vs 8.33 ± 4.93, p = 0.4836). There was no difference in mean scores between videos published before or after 2020 (7.59 ± 4.85 vs. 10.55 ± 4.23, p = 0.0908). A majority of videos failed to provide patient position (52%), intraoperative findings (56%), operative time (63%), graphic aids (74%), and audio/written commentary (52%). A positive association was demonstrated between scores and number of likes (rs = 0.59, p = 0.0011) and video length (rs = 0.39, p = 0.0421), but not number of views (rs = 0.17, p = 0.3991).

Conclusion: The majority of available YouTube® videos on laparoscopic jejunostomy fail to meet the basic educational needs of surgical trainees, and there is no difference between those produced by academic centers or independent physicians. Similarly, there has been no improvement in video quality following the release of the scoring tool. Standardization of laparoscopic jejunostomy training videos with the LAP-VEGaS score, as well as increased publicity of the metric, can ensure that videos are of appropriate educational value with logical structure.